You are on page 1of 161

ĐỖ MINH TRIẾT

ÔN THI OLYMPIC TOÁN ĐẠI


SỐ SINH VIÊN VIỆT NAM
TOÀN TẬP

tủ sách -MATHTASY-
Copyright © 2017 Đỗ Minh Triết

XUẤT BẢN BỞI MATHTASY

facebook.com/mathtasyvn

MATHTASY@GMAIL.COM
LỜI NÓI ĐẦU

Quyển sách nhỏ bé này được tích cóp trong quá trình học tập của bản thân của tác giả, là kết
quả của những năm ngồi trên ghế nhà trường Sư phạm. Giờ đây mong nó sẽ có ích cho các em
sinh viên thế hệ sau này.

Sai sót là điều không tránh được, mọi ý kiến, đóng góp xin liên hệ qua

Đỗ Minh Triết

Website: facebook.com/mathtasyvn | toanhockithu.wordpress.com

Facebook cá nhân: facebook.com/mtrietdo

E-mail: mathtasy@gmail.com | trietdo@hotmail.com

Sách được cấp miễn phí tại Mathtasy, vui lòng không sao chép cũng như đem qua các website,
forum khác, nếu có vui lòng dẫn link trực tiếp đến fanpage Mathtasy.

Vũng Tàu, Ngày 11 tháng 7 năm 2017

Tác giả

Đỗ Minh Triết
MỤC LỤC

PHẦN I. SỐ PHỨC (Complex number) .................................................................. 11

1.1. Kiến thức cơ bản về số phức ℂ........................................................................................ 11

1.1.1. Dạng lượng giác của số phức..................................................................................... 11

1.1.2. Dạng mũ của số phức ............................................................................................... 11

1.1.3. Luỹ thừa và khai căn số phức ................................................................................... 12

1.2. Tích vô hướng (inner product, scalar product) của hai vector trên trường số phức ......... 12

PHẦN II. MỘT SỐ KIẾN THỨC CƠ BẢN VỀ MA TRẬN.............................. II-14

2.1. Định nghĩa các ma trận .............................................................................................. II-14

2.1.1. Ma trận nghịch đảo, ma trận khả nghịch


(The Inverse of the Matrix, Matrix Inversion) .......................................................... II-14

2.1.2. Ma trận chuyển vị (Transpose of the Matrix) ....................................................... II-14

2.1.3. Ma trận liên hợp ................................................................................................... II-14

2.1.4. Ma trận chuyển vị liên hợp ................................................................................... II-15

2.1.5. Ma trận đồng dạng ............................................................................................... II-15

2.1.6. Ma trận xác định dương (Positive Definite Matrix) .............................................. II-15

2.1.7. Ma trận đối xứng (Symmetric Matrix).................................................................. II-16

2.1.8. Ma trận phản đối xứng ......................................................................................... II-16

2.1.9. Ma trận trực giao (Orthogonal Matrix) ................................................................ II-16

2.1.10. Ma trận unita (Unita Matrix) ............................................................................. II-17

2.1.11. Ma trận Hermite (đối xứng) (Hermite Matrix).................................................... II-17

2.1.12. Ma trận Hermite đối xứng lệch ........................................................................... II-18

2.1.13. Ma trận băng ...................................................................................................... II-19

2.1.14. Ma trận phụ hợp (Adjunct Matrix, Adjutage Matrix) ........................................ II-19

2.1.15. Ma trận vandermonde (Vandermonde Matrix).................................................... II-19

2.2. Bài tập ........................................................................................................................ II-20


PHẦN III. MA TRẬN KHỐI ............................................................................. III-32

3.1. Cơ sở lý thuyết .......................................................................................................... III-32

3.2. Bài tập ....................................................................................................................... III-32

PHẦN IV. MA TRẬN LUỸ LINH (Nilpotent Matrix) .....................................IV-39

4.1. Định nghĩa và tính chất ............................................................................................. IV-39

4.2. Bài tập ....................................................................................................................... IV-41

PHẦN V. VẾT CỦA MA TRẬN (Trace of the Matrix) .................................... V-46

5.1. Định nghĩa và tính chất .............................................................................................. V-46

5.2. Bài tập ........................................................................................................................ V-46

PHẦN VI. HẠNG CỦA MA TRẬN (Rank of the matrix) ...............................VI-49

6.1. Định nghĩa và tính chất ............................................................................................. VI-49

6.2. Bài tập ....................................................................................................................... VI-49

PHẦN VII. GIÁ TRỊ RIÊNG, VECTOR RIÊNG (Eigenvalue, Eigenvector) VII-62

7.1. Cơ sở lý thuyết .........................................................................................................VII-62

7.2. Bài tập ......................................................................................................................VII-68

PHẦN VIII. ĐA THỨC ĐẶC TRƯNG, ĐA THỨC TỐI TIỂU ..................... VIII-87

8.1. Cơ sở lý thuyết ....................................................................................................... VIII-87

8.2. Bài tập .................................................................................................................... VIII-87

PHẦN IX. ĐỊNH THỨC (Determinant) ............................................................ IX-93

9.1. Cơ sở lý thuyết .......................................................................................................... IX-93

9.1.1. Tính định thức .................................................................................................... IX-93

9.1.2. Sử dụng định nghĩa định thức giải một số bài toán liên quan ............................ IX-117
9.2. Bài tập ..................................................................................................................... IX-118

PHẦN X. LUỸ THỪA MA TRẬN (Power of the Matrix)............................... X-125

10.1. Luỹ thừa ma trận cấp 3 .......................................................................................... X-125

10.2. Luỹ thừa ma trận cấp 2 .......................................................................................... X-131

10.3. Luỹ thừa ma trận cấp cao (𝑛𝑛 ≥ 4) .......................................................................... X-135

10.4. Luỹ thừa ma trận qua các bài toán về dãy truy hồi tuyến tính............................... X-138

PHẦN XI. BÀI TOÁN XÁC ĐỊNH MA TRẬN .............................................. XI-144

11.1. Tìm mọi ma trận 𝑋𝑋 giao hoán


hoặc phản giao hoán với một ma trận 𝐴𝐴 cho trước. ........................................................ XI-144

11.2. Phương trình ma trận ............................................................................................ XI-148

11.3. Các dạng khác ........................................................................................................ XI-151

PHẦN XII. HỆ PHƯƠNG TRÌNH TUYẾN TÍNH ....................................... XII-153

PHẦN XIII. MỘT SỐ BÀI TOÁN KHÁC.................................................... XIII-156


PHẦN I. SỐ PHỨC (Complex number)

1.1. Kiến thức cơ bản về số phức ℂ

1.1.1. Dạng lượng giác của số phức


a) Đưa số phức dạng đại số (algebraic form) về số phức dạng lượng giác
(trigonometric form)
Giả sử ta muốn đưa số phức 𝑧𝑧 = 𝑎𝑎 + 𝑏𝑏𝑏𝑏 (𝑎𝑎, 𝑏𝑏 ∈ ℝ) về dạng lượng giác dạng 𝑟𝑟(cos 𝜑𝜑 + 𝑖𝑖 sin 𝜑𝜑), thì

modun 𝑟𝑟 = 𝑎𝑎2 + 𝑏𝑏2 và argument 𝜑𝜑 được xác định như sau:
Rõ ràng là
𝑎𝑎 = 𝑟𝑟 cos 𝜑𝜑
� 𝑏𝑏 = 𝑟𝑟 sin 𝜑𝜑

Ta xét các trường hợp:


- Nếu 𝑎𝑎 = 0, 𝑏𝑏 ≠ 0 thì dễ thấy
𝜋𝜋
⎧ , 𝑏𝑏 > 0
�2
𝜑𝜑 = ⎨3𝜋𝜋
� , 𝑏𝑏 < 0
⎩2
- Nếu 𝑎𝑎 ≠ 0 thì từ tan 𝜑𝜑 = 𝑎𝑎𝑏𝑏 , ta được
𝑏𝑏
𝜑𝜑 = arctan + 𝑘𝑘𝑘𝑘
𝑎𝑎
trong đó
0 nếu 𝑎𝑎 > 0, 𝑏𝑏 ≥ 0 (𝑎𝑎, 𝑏𝑏) ∈ 𝐼𝐼
𝑘𝑘 = �1 nếu 𝑎𝑎 < 0, 𝑏𝑏 ∈ ℝ hay (𝑎𝑎, 𝑏𝑏) ∈ 𝐼𝐼𝐼𝐼, 𝐼𝐼𝐼𝐼𝐼𝐼
2 nếu 𝑎𝑎 > 0, 𝑏𝑏 < 0 (𝑎𝑎, 𝑏𝑏) ∈ 𝐼𝐼𝐼𝐼
b) Các phép Toán
- Phép nhân:
𝑧𝑧1 𝑧𝑧2 = 𝑟𝑟1 (cos 𝜑𝜑1 + 𝑖𝑖 sin 𝜑𝜑1 ). 𝑟𝑟2 (cos 𝜑𝜑2 + 𝑖𝑖 sin 𝜑𝜑2 ) = 𝑟𝑟1 𝑟𝑟2 [cos(𝜑𝜑1 + 𝜑𝜑2 ) + 𝑖𝑖 sin(𝜑𝜑1 + 𝜑𝜑2 )]
- Phép chia:
𝑧𝑧1 𝑟𝑟1
= [cos(𝜑𝜑1 − 𝜑𝜑2 ) + 𝑖𝑖 sin(𝜑𝜑1 − 𝜑𝜑2 )] (𝑧𝑧2 ≠ 0)
𝑧𝑧2 𝑟𝑟2
- Luỹ thừa (Định lý De Moivre):
∀𝑛𝑛 ∈ ℕ: 𝑧𝑧𝑛𝑛 = [𝑟𝑟(cos 𝜑𝜑 + 𝑖𝑖 sin 𝜑𝜑)]𝑛𝑛 = 𝑟𝑟𝑛𝑛 (cos 𝑛𝑛𝑛𝑛 + 𝑖𝑖 sin 𝑛𝑛𝑛𝑛)

1.1.2. Dạng mũ của số phức

Đỗ Minh Triết 11 - MATHTASY -


Ôn thi Olympic Toán Đại số Phần I: Số phức

Công thức Euler: cos 𝜑𝜑 + 𝑖𝑖 sin 𝜑𝜑 = 𝑒𝑒𝑖𝑖𝑖𝑖


Từ đó 𝑧𝑧 = 𝑟𝑟(cos 𝜑𝜑 + 𝑖𝑖 sin 𝜑𝜑) = 𝑟𝑟𝑒𝑒𝑖𝑖𝑖𝑖
Làm việc với số phức dưới dạng mũ có rất nhiều tiện lợi. Để đưa số phức về dạng mũ, cần phải
xác định modun và argument của nó.

1.1.3. Luỹ thừa và khai căn số phức


a) Luỹ thừa với số mũ 𝒏𝒏 nguyên dương
Như trên đã nói, ta có thể đưa số phức về dạng lượng giác hay dạng mũ rồi từ đó tính luỹ thừa
được dễ dàng.
b) Căn bậc 𝒏𝒏 (𝒏𝒏𝒕𝒕𝒕𝒕 root) của số phức
Trước hết, căn bậc 𝑛𝑛 của đơn vị là 𝑛𝑛 số phức phân biệt sau
2𝑘𝑘𝑘𝑘 2𝑘𝑘𝑘𝑘 2𝑘𝑘𝑘𝑘 ��������������������
𝑒𝑒𝑖𝑖𝑛𝑛 = cos + 𝑖𝑖 sin , 𝑘𝑘 = 0, 𝑛𝑛 − 1
𝑛𝑛 𝑛𝑛
Định lí: Cho 𝑧𝑧 = 𝑟𝑟(cos 𝜑𝜑 + 𝑖𝑖 sin 𝜑𝜑) = 𝑟𝑟𝑒𝑒𝑖𝑖𝑖𝑖 là số phức với 𝑟𝑟 > 0 và 𝜑𝜑 ∈ [0,2𝜋𝜋), khi đó căn bậc 𝑛𝑛
của 𝑧𝑧 là 𝑛𝑛 số phức phân biệt
√ 𝜑𝜑 + 2𝑘𝑘𝑘𝑘 𝜑𝜑 + 2𝑘𝑘𝑘𝑘 √ 𝜑𝜑+2𝑘𝑘𝑘𝑘
𝑧𝑧𝑘𝑘 = 𝑛𝑛 𝑟𝑟 �cos + 𝑖𝑖 sin ��������������������
� = 𝑛𝑛 𝑟𝑟𝑒𝑒𝑖𝑖 𝑛𝑛 , 𝑘𝑘 = 0, 𝑛𝑛 − 1
𝑛𝑛 𝑛𝑛

1.2. Tích vô hướng (inner product, scalar product) của hai vector trên
trường số phức

Mục đích của chúng ta là trang bị tích vô hướng cho các không gian vector trên trường số phức.
Tuy nhiên, nếu định nghĩa tích vô hướng như trong trường hợp 𝕂𝕂 là trường số thực thì gặp một
số trở ngại chẳng hạn:
〈𝑥𝑥, 𝑥𝑥〉 không hẳn thuộc ℝ, do đó không đảm bảo 〈𝑥𝑥, 𝑥𝑥〉 ≥ 0 ∀𝑥𝑥
Có thể xảy ra trường hợp 𝑥𝑥 ≠ 0 nhưng 〈𝑥𝑥, 𝑥𝑥〉 = 0.
Ví dụ: (1, 𝑖𝑖) ≠ 0, nhưng 〈(1, 𝑖𝑖), (1, 𝑖𝑖)〉 = 1 + 𝑖𝑖2 = 0
Do đó, cần một định nghĩa mới thích hợp cho tích vô hướng trong trường số phức ℂ.
Định nghĩa: 𝑉𝑉 là một không gian vector trên ℂ. Ta gọi một tích vô hướng trên 𝑉𝑉 là một ánh
xạ:
〈, 〉: 𝑉𝑉 × 𝑉𝑉 → ℂ
(𝑥𝑥, 𝑦𝑦) ↦ 〈𝑥𝑥, 𝑦𝑦〉
thỏa các điều kiện sau:

Đỗ Minh Triết 12 - MATHTASY -


Ôn thi Olympic Toán Đại số Phần I: Số phức

∀𝑥𝑥, 𝑥𝑥′ , 𝑦𝑦 ∈ 𝑉𝑉 , 𝑘𝑘 ∈ ℂ thì


��������������
i) 〈𝑥𝑥, 𝑦𝑦〉 = 〈𝑦𝑦, 𝑥𝑥〉
ii) 〈𝑥𝑥 + 𝑥𝑥′ , 𝑦𝑦〉 = 〈𝑥𝑥, 𝑦𝑦〉 + 〈𝑥𝑥′ , 𝑦𝑦〉
iii) 〈𝑘𝑘𝑘𝑘, 𝑦𝑦〉 = 𝑘𝑘〈𝑥𝑥, 𝑦𝑦〉
iv) Nếu 𝑥𝑥 ≠ 0 thì 〈𝑥𝑥, 𝑥𝑥〉 > 0
�����������������
v) 〈𝑥𝑥, 𝑘𝑘𝑘𝑘〉 = 〈𝑘𝑘𝑘𝑘, �����������������
𝑥𝑥〉 = 𝑘𝑘〈𝑦𝑦, ��������������
𝑥𝑥〉 = 𝑘𝑘̅〈𝑦𝑦, 𝑥𝑥〉 = 𝑘𝑘̅〈𝑥𝑥, 𝑦𝑦〉
Nhận xét:
Điều kiện i) đảm bảo 〈𝑥𝑥, 𝑥𝑥〉 ∈ ℝ
������������������������������������
〈𝑥𝑥, 𝛼𝛼𝑦𝑦1 + 𝛽𝛽𝑦𝑦2 〉 = 〈𝛼𝛼𝑦𝑦 �������������������� �������������������� ̅
1 + 𝛽𝛽𝑦𝑦2 , 𝑥𝑥〉 = 𝛼𝛼〈𝑦𝑦1 , 𝑥𝑥〉 + 𝛽𝛽〈𝑦𝑦2 , 𝑥𝑥〉 = 𝛼𝛼̅〈𝑥𝑥, 𝑦𝑦1 〉 + 𝛽𝛽 〈𝑥𝑥, 𝑦𝑦2 〉

Ánh xạ định nghĩa như trên được gọi là dạng song tuyến tính đối xứng liên hợp xác định trên 𝑉𝑉 .
Khi đó cặp 𝑈𝑈 = (𝑉𝑉 , 〈, 〉) được gọi là một không gian Unita.
Ví dụ: 𝑉𝑉 = ℂ𝑛𝑛 thì ∀ 𝑥𝑥 = (𝑥𝑥1 , … , 𝑥𝑥𝑛𝑛 ), 𝑦𝑦 = (𝑦𝑦1 , … , 𝑦𝑦𝑛𝑛 ) ∈ 𝑉𝑉 thì
𝑛𝑛
〈𝑥𝑥, 𝑦𝑦〉 = � 𝑥𝑥𝑖𝑖 �����
𝑦𝑦𝑖𝑖
𝑖𝑖=1

Khi đó 𝑈𝑈 = (𝑉𝑉 , 〈, 〉) được gọi là không gian Unita.

Đỗ Minh Triết 13 - MATHTASY -


PHẦN II. MỘT SỐ KIẾN THỨC CƠ BẢN VỀ MA TRẬN

2.1. Định nghĩa các ma trận

2.1.1. Ma trận nghịch đảo, ma trận khả nghịch


(The Inverse of the Matrix, Matrix Inversion)
Định nghĩa: Cho 𝐴𝐴, 𝐵𝐵 ∈ 𝑀𝑀𝑛𝑛 (𝕂𝕂), nếu 𝐴𝐴𝐴𝐴 = 𝐵𝐵𝐵𝐵 = 𝐼𝐼 thì ta nói 𝐴𝐴 là ma trận nghịch đảo của ma
trận 𝐵𝐵 và ngược lại, ta còn nói 𝐴𝐴, 𝐵𝐵 là hai ma trận nghịch đảo của nhau. Ký hiệu nghịch đảo của
𝐴𝐴 là 𝐴𝐴−1 .
Nếu det 𝐴𝐴 ≠ 0 thì 𝐴𝐴 được gọi là ma trận khả nghịch (hay không suy biến).
Tính chất:
- Theo định nghĩa, hiển nhiên 2 ma trận là nghịch đảo của nhau thì giao hoán: 𝐴𝐴𝐴𝐴−1 = 𝐴𝐴−1 𝐴𝐴 =
𝐼𝐼.
- (𝑘𝑘𝑘𝑘)−1 = 𝑘𝑘1 𝐴𝐴−1 (𝑘𝑘 ≠ 0)
- Nghịch đảo của nghịch đảo một ma trận là chính nó: (𝐴𝐴−1 )−1 = 𝐴𝐴
- Với hai ma trận 𝐴𝐴, 𝐵𝐵 khả nghịch bất kì, ta luôn có: (𝐴𝐴𝐴𝐴)−1 = 𝐵𝐵−1 𝐴𝐴−1
−1
- det(𝐴𝐴−1 ) = det 𝐴𝐴

2.1.2. Ma trận chuyển vị


(Transpose of the Matrix)
Định nghĩa: Cho 𝐴𝐴 = �𝑎𝑎𝑖𝑖𝑖𝑖 � ∈ 𝑀𝑀𝑛𝑛 (𝕂𝕂), ma trận chuyển vị của 𝐴𝐴 là ma trận mà các hàng của nó
tương ứng là các cột của 𝐴𝐴, ký hiệu 𝐴𝐴𝑇𝑇 = �𝑎𝑎𝑗𝑗𝑗𝑗 �.
Tính chất:
- Chuyển vị của chuyển vị một ma trận là chính nó: (𝐴𝐴𝑇𝑇 )𝑇𝑇 = 𝐴𝐴
- Hiển nhiên rằng: (𝑘𝑘𝑘𝑘)𝑇𝑇 = 𝑘𝑘𝐴𝐴𝑇𝑇
- Chuyển vị của tổng là tổng các chuyển vị: (𝐴𝐴 + 𝐵𝐵)𝑇𝑇 = 𝐴𝐴𝑇𝑇 + 𝐵𝐵𝑇𝑇
- Với hai ma trận 𝐴𝐴, 𝐵𝐵 bất kì, ta luôn có: (𝐴𝐴𝐴𝐴)𝑇𝑇 = 𝐵𝐵𝑇𝑇 𝐴𝐴𝑇𝑇 , từ đó (𝐴𝐴2 )𝑇𝑇 = (𝐴𝐴𝑇𝑇 )2
- Nghịch đảo ma trận chuyển vị bằng chuyển vị nghịch đảo ma trận đó: (𝐴𝐴𝑇𝑇 )−1 = (𝐴𝐴−1 )𝑇𝑇
- Định thức của một ma trận bằng định thức ma trận chuyển vị của nó: det 𝐴𝐴 = det(𝐴𝐴𝑇𝑇 )

2.1.3. Ma trận liên hợp

Đỗ Minh Triết II-14 - MATHTASY -


Ôn thi Olympic Toán Đại số Phần II: Một số kiến thức cơ bản về Ma trận

Định nghĩa: Cho 𝐴𝐴 ∈ 𝑀𝑀𝑛𝑛 (𝕂𝕂), ma trận liên hợp của 𝐴𝐴 là ma trận có các phần tử tương ứng liên
hợp với các phần tử của 𝐴𝐴, ký hiệu 𝐴𝐴.̅
Nếu 𝐴𝐴 ∈ 𝑀𝑀𝑛𝑛 (ℝ) là ma trận thực thì ma trận liên hợp của 𝐴𝐴 cũng là chính nó, tức là 𝐴𝐴 = 𝐴𝐴 ̅ nên
ta không xét gì đến ma trận liên hợp trên trường ℝ.
Tính chất:
- 𝐴𝐴 + 𝐵𝐵 = 𝐴𝐴 ̅ + 𝐵𝐵
���������������� ����
�������� = 𝑘𝑘̅𝐴𝐴 ̅
- 𝑘𝑘𝑘𝑘
��������� = 𝐵𝐵
- 𝐴𝐴𝐴𝐴 ����𝐴𝐴 ̅
−1
- �𝐴𝐴�̅ �����������
= 𝐴𝐴 −1

��������
- 𝐴𝐴 ̅
𝑇𝑇 = 𝐴𝐴𝑇𝑇

2.1.4. Ma trận chuyển vị liên hợp


��������
Định nghĩa: Đơn giản đó là ma trận vuông 𝐴𝐴 lấy chuyển vị rồi liên hợp. Ký hiệu 𝐴𝐴∗ = 𝐴𝐴 𝑇𝑇 =

𝐴𝐴𝑇𝑇̅ .
Nếu 𝐴𝐴 ∈ 𝑀𝑀𝑛𝑛 (ℝ) là ma trận thực thì ma trận chuyển vị liên hợp của nó là ma trận chuyển vị, tức
là 𝐴𝐴∗ = 𝐴𝐴𝑇𝑇 .
Tính chất:
- (𝐴𝐴∗ )∗ = 𝐴𝐴
- (𝐴𝐴 + 𝐵𝐵)∗ = 𝐴𝐴∗ + 𝐵𝐵∗
- (𝑘𝑘𝑘𝑘)∗ = 𝑘𝑘̅𝐴𝐴∗

2.1.5. Ma trận đồng dạng


Định nghĩa: Hai ma trận vuông 𝐴𝐴, 𝐵𝐵 ∈ 𝑀𝑀𝑛𝑛 (𝕂𝕂) được gọi là đồng dạng nếu tồn tại một ma trận
𝑃𝑃 không suy biến sao cho 𝐴𝐴 = 𝑃𝑃𝑃𝑃𝑃𝑃 −1 , ký hiệu 𝐴𝐴 ∽ 𝐵𝐵.
Tính chất:
Hai ma trận đồng dạng có cùng định thức, giá trị riêng, hạng và đa thức đặc trưng. Muốn cho
hai ma trận trực giao hoặc Unita hoặc đối xứng là đồng dạng thì điều kiện cần và đủ là chúng
phải có chung một hệ các giá trị riêng.

2.1.6. Ma trận xác định dương


(Positive Definite Matrix)

Đỗ Minh Triết II-15 - MATHTASY -


Ôn thi Olympic Toán Đại số Phần II: Một số kiến thức cơ bản về Ma trận

Một ma trận vuông 𝐴𝐴 ∈ 𝑀𝑀𝑛𝑛 (𝕂𝕂) cấp 𝑛𝑛 được gọi là xác định dương nếu với một vector 𝑥𝑥 ≠ 0 bất
kì thì tích vô hướng của 𝑥𝑥 và 𝐴𝐴𝐴𝐴 dương: 〈𝐴𝐴𝐴𝐴, 𝑥𝑥〉 > 0. Chú ý là ma trận xác định dương thì khả
nghịch.
Với 𝐴𝐴 ∈ 𝑀𝑀𝑛𝑛 (ℝ) là ma trận thực thì là 𝑥𝑥𝑇𝑇 𝐴𝐴𝐴𝐴 > 0, 𝑥𝑥 ≠ 0.

2.1.7. Ma trận đối xứng


(Symmetric Matrix)
Định nghĩa: 𝐴𝐴 = �𝑎𝑎𝑖𝑖𝑖𝑖 � ∈ 𝑀𝑀𝑛𝑛 (𝕂𝕂) được gọi là đối xứng nếu 𝐴𝐴 = 𝐴𝐴𝑇𝑇 , hay 𝑎𝑎𝑖𝑖𝑖𝑖 = 𝑎𝑎𝑗𝑗𝑗𝑗 . Ma trận đối
xứng là ma trận của một dạng toàn phương.
Tính chất 1:
- Nếu 𝐴𝐴 đối xứng, không suy biến thì 𝐴𝐴−1 cũng là ma trận đối xứng: (𝐴𝐴−1 )𝑇𝑇 = (𝐴𝐴𝑇𝑇 )−1 = 𝐴𝐴−1
- Nếu 𝐴𝐴 đối xứng thì 𝑘𝑘𝑘𝑘 cũng đối xứng: 𝑘𝑘𝑘𝑘 = (𝑘𝑘𝑘𝑘)𝑇𝑇
- Nếu 𝐴𝐴, 𝐵𝐵 đối xứng thì 𝐴𝐴 + 𝐵𝐵 cũng đối xứng: 𝐴𝐴 = 𝐴𝐴𝑇𝑇 , 𝐵𝐵 = 𝐵𝐵𝑇𝑇 ⇒ (𝐴𝐴 + 𝐵𝐵) = 𝐴𝐴𝑇𝑇 + 𝐵𝐵𝑇𝑇 =
(𝐴𝐴 + 𝐵𝐵)𝑇𝑇
- Tích hai ma trận đối xứng là một ma trận đối xứng nếu và chỉ nếu chúng giao hoán.
Tính chất 2: Ma trận thực, đối xứng có
- Các giá trị riêng đều là số thực.
- Các vector riêng ứng với các trị riêng phân biệt thì trực giao nhau và độc lập tuyến tính (vuông
góc với nhau theo nghĩa tích vô hướng của chúng bằng 0).
- Luôn chéo hóa được.

2.1.8. Ma trận phản đối xứng


𝐴𝐴 = �𝑎𝑎𝑖𝑖𝑖𝑖 � ∈ 𝑀𝑀𝑛𝑛 (𝕂𝕂) được gọi là phản đối xứng nếu 𝐴𝐴 = −𝐴𝐴𝑇𝑇 , hay 𝑎𝑎𝑖𝑖𝑖𝑖 = −𝑎𝑎𝑗𝑗𝑗𝑗 .

2.1.9. Ma trận trực giao


(Orthogonal Matrix)
Định nghĩa: Ma trận 𝐴𝐴 = �𝑎𝑎𝑖𝑖𝑖𝑖 � ∈ 𝑀𝑀𝑛𝑛 (ℝ) được gọi là ma trận trực giao nếu các cột tạo thành
một hệ vectơ trực chuẩn, nghĩa là:
𝑛𝑛
0 nếu 𝑗𝑗 ≠ 𝑘𝑘
� 𝑎𝑎𝑖𝑖𝑖𝑖 𝑎𝑎𝑖𝑖𝑖𝑖 = 𝛿𝛿𝑖𝑖𝑖𝑖 = �
𝑖𝑖=1 1 nếu 𝑗𝑗 = 𝑘𝑘
Nói cách khác, 𝐴𝐴 là ma trận trực giao khi và chỉ khi 𝐴𝐴𝐴𝐴𝑇𝑇 = 𝐴𝐴𝑇𝑇 𝐴𝐴 = 𝐼𝐼 (hay 𝐴𝐴−1 = 𝐴𝐴𝑇𝑇 hoặc 𝐴𝐴 =
(𝐴𝐴𝑇𝑇 )−1 ). Ma trận trực giao là ma trận chuyển từ một cơ sở trực chuẩn này sang một cơ sở trực
chuẩn khác trong không gian Euclide. Ma trận trực giao cũng là ma trận các phép biến đổi trực

Đỗ Minh Triết II-16 - MATHTASY -


Ôn thi Olympic Toán Đại số Phần II: Một số kiến thức cơ bản về Ma trận

giao của một không gian Euclide trong một cơ sở trực chuẩn. Tập các ma trận trực giao cấp 𝑛𝑛
lập thành một nhóm gọi là nhóm trực giao, kí hiệu 𝑂𝑂𝑛𝑛 . Tập các ma trận trực giao có định thức
bằng 1 lập thành một nhóm con của 𝑂𝑂𝑛𝑛 , kí hiệu 𝑆𝑆𝑂𝑂𝑛𝑛 .
Tính chất:
- det 𝐴𝐴 = ±1
- Nếu 𝐴𝐴, 𝐵𝐵 là các ma trận vuông trực giao thì 𝐴𝐴𝑇𝑇 , 𝐴𝐴−1 , 𝐴𝐴𝐴𝐴 cũng là các ma trận trực giao:
(𝐴𝐴𝑇𝑇 )−1 = (𝐴𝐴𝑇𝑇 )𝑇𝑇 = 𝐴𝐴 ; 𝐴𝐴 = (𝐴𝐴−1 )−1 = (𝐴𝐴−1 )𝑇𝑇 = (𝐴𝐴𝑇𝑇 )−1 ; (𝐴𝐴𝐴𝐴)−1 = 𝐵𝐵−1 𝐴𝐴−1 = 𝐵𝐵𝑇𝑇 𝐴𝐴𝑇𝑇
= (𝐴𝐴𝐴𝐴)𝑇𝑇

2.1.10. Ma trận unita


(Unita Matrix)
Định nghĩa: 𝐴𝐴 ∈ 𝑀𝑀𝑛𝑛 (ℂ) không suy biến thoả mãn điều kiện 𝐴𝐴𝐴𝐴∗ = 𝐴𝐴∗ 𝐴𝐴 = 𝐼𝐼 (tức là 𝐴𝐴−1 = 𝐴𝐴∗
hay 𝐴𝐴 = 𝐴𝐴∗ −1 ).
Nếu 𝐴𝐴 là ma trận thực thì 𝐴𝐴∗ = 𝐴𝐴𝑇𝑇 = 𝐴𝐴−1 , tức 𝐴𝐴 là ma trận trực giao.
Ví dụ:
Ma trận 𝐴𝐴 = �𝑒𝑒
𝑖𝑖𝑖𝑖
0 � và 𝐵𝐵 = � cos 𝛼𝛼 −𝑖𝑖 sin 𝛼𝛼
� là ma trận Unita.
0 𝑒𝑒𝑖𝑖𝑖𝑖 𝑖𝑖 sin 𝛼𝛼 cos 𝛼𝛼
Tính chất:
- Mọi ma trận trực giao là ma trận Unita.
- Tích của hai ma trận Unita là ma trận Unita
- Ma trận nghịch đảo của một ma trận Unita là ma trận Unita.
- Định thức của ma trận Unita là một số phức có môđun bằng 1: det 𝐴𝐴 = 𝑎𝑎 + 𝑏𝑏𝑖𝑖 với 𝑟𝑟 =

𝑎𝑎2 + 𝑏𝑏2 = 1.
- Tất cả các giá trị riêng của ma trận Unita cũng có môđun bằng 1.

2.1.11. Ma trận Hermite (đối xứng)


(Hermite Matrix)
Định nghĩa: 𝐴𝐴 ∈ 𝑀𝑀𝑛𝑛 (ℂ) được gọi là ma trận Hermite (đối xứng) nếu 𝐴𝐴 = 𝐴𝐴∗ .
Nếu 𝐴𝐴 là ma trận thực thì 𝐴𝐴 = 𝐴𝐴𝑇𝑇 , tức 𝐴𝐴 là ma trận đối xứng.
Ví dụ: Với 𝐴𝐴, 𝐵𝐵 ∈ 𝑀𝑀2 (ℂ):
𝑖𝑖 3 − 𝑖𝑖 −𝑖𝑖 3 + 𝑖𝑖 −𝑖𝑖 2
𝐴𝐴 = � � → 𝐴𝐴 ̅ = � � → 𝐴𝐴𝑇𝑇̅ = � � ≠ 𝐴𝐴
2 1 + 𝑖𝑖 2 1 − 𝑖𝑖 3 + 𝑖𝑖 1 − 𝑖𝑖
1 3 − 𝑖𝑖 ���� = � 1 3 + 𝑖𝑖 ����𝑇𝑇 = � 1 3 − 𝑖𝑖
𝐵𝐵 = � � → 𝐵𝐵 � → 𝐵𝐵 � = 𝐵𝐵
3 + 𝑖𝑖 6 3 − 𝑖𝑖 6 3 + 𝑖𝑖 6

Đỗ Minh Triết II-17 - MATHTASY -


Ôn thi Olympic Toán Đại số Phần II: Một số kiến thức cơ bản về Ma trận

Tính chất: Như tính chất của ma trận đối xứng đã nói ở trên.
Định lí: Ma trận Hermite chéo hóa được nhờ ma trận trực giao Unita, ký hiệu: 𝑈𝑈 .
Ví dụ:
1 1 2
𝐴𝐴 = �1 1 2�
2 2 4
*Đa thức đặc trưng của 𝐴𝐴: 𝑃𝑃𝐴𝐴 (𝑥𝑥) = −𝑥𝑥2 (𝑥𝑥 − 6)
*Cơ sở của không gian riêng 𝐸𝐸0 ứng với trị riêng 𝑥𝑥 = 0 là
𝐵𝐵 = {𝑋𝑋1 = (−1,1,0), 𝑋𝑋2 = (−2,0,1)}
Cơ sở của không gian riêng 𝐸𝐸6 ứng với trị riêng 𝑥𝑥 = 0 là
𝐶𝐶 = {𝑋𝑋3 = (1,1,2)}
Các vector riêng ứng với các trị riêng phân biệt vuông góc với nhau.
*Trực giao các vector trong không gian 𝐸𝐸0 (tích vô hướng của 2 vector khác nhau bằng 0)

Đặt 𝑌𝑌1 = 𝑋𝑋1 = (−1,1,0) ⇒ �|𝑌𝑌1 |� = 2

𝑌𝑌2 = 𝑋𝑋2 − 𝑋𝑋𝑌𝑌2 𝑌𝑌2 1 𝑌𝑌1 = (−2,0,1) − (−1,1,0) = (−1, −1,1) ⇒ �|𝑌𝑌2 |� = 3
1

*Trực chuẩn cơ sở 𝐵𝐵, 𝐶𝐶 (độ dài mỗi vector bằng 1 và tích vô hướng của 2 vector khác nhau bằng
0)
𝑌𝑌1 1 −1 1
𝑍𝑍1 = = √ (−1,1,0) = �√ , √ , 0�
�|𝑌𝑌1 |� 2 2 2
𝑌𝑌2 1 −1 −1 1
𝑍𝑍2 = = √ (−1, −1,1) = �√ , √ , √ �
�|𝑌𝑌2 |� 3 3 3 3
𝑌𝑌3 1 1 1 2
𝑍𝑍3 = = √ (1,1,2) = �√ , √ , √ �
�|𝑌𝑌3 |� 6 6 6 6
−1
√ −1
√ √1
2 3 6

⎜ ⎞
Vậy ma trận làm chéo hóa 𝐴𝐴 là ma trận Unita 𝑈𝑈 = (𝑍𝑍1 𝑍𝑍2 𝑍𝑍3 ) = ⎜ 1 −1 √ ⎟
1
6⎟
√ √
⎜ 2 3 ⎟
√1 √2
⎝0 3 6⎠

0 0 0
và 𝑈𝑈 −1 𝐴𝐴𝐴𝐴 = 𝑈𝑈 𝑇𝑇 𝐴𝐴𝐴𝐴 = �0 0 0�.
0 0 6

2.1.12. Ma trận Hermite đối xứng lệch


��������
Định nghĩa: 𝐴𝐴 ∈ 𝑀𝑀𝑛𝑛 (ℂ) được gọi là ma trận Hermite đối xứng lệch nếu 𝐴𝐴 = −𝐴𝐴 𝑇𝑇 = −𝐴𝐴∗ .

Nếu 𝐴𝐴 ∈ 𝑀𝑀𝑛𝑛 (ℝ) thì 𝐴𝐴 là ma trận phản đối xứng.


Tính chất:

Đỗ Minh Triết II-18 - MATHTASY -


Ôn thi Olympic Toán Đại số Phần II: Một số kiến thức cơ bản về Ma trận

- Mọi giá trị riêng của một phép biến đổi tuyến tính đối xứng lệch đều là số thuần ảo (số phức có
phần thực bằng 0).

2.1.13. Ma trận băng


Là ma trận có các phần tử trên đường chéo chính và một số đường chéo phụ hai bên đường chéo
chính khác 0, các phần tử còn lại bằng 0. Độ rộng của băng được tính bằng số đường chéo phụ ở
một bên đường chéo chính.

2.1.14. Ma trận phụ hợp


(Adjunct Matrix, Adjutage Matrix)
Ma trận phụ hợp của ma trận 𝐴𝐴 ∈ 𝑀𝑀𝑛𝑛 (𝕂𝕂) được định nghĩa và ký hiệu như sau:
𝐴𝐴11 … 𝐴𝐴1𝑛𝑛

𝐴𝐴 = � ⋮ ⋱ ⋮ �
𝐴𝐴𝑛𝑛1 … 𝐴𝐴𝑛𝑛𝑛𝑛
trong đó, 𝐴𝐴𝑖𝑖𝑖𝑖 là phần bù đại số của phần tử 𝑎𝑎𝑖𝑖𝑖𝑖 (cofactor of the element 𝑎𝑎𝑖𝑖𝑖𝑖 ) của 𝐴𝐴, ngoài ra ta
còn ký hiệu adj(𝐴𝐴) = 𝐴𝐴′ 𝑇𝑇 .
𝐴𝐴. 𝐴𝐴′ 𝑇𝑇 = 𝐴𝐴′ 𝑇𝑇 . 𝐴𝐴 = (det 𝐴𝐴)𝐼𝐼 và nếu A khả nghịch thì 𝐴𝐴−1 = det1 𝐴𝐴 . 𝐴𝐴′ 𝑇𝑇 .
Bổ đề: Cho 𝐴𝐴 = �𝑎𝑎𝑖𝑖𝑖𝑖 � ∈ 𝑀𝑀𝑛𝑛 (𝕂𝕂), khi đó
𝑛𝑛
nếu 𝑗𝑗 = 𝑘𝑘
∀(𝑖𝑖, 𝑗𝑗) ∈ {1,2, … , 𝑛𝑛}2 , � 𝑎𝑎𝑖𝑖𝑖𝑖 𝐴𝐴𝑖𝑖𝑖𝑖 = �det 𝐴𝐴
𝑖𝑖=1
0 nếu 𝑗𝑗 ≠ 𝑘𝑘

2.1.15. Ma trận vandermonde


(Vandermonde Matrix)
Định nghĩa: 𝐴𝐴 ∈ 𝑀𝑀𝑛𝑛 (𝕂𝕂) được gọi là ma trận Vandermone khi các phần tử nằm trên mỗi cột
(hay mỗi hàng) lập thành một cấp số nhân số có hạng đầu bằng 1, công bội tương ứng là 𝑎𝑎1 , … , 𝑎𝑎𝑛𝑛
1 1 … 1 1

⎜ 𝑎𝑎 𝑎𝑎2 … 𝑎𝑎𝑛𝑛−1 𝑎𝑎𝑛𝑛 ⎞⎟
⎜ 12 ⎟
𝐴𝐴 = ⎜
⎜ 𝑎𝑎1 𝑎𝑎22 … 𝑎𝑎2𝑛𝑛−1 𝑎𝑎2𝑛𝑛 ⎟


⎜ ⋮ ⎟
⋮ … ⋮ ⋮ ⎟
𝑛𝑛−1
⎝ 𝑎𝑎𝑛𝑛−1
1 𝑎𝑎𝑛𝑛−1
2 … 𝑎𝑎𝑛𝑛−1 𝑎𝑎𝑛𝑛−1
𝑛𝑛 ⎠
Tính chất:
- det 𝐴𝐴 = ∏1≤𝑗𝑗<𝑖𝑖≤𝑛𝑛�𝑎𝑎𝑖𝑖 − 𝑎𝑎𝑗𝑗 �, dễ thấy rằng det 𝐴𝐴 ≠ 0 khi và chỉ khi các công bội 𝑎𝑎1 , … , 𝑎𝑎𝑛𝑛 đôi một
phân biệt, det 𝐴𝐴 được gọi là định thức Vandermonde.
Ta có bảng đối chiếu giữa 𝑀𝑀𝑛𝑛 (ℂ) và 𝑀𝑀𝑛𝑛 (ℝ):

Đỗ Minh Triết II-19 - MATHTASY -


Ôn thi Olympic Toán Đại số Phần II: Một số kiến thức cơ bản về Ma trận

𝐴𝐴 ∈ 𝑀𝑀𝑛𝑛 (ℂ) 𝐴𝐴 ∈ 𝑀𝑀𝑛𝑛 (ℝ)


Liên hợp: 𝐴𝐴 ̅ Chính nó: 𝐴𝐴 = 𝐴𝐴 ̅
��������
Chuyển vị liên hợp: 𝐴𝐴∗ = 𝐴𝐴 𝑇𝑇 Chuyển vị: 𝐴𝐴∗ = 𝐴𝐴𝑇𝑇
Unita: 𝐴𝐴 = 𝐴𝐴∗ −1 Trực giao: 𝐴𝐴 = (𝐴𝐴𝑇𝑇 )−1
Hermite (đối xứng): 𝐴𝐴 = 𝐴𝐴∗ Đối xứng: 𝐴𝐴 = 𝐴𝐴𝑇𝑇
Hermite (đối xứng lệch): 𝐴𝐴 = Phản đối xứng: 𝐴𝐴 =
−𝐴𝐴∗ −𝐴𝐴𝑇𝑇

2.2. Bài tập

Bài 2.1. Chứng minh rằng


a) Mọi ma trận cấp 𝑛𝑛 đều có thể phân tích được thành tổng của một ma trận đối xứng và một ma
trận phản đối xứng.
b) Tích hai ma trận đối xứng là một ma trận đối xứng nếu và chỉ nếu chúng giao hoán. Lấy ví dụ
chứng tỏ tích hai ma trận đối xứng có thể không đối xứng.
c) Tích hai ma trận phản đối xứng 𝐴𝐴 và 𝐵𝐵 là một ma trận phản đối xứng nếu và chỉ nếu 𝐴𝐴𝐴𝐴 = −𝐵𝐵𝐵𝐵.
Lấy ví dụ chứng tỏ tích hai ma trận phản đối xứng có thể không phản đối xứng.
d) Nghịch đảo (nếu có) của một ma trận đối xứng là một ma trận đối xứng.
e) Nghịch đảo (nếu có) của một ma trận phản đối xứng là một ma trận phản đối xứng.
f) Định thức của ma trận phản đối xứng cấp lẻ bằng 0.

Chứng minh.

a) 𝐴𝐴 = 12 [(𝐴𝐴 + 𝐴𝐴𝑇𝑇 ) + (𝐴𝐴 − 𝐴𝐴𝑇𝑇 )]


b) Giả sử 𝐴𝐴, 𝐵𝐵 là hai ma trận đối xứng, ta có
𝐴𝐴𝐴𝐴 = (𝐴𝐴𝐴𝐴)𝑇𝑇 ⇔ 𝐴𝐴𝐴𝐴 = 𝐵𝐵𝑇𝑇 𝐴𝐴𝑇𝑇 �tính chất của ma trận� ⇔ 𝐴𝐴𝐴𝐴 = 𝐵𝐵𝐵𝐵 �𝐴𝐴, 𝐵𝐵 đối xứng�
c) Giả sử 𝐴𝐴, 𝐵𝐵 là hai ma trận phản đối xứng, ta có
𝐴𝐴𝐴𝐴 = −(𝐴𝐴𝐴𝐴)𝑇𝑇 ⇔ 𝐴𝐴𝐴𝐴 = −𝐵𝐵𝑇𝑇 𝐴𝐴𝑇𝑇 �tính chất của ma trận� ⇔ 𝐴𝐴𝐴𝐴 = −𝐵𝐵𝐵𝐵�𝐴𝐴, 𝐵𝐵 phản đối xứng�
d) Giả sử 𝐴𝐴 là ma trận đối xứng khả nghịch, ta có
𝐴𝐴−1 = (𝐴𝐴𝑇𝑇 )−1 �𝐴𝐴 đối xứng� = (𝐴𝐴−1 )𝑇𝑇 �tính chất của ma trận�
e) Giả sử 𝐴𝐴 là ma trận phản đối xứng khả nghịch, ta có
𝐴𝐴−1 = (−𝐴𝐴𝑇𝑇 )−1 �𝐴𝐴 phản đối xứng� = −(𝐴𝐴−1 )𝑇𝑇 �tính chất của ma trận�
f) Giả sử 𝐴𝐴 là ma trận phản đối xứng cấp 𝑛𝑛 lẻ, ta có

Đỗ Minh Triết II-20 - MATHTASY -


Ôn thi Olympic Toán Đại số Phần II: Một số kiến thức cơ bản về Ma trận

|𝐴𝐴| = |𝐴𝐴𝑇𝑇 | (tı́nh chất định thứ c) = |−𝐴𝐴𝑇𝑇 | �𝐴𝐴 phản đối xứng� = (−1)𝑛𝑛 |𝐴𝐴𝑇𝑇 | = −|𝐴𝐴𝑇𝑇 | (do 𝑛𝑛 lẻ)
Suy ra |𝐴𝐴| = 0.

Bài 2.2. Cho 𝐴𝐴 là ma trận vuông cấp 𝑛𝑛 thoả 𝐴𝐴2 + 𝐴𝐴𝑇𝑇 = 𝐼𝐼. Chứng minh rẳng 𝐴𝐴4 − 2𝐴𝐴2 +
𝐴𝐴 = 0.

Chứng minh.

Từ giả thiết 𝐴𝐴2 + 𝐴𝐴𝑇𝑇 = 𝐼𝐼 ⇒ (𝐴𝐴2 )𝑇𝑇 = (𝐼𝐼 − 𝐴𝐴𝑇𝑇 )𝑇𝑇 = 𝐼𝐼 − 𝐴𝐴. Mặt khác (𝐴𝐴2 )𝑇𝑇 = (𝐴𝐴𝐴𝐴)𝑇𝑇 = 𝐴𝐴𝑇𝑇 𝐴𝐴𝑇𝑇 =
(𝐴𝐴𝑇𝑇 )2 .
Từ đó, ta có
𝐴𝐴4 − 2𝐴𝐴2 + 𝐴𝐴 = 𝐴𝐴2 (𝐴𝐴2 − 2𝐼𝐼) + 𝐴𝐴 = 𝐴𝐴2 (−𝐴𝐴𝑇𝑇 − 𝐼𝐼) + 𝐴𝐴 = −(𝐼𝐼 − 𝐴𝐴𝑇𝑇 )(𝐼𝐼 + 𝐴𝐴𝑇𝑇 ) + 𝐴𝐴
= −𝐼𝐼 + (𝐴𝐴𝑇𝑇 )2 + 𝐴𝐴
= −𝐼𝐼 + (𝐴𝐴2 )𝑇𝑇 + 𝐴𝐴 = −𝐼𝐼 + 𝐼𝐼 − 𝐴𝐴 + 𝐴𝐴 = 0

Bài 2.3. (Câu A2 Putnam 1991) Cho 𝐴𝐴 và 𝐵𝐵 là các ma trận vuông thực cấp 𝑛𝑛 thỏa mãn 𝐴𝐴 ≠
𝐵𝐵, 𝐴𝐴 = 𝐵𝐵 và 𝐴𝐴 𝐵𝐵 = 𝐵𝐵 𝐴𝐴. Chứng minh ma trận 𝐴𝐴2 + 𝐵𝐵2 không khả nghịch.
3 3 2 2

Chứng minh.

Ta có (𝐴𝐴2 + 𝐵𝐵2 )(𝐴𝐴 − 𝐵𝐵) = 𝐴𝐴3 − 𝐵𝐵3 + 𝐵𝐵2 𝐴𝐴 − 𝐴𝐴2 𝐵𝐵 = 0.


Nếu 𝐴𝐴2 + 𝐵𝐵2 khả nghịch thì 𝐴𝐴 − 𝐵𝐵 = 0, mâu thuẫn giả thiết. Vậy 𝐴𝐴2 + 𝐵𝐵2 không khả nghịch.

Bài 2.4. Giả sử 𝐴𝐴, 𝐵𝐵 là hai ma trận vuông cấp 𝑛𝑛 ∈ ℕ∗ khả nghịch thoả mãn 𝐴𝐴𝐴𝐴 + 𝐵𝐵𝐵𝐵 = 0.
Chứng minh 𝑛𝑛 là một số chẵn. Tìm một ví dụ về hai ma trận 𝐴𝐴, 𝐵𝐵 khả nghịch thoả 𝐴𝐴𝐴𝐴 + 𝐵𝐵𝐵𝐵 = 0.

Chứng minh.

Ta có |𝐴𝐴𝐴𝐴| = |−𝐵𝐵𝐵𝐵| = (−1)𝑛𝑛 |𝐵𝐵𝐵𝐵| = (−1)𝑛𝑛 |𝐴𝐴𝐴𝐴| do 𝐴𝐴, 𝐵𝐵 khả nghịch nên ta c ó 1 = (−1)𝑛𝑛 suy ra
𝑛𝑛 phải là số chẵn.
0 1 0 −1 1 0 −1 0
Lấy 𝐴𝐴 = � � , 𝐵𝐵 = � �, khi đó 𝐴𝐴𝐴𝐴 + 𝐵𝐵𝐴𝐴 = � �+� �=0
1 0 1 0 0 0 0 0

Bài 2.5. Chứng tỏ hai ma trận sau đồng dạng trên ℂ


cos 𝜃𝜃 − sin 𝜃𝜃 𝑖𝑖𝑖𝑖
𝐴𝐴 = � � ; 𝐵𝐵 = �𝑒𝑒 0 �
sin 𝜃𝜃 cos 𝜃𝜃 0 𝑒𝑒−𝑖𝑖𝑖𝑖
Chứng minh.

Đỗ Minh Triết II-21 - MATHTASY -


Ôn thi Olympic Toán Đại số Phần II: Một số kiến thức cơ bản về Ma trận

Bài 2.6. Cho 𝑛𝑛 ∈ ℕ, 𝐴𝐴 là ma trận thực, phản đối xứng cấp 𝑛𝑛. Chứng minh 𝐼𝐼 + 𝐴𝐴 khả
nghịch.

Chứng minh.
𝑥𝑥1
Xét hệ (𝐼𝐼 + 𝐴𝐴)𝑥𝑥 = 0 với 𝑥𝑥 = � ⋮ � ∈ ℝ𝑛𝑛 , ta có
𝑥𝑥𝑛𝑛
𝐴𝐴𝐴𝐴 = −𝑥𝑥 ⇔ (𝐴𝐴𝐴𝐴)𝑇𝑇 = −𝑥𝑥𝑇𝑇 ⇔ 𝑥𝑥𝑇𝑇 𝐴𝐴𝑇𝑇 = −𝑥𝑥𝑇𝑇 ⇔ 𝑥𝑥𝑇𝑇 𝐴𝐴𝑇𝑇 𝑥𝑥 = −𝑥𝑥𝑇𝑇 𝑥𝑥
Do 𝐴𝐴𝑇𝑇 = −𝐴𝐴 và 𝐴𝐴𝐴𝐴 = −𝑥𝑥 nên ta có 𝑥𝑥𝑇𝑇 𝐴𝐴𝐴𝐴 = 𝑥𝑥𝑇𝑇 𝑥𝑥 ⇔ 𝑥𝑥𝑇𝑇 𝑥𝑥 = −𝑥𝑥𝑇𝑇 𝑥𝑥 ⇔ 𝑥𝑥𝑇𝑇 𝑥𝑥 = 0
𝑥𝑥1
⇔( 1 𝑥𝑥 … 𝑥𝑥 𝑛𝑛 ) � ⋮ � = 𝑥𝑥1 +. . . +𝑥𝑥𝑛𝑛 = 0 ⇔ 𝑥𝑥1 =. . . = 𝑥𝑥𝑛𝑛 = 0
2 2

𝑥𝑥𝑛𝑛
Hệ (𝐼𝐼 + 𝐴𝐴)𝑥𝑥 = 0 chỉ có nghiệm tầm thường nên 𝐼𝐼 + 𝐴𝐴 khả nghịch.

Bài 2.7. Cho 𝐴𝐴 ∈ 𝑀𝑀𝑛𝑛 (𝕂𝕂). Chứng minh rằng


a) Nếu (𝑋𝑋𝑋𝑋)2 = 0 ∀ 𝑋𝑋 ∈ 𝑀𝑀𝑛𝑛 (𝕂𝕂) thì 𝐴𝐴 = 0.
b) Nếu 𝐴𝐴𝐴𝐴 = 𝐷𝐷𝐷𝐷 với 𝐷𝐷 = diag(𝜆𝜆1 , … , 𝜆𝜆𝑛𝑛 ) là ma trận đường chéo có các phần tử chéo khác nhau thì
𝐴𝐴 cũng là ma trận đường chéo.

Chứng minh.

a) Lần lượt chọn 𝑛𝑛2 ma trận 𝑋𝑋𝑖𝑖𝑖𝑖 = �𝑥𝑥𝑖𝑖𝑖𝑖 � chỉ có một phần tử 𝑥𝑥𝑖𝑖𝑖𝑖 = 1, mọi phần tử khác đều bằng
0. Khi đó ma trận 𝑋𝑋𝑖𝑖𝑖𝑖 𝐴𝐴 là ma trận có hàng 𝑖𝑖 là hàng 𝑗𝑗 của 𝐴𝐴, mọi hàng khác đều bằng 0. Từ
�𝑋𝑋𝑖𝑖𝑖𝑖 𝐴𝐴�2 = 0 ta suy ra 𝑎𝑎2𝑗𝑗𝑗𝑗 = 0 dẫn đến 𝑎𝑎2𝑗𝑗𝑗𝑗 = 0. Vậy 𝐴𝐴 = 0.
b) So sánh các hàng 𝑖𝑖 của 𝐴𝐴𝐴𝐴 và 𝐷𝐷𝐷𝐷, ta có
�𝜆𝜆1 𝑎𝑎𝑖𝑖1 , 𝜆𝜆2 𝑎𝑎𝑖𝑖2 , … 𝜆𝜆𝑖𝑖−1 𝑎𝑎𝑖𝑖,𝑖𝑖−1 , 𝜆𝜆𝑖𝑖 𝑎𝑎𝑖𝑖𝑖𝑖 , 𝜆𝜆𝑖𝑖+1 𝑎𝑎𝑖𝑖,𝑖𝑖+1 , … , 𝜆𝜆𝑛𝑛−1 𝑎𝑎𝑖𝑖,𝑛𝑛−1 , 𝜆𝜆𝑛𝑛 𝑎𝑎𝑖𝑖𝑖𝑖 �
= �𝜆𝜆𝑖𝑖 𝑎𝑎𝑖𝑖1 , 𝜆𝜆𝑖𝑖 𝑎𝑎𝑖𝑖2 , … , 𝜆𝜆𝑖𝑖 𝑎𝑎𝑖𝑖,𝑖𝑖−1 , 𝜆𝜆𝑖𝑖 𝑎𝑎𝑖𝑖𝑖𝑖 , 𝜆𝜆𝑖𝑖 𝑎𝑎𝑖𝑖,𝑖𝑖+1 , … 𝜆𝜆𝑖𝑖 𝑎𝑎𝑖𝑖,𝑛𝑛−1 , 𝜆𝜆𝑖𝑖 𝑎𝑎𝑖𝑖𝑖𝑖 �
do các 𝜆𝜆𝑖𝑖 khác nhau nên 𝑎𝑎𝑖𝑖1 = 𝑎𝑎𝑖𝑖2 =. . . 𝑎𝑎𝑖𝑖,𝑖𝑖−1 = 𝑎𝑎𝑖𝑖,𝑖𝑖+1 =. . . = 𝑎𝑎𝑖𝑖,𝑛𝑛−1 = 𝑎𝑎𝑖𝑖𝑖𝑖 = 0, tức là các phần tử
ngoài đường chéo chính của 𝐴𝐴 đều bằng 0 nên 𝐴𝐴 là ma trận đường chéo.

Bài 2.8. Cho 𝐴𝐴 là ma trận cấp 𝑛𝑛 thoả 𝐴𝐴𝑛𝑛 = 𝛼𝛼𝛼𝛼 với 𝛼𝛼 là số thực khác 1 và -1. Chứng minh
𝐴𝐴 + 𝐼𝐼 khả nghịch.

Chứng minh.

Ta có 𝐴𝐴𝑛𝑛 = (𝐴𝐴 + 𝐼𝐼 − 𝐼𝐼)𝑛𝑛 = (𝐴𝐴 + 𝐼𝐼)𝐶𝐶 + (−1)𝑛𝑛 𝐼𝐼, mặt khác 𝐴𝐴𝑛𝑛 = 𝛼𝛼(𝐴𝐴 + 𝐼𝐼 − 𝐼𝐼), từ đó
(𝐴𝐴 + 𝐼𝐼)(𝐶𝐶 − 𝛼𝛼𝛼𝛼) = [(−1)𝑛𝑛+1 − 𝛼𝛼]𝐼𝐼
Vì 𝛼𝛼 ≠ ±1 suy ra 𝐴𝐴 + 𝐼𝐼 khả nghịch và (𝐴𝐴 + 𝐼𝐼)−1 = (−1)𝑛𝑛+1
1
−𝛼𝛼
(𝐶𝐶 − 𝛼𝛼𝛼𝛼).

Đỗ Minh Triết II-22 - MATHTASY -


Ôn thi Olympic Toán Đại số Phần II: Một số kiến thức cơ bản về Ma trận

Bài 2.9. Cho 𝐴𝐴 = �𝑎𝑎𝑖𝑖𝑖𝑖 �𝑛𝑛 là ma trận trực giao, 𝑛𝑛 lẻ. Chứng minh rằng |tr 𝐴𝐴| ≤ 𝑛𝑛 và
det(𝐴𝐴2 − 𝐼𝐼) = 0.

Chứng minh.

Ta có 𝐴𝐴𝑇𝑇 𝐴𝐴 = 𝐼𝐼, so sánh các phần tử trên đường chéo chính của hai bên, ta thấy rằng tổng bình
phương các phần tử trên một hàng của 𝐴𝐴 bằng 1, suy ra mọi phần tử của 𝐴𝐴 có trị tuyệt đối không
quá 1, vậy |tr 𝐴𝐴| ≤ 𝑛𝑛.
Vì 𝑛𝑛 lẻ nên 𝐴𝐴 có ít nhất một giá trị riêng 𝜆𝜆 thực và vector riêng 𝑥𝑥 tương ứng, tức là 𝐴𝐴𝐴𝐴 = 𝜆𝜆𝜆𝜆.
Từ 𝐴𝐴𝑇𝑇 𝐴𝐴 = 𝐼𝐼, ta có
𝑥𝑥𝑇𝑇 𝐴𝐴𝑇𝑇 𝐴𝐴𝐴𝐴 = 𝑥𝑥𝑇𝑇 𝑥𝑥 ⇔ (𝐴𝐴𝐴𝐴)𝑇𝑇 𝐴𝐴𝐴𝐴 = 𝑥𝑥𝑇𝑇 𝑥𝑥 ⇔ 𝜆𝜆2 𝑥𝑥𝑇𝑇 𝑥𝑥 = 𝑥𝑥𝑇𝑇 𝑥𝑥 ⇒ 𝜆𝜆2 = 1
Mặt khác 𝜆𝜆2 là giá trị riêng của 𝐴𝐴2 , từ đó suy ra det(𝐴𝐴2 − 𝐼𝐼) = 0.

Bài 2.10. (Câu 4 Olympic 1997) Tìm ma trận nghịch đảo của ma trận sau
𝐶𝐶00 2𝐶𝐶10 22 𝐶𝐶20 … 2𝑛𝑛 𝐶𝐶𝑛𝑛0

⎜0 ⎞
𝑀𝑀 = ⎜ 𝐶𝐶11 2𝐶𝐶21 … 2𝑛𝑛−1 𝐶𝐶𝑛𝑛1 ⎟


⎜ ⎟
⋮ ⋮ ⋮ … ⋮ ⎟
𝑛𝑛
⎝ 0 0 0 … 𝐶𝐶𝑛𝑛 ⎠

Chứng minh.

Đặt 𝑋𝑋 = (1 𝑥𝑥 𝑥𝑥2 ���� = (1


… 𝑥𝑥𝑛𝑛 ) và 𝑋𝑋 2 + 𝑥𝑥 (2 + 𝑥𝑥)2 … (2 + 𝑥𝑥)𝑛𝑛
Dễ thấy với mọi 𝑥𝑥 ∈ ℝ, ta có
����
𝑋𝑋𝑋𝑋 = 𝑋𝑋 ���� 𝑀𝑀 −1
hay 𝑋𝑋 = 𝑋𝑋
Đặt 𝑥𝑥 = 𝑡𝑡 − 2, ta được
−1
(1 𝑡𝑡 − 2 (𝑡𝑡 − 2)2 … (𝑡𝑡 − 2)𝑛𝑛 ) = (1 𝑡𝑡 𝑡𝑡2 … 𝑡𝑡𝑛𝑛 )𝑀𝑀
Suy ra
𝐶𝐶00 −2𝐶𝐶10 (−2)2 𝐶𝐶20 … (−2)𝑛𝑛 𝐶𝐶𝑛𝑛0

⎜ ⎞
⎜0 𝐶𝐶11 −2𝐶𝐶21 … (−2)𝑛𝑛−1 𝐶𝐶𝑛𝑛1 ⎟

𝑀𝑀 −1 =⎜

⎜ 0 0 𝐶𝐶22

… (−2)𝑛𝑛−2 𝐶𝐶𝑛𝑛2 ⎟


⎜ ⋮ ⎟

⋮ ⋮ ⋱ ⋮
𝑛𝑛
⎝0 0 0 … 𝐶𝐶𝑛𝑛 ⎠

Bài 2.11. (Câu 3 đề chọn đội tuyển ĐH Kinh tế Quốc dân 2004) Cho 𝐴𝐴 là ma trận vuông cấp
𝑛𝑛 thoả mãn 𝐴𝐴𝐴𝐴 + 𝐵𝐵𝐵𝐵 = 0, trong đó 𝐵𝐵 = 𝐴𝐴𝐴𝐴 − 𝑋𝑋𝑋𝑋 với 𝑋𝑋 là ma trận vuông cấp 𝑛𝑛 tuỳ ý. Chứng
minh 𝐴𝐴2 là ma trận có dạng 𝑘𝑘𝑘𝑘.

Chứng minh.

Từ giả thiết, ta có

Đỗ Minh Triết II-23 - MATHTASY -


Ôn thi Olympic Toán Đại số Phần II: Một số kiến thức cơ bản về Ma trận

𝐴𝐴𝐴𝐴 + 𝐵𝐵𝐵𝐵 = 𝐴𝐴(𝐴𝐴𝐴𝐴 − 𝑋𝑋𝑋𝑋) + (𝐴𝐴𝐴𝐴 − 𝑋𝑋𝑋𝑋)𝐴𝐴 = 𝐴𝐴2 𝑋𝑋 − 𝐴𝐴𝐴𝐴𝐴𝐴 + 𝐴𝐴𝐴𝐴𝐴𝐴 − 𝑋𝑋𝐴𝐴2 = 𝐴𝐴2 𝑋𝑋 − 𝑋𝑋𝐴𝐴2 = 0
hay 𝐴𝐴2 𝑋𝑋 = 𝑋𝑋𝐴𝐴2
Ký hiệu 𝐴𝐴 = �𝑎𝑎𝑖𝑖𝑗𝑗 �. Vì 𝑋𝑋 tuỳ ý nên trước tiên, ta chọn 𝑋𝑋 lần lượt là 𝑛𝑛 ma trận 𝑋𝑋𝑖𝑖𝑖𝑖 = �𝑥𝑥𝑖𝑖𝑖𝑖 �, trong
đó phần tử 𝑥𝑥𝑖𝑖𝑖𝑖 = 1, mọi phần tử khác đều bằng 0, so sánh hai vế của đẳng thức 𝐴𝐴2 𝑋𝑋 = 𝑋𝑋𝐴𝐴2 , ta
thu được 𝐴𝐴 là ma trận đường chéo 𝐴𝐴 = diag(𝑎𝑎11 , … , 𝑎𝑎𝑛𝑛𝑛𝑛 ).
Tiếp theo, ta chọn 𝑋𝑋 lần lượt là 𝑛𝑛 ma trận 𝑋𝑋𝑖𝑖,𝑛𝑛−𝑖𝑖+1 = �𝑥𝑥𝑖𝑖𝑖𝑖 �, trong đó phần tử 𝑥𝑥𝑖𝑖,𝑛𝑛−𝑖𝑖+1 = 1, mọi
phần tử khác đều bằng 0, lại so sánh hai vế của đẳng thức 𝐴𝐴2 𝑋𝑋 = 𝑋𝑋𝐴𝐴2 , ta thu được 𝑎𝑎11 =. . . =
𝑎𝑎𝑛𝑛𝑛𝑛 .
Vậy 𝐴𝐴2 = 𝑘𝑘𝑘𝑘.

Bài 2.12. (Câu 4 Olympic 2006) Cho 𝐴𝐴 là ma trận vuông cấp 𝑛𝑛 sao cho mỗi dòng của nó
chứa đúng hai phần tử khác 0, trong đó phần tử nằm ở đường chéo chính bằng 2006, phần tử còn lại
bằng 1. Chứng minh 𝐴𝐴 khả nghịch.

Chứng minh.

Đặt 𝐴𝐴 = �𝑎𝑎𝑖𝑖𝑖𝑖 �, ta chứng minh bằng phản chứng. Giả sử 𝐴𝐴 suy biến, ký hiệu 𝑐𝑐𝑖𝑖 là cột thứ 𝑖𝑖 của 𝐴𝐴,
khi đó các cột 𝑐𝑐1 , 𝑐𝑐2 , … , 𝑐𝑐𝑛𝑛 của 𝐴𝐴 là phụ thuộc tuyến tính trong ℝ𝑛𝑛 . Do đó phải có một tổ hợp
tuyến tính
𝜆𝜆1 𝑐𝑐1 + 𝜆𝜆2 𝑐𝑐2 +. . . +𝜆𝜆𝑛𝑛−1 𝑐𝑐𝑛𝑛−1 + 𝜆𝜆𝑛𝑛 𝑐𝑐𝑛𝑛 = 0
trong đó có ít nhất một hệ số khác 0. Giả sử |𝜆𝜆𝑚𝑚 | = max{|𝜆𝜆1 |, |𝜆𝜆2 |, … , |𝜆𝜆𝑛𝑛 |}, hiển nhiên là |𝜆𝜆𝑚𝑚 | >
0. Giả sử hai phần tử khác 0 của dòng 𝑚𝑚 là 𝑎𝑎𝑚𝑚𝑚𝑚 = 2006, 𝑎𝑎𝑚𝑚𝑚𝑚 = 1 (1 ≤ 𝑝𝑝 ≤ 𝑛𝑛, 𝑝𝑝 ≠ 𝑚𝑚), ta có
𝜆𝜆1 𝑎𝑎𝑚𝑚1 + 𝜆𝜆2 𝑎𝑎𝑚𝑚2 +. . . +𝜆𝜆𝑛𝑛 𝑎𝑎𝑚𝑚𝑚𝑚 = 𝜆𝜆𝑚𝑚 2006 + 𝜆𝜆𝑝𝑝 = 0
Suy ra
�𝜆𝜆𝑝𝑝 � = 2006|𝜆𝜆𝑚𝑚 | > |𝜆𝜆𝑚𝑚 |
mâu thuẫn cách chọn |𝜆𝜆𝑚𝑚 |. Vậy 𝐴𝐴 khả nghịch.
(Có thể xem thêm chứng minh 𝑑𝑑𝑑𝑑𝑑𝑑 𝐴𝐴 > 0 ở bài tập phần Giá trị riêng, vector riêng).

Bài 2.13. (Câu 5 Olympic 2006) Cho 𝐴𝐴 là ma trận vuông cấp 𝑛𝑛 hạng 1 thoả 𝐴𝐴2006 = 𝐴𝐴.
Chứng minh ma trận 𝐼𝐼 − 𝐴𝐴 suy biến.

Chứng minh.

Cách 1:

Vì rank 𝐴𝐴 = 1 nên tồn tại một dòng khác 0 trong 𝐴𝐴 và các dòng còn lại đều biểu diễn tuyến tính
được qua dòng này. Do đó ma trận 𝐴𝐴 có dạng sau:

Đỗ Minh Triết II-24 - MATHTASY -


Ôn thi Olympic Toán Đại số Phần II: Một số kiến thức cơ bản về Ma trận

𝜆𝜆1 𝑥𝑥1 𝜆𝜆1 𝑥𝑥2 … 𝜆𝜆1 𝑥𝑥𝑛𝑛



⎜ 𝜆𝜆2 𝑥𝑥1 𝜆𝜆2 𝑥𝑥2 … 𝜆𝜆𝑛𝑛 𝑥𝑥𝑛𝑛 ⎞⎟

⎜ ⎟
𝐴𝐴 = ⎜ … … … … ⎟ ⎟≠0

⎜ 𝜆𝜆𝑖𝑖 𝑥𝑥1 𝜆𝜆𝑖𝑖 𝑥𝑥2 … 𝜆𝜆𝑖𝑖 𝑥𝑥𝑛𝑛 ⎟


⎜ … ⎟
… … … ⎟
⎝𝜆𝜆𝑛𝑛 𝑥𝑥1 𝜆𝜆𝑛𝑛 𝑥𝑥2 … 𝜆𝜆𝑛𝑛 𝑥𝑥𝑛𝑛 ⎠
���������
Không mất tính tổng quát, giả sử dòng khác 0 đó là dòng đầu và 𝜆𝜆𝑖𝑖 ≠ 0 �𝑖𝑖 = 1, 𝑛𝑛�.
𝜆𝜆1 𝑥𝑥1
Đặt 𝑈𝑈 = � ⋮ � ≠ 0, 𝑉𝑉 = � ⋮ � ≠ 0.
𝜆𝜆𝑛𝑛 𝑥𝑥𝑛𝑛
Khi đó 𝐴𝐴 = 𝑈𝑈 𝑇𝑇 𝑉𝑉 và 𝑉𝑉 𝑈𝑈 𝑇𝑇 = 𝜆𝜆1 𝑥𝑥1 + 𝜆𝜆2 𝑥𝑥2 +. . . +𝜆𝜆𝑛𝑛 𝑥𝑥𝑛𝑛 = tr 𝐴𝐴 = 𝑥𝑥.
Ta có 𝐴𝐴2 = (𝑈𝑈 𝑇𝑇 𝑉𝑉 )(𝑈𝑈 𝑇𝑇 𝑉𝑉 ) = 𝑈𝑈 𝑇𝑇 (𝑉𝑉 𝑈𝑈 𝑇𝑇 )𝑉𝑉 = 𝑥𝑥𝑈𝑈 𝑇𝑇 𝑉𝑉 = 𝑥𝑥𝑥𝑥.
Bằng quy nạp, ta có 𝐴𝐴2006 = 𝑥𝑥2005 𝐴𝐴 mà 𝐴𝐴2006 = 𝐴𝐴 suy ra (1 − 𝑥𝑥2005 )𝐴𝐴 = 0, do 𝐴𝐴 ≠ 0 nên 𝑥𝑥 = 1.
Từ đó 𝐴𝐴2 = 𝐴𝐴 hay 𝐴𝐴(𝐼𝐼 − 𝐴𝐴) = 0. Nếu 𝐼𝐼 − 𝐴𝐴 suy biến thì 𝐴𝐴 = 0, mâu thuẫn.
Vậy 𝐼𝐼 − 𝐴𝐴 khả nghịch.

Cách 2: Sử dụng ánh xạ tuyến tính

Với 𝑛𝑛 = 1 thì hiển nhiên. Giả sử 𝑛𝑛 ≥ 2, xét ánh xạ


𝜑𝜑: ℝ𝑛𝑛 → ℝ𝑛𝑛
𝑥𝑥 ↦ 𝜑𝜑(𝑥𝑥) = 𝐴𝐴𝐴𝐴
Khi đó 𝜑𝜑(ℝ𝑛𝑛 ) là không gian con của ℝ𝑛𝑛 có số chiều bằng 1 (do rank 𝐴𝐴 = 1). Gọi 𝑒𝑒0 là một vector
khác 0 bất kì của 𝜑𝜑(ℝ𝑛𝑛 ). Khi đó 𝐴𝐴𝑒𝑒0 = 𝑎𝑎𝑒𝑒0 (𝑎𝑎 ∈ ℝ∗ ). Mặt khác, do 𝐴𝐴2006 = 𝐴𝐴 nên ta có
𝐴𝐴𝑒𝑒0 = 𝐴𝐴2006 𝑒𝑒0 = 𝐴𝐴2005 𝑎𝑎𝑒𝑒0 = 𝐴𝐴2004 𝑎𝑎2 𝑒𝑒0 =. . . = 𝑎𝑎2006 𝑒𝑒0
Từ đó, ta thu được đẳng thức 𝑎𝑎𝑒𝑒0 = 𝑎𝑎2006 𝑒𝑒0 hay (𝑎𝑎2006 − 𝑎𝑎)𝑒𝑒0 = 0 suy ra 𝑎𝑎 = 1. Như vậy 𝐴𝐴𝑒𝑒0 =
𝑒𝑒0 hay (𝐼𝐼 − 𝐴𝐴)𝑒𝑒0 = 0 nghĩa là hệ phương trình tuyến tính có nghiệm không tầm thường (𝑒𝑒0 ≠ 0)
nên ma trận 𝐼𝐼 − 𝐴𝐴 suy biến.

Bài 2.14. (Câu 7 Virginia Tech Regional Mathematics Contest 1991) Cho 𝐴𝐴 = �𝑎𝑎𝑖𝑖𝑖𝑖 �, 𝐵𝐵 =
�𝑏𝑏𝑖𝑖𝑖𝑖 � là các ma trận vuông cấp 𝑛𝑛, 𝐴𝐴 khả nghịch, 𝐴𝐴𝑡𝑡 , 𝐵𝐵𝑡𝑡 là ma trận mà hàng cuối của 𝐴𝐴, 𝐵𝐵 được nhân
thêm một số 𝑡𝑡 ≠ 0. Chứng minh với mọi 𝑛𝑛 thì 𝐴𝐴−1 −1
𝑡𝑡 𝐵𝐵𝑡𝑡 = 𝐴𝐴 𝐵𝐵.

Chứng minh

Cách 1:

Ký hiệu
𝑎𝑎11 … 𝑎𝑎1𝑛𝑛 𝑏𝑏11 … 𝑏𝑏1𝑛𝑛
𝐴𝐴 = � ⋮ ⋱ ⋮ � ; 𝐵𝐵 = � ⋮ ⋱ ⋮ �
𝑎𝑎𝑛𝑛1 … 𝑎𝑎𝑛𝑛𝑛𝑛 𝑏𝑏𝑛𝑛1 … 𝑏𝑏𝑛𝑛𝑛𝑛

Đỗ Minh Triết II-25 - MATHTASY -


Ôn thi Olympic Toán Đại số Phần II: Một số kiến thức cơ bản về Ma trận

thì khi đó
𝑎𝑎11 … 𝑎𝑎1𝑛𝑛 𝑏𝑏11 … 𝑏𝑏1𝑛𝑛
⎛ ⋮ ⋱ ⋮ ⎞ ⎛ ⋮ ⋱ ⋮ ⎞

𝐴𝐴𝑡𝑡 = ⎜ ⎟ ; 𝐵𝐵 = ⎜
⎜ ⎟
⎜𝑎𝑎𝑛𝑛−1,1 … 𝑎𝑎𝑛𝑛−1,𝑛𝑛 ⎟
⎟ 𝑡𝑡 ⎜𝑏𝑏𝑛𝑛−1,1 … 𝑏𝑏𝑛𝑛−1,𝑛𝑛 ⎟⎟
⎝ 𝑡𝑡𝑎𝑎𝑛𝑛1 … 𝑡𝑡𝑎𝑎𝑛𝑛𝑛𝑛 ⎠ ⎝ 𝑡𝑡𝑏𝑏𝑛𝑛1 … 𝑡𝑡𝑏𝑏𝑛𝑛𝑛𝑛 ⎠
Do 𝑡𝑡 ≠ 0 nên 𝐴𝐴𝑡𝑡 khả nghịch và
1
⎛ 𝐴𝐴11 … 𝐴𝐴𝑛𝑛−1,1 𝐴𝐴
𝐴𝐴11 … 𝐴𝐴𝑛𝑛1 ⎜ 𝑡𝑡 𝑛𝑛1 ⎞ ⎟
𝐴𝐴−1 =� ⋮ ⋱ −1 ⎜
⋮ � ; 𝐴𝐴𝑡𝑡 = ⎜ ⋮ ⋱ ⋮ ⋮ ⎟ ⎟

⎜ ⎟

𝐴𝐴1𝑛𝑛 … 𝐴𝐴𝑛𝑛𝑛𝑛 1
𝐴𝐴 … 𝐴𝐴𝑛𝑛−1,𝑛𝑛 𝐴𝐴𝑛𝑛𝑛𝑛 ⎠
⎝ 1𝑛𝑛 𝑡𝑡
Tính toán và so sánh 𝐴𝐴−1
𝑡𝑡 𝐵𝐵𝑡𝑡 với 𝐴𝐴 𝐵𝐵 , ta có điều phải chứng minh.
−1

Cách 2:
1 … 0 0
⎛ ⋮ ⋱ ⋮ ⋮ ⎞ thì ta có 𝐴𝐴 = 𝐷𝐷𝐷𝐷, 𝐵𝐵 = 𝐷𝐷𝐷𝐷, khi đó do 𝑡𝑡 ≠ 0 nên 𝐷𝐷 khả nghịch và
Đặt 𝐷𝐷 = ⎜
⎜0 … 1 0⎟ ⎟ 𝑡𝑡 𝑡𝑡

⎝0 … 0 𝑡𝑡 ⎠
𝐴𝐴−1 −1 −1 −1 −1
𝑡𝑡 𝐵𝐵𝑡𝑡 = (𝐷𝐷𝐷𝐷) 𝐷𝐷𝐷𝐷 = 𝐴𝐴 𝐷𝐷 𝐷𝐷𝐷𝐷 = 𝐴𝐴 𝐵𝐵

Bài 2.15. (Câu 3 đề chọn đội tuyển ĐH Kinh tế Quốc dân 2009)
Cho 𝐴𝐴 làm ma trận khả nghịch, ma trận 𝐴𝐴−1 sẽ thay đổi ra sao nếu ta thực hiện các phép biến đổi sau
đây trên 𝐴𝐴:
a) Đổi chỗ dòng 𝑖𝑖, 𝑗𝑗 cho nhau.
b) Nhân dòng 𝑖𝑖 với số thực 𝛼𝛼 ≠ 0.
c) Cộng một bội của dòng 𝑗𝑗 vào dòng 𝑖𝑖.

Chứng minh.

Ký hiệu
𝑎𝑎11 𝑎𝑎12 … 𝑎𝑎1,𝑛𝑛−1 𝑎𝑎1𝑛𝑛
⎛ 𝑎𝑎21 𝑎𝑎22 … 𝑎𝑎2,𝑛𝑛−1 𝑎𝑎2𝑛𝑛 ⎞

⎜ ⎟
⎜ ⋮ ⋮ … ⋮ ⋮ ⎟ ⎟

⎜ ⎟
𝑎𝑎 𝑎𝑎𝑖𝑖2 … 𝑎𝑎𝑖𝑖,𝑛𝑛−1 𝑎𝑎𝑖𝑖𝑖𝑖 ⎟
𝐴𝐴 = ⎜ ⎟
𝑖𝑖1

⎜ ⋮ ⋮ … ⋮ ⋮ ⎟ ⎟

⎜ ⎟
⎜ 𝑎𝑎𝑗𝑗1 𝑎𝑎𝑗𝑗2 … 𝑎𝑎𝑗𝑗,𝑛𝑛−1 𝑎𝑎𝑗𝑗𝑗𝑗 ⎟


⎜ ⋮ ⎟
⋮ … ⋮ ⋮ ⎟
⎝𝑎𝑎𝑛𝑛1 𝑎𝑎𝑛𝑛2 … 𝑎𝑎𝑛𝑛,𝑛𝑛−1 𝑎𝑎𝑛𝑛𝑛𝑛 ⎠

Đỗ Minh Triết II-26 - MATHTASY -


Ôn thi Olympic Toán Đại số Phần II: Một số kiến thức cơ bản về Ma trận

𝐴𝐴11 𝐴𝐴21 … 𝐴𝐴𝑖𝑖1 … 𝐴𝐴𝑗𝑗1 … 𝐴𝐴𝑛𝑛−1,1 𝐴𝐴𝑛𝑛1



⎜ 𝐴𝐴12 𝐴𝐴22 … 𝐴𝐴𝑖𝑖2 … 𝐴𝐴𝑗𝑗2 … 𝐴𝐴𝑛𝑛−1,2 𝐴𝐴𝑛𝑛2 ⎞ ⎟
1 ⎜ ⎜ ⎟
𝐴𝐴−1 = ⎜ ⋮ ⋮ … ⋮ … ⋮ … ⋮ ⋮ ⎟ ⎟
|𝐴𝐴| ⎜
⎜ ⎟
𝐴𝐴𝑛𝑛,𝑛𝑛−1 ⎟
⎜𝐴𝐴1,𝑛𝑛−1 𝐴𝐴2,𝑛𝑛−1 … 𝐴𝐴𝑖𝑖,𝑛𝑛−1 … 𝐴𝐴𝑖𝑖,𝑛𝑛−1 … 𝐴𝐴𝑛𝑛−1,𝑛𝑛−1 ⎟
⎝ 𝐴𝐴1𝑛𝑛 𝐴𝐴2𝑛𝑛 … 𝐴𝐴𝑖𝑖𝑖𝑖 … 𝐴𝐴𝑗𝑗𝑗𝑗 … 𝐴𝐴𝑛𝑛−1,𝑛𝑛 𝐴𝐴𝑛𝑛𝑛𝑛 ⎠
trong đó 𝐴𝐴𝑖𝑖𝑖𝑖 , (𝑖𝑖, 𝑗𝑗) ∈ {1, … , 𝑛𝑛}2 là các phần bù đại số ứng với phần tử 𝑎𝑎𝑖𝑖𝑖𝑖 .

Cách 1:

a) Đổi chỗ dòng 𝑖𝑖, 𝑗𝑗 của 𝐴𝐴 cho nhau


𝑎𝑎11 𝑎𝑎12 … 𝑎𝑎1,𝑛𝑛−1 𝑎𝑎1𝑛𝑛
⎛ 𝑎𝑎21 𝑎𝑎22 … 𝑎𝑎2,𝑛𝑛−1 𝑎𝑎2𝑛𝑛 ⎞

⎜ ⎟
⎜ ⋮ ⋮ … ⋮ ⋮ ⎟ ⎟

⎜ ⎟
𝑎𝑎 𝑎𝑎𝑗𝑗2 … 𝑎𝑎𝑗𝑗,𝑛𝑛−1 𝑎𝑎𝑗𝑗𝑗𝑗 ⎟
𝐴𝐴 → 𝐴𝐴1 = ⎜ ⎟
𝑗𝑗1

⎜ ⋮ ⋮ … ⋮ ⋮ ⎟ ⎟

⎜ ⎟
⎜ 𝑎𝑎 𝑖𝑖1 𝑎𝑎𝑖𝑖2 … 𝑎𝑎𝑖𝑖,𝑛𝑛−1 𝑎𝑎𝑖𝑖𝑖𝑖 ⎟


⎜ ⋮ ⎟
⋮ … ⋮ ⋮ ⎟
⎝𝑎𝑎𝑛𝑛1 𝑎𝑎𝑛𝑛2 … 𝑎𝑎𝑛𝑛,𝑛𝑛−1 𝑎𝑎𝑛𝑛𝑛𝑛 ⎠
Khi đó:
- |𝐴𝐴1 | = −|𝐴𝐴|.
- Các phần bù đại số ngoài dòng 𝑖𝑖, 𝑗𝑗 bị đổi dấu.
- Các phần bù đại số của dòng 𝑖𝑖, 𝑗𝑗 bị hoán đổi về định thức con bù, còn dấu của chúng không đổi.
Như vậy, ma trận khả nghịch bị biến đổi như sau
𝐴𝐴−1 → 𝐴𝐴−1
1

−𝐴𝐴11 −𝐴𝐴21 … (−1)𝑖𝑖+1 𝑀𝑀𝑗𝑗1 … (−1)𝑗𝑗+1 𝑀𝑀𝑖𝑖1 … −𝐴𝐴𝑛𝑛−1,1 −𝐴𝐴𝑛𝑛1



⎜ −𝐴𝐴12 ⎞
−𝐴𝐴22 … (−1) 𝑖𝑖+2
𝑀𝑀𝑗𝑗2 … (−1) 𝑀𝑀𝑖𝑖2 𝑗𝑗+2
… −𝐴𝐴𝑛𝑛−1,2 −𝐴𝐴𝑛𝑛2 ⎟
1 ⎜ ⎜




= ⋮ ⋮ … ⋮ … ⋮ … ⋮ ⋮
−|𝐴𝐴| ⎜




⎜−𝐴𝐴1,𝑛𝑛−1 −𝐴𝐴2,𝑛𝑛−1 … (−1)𝑖𝑖+𝑛𝑛−1 𝑀𝑀𝑗𝑗,𝑛𝑛−1 … (−1)𝑗𝑗+𝑛𝑛−1 𝑀𝑀𝑖𝑖,𝑛𝑛−1 … −𝐴𝐴𝑛𝑛−1,𝑛𝑛−1 −𝐴𝐴𝑛𝑛,𝑛𝑛−1 ⎟

⎝ −𝐴𝐴1𝑛𝑛 −𝐴𝐴2𝑛𝑛 … (−1)𝑖𝑖+𝑛𝑛 𝑀𝑀𝑗𝑗𝑗𝑗 … (−1)𝑗𝑗+𝑛𝑛 𝑀𝑀𝑖𝑖𝑖𝑖 … −𝐴𝐴𝑛𝑛−1,𝑛𝑛 −𝐴𝐴𝑛𝑛𝑛𝑛 ⎠
𝐴𝐴11 𝐴𝐴21 … 𝐴𝐴𝑗𝑗1 … 𝐴𝐴11 … 𝐴𝐴𝑛𝑛−1,1 𝐴𝐴𝑛𝑛1

⎜ 𝐴𝐴12 𝐴𝐴22 … 𝐴𝐴𝑗𝑗2 … 𝐴𝐴𝑖𝑖2 … 𝐴𝐴𝑛𝑛−1,2 𝐴𝐴𝑛𝑛2 ⎞ ⎟
1 ⎜ ⎜ ⎟
= ⎜ ⋮ ⋮ … ⋮ … ⋮ … ⋮ ⋮ ⎟ ⎟
|𝐴𝐴| ⎜
⎜ ⎟
𝐴𝐴𝑛𝑛,𝑛𝑛−1 ⎟
⎜𝐴𝐴 1,𝑛𝑛−1 𝐴𝐴2,𝑛𝑛−1 … 𝐴𝐴𝑗𝑗,𝑛𝑛−1 … 𝐴𝐴𝑖𝑖,𝑛𝑛−1 … 𝐴𝐴𝑛𝑛−1,𝑛𝑛−1 ⎟
⎝ 𝐴𝐴1𝑛𝑛 𝐴𝐴2𝑛𝑛 … 𝐴𝐴𝑗𝑗𝑗𝑗 … 𝐴𝐴𝑖𝑖𝑖𝑖 … 𝐴𝐴𝑛𝑛−1,𝑛𝑛 𝐴𝐴𝑛𝑛𝑛𝑛 ⎠
b) Dòng 𝑖𝑖 của 𝐴𝐴 được nhân thêm một số thực 𝛼𝛼 ≠ 0

Đỗ Minh Triết II-27 - MATHTASY -


Ôn thi Olympic Toán Đại số Phần II: Một số kiến thức cơ bản về Ma trận

𝑎𝑎11 𝑎𝑎12 … 𝑎𝑎1,𝑛𝑛−1 𝑎𝑎1𝑛𝑛


⎛ 𝑎𝑎21 𝑎𝑎22 … 𝑎𝑎2,𝑛𝑛−1 𝑎𝑎2𝑛𝑛 ⎞

⎜ ⎟
⎜ ⋮ ⋮ … ⋮ ⋮ ⎟ ⎟

⎜ ⎟
𝛼𝛼𝑎𝑎 𝛼𝛼𝑎𝑎𝑖𝑖2 … 𝛼𝛼𝑎𝑎𝑖𝑖,𝑛𝑛−1 𝛼𝛼𝑎𝑎𝑖𝑖𝑖𝑖 ⎟
𝐴𝐴 → 𝐴𝐴2 = ⎜ ⎟
𝑖𝑖1

⎜ ⋮ ⋮ … ⋮ ⋮ ⎟ ⎟

⎜ ⎟
⎜ 𝑎𝑎𝑗𝑗1 𝑎𝑎𝑗𝑗2 … 𝑎𝑎𝑗𝑗,𝑛𝑛−1 𝑎𝑎𝑗𝑗𝑗𝑗 ⎟⎟

⎜ ⋮ ⎟
⋮ … ⋮ ⋮ ⎟
⎝ 𝑎𝑎𝑛𝑛1 𝑎𝑎𝑛𝑛2 … 𝑎𝑎𝑛𝑛,𝑛𝑛−1 𝑎𝑎𝑛𝑛𝑛𝑛 ⎠
Khi đó:
- |𝐴𝐴2 | = 𝛼𝛼|𝐴𝐴|
- Các phần bù đại số của dòng 𝑖𝑖 không đổi.
- Các phần bù đại số ngoài dòng 𝑖𝑖 được nhân thêm 𝛼𝛼.
𝛼𝛼𝐴𝐴11 𝛼𝛼𝐴𝐴21 … 𝐴𝐴𝑖𝑖1 … 𝛼𝛼𝐴𝐴𝑗𝑗1 … 𝛼𝛼𝐴𝐴𝑛𝑛−1,1 𝛼𝛼𝐴𝐴𝑛𝑛1

⎜ 𝛼𝛼𝐴𝐴12 𝛼𝛼𝐴𝐴22 … 𝐴𝐴𝑖𝑖2 … 𝛼𝛼𝐴𝐴𝑗𝑗2 … 𝛼𝛼𝐴𝐴𝑛𝑛−1,2 𝛼𝛼𝐴𝐴𝑛𝑛2 ⎞ ⎟
1 ⎜ ⎜ ⎟

𝐴𝐴−1 → 𝐴𝐴−1 = ⎜ ⋮ ⋮ … ⋮ … ⋮ … ⋮ ⋮ ⎟
2
𝛼𝛼|𝐴𝐴| ⎜
⎜ ⎟
𝛼𝛼𝐴𝐴𝑛𝑛,𝑛𝑛−1 ⎟
⎜𝛼𝛼𝐴𝐴 1,𝑛𝑛−1 𝛼𝛼𝐴𝐴2,𝑛𝑛−1 … 𝐴𝐴𝑖𝑖,𝑛𝑛−1 … 𝛼𝛼𝐴𝐴𝑖𝑖,𝑛𝑛−1 … 𝛼𝛼𝐴𝐴𝑛𝑛−1,𝑛𝑛−1 ⎟
⎝ 𝛼𝛼𝐴𝐴1𝑛𝑛 𝛼𝛼𝐴𝐴2𝑛𝑛 … 𝐴𝐴𝑖𝑖𝑖𝑖 … 𝛼𝛼𝐴𝐴𝑗𝑗𝑗𝑗 … 𝛼𝛼𝐴𝐴𝑛𝑛−1,𝑛𝑛 𝛼𝛼𝐴𝐴𝑛𝑛𝑛𝑛 ⎠
1
⎛ 𝐴𝐴11 𝐴𝐴21 … 𝐴𝐴 … 𝐴𝐴𝑗𝑗1 … 𝐴𝐴𝑛𝑛−1,1 𝐴𝐴𝑛𝑛1 ⎞
⎜ 𝛼𝛼 𝑖𝑖1 ⎟

⎜ 1 ⎟


⎜ 𝐴𝐴12 𝐴𝐴22 … 𝐴𝐴 … 𝐴𝐴𝑗𝑗2 … 𝐴𝐴𝑛𝑛−1,2 𝐴𝐴𝑛𝑛2 ⎟ ⎟
1 ⎜ ⎜ 𝛼𝛼 𝑖𝑖2 ⎟
= ⎜ ⋮ ⋮ … ⋮ … ⋮ … ⋮ ⋮ ⎟ ⎟
|𝐴𝐴| ⎜
⎜ 1



⎜𝐴𝐴1,𝑛𝑛−1 𝐴𝐴2,𝑛𝑛−1 … 𝐴𝐴 … 𝐴𝐴𝑖𝑖,𝑛𝑛−1 … 𝐴𝐴𝑛𝑛−1,𝑛𝑛−1 𝐴𝐴𝑛𝑛,𝑛𝑛−1 ⎟


⎜ 𝛼𝛼 𝑖𝑖,𝑛𝑛−1 ⎟

⎜ 1 ⎟
⎝ 𝐴𝐴1𝑛𝑛 𝐴𝐴2𝑛𝑛 … 𝐴𝐴
𝛼𝛼 𝑖𝑖𝑖𝑖
… 𝐴𝐴𝑗𝑗𝑗𝑗 … 𝐴𝐴𝑛𝑛−1,𝑛𝑛 𝐴𝐴𝑛𝑛𝑛𝑛 ⎠

c) Giả sử ta cộng 𝑘𝑘 lần dòng 𝑗𝑗 vào dòng 𝑖𝑖 của ma trận 𝐴𝐴, tức là
𝑎𝑎11 𝑎𝑎12 … 𝑎𝑎1,𝑛𝑛−1 𝑎𝑎1𝑛𝑛
⎛ 𝑎𝑎21 𝑎𝑎22 … 𝑎𝑎2,𝑛𝑛−1 𝑎𝑎2𝑛𝑛 ⎞

⎜ ⎟

⎜ ⋮ ⋮ … ⋮ ⋮ ⎟

⎜ ⎟
𝑎𝑎 + 𝑘𝑘𝑎𝑎 𝑎𝑎 + 𝑘𝑘𝑎𝑎𝑗𝑗2 … 𝑎𝑎𝑖𝑖,𝑛𝑛−1 + 𝑘𝑘𝑎𝑎𝑗𝑗,−1 𝑎𝑎𝑖𝑖𝑖𝑖 + 𝑘𝑘𝑎𝑎𝑗𝑗𝑗𝑗 ⎟
𝐴𝐴 → 𝐴𝐴3 = ⎜ ⎟
𝑖𝑖1 𝑗𝑗1 𝑖𝑖2

⎜ ⋮ ⋮ … ⋮ ⋮ ⎟


⎜ ⎟

⎜ 𝑎𝑎 𝑗𝑗1 𝑎𝑎 𝑗𝑗2 … 𝑎𝑎𝑗𝑗,𝑛𝑛−1 𝑎𝑎𝑗𝑗𝑗𝑗 ⎟

⎜ ⎟

⋮ ⋮ … ⋮ ⋮
⎝ 𝑎𝑎𝑛𝑛1 𝑎𝑎𝑛𝑛2 … 𝑎𝑎𝑛𝑛,𝑛𝑛−1 𝑎𝑎𝑛𝑛𝑛𝑛 ⎠
Khi đó:
- Hiển nhiên là |𝐴𝐴3 | = |𝐴𝐴|.
- Các phần bù đại số của dòng 𝑖𝑖 không thay đổi.
- Các phần bù đại số của những phần tử ở dòng 𝑗𝑗 thay đổi, sử dụng tính đa tuyến tính để tách
���������
định thức, ta được 𝐴𝐴3 = 𝐴𝐴𝑗𝑗𝑗𝑗 − 𝑘𝑘𝐴𝐴𝑖𝑖𝑖𝑖 , 𝑘𝑘 = 1, 𝑛𝑛.
𝑗𝑗𝑗𝑗

Đỗ Minh Triết II-28 - MATHTASY -


Ôn thi Olympic Toán Đại số Phần II: Một số kiến thức cơ bản về Ma trận

- Mọi phần bù đại số còn lại không thay đổi, vì là những định thức được biến đổi sơ cấp cộng vào
một dòng một bội lần dòng khác.
𝐴𝐴11 𝐴𝐴21 … 𝐴𝐴𝑖𝑖1 … 𝐴𝐴𝑗𝑗1 − 𝑘𝑘𝐴𝐴𝑖𝑖1 … 𝐴𝐴𝑛𝑛−1,1 𝐴𝐴𝑛𝑛1

⎜ 𝐴𝐴12 𝐴𝐴22 … 𝐴𝐴𝑖𝑖2 … 𝐴𝐴𝑗𝑗2 − 𝑘𝑘𝐴𝐴𝑖𝑖2 … 𝐴𝐴𝑛𝑛−1,2 𝐴𝐴𝑛𝑛2 ⎞ ⎟
1 ⎜ ⎜ ⎟
𝐴𝐴−1 → 𝐴𝐴−1 = ⎜ ⋮ ⋮ … ⋮ … ⋮ … ⋮ ⋮ ⎟ ⎟
3
|𝐴𝐴| ⎜
⎜ ⎟
𝐴𝐴𝑛𝑛,𝑛𝑛−1 ⎟
⎜𝐴𝐴 1,𝑛𝑛−1 𝐴𝐴2,𝑛𝑛−1 … 𝐴𝐴𝑖𝑖,𝑛𝑛−1 … 𝐴𝐴𝑖𝑖,𝑛𝑛−1 − 𝑘𝑘𝐴𝐴𝑖𝑖,𝑛𝑛−1 … 𝐴𝐴𝑛𝑛−1,𝑛𝑛−1 ⎟
⎝ 𝐴𝐴1𝑛𝑛 𝐴𝐴2𝑛𝑛 … 𝐴𝐴𝑖𝑖𝑖𝑖 … 𝐴𝐴𝑗𝑗𝑗𝑗 − 𝑘𝑘𝐴𝐴𝑖𝑖𝑖𝑖 … 𝐴𝐴𝑛𝑛−1,𝑛𝑛 𝐴𝐴𝑛𝑛𝑛𝑛 ⎠

Cách 2:

a) Ta có 𝐴𝐴1 = 𝐷𝐷𝐷𝐷, trong đó 𝐷𝐷 là ma trận mà dòng 𝑖𝑖, 𝑗𝑗 đổi chỗ cho nhau trong ma trận đơn vị
và dễ thấy 𝐷𝐷 = 𝐷𝐷−1 , suy ra 𝐴𝐴−1
1 = (𝐷𝐷𝐷𝐷)
−1
= 𝐴𝐴−1 𝐷𝐷−1 = 𝐴𝐴−1 𝐷𝐷 cho kết quả như trên.
b) Ta có 𝐴𝐴2 = 𝑀𝑀𝑀𝑀, trong đó 𝑀𝑀 là ma trận đường chéo có phần tử 𝑎𝑎𝑖𝑖𝑖𝑖 = 𝛼𝛼, các phần tử chéo còn
lại bằng 1. Khi đó 𝑀𝑀 −1 cũng là ma trận đường chéo có phần tử 𝑎𝑎𝑖𝑖𝑖𝑖 = 𝛼𝛼1 , các phần tử chéo còn
lại bằng 1, suy ra 𝐴𝐴−1
2 = (𝑀𝑀𝑀𝑀)
−1
= 𝐴𝐴−1 𝑀𝑀 −1 cho kết quả trên.
c) Ta có 𝐴𝐴3 = 𝑇𝑇𝑇𝑇, trong đó 𝑇𝑇 là ma trận mà mọi phần tử trên đường chéo chính bằng 1 và phần
tử ở hàng 𝑖𝑖 cột 𝑗𝑗 là 𝑘𝑘 và dễ thấy 𝑇𝑇 −1 cũng là ma trận mà mọi phần tử trên đường chéo chính
bằng 1 nhưng phần tử ở hàng 𝑖𝑖 cột 𝑗𝑗 là – 𝑘𝑘, suy ra 𝐴𝐴−1
3 = (𝑇𝑇𝑇𝑇)
−1
= 𝐴𝐴−1 𝑇𝑇 −1 cho kết quả như trên.

Bài 2.16. (Câu 4 Olympic 2008) Cho các số thực 𝑎𝑎1 , 𝑎𝑎2 , … , 𝑎𝑎2008 . Chứng minh rằng tồn tại
các ma trận thực vuông cấp 𝑛𝑛 (𝑛𝑛 > 1) 𝐴𝐴1 , 𝐴𝐴2 , … , 𝐴𝐴2008 thỏa mãn
2008
det 𝐴𝐴𝑘𝑘 = 𝑎𝑎𝑘𝑘 , 𝑘𝑘 = 1, … ,2008 và det �� 𝐴𝐴𝑘𝑘 � = 2009
𝐾𝐾=1

Chứng minh.

Đặt 𝑠𝑠 = ∑2008 𝑎𝑎 , 𝑏𝑏 = 2008𝑠𝑠 − 2009


𝑘𝑘=1 𝑘𝑘 𝑛𝑛−2 . Xét các ma trận cấp 𝑛𝑛 sau
2008
𝑎𝑎1 1 0 … 0 𝑎𝑎2 0 0 … 0 𝑎𝑎𝑘𝑘 0 0 … 0
⎛ 0 1 0 … 0⎟ ⎞ ⎛ 𝑏𝑏 1 0 … 0⎞ ⎛ 0 1 0 … 0⎞

𝐴𝐴1 = ⎜
⎜ ⎟ , 𝐴𝐴 = ⎜
⎜ ⎟
⎟, ⎜
𝐴𝐴𝑘𝑘 = ⎜



⎜ 0 0 1 … 0⎟ ⎟ 2 ⎜
⎜ 0 0 1 … 0⎟⎟ ⎜
⎜ 0 0 1 … 0⎟⎟ (𝑘𝑘
⋮ ⋮ ⋮ ⋱ ⋮⎟ ⎜ ⋮ ⋮ ⋮ ⋱ ⋮⎟ ⎜ ⋮ ⋮ ⋮ ⋱ ⋮⎟
⎝ 0 0 0 … 1⎠ ⎝0 0 0 … 1⎠ ⎝ 0 0 0 … 1⎠
= 3,4, … 2008)
Do đó det 𝐴𝐴𝑘𝑘 = 𝑎𝑎𝑘𝑘 , 𝑘𝑘 = 1, … ,2008. Mặt khác:
𝑠𝑠 1 0 …… 0
2008 ⎛
⎜ 𝑏𝑏 2008 0 …… 0 ⎞

� 𝐴𝐴𝑘𝑘 = ⎜
⎜ 0 0 2008 …… 0 ⎟

𝑘𝑘=1
⎜ ⋮ ⋮ ⋮ ⋱ ⋮ ⎟
⎝0 0 0 … 2008 ⎠
Khai triển Laplace theo cột thứ nhất, ta được:

Đỗ Minh Triết II-29 - MATHTASY -


Ôn thi Olympic Toán Đại số Phần II: Một số kiến thức cơ bản về Ma trận

2008
det �� 𝐴𝐴𝑘𝑘 � = 𝑠𝑠. 2008𝑛𝑛−1 − 𝑏𝑏. 2008𝑛𝑛−2 = 2009
𝑘𝑘=1

Bài 2.17. (Câu 3 Olympic Quốc tế 2000)


Cho 𝐴𝐴, 𝐵𝐵 ∈ 𝑀𝑀𝑛𝑛 (ℂ) và rank(𝐴𝐴𝐴𝐴 − 𝐵𝐵𝐵𝐵) = 1. Chứng minh (𝐴𝐴𝐴𝐴 − 𝐵𝐵𝐵𝐵)2 = 0.

Chứng minh.

Vì rank(𝐴𝐴𝐴𝐴 − 𝐵𝐵𝐵𝐵) = 1 nên tồn tại vector dòng khác 0 trong 𝐴𝐴 và các vector dòng còn lại đều
biểu diễn tuyến tính được qua dòng này. Do đó ma trận 𝐴𝐴𝐴𝐴 − 𝐵𝐵𝐵𝐵 có dạng sau:
𝜆𝜆1 𝑥𝑥1 𝜆𝜆1 𝑥𝑥2 … 𝜆𝜆1 𝑥𝑥𝑛𝑛

⎜ 𝜆𝜆2 𝑥𝑥1 𝜆𝜆2 𝑥𝑥2 … 𝜆𝜆𝑛𝑛 𝑥𝑥𝑛𝑛 ⎞⎟

⎜ ⎟
… … … … ⎟
𝐴𝐴𝐴𝐴 − 𝐵𝐵𝐵𝐵 = ⎜
⎜ ⎟≠0
⎜ 𝜆𝜆𝑖𝑖 𝑥𝑥1 𝜆𝜆𝑖𝑖 𝑥𝑥2 … 𝜆𝜆𝑖𝑖 𝑥𝑥𝑛𝑛 ⎟


⎜ … ⎟
… … … ⎟
⎝𝜆𝜆𝑛𝑛 𝑥𝑥1 𝜆𝜆𝑛𝑛 𝑥𝑥2 … 𝜆𝜆𝑛𝑛 𝑥𝑥𝑛𝑛 ⎠
���������
Không mất tính tổng quát, giả sử dòng khác 0 đó là dòng đầu và 𝜆𝜆𝑖𝑖 ≠ 0 �𝑖𝑖 = 1, 𝑛𝑛�.
𝜆𝜆1 𝑥𝑥1
Đặt 𝑈𝑈 = � ⋮ � ≠ 0, 𝑉𝑉 = � ⋮ � ≠ 0.
𝜆𝜆𝑛𝑛 𝑥𝑥𝑛𝑛
Khi đó 𝐴𝐴𝐴𝐴 − 𝐵𝐵𝐵𝐵 = 𝑈𝑈 𝑇𝑇 𝑉𝑉 và 𝑉𝑉 𝑈𝑈 𝑇𝑇 = 𝜆𝜆1 𝑥𝑥1 + 𝜆𝜆2 𝑥𝑥2 +. . . +𝜆𝜆𝑛𝑛 𝑥𝑥𝑛𝑛 = tr(𝐴𝐴𝐴𝐴 − 𝐵𝐵𝐵𝐵) = 0
Ta có (𝐴𝐴𝐴𝐴 − 𝐵𝐵𝐵𝐵)2 = (𝑈𝑈 𝑇𝑇 𝑉𝑉 )(𝑈𝑈 𝑇𝑇 𝑉𝑉 ) = 𝑈𝑈 𝑇𝑇 (𝑉𝑉 𝑈𝑈 𝑇𝑇 )𝑉𝑉 = 𝑈𝑈 𝑇𝑇 tr(𝐴𝐴𝐴𝐴 − 𝐵𝐵𝐵𝐵) 𝑉𝑉 = 0.

Bài 2.18. Cho các ma trận khả nghịch 𝐴𝐴, 𝐵𝐵 ∈ 𝑀𝑀𝑛𝑛 (ℝ) và số thực 𝑐𝑐 thoả 𝑐𝑐2 ≠ 1. chứng minh
rằng 𝑐𝑐𝐴𝐴𝑇𝑇 + 𝐵𝐵−1 𝐴𝐴𝐵𝐵𝑇𝑇 ≠ 0.

Chứng minh.

Ta chứng minh bằng phản chứng, giả sử 𝑐𝑐𝐴𝐴𝑇𝑇 + 𝐵𝐵−1 𝐴𝐴𝐵𝐵𝑇𝑇 = 0 ⇔ 𝑐𝑐𝐴𝐴𝑇𝑇 = −𝐵𝐵−1 𝐴𝐴𝐵𝐵𝑇𝑇
Lấy định thức hai vế, ta được
det(𝑐𝑐𝐴𝐴𝑇𝑇 ) = det(−𝐵𝐵−1 𝐴𝐴𝐵𝐵𝑇𝑇 ) ⇔ 𝑐𝑐𝑛𝑛 det(𝐴𝐴𝑇𝑇 ) = (−1)𝑛𝑛 det(𝐵𝐵−1 ) det 𝐴𝐴 det(𝐵𝐵𝑇𝑇 )
⇔ 𝑐𝑐𝑛𝑛 det 𝐴𝐴 = (−1)𝑛𝑛 det(𝐵𝐵−1 ) det 𝐵𝐵 det 𝐴𝐴 ⇔ 𝑐𝑐𝑛𝑛 det 𝐴𝐴 = (−1)𝑛𝑛 det 𝐴𝐴 ⇔ 𝑐𝑐𝑛𝑛 = (−1)𝑛𝑛
Điều này mâu thuẫn với giả thiết 𝑐𝑐 ≠ ±1 dù 𝑛𝑛 chẵn hay lẻ. Vậy 𝑐𝑐𝐴𝐴𝑇𝑇 + 𝐵𝐵−1 𝐴𝐴𝐵𝐵𝑇𝑇 ≠ 0.

Bài 2.19. Chứng minh rằng nếu 𝐴𝐴 là ma trận vuông cấp 𝑛𝑛 thoả 𝑎𝑎𝐴𝐴2 + 𝑏𝑏𝑏𝑏 + 𝑐𝑐𝑐𝑐 = 0 thì 𝐴𝐴 −
𝑘𝑘𝑘𝑘 khả nghịch và tính (𝐴𝐴 − 𝑘𝑘𝑘𝑘)−1 .

Chứng minh.

Ta có 𝑎𝑎𝐴𝐴2 + 𝑏𝑏𝑏𝑏 + 𝑐𝑐𝑐𝑐 = 0 ⇔ 𝑎𝑎(𝐴𝐴2 − 2𝑘𝑘𝑘𝑘 + 𝑘𝑘2 𝐼𝐼) + (𝑏𝑏 − 2𝑎𝑎𝑎𝑎)𝐴𝐴 + 𝑐𝑐𝑐𝑐 − 𝑎𝑎𝑘𝑘2 𝐼𝐼 = 0
⇔ 𝑎𝑎(𝐴𝐴 − 𝑘𝑘𝑘𝑘)2 + (𝑏𝑏 − 2𝑎𝑎𝑎𝑎)(𝐴𝐴 − 𝑘𝑘𝑘𝑘) + 𝑐𝑐𝑐𝑐 + 𝑘𝑘𝑘𝑘𝑘𝑘 − 3𝑎𝑎𝑘𝑘2 𝐼𝐼 = 0

Đỗ Minh Triết II-30 - MATHTASY -


Ôn thi Olympic Toán Đại số Phần II: Một số kiến thức cơ bản về Ma trận

⇔ (𝐴𝐴 − 𝑘𝑘𝑘𝑘)[𝑎𝑎(𝐴𝐴 − 𝑘𝑘𝑘𝑘) + (𝑏𝑏 − 2𝑎𝑎𝑎𝑎)𝐼𝐼] = −(𝑐𝑐 + 𝑘𝑘𝑘𝑘 − 3𝑎𝑎𝑘𝑘2 )𝐼𝐼


Suy ra 𝐴𝐴 − 𝑘𝑘𝑘𝑘 khả nghịch và (𝐴𝐴 − 𝑘𝑘𝑘𝑘)−1 = 𝑐𝑐+𝑘𝑘𝑘𝑘−3𝑎𝑎𝑘𝑘
−1
2 [𝑎𝑎(𝐴𝐴 − 𝑘𝑘𝑘𝑘) + (𝑏𝑏 − 2𝑎𝑎𝑎𝑎)𝐼𝐼].

Bài 2.20. Cho 𝐴𝐴1 , 𝐴𝐴2 , 𝐵𝐵, 𝐶𝐶 ∈ 𝑀𝑀2 (𝕂𝕂). Chứng minh 𝐴𝐴1 (𝐵𝐵𝐵𝐵 − 𝐶𝐶𝐶𝐶)2 𝐴𝐴2 =
𝐴𝐴1 𝐴𝐴2 (𝐵𝐵𝐵𝐵 − 𝐶𝐶𝐶𝐶)2 .

Chứng minh.
𝑎𝑎 𝑏𝑏
Vì tr(𝐵𝐵𝐵𝐵 − 𝐶𝐶𝐶𝐶) = 0 nên đặt 𝐵𝐵𝐵𝐵 − 𝐶𝐶𝐶𝐶 = � �, khi đó (𝐵𝐵𝐵𝐵 − 𝐶𝐶𝐶𝐶)2 = (𝑎𝑎2 + 𝑏𝑏𝑏𝑏)𝐼𝐼.
𝑐𝑐 −𝑎𝑎
Do đó
𝐴𝐴1 (𝐵𝐵𝐵𝐵 − 𝐶𝐶𝐶𝐶)2 𝐴𝐴2 = 𝐴𝐴1 (𝑎𝑎2 + 𝑏𝑏𝑏𝑏)𝐼𝐼𝐴𝐴2 = 𝐴𝐴1 𝐴𝐴2 (𝑎𝑎2 + 𝑏𝑏𝑏𝑏)𝐼𝐼 = 𝐴𝐴1 𝐴𝐴2 (𝐵𝐵𝐵𝐵 − 𝐶𝐶𝐶𝐶)2

Bài 2.21. Câu 21 (Đề chọn đội tuyển Đại học Vinh 2003) : Cho 𝐴𝐴, 𝐵𝐵, 𝐶𝐶 là ba ma trận
vuông cấp 2003. Biết 𝐶𝐶 khả nghịch và 𝐴𝐴𝐴𝐴 = 𝐶𝐶𝐶𝐶. Các phần tử trên đường chéo chính của 𝐴𝐴 là 0 hoặc
1, các phần tử trên đường chéo chính của 𝐵𝐵 là 0 hoặc -1. Tìm số phần tử 0 trên đường chéo chính của
𝐴𝐴 và 𝐵𝐵.

Chứng minh.

Từ 𝐴𝐴𝐴𝐴 = 𝐶𝐶𝐶𝐶 và 𝐶𝐶 khả nghịch, ta có 𝐴𝐴 = 𝐶𝐶𝐶𝐶𝐶𝐶 −1 . Suy ra A và B là hai ma trận đồng dạng nên
tr 𝐴𝐴 = tr 𝐵𝐵.
Kết hợp với điều dễ thấy là tr 𝐴𝐴 ≥ 0 và tr 𝐵𝐵 ≤ 0, suy ra tr 𝐴𝐴 = tr 𝐵𝐵 = 0.
Vậy các phần tử chéo của 𝐴𝐴 và 𝐵𝐵 là 0 cả.

Bài 2.22. Cho ma trận vuông 𝐴𝐴 = �𝑎𝑎𝑖𝑖𝑖𝑖 � khả nghịch có mọi phần tử đều dương. Chứng minh
rằng số phần tử bằng 0 của ma trận 𝐴𝐴−1 không quá 𝑛𝑛2 − 2𝑛𝑛.

Chứng minh.

Ký hiệu 𝐴𝐴−1 = �𝑏𝑏𝑖𝑖𝑖𝑖 � thì từ 𝐴𝐴−1 𝐴𝐴 = 𝐼𝐼 ta có


𝑛𝑛
� 𝑏𝑏𝑖𝑖𝑖𝑖 𝑎𝑎𝑘𝑘𝑘𝑘 = 0 ∀𝑖𝑖 ≠ 𝑗𝑗 hay 𝑏𝑏𝑖𝑖1 𝑎𝑎1𝑗𝑗 + 𝑏𝑏𝑖𝑖2 𝑎𝑎2𝑗𝑗 +. . . +𝑏𝑏𝑖𝑖,𝑛𝑛−1 𝑎𝑎𝑛𝑛−1,𝑗𝑗 + 𝑏𝑏𝑖𝑖𝑖𝑖 𝑎𝑎𝑛𝑛𝑛𝑛 = 0
𝑘𝑘=1

Hiển nhiên là trong một dòng của 𝐴𝐴−1 có ít nhất một phần tử khác 0, nhưng vì 𝑎𝑎𝑖𝑖𝑖𝑖 > 0 ∀𝑖𝑖, 𝑗𝑗 nên
từ đẳng thức trên số phần tử khác 0 trong một dòng của 𝐴𝐴−1 phải là 2. Suy ra số phần tử bằng
0 của ma trận 𝐴𝐴−1 không quá 𝑛𝑛2 − 2𝑛𝑛.

Đỗ Minh Triết II-31 - MATHTASY -


PHẦN III. MA TRẬN KHỐI

3.1. Cơ sở lý thuyết

Thông thường, ta chỉ dùng đến ma trận khối cỡ 2 × 1, 1 × 2, 2 × 2, với 𝐴𝐴, 𝐵𝐵, 𝐶𝐶, 𝐷𝐷, 𝐸𝐸, 𝐹𝐹 , 𝐺𝐺, 𝐻𝐻 ∈
𝑀𝑀𝑛𝑛 (𝕂𝕂):

�𝐴𝐴� (𝐶𝐶 𝐴𝐴𝐴𝐴 𝐴𝐴𝐴𝐴� ; ( 𝐶𝐶


𝐷𝐷) = � 𝐴𝐴 𝐵𝐵) � � = (𝐴𝐴𝐴𝐴 + 𝐶𝐶𝐶𝐶)
𝐵𝐵 𝐵𝐵𝐵𝐵 𝐵𝐵𝐵𝐵 𝐷𝐷
𝐸𝐸 𝐹𝐹 𝐴𝐴𝐴𝐴 + 𝐵𝐵𝐵𝐵 𝐴𝐴𝐴𝐴 + 𝐵𝐵𝐵𝐵
�𝐴𝐴 𝐵𝐵 � � �=� �
𝐶𝐶 𝐷𝐷 𝐺𝐺 𝐻𝐻 𝐶𝐶𝐶𝐶 + 𝐷𝐷𝐷𝐷 𝐶𝐶𝐶𝐶 + 𝐷𝐷𝐷𝐷
rank 𝐴𝐴 + rank 𝐵𝐵 ≤ rank �𝐴𝐴 0 �
∗ 𝐵𝐵
0 𝐵𝐵
rank 𝐴𝐴 + rank 𝐵𝐵 = rank �𝐴𝐴 0 � = rank � �
0 𝐵𝐵 𝐴𝐴 ∗
Tính định thức của một ma trận khối không như tính định thức của ma trận thông thường. Khi
ma trận khối có dạng tam giác (trên, dưới, đường chéo) thì định thức của nó bằng tích các định
thức của ma trận nằm trên đường chéo chính (trong phần Định thức sẽ nói rõ điều này).

3.2. Bài tập

Bài 3.1. (Câu 1 ngày thứ hai Olympic Quốc tế 2004)


𝐼𝐼 −𝐼𝐼
Cho 𝐴𝐴 là ma trận cỡ 4×2 và 𝐵𝐵 là ma trận cỡ 2 × 4 thoả 𝐴𝐴𝐴𝐴 = � �. Tìm 𝐵𝐵𝐵𝐵.
−𝐼𝐼 𝐼𝐼
Giải.

Cách 1:

Giả sử 𝐴𝐴 = �𝐶𝐶 � , 𝐵𝐵 = (𝐸𝐸 𝐹𝐹 ) với 𝐶𝐶, 𝐷𝐷, 𝐸𝐸, 𝐹𝐹 là ma trận vuông cấp 2 thì ta có
𝐷𝐷
𝐵𝐵𝐵𝐵 = (𝐸𝐸𝐸𝐸 + 𝐹𝐹𝐹𝐹) , 𝐴𝐴𝐴𝐴 = � 𝐶𝐶𝐶𝐶 𝐶𝐶𝐶𝐶 � = � 𝐼𝐼 −𝐼𝐼

𝐷𝐷𝐷𝐷 𝐷𝐷𝐷𝐷 −𝐼𝐼 𝐼𝐼
⇒ 𝐶𝐶𝐶𝐶 = 𝐼𝐼 = 𝐸𝐸𝐸𝐸, 𝐷𝐷𝐷𝐷 = 𝐼𝐼 = 𝐹𝐹𝐹𝐹
Do đó 𝐵𝐵𝐵𝐵 = 𝐸𝐸𝐸𝐸 + 𝐹𝐹𝐹𝐹 = 2𝐼𝐼2

Cách 2:

Ta có rank(𝐴𝐴𝐴𝐴) = 2, (𝐴𝐴𝐴𝐴)2 = 2𝐴𝐴𝐴𝐴


⇒ 2 = rank(𝐴𝐴𝐴𝐴) = rank(2𝐴𝐴𝐴𝐴) = rank[(𝐴𝐴𝐴𝐴)2 ] = rank[𝐴𝐴(𝐵𝐵𝐵𝐵)𝐵𝐵] ≤ rank 𝐵𝐵𝐵𝐵 ≤ 2
do 𝐵𝐵𝐵𝐵 là ma trận vuông cấp 2 ⇒ rank(𝐵𝐵𝐵𝐵) = 2 hay 𝐵𝐵𝐵𝐵 khả nghịch.

Đỗ Minh Triết III-32 - MATHTASY -


Ôn thi Olympic Toán Đại số Phần III: Ma trận khối

Mặt khác (𝐵𝐵𝐵𝐵)3 = 𝐵𝐵𝐵𝐵𝐵𝐵𝐵𝐵𝐵𝐵𝐵𝐵 = 𝐵𝐵(𝐴𝐴𝐴𝐴)2 𝐴𝐴 = 2𝐵𝐵𝐵𝐵𝐵𝐵𝐵𝐵 = 2(𝐵𝐵𝐵𝐵)2 . Vì 𝐵𝐵𝐵𝐵 khả nghịch nên 𝐵𝐵𝐵𝐵 =
2𝐼𝐼2 .

Bài 3.2. (Câu 6b Putnam 1986 - Câu 5 đề chọn đội tuyển CĐ SP BR-VT 2012)
Cho 𝐴𝐴, 𝐵𝐵, 𝐶𝐶, 𝐷𝐷 ∈ 𝑀𝑀𝑛𝑛 (ℝ) thoả 𝐴𝐴𝐵𝐵𝑇𝑇 = 𝐶𝐶𝐷𝐷𝑇𝑇 là các ma trận đối xứng và 𝐴𝐴𝐷𝐷𝑇𝑇 − 𝐵𝐵𝐶𝐶 𝑇𝑇 = 𝐼𝐼. Chứng
minh 𝐴𝐴𝑇𝑇 𝐷𝐷 − 𝐶𝐶 𝑇𝑇 𝐵𝐵 = 𝐼𝐼.

Chứng minh.

𝐴𝐴𝐵𝐵𝑇𝑇 = (𝐴𝐴𝐵𝐵𝑇𝑇 )𝑇𝑇 = 𝐵𝐵𝐴𝐴𝑇𝑇


Do 𝐴𝐴𝐵𝐵𝑇𝑇 và 𝐶𝐶𝐷𝐷𝑇𝑇 là các ma trận đối xứng nên �
𝐶𝐶𝐷𝐷𝑇𝑇 = (𝐶𝐶𝐷𝐷𝑇𝑇 )𝑇𝑇 = 𝐷𝐷𝐶𝐶 𝑇𝑇
Mặt khác, 𝐴𝐴𝐷𝐷𝑇𝑇 − 𝐵𝐵𝐶𝐶 𝑇𝑇 = 𝐼𝐼 ⇒ 𝐼𝐼 = (𝐴𝐴𝐷𝐷𝑇𝑇 − 𝐵𝐵𝐶𝐶 𝑇𝑇 )𝑇𝑇 = (𝐴𝐴𝐷𝐷𝑇𝑇 )𝑇𝑇 − (𝐵𝐵𝐶𝐶 𝑇𝑇 )𝑇𝑇 = 𝐷𝐷𝐴𝐴𝑇𝑇 − 𝐶𝐶𝐵𝐵𝑇𝑇 .
Ta có
⎧ 𝐴𝐴𝐵𝐵𝑇𝑇 − 𝐵𝐵𝐴𝐴𝑇𝑇 =0
�𝐶𝐶𝐷𝐷𝑇𝑇 − 𝐷𝐷𝐶𝐶 𝑇𝑇 = 0 ⇒ �𝐴𝐴 𝐵𝐵 � � 𝐷𝐷𝑇𝑇 −𝐵𝐵𝑇𝑇 � = �𝐼𝐼 0
⎨𝐴𝐴𝐷𝐷𝑇𝑇 �
� − 𝐵𝐵𝐶𝐶 𝑇𝑇 = 𝐼𝐼 𝐶𝐶 𝐷𝐷 −𝐶𝐶 𝑇𝑇 𝐴𝐴𝑇𝑇 0 𝐼𝐼
⎩𝐷𝐷𝐴𝐴𝑇𝑇 − 𝐶𝐶𝐵𝐵𝑇𝑇 = 𝐼𝐼
Suy ra
𝐷𝐷𝑇𝑇 −𝐵𝐵𝑇𝑇 � �𝐴𝐴 𝐵𝐵 � = �𝐼𝐼 0
� ⇒ 𝐴𝐴𝑇𝑇 𝐷𝐷 − 𝐶𝐶 𝑇𝑇 𝐵𝐵 = 𝐼𝐼.

−𝐶𝐶 𝑇𝑇 𝐴𝐴𝑇𝑇 𝐶𝐶 𝐷𝐷 0 𝐼𝐼

Bài 3.3. (Câu 4 Olympic 1993)


Cho 𝐴𝐴, 𝐵𝐵 là các ma trận vuông cấp 𝑛𝑛 sao cho tổng, hiệu của chúng là các ma trận không suy biến.
Chứng minh rằng �𝐴𝐴 𝐵𝐵� cũng là ma trận không suy biến.
𝐵𝐵 𝐴𝐴
Chứng minh.

Cách 1:

Ta có �𝐴𝐴 𝐵𝐵� là một định thức cấp 2𝑛𝑛.


𝐵𝐵 𝐴𝐴
Nhân -1 vào cột 𝑛𝑛 + 𝑖𝑖 rồi cộng vào cột 𝑖𝑖 (𝑖𝑖 = 1, … , 𝑛𝑛)

�𝐴𝐴 𝐵𝐵� = �𝐴𝐴 − 𝐵𝐵 𝐵𝐵�


𝐵𝐵 𝐴𝐴 𝐵𝐵 − 𝐴𝐴 𝐴𝐴
Nhân 1 vào hàng 𝑛𝑛 + 𝑖𝑖 rồi cộng vào hàng 𝑖𝑖 (𝑖𝑖 = 1, … , 𝑛𝑛)

�𝐴𝐴 − 𝐵𝐵 𝐵𝐵� = � 0 𝐴𝐴 + 𝐵𝐵�


𝐵𝐵 − 𝐴𝐴 𝐴𝐴 𝐵𝐵 − 𝐴𝐴 𝐴𝐴
Khai triển Laplace theo 𝑛𝑛 hàng đầu

� 0 𝐴𝐴 + 𝐵𝐵� = (−1)𝑛𝑛2 |𝐵𝐵 − 𝐴𝐴||𝐴𝐴 + 𝐵𝐵| ≠ 0


𝐵𝐵 − 𝐴𝐴 𝐴𝐴

Đỗ Minh Triết III-33 - MATHTASY -


Ôn thi Olympic Toán Đại số Phần III: Ma trận khối

Cách 2:
𝐼𝐼 0 𝐴𝐴 𝐵𝐵 𝐼𝐼 0 𝐴𝐴 − 𝐵𝐵 0

�� �� �=� �
𝐼𝐼 𝐼𝐼 𝐵𝐵 𝐴𝐴 −𝐼𝐼 𝐼𝐼 0 𝐴𝐴 + 𝐵𝐵
Lấy định thức hai vế, suy ra điều phải chứng minh.

Cách 3:

Xét hệ

�𝐴𝐴 𝐵𝐵� �𝑥𝑥� = 0 ⇔ �𝐴𝐴𝐴𝐴 + 𝐵𝐵𝐵𝐵 = 0 ⇔ (𝐴𝐴 + 𝐵𝐵)(𝑥𝑥 + 𝑦𝑦) = 0



𝐵𝐵 𝐴𝐴 𝑦𝑦 𝐵𝐵𝐵𝐵 + 𝐴𝐴𝐴𝐴 = 0 (𝐴𝐴 − 𝐵𝐵)(𝑥𝑥 − 𝑦𝑦) = 0
Do |𝐴𝐴 + 𝐵𝐵| ≠ 0 nên 𝑥𝑥 + 𝑦𝑦 = 0 và |𝐴𝐴 − 𝐵𝐵| ≠ 0 nên 𝑥𝑥 − 𝑦𝑦 = 0

Suy ra 𝑥𝑥 = 𝑦𝑦 = 0. Nói cách khác, hệ chỉ có nghiệm tầm thường nên �𝐴𝐴 𝐵𝐵� ≠ 0.
𝐵𝐵 𝐴𝐴

Bài 3.4. (artofproblemsolving.com)

Cho 𝐴𝐴, 𝐵𝐵, 𝐶𝐶, 𝐷𝐷 ∈ 𝑀𝑀𝑛𝑛 (ℂ) giao hoán với nhau từng đôi một. Đặt 𝑋𝑋 = �𝐴𝐴 𝐵𝐵 �. Chứng minh 𝑋𝑋 khả
𝐶𝐶 𝐷𝐷
nghịch khi và chỉ khi (𝐴𝐴𝐴𝐴 − 𝐵𝐵𝐵𝐵) khả nghịch.

Chứng minh.

Ta chứng minh det 𝑋𝑋 = det �𝐴𝐴 𝐵𝐵 �:


𝐶𝐶 𝐷𝐷
Giả sử 𝑥𝑥 không là giá trị riêng của 𝐷𝐷, khi đó (𝐷𝐷 − 𝑥𝑥𝑥𝑥) khả nghịch. Ta có
𝐷𝐷 − 𝑥𝑥𝑥𝑥 −𝐵𝐵 𝐴𝐴 𝐵𝐵 (𝐷𝐷 − 𝑥𝑥𝑥𝑥)𝐴𝐴 − 𝐵𝐵𝐵𝐵 0
� �� �=� �
0 𝐼𝐼 𝐶𝐶 𝐷𝐷 − 𝑥𝑥𝑥𝑥 𝐶𝐶 𝐷𝐷 − 𝑥𝑥𝑥𝑥
Lấy định thức hai vế, ta được
𝐴𝐴 𝐵𝐵
det(𝐷𝐷 − 𝑥𝑥𝑥𝑥) det � � = det[(𝐷𝐷 − 𝑥𝑥𝑥𝑥)𝐴𝐴 − 𝐵𝐵𝐵𝐵] det(𝐷𝐷 − 𝑥𝑥𝑥𝑥)
𝐶𝐶 𝐷𝐷 − 𝑥𝑥𝑥𝑥
⇔ det �𝐴𝐴 𝐵𝐵 � = det[(𝐷𝐷 − 𝑥𝑥𝑥𝑥)𝐴𝐴 − 𝐵𝐵𝐵𝐵]
𝐶𝐶 𝐷𝐷 − 𝑥𝑥𝑥𝑥
Xét đa thức 𝑃𝑃 (𝑥𝑥) = det � 𝐴𝐴 𝐵𝐵 � − det[(𝐷𝐷 − 𝑥𝑥𝑥𝑥)𝐴𝐴 − 𝐵𝐵𝐵𝐵]
𝐶𝐶 𝐷𝐷 − 𝑥𝑥𝑥𝑥
Dễ thấy deg 𝑃𝑃 (𝑥𝑥) ≤ 𝑛𝑛 và 𝑃𝑃 (𝑥𝑥) = 0 với mọi 𝑥𝑥 không là là giá trị riêng của 𝐷𝐷. Suy ra 𝑃𝑃 (𝑥𝑥) ≡ 0
hay 𝑃𝑃 (𝑥𝑥) = 0 ∀𝑥𝑥 ∈ ℂ.
Từ đó 𝑃𝑃 (0) = 0, ta có điều phải chứng minh.

Bài 3.5. (artofproblemsolving.com)


𝐴𝐴 𝐵𝐵 𝐶𝐶
Cho 𝐴𝐴, 𝐵𝐵, 𝐶𝐶 ∈ 𝑀𝑀𝑛𝑛 (ℂ) giao hoán với nhau từng đôi một. Đặt 𝑀𝑀 = �𝐶𝐶 𝐴𝐴 𝐵𝐵 �. Chứng minh 𝑀𝑀
𝐵𝐵 𝐶𝐶 𝐴𝐴
khả nghịch khi và chỉ khi (𝐴𝐴 + 𝐵𝐵 + 𝐶𝐶 − 3𝐴𝐴𝐴𝐴𝐴𝐴) khả nghịch.
3 3 3

Đỗ Minh Triết III-34 - MATHTASY -


Ôn thi Olympic Toán Đại số Phần III: Ma trận khối

Chứng minh.
𝐴𝐴 𝐵𝐵 𝐶𝐶
Ta chứng minh det 𝑀𝑀 = det �𝐶𝐶 𝐴𝐴 𝐵𝐵 �
𝐵𝐵 𝐶𝐶 𝐴𝐴
Giả sử 𝑥𝑥 không là giá trị riêng của 𝐴𝐴, khi đó 𝐴𝐴1 = (𝐴𝐴 − 𝑥𝑥𝑥𝑥) khả nghịch. Ta có
𝐴𝐴1 𝐵𝐵 𝐶𝐶 𝐼𝐼 0 0 𝐴𝐴1 𝐵𝐵 𝐶𝐶

det ⎜ 𝐶𝐶 𝐴𝐴1 ⎞ ⎛
𝐵𝐵 ⎟ = det ⎜−𝐴𝐴−1
1 𝐶𝐶 𝐼𝐼 ⎞ ⎛
0⎟ det ⎜ 𝐶𝐶 𝐴𝐴1 𝐵𝐵 ⎞

⎝ 𝐵𝐵 𝐶𝐶 𝐴𝐴1 ⎠ ⎝−𝐴𝐴−1
1 𝐵𝐵 0 𝐼𝐼⎠ ⎝ 𝐵𝐵 𝐶𝐶 𝐴𝐴1 ⎠
𝐴𝐴1 𝐵𝐵 𝐶𝐶

= det ⎜ 0 𝐴𝐴1 − 𝐵𝐵𝐵𝐵𝐴𝐴−1 𝐵𝐵 − 𝐶𝐶 2 𝐴𝐴−1 ⎞
1 1 ⎟
⎝0 𝐶𝐶 − 𝐵𝐵2 𝐴𝐴−1
1 𝐴𝐴1 − 𝐵𝐵𝐵𝐵𝐴𝐴−11 ⎠

= det 𝐴𝐴1 det[(𝐴𝐴1 − 𝐵𝐵𝐵𝐵𝐴𝐴−1 −1 2 −1 2 −1


1 )(𝐴𝐴1 − 𝐵𝐵𝐵𝐵𝐴𝐴1 ) − (𝐶𝐶 − 𝐵𝐵 𝐴𝐴1 )(𝐵𝐵 − 𝐶𝐶 𝐴𝐴1 )]

= det[𝐴𝐴1 (𝐴𝐴21 + 𝐵𝐵3 𝐴𝐴−1 3 −1 3 3 3


1 + 𝐶𝐶 𝐴𝐴1 − 3𝐵𝐵𝐵𝐵)] = det(𝐴𝐴1 + 𝐵𝐵 + 𝐶𝐶 − 3𝐴𝐴1 𝐵𝐵)
𝐴𝐴1 𝐵𝐵 𝐶𝐶
Xét đa thức 𝑃𝑃 (𝑥𝑥) = det ⎛
⎜ 𝐶𝐶 𝐴𝐴1 𝐵𝐵 ⎞
⎟ − det(𝐴𝐴31 + 𝐵𝐵3 + 𝐶𝐶 3 − 3𝐴𝐴1 𝐵𝐵)
⎝ 𝐵𝐵 𝐶𝐶 𝐴𝐴1 ⎠
Dễ thấy deg 𝑃𝑃 (𝑥𝑥) ≤ 3𝑛𝑛 và 𝑃𝑃 (𝑥𝑥) = 0 với mọi 𝑥𝑥 không là là giá trị riêng của 𝐴𝐴. Suy ra 𝑃𝑃 (𝑥𝑥) ≡ 0
hay 𝑃𝑃 (𝑥𝑥) = 0 ∀𝑥𝑥 ∈ ℂ.
Từ đó 𝑃𝑃 (0) = 0, ta có điều phải chứng minh.

Bài 3.6. Cho các ma trận vuông không suy biến 𝐴𝐴 = �𝑎𝑎𝑖𝑖𝑖𝑖 �𝑛𝑛 , 𝐵𝐵 = �𝑏𝑏𝑖𝑖𝑖𝑖 �𝑚𝑚 . Tính
𝑏𝑏 𝐴𝐴 𝑏𝑏12 𝐴𝐴 … 𝑏𝑏1𝑚𝑚 𝐴𝐴
� 11 �
𝑏𝑏21 𝐴𝐴 𝑏𝑏22 𝐴𝐴 … 𝑏𝑏2𝑚𝑚 𝐴𝐴
|𝑀𝑀| = � �
� ⋮ ⋮ ⋱ ⋮ �
�𝑏𝑏𝑚𝑚1 𝐴𝐴 𝑏𝑏𝑚𝑚2 𝐴𝐴 … 𝑏𝑏𝑚𝑚𝑚𝑚 𝐴𝐴�
theo |𝐴𝐴| và |𝐵𝐵|.

Giải.

Dùng các phép biến đổi sơ cấp theo dòng , ta có


|𝐵𝐵| = |diag(𝑐𝑐1 , 𝑐𝑐2 , … , 𝑐𝑐𝑚𝑚 )| = 𝑐𝑐1 … 𝑐𝑐𝑚𝑚
điều này thực hiện được do 𝐵𝐵 khả nghịch.
Nếu ta áp dụng một phép biến đổi sơ cấp nào đó với dòng 𝑖𝑖, 𝑘𝑘 của |𝐵𝐵| thì ta cũng áp dụng phép
biến đổi đó cho tất cả các dòng của khối (𝑏𝑏𝑖𝑖1 𝐴𝐴 𝑏𝑏𝑖𝑖2 𝐴𝐴 … 𝑏𝑏𝑖𝑖𝑖𝑖 𝐴𝐴) và (𝑏𝑏𝑘𝑘1 𝐴𝐴 𝑏𝑏𝑘𝑘2 𝐴𝐴 … 𝑏𝑏𝑘𝑘𝑘𝑘 𝐴𝐴).
Khi đó ta được
𝑐𝑐 𝐴𝐴 0 … 0
� 1 �
0 𝑐𝑐2 𝐴𝐴 … 0
|𝑀𝑀| = � � = (𝑐𝑐1 … 𝑐𝑐𝑚𝑚 )𝑛𝑛 |𝐴𝐴|𝑚𝑚 = |𝐴𝐴|𝑚𝑚 |𝐵𝐵|𝑛𝑛
� ⋮ ⋮ ⋱ ⋮ �
� 0 0 0 𝑐𝑐 𝑚𝑚 𝐴𝐴 �

Đỗ Minh Triết III-35 - MATHTASY -


Ôn thi Olympic Toán Đại số Phần III: Ma trận khối

Bài 3.7. (Câu 1 Đề Virginia Tech Regional Mathematics Contest 2004)

Cho 𝐴𝐴, 𝐵𝐵, 𝐶𝐶 là các ma trận vuông thực cấp 2 và đặt 𝑀𝑀 = � 𝐼𝐼 𝐴𝐴� , 𝑁𝑁 = � 𝐼𝐼 𝐵𝐵 �. Nếu 𝑀𝑀 khả
𝐵𝐵 𝐶𝐶 𝐴𝐴 𝐶𝐶
nghịch thì 𝑁𝑁 có khả nghịch không?

Giải.
0 1 0 0 1 0
Chọn 𝐴𝐴 = � � , 𝐵𝐵 = � � , 𝐶𝐶 = � � thì det 𝑀𝑀 = det � 𝐼𝐼 𝐴𝐴 � ≠ 0 nhưng det 𝑁𝑁 =
0 0 1 0 0 0 𝐵𝐵 𝐶𝐶
𝐼𝐼 𝐵𝐵
det � � = 0.
𝐴𝐴 𝐶𝐶

Bài 3.8. (Định lý phân tích hạng) Cho ma trận 𝐴𝐴𝑚𝑚×𝑛𝑛 hạng 𝑟𝑟 ≠ 0,. Chứng minh tồn tại hai
ma trận 𝐵𝐵𝑚𝑚×𝑟𝑟 , 𝐶𝐶𝑟𝑟×𝑛𝑛 sao cho rank 𝐵𝐵 = rank 𝐶𝐶 = 𝑟𝑟 và 𝐴𝐴 = 𝐵𝐵𝐵𝐵.

Chứng minh.

Do rank 𝐴𝐴 = 𝑟𝑟 nên không gian sinh bởi họ các vector cột của 𝐴𝐴 có hạng là 𝑟𝑟.
Gọi 𝐵𝐵𝑚𝑚×𝑟𝑟 với 𝑟𝑟 cột là 𝑟𝑟 cột cơ sở của không gian này lấy từ các cột của 𝐴𝐴. Khi đó mỗi cột của 𝐴𝐴
là một tổ hợp tuyến tính các cột của 𝐵𝐵 nên tồn tại ma trận 𝐶𝐶𝑟𝑟×𝑛𝑛 thoả 𝐴𝐴 = 𝐵𝐵𝐵𝐵 (giả sử các cột
của 𝐵𝐵 là 𝑏𝑏1 , 𝑏𝑏2 , … , 𝑏𝑏𝑟𝑟 và cột thứ 𝑗𝑗 của ma trận 𝐴𝐴 có sự biểu diễn 𝑐𝑐𝑗𝑗 (𝐴𝐴) = 𝑐𝑐1𝑗𝑗 𝑏𝑏1 + 𝑐𝑐2𝑗𝑗 𝑏𝑏2 +. . . +𝑐𝑐𝑟𝑟𝑟𝑟 𝑏𝑏𝑟𝑟
thì 𝐶𝐶 = �𝑐𝑐𝑖𝑖𝑖𝑖 � là ma trận cần tìm).
Bây giờ, do cách xây dựng như trên nên rank 𝐵𝐵 = 𝑟𝑟. Hơn nữa, 𝑟𝑟 = rank 𝐴𝐴 = rank(𝐵𝐵𝐵𝐵) ≤ rank 𝐶𝐶
và 𝐶𝐶 có 𝑟𝑟 hàng nên rank 𝐶𝐶 = 𝑟𝑟.

Bài 3.9.
Cho ma trận 𝐴𝐴𝑚𝑚×𝑛𝑛 hạng 𝑛𝑛. Chứng minh rằng tồn tại ma trận 𝑋𝑋 sao cho 𝑋𝑋𝑋𝑋 = 𝐼𝐼𝑛𝑛 .

Chứng minh.

Nếu 𝑚𝑚 = 𝑛𝑛 thì 𝐴𝐴 là ma trận vuông khả nghịch nên 𝑋𝑋 = 𝐴𝐴−1 .


Dễ thấy, 𝐴𝐴𝑚𝑚×𝑛𝑛 hạng 𝑛𝑛 không thể xảy ra khi 𝑚𝑚 < 𝑛𝑛.
Xét 𝑚𝑚 > 𝑛𝑛, khi đó 𝑛𝑛 cột của 𝐴𝐴 lập thành một họ vector độc lập tuyến tính trong 𝕂𝕂𝑚𝑚 , do đó ta
có thể bổ sung thêm 𝑚𝑚 − 𝑛𝑛 cột để được một cơ sở trong 𝕂𝕂𝑚𝑚 . Khi đó ta được một ma trận vuông
cấp 𝑚𝑚 khả nghịch có dạng [ 𝐴𝐴 𝐵𝐵 ] với 𝐵𝐵𝑚𝑚×(𝑚𝑚−𝑛𝑛) là ma trận gồm 𝑚𝑚 − 𝑛𝑛 vector cột bổ sung đó.
Biểu diễn nghịch đảo của ma trận này dưới dạng ma trận khối
𝑋𝑋
[ 𝐴𝐴 𝐵𝐵 ]−1 = � � trong đó 𝑋𝑋𝑛𝑛×𝑚𝑚 , 𝐶𝐶(𝑚𝑚−𝑛𝑛)×𝑚𝑚
𝐶𝐶

Đỗ Minh Triết III-36 - MATHTASY -


Ôn thi Olympic Toán Đại số Phần III: Ma trận khối

Khi đó
𝑋𝑋
� � [ 𝐴𝐴 𝐵𝐵 ] = 𝐼𝐼𝑚𝑚
𝐶𝐶
Cụ thể như sau
𝑋𝑋𝑛𝑛×𝑚𝑚 (𝑋𝑋𝑋𝑋)𝑛𝑛 (𝑋𝑋𝑋𝑋)𝑛𝑛×(𝑚𝑚−𝑛𝑛) 𝐼𝐼𝑛𝑛 0𝑛𝑛×(𝑚𝑚−𝑛𝑛)
� � [ 𝐴𝐴𝑚𝑚×𝑛𝑛 𝐵𝐵𝑚𝑚×(𝑚𝑚−𝑛𝑛) ] = � �=� � = 𝐼𝐼𝑚𝑚
𝐶𝐶(𝑚𝑚−𝑛𝑛)×𝑚𝑚 (𝐶𝐶𝐴𝐴)(𝑚𝑚−𝑛𝑛)×𝑛𝑛 (𝐶𝐶𝐶𝐶)(𝑚𝑚−𝑛𝑛) 0(𝑚𝑚−𝑛𝑛)×𝑛𝑛 𝐼𝐼(𝑚𝑚−𝑛𝑛)
⇒ 𝑋𝑋𝑋𝑋 = 𝐼𝐼𝑛𝑛

Bài 3.10. Cho ma trận 𝐴𝐴𝑚𝑚×𝑛𝑛 hạng 𝑛𝑛. Chứng minh rằng tồn tại ma trận 𝑌𝑌 khả nghịch sao
cho
𝐼𝐼
𝑌𝑌𝑌𝑌 = � 𝑛𝑛 �
0
Chứng minh.
𝑋𝑋
Làm như chứng minh trên thì 𝑌𝑌 = [ 𝐴𝐴 𝐵𝐵 ]−1 = � � là ma trận cần tìm. Thật vậy
𝐶𝐶
𝑋𝑋𝑛𝑛×𝑚𝑚 (𝑋𝑋𝑋𝑋)𝑛𝑛×𝑛𝑛 𝐼𝐼𝑛𝑛
𝑌𝑌𝑌𝑌 = � � [𝐴𝐴𝑚𝑚×𝑛𝑛 ] = � �=� �
𝐶𝐶(𝑚𝑚−𝑛𝑛)×𝑚𝑚 (𝐶𝐶𝐶𝐶)(𝑚𝑚−𝑛𝑛)×𝑛𝑛 0(𝑚𝑚−𝑛𝑛)×𝑛𝑛

Bài 3.11. Cho ma trận 𝐴𝐴𝑚𝑚×𝑛𝑛 hạng 𝑚𝑚. Chứng minh rằng tồn tại ma trận 𝑋𝑋 sao cho 𝐴𝐴𝐴𝐴 =
𝐼𝐼𝑚𝑚 .

Chứng minh.

Nếu 𝑚𝑚 = 𝑛𝑛 thì 𝐴𝐴 là ma trận vuông khả nghịch nên 𝑋𝑋 = 𝐴𝐴−1 .


Dễ thấy, 𝐴𝐴𝑚𝑚×𝑛𝑛 hạng 𝑛𝑛 không thể xảy ra khi 𝑚𝑚 > 𝑛𝑛.
Xét 𝑚𝑚 < 𝑛𝑛, khi đó 𝑚𝑚 hàng của 𝐴𝐴 lập thành một họ vector độc lập tuyến tính trong 𝕂𝕂𝑚𝑚 , do đó ta
có thể bổ sung thêm 𝑛𝑛 − 𝑚𝑚 hàng để được một cơ sở trong 𝕂𝕂𝑚𝑚 . Khi đó ta được một ma trận vuông
cấp 𝑛𝑛 khả nghịch có dạng � 𝐴𝐴 � với 𝐵𝐵(𝑛𝑛−𝑚𝑚)×𝑛𝑛 là ma trận gồm 𝑛𝑛 − 𝑚𝑚 hàng bổ sung đó. Biểu diễn
𝐵𝐵
nghịch đảo của ma trận này dưới dạng ma trận khối
−1
� 𝐴𝐴 � = [ 𝑋𝑋 𝐶𝐶 ] trong đó 𝑋𝑋𝑛𝑛×𝑚𝑚 , 𝐶𝐶𝑛𝑛×(𝑛𝑛−𝑚𝑚)
𝐵𝐵
Khi đó

� 𝐴𝐴 � [ 𝑋𝑋 𝐶𝐶 ] = 𝐼𝐼𝑛𝑛
𝐵𝐵
Cụ thể như sau
𝐴𝐴𝑚𝑚×𝑛𝑛 (𝐴𝐴𝐴𝐴)𝑚𝑚 (𝐴𝐴𝐴𝐴)𝑚𝑚×(𝑛𝑛−𝑚𝑚) 𝐼𝐼𝑚𝑚 0𝑚𝑚×(𝑛𝑛−𝑚𝑚)
� � [𝑋𝑋𝑛𝑛×𝑚𝑚 𝐶𝐶𝑛𝑛×(𝑛𝑛−𝑚𝑚) ] = � �=� � = 𝐼𝐼𝑚𝑚
𝐵𝐵(𝑛𝑛−𝑚𝑚)×𝑛𝑛 (𝐵𝐵𝐵𝐵)(𝑛𝑛−𝑚𝑚)×𝑚𝑚 (𝐵𝐵𝐵𝐵)(𝑛𝑛−𝑚𝑚) 0(𝑛𝑛−𝑚𝑚)×𝑚𝑚 𝐼𝐼𝑛𝑛−𝑚𝑚

Đỗ Minh Triết III-37 - MATHTASY -


Ôn thi Olympic Toán Đại số Phần III: Ma trận khối

⇒ 𝐴𝐴𝐴𝐴 = 𝐼𝐼𝑚𝑚 .

Bài 3.12. Cho ma trận 𝐴𝐴𝑚𝑚×𝑛𝑛 hạng 𝑚𝑚. Chứng minh rằng tồn tại ma trận 𝑌𝑌 khả nghịch sao
cho
𝐴𝐴𝐴𝐴 = [𝐼𝐼𝑚𝑚 0]

Chứng minh.
−1
Làm như chứng minh trên thì 𝑌𝑌 = � 𝐴𝐴 � = [ 𝑋𝑋 𝐶𝐶 ] là ma trận cần tìm. Thật vậy
𝐵𝐵
𝐴𝐴𝐴𝐴 = [𝐴𝐴𝑚𝑚×𝑛𝑛 ][𝑋𝑋𝑛𝑛×𝑚𝑚 𝐶𝐶𝑛𝑛×(𝑛𝑛−𝑚𝑚) ] = [ (𝐴𝐴𝐴𝐴)𝑚𝑚 (𝐴𝐴𝐴𝐴)𝑚𝑚×(𝑛𝑛−𝑚𝑚) ] = [ 𝐼𝐼𝑚𝑚 0𝑚𝑚×(𝑛𝑛−𝑚𝑚) ]

Bài 3.13. Cho ma trận 𝐴𝐴𝑚𝑚×𝑛𝑛 có hạng 𝑟𝑟. Chứng minh rằng tồn tại các ma trận khả nghịch
𝐼𝐼 0
𝑃𝑃 , 𝑄𝑄 sao cho 𝑃𝑃𝑃𝑃𝑃𝑃 = � 𝑟𝑟 �.
0 0
Chứng minh.

Theo chứng minh của các câu trước, ta có:


- Thứ nhất, do ma trận 𝐴𝐴𝑚𝑚×𝑛𝑛 hạng là 𝑟𝑟 nên có sự phân tích 𝐴𝐴 = 𝐵𝐵𝐵𝐵 với 𝐵𝐵𝑚𝑚×𝑟𝑟 , 𝐶𝐶𝑟𝑟×𝑛𝑛 cùng
hạng 𝑟𝑟.
𝐼𝐼𝑟𝑟
- Thứ hai, tồn tại các ma trận khả nghịch 𝑃𝑃 , 𝑄𝑄 sao cho 𝑃𝑃𝑃𝑃 = � � và 𝐶𝐶𝐶𝐶 = [𝐼𝐼𝑟𝑟 0].
0
𝐼𝐼𝑟𝑟 𝐼𝐼 0
Vậy 𝑃𝑃𝑃𝑃𝑃𝑃 = 𝑃𝑃 (𝐵𝐵𝐵𝐵)𝑄𝑄 = (𝑃𝑃𝑃𝑃)(𝐶𝐶𝐶𝐶) = � � [𝐼𝐼𝑟𝑟 0] = � 𝑟𝑟 �.
0 0 0

Bài 3.14. Giả sử 𝐴𝐴 là ma trận cấp 𝑛𝑛 hạng 𝑟𝑟. Tìm số nghiệm độc lập tuyến tính của hệ
phương trình 𝐴𝐴𝐴𝐴 = 0 với 𝑋𝑋 là ma trận cấp 𝑛𝑛.

Giải.

Do 𝐴𝐴 là ma trận cấp 𝑛𝑛 hạng 𝑟𝑟 nên tồn tại các ma trận 𝑃𝑃 , 𝑄𝑄 khả nghịch sao cho 𝐴𝐴 = 𝑃𝑃 𝐼𝐼𝑟𝑟,𝑛𝑛 𝑄𝑄 với
𝐼𝐼𝑟𝑟 0
𝐼𝐼𝑟𝑟,𝑛𝑛 = � �
0 0
Khi đó hệ 𝐴𝐴𝐴𝐴 = 0 tương đương 𝐼𝐼𝑟𝑟,𝑛𝑛 𝑄𝑄𝑄𝑄 = 0, khi đó 𝑄𝑄𝑄𝑄 phải có dạng sau
𝑟𝑟 𝑛𝑛 − 𝑟𝑟
𝑄𝑄𝑄𝑄 = 𝑟𝑟 0 0
� �
𝑛𝑛 − 𝑟𝑟 𝑌𝑌1 𝑌𝑌2
Suy ra số nghiệm độc lập tuyến tính của hệ 𝐼𝐼𝑟𝑟,𝑛𝑛 𝑄𝑄𝑄𝑄 = 0 cũng như 𝐴𝐴𝐴𝐴 = 0 là 𝑛𝑛(𝑛𝑛 − 𝑟𝑟).

Đỗ Minh Triết III-38 - MATHTASY -


PHẦN IV. MA TRẬN LUỸ LINH (Nilpotent Matrix)

4.1. Định nghĩa và tính chất

Định nghĩa: Cho 𝐴𝐴 là ma trận vuông cấp 𝑛𝑛, 𝐴𝐴 được gọi là ma trận luỹ linh nếu tồn tại số nguyên
dương 𝑞𝑞 sao cho 𝐴𝐴𝑞𝑞 = 0.
Nếu 𝐴𝐴𝑞𝑞 = 0 thì ta cũng có 𝐴𝐴𝑚𝑚 = 0 với mọi số tự nhiên 𝑚𝑚 thoả 𝑚𝑚 ≥ 𝑞𝑞.
Số nguyên dương 𝑘𝑘 được gọi là cấp luỹ linh của ma trận 𝐴𝐴 nếu 𝐴𝐴𝑘𝑘 = 0, và 𝐴𝐴𝑘𝑘−1 ≠ 0.
Ma trận 𝐴𝐴 được gọi là ma trận luỹ linh đơn nếu ma trận 𝐴𝐴– 𝐼𝐼 luỹ linh.
Một số tính chất đặc trưng
Cho 𝐴𝐴 ∈ 𝑀𝑀𝑛𝑛 (𝕂𝕂), các mệnh đề sau là tương đương:
1. 𝐴𝐴 lũy linh.
2. Đa thức tối tiểu của 𝐴𝐴 là 𝜆𝜆𝑘𝑘 với số nguyên dương 𝑘𝑘 ≤ 𝑛𝑛.
3. Đa thức đặc trưng của 𝐴𝐴 là 𝜆𝜆𝑛𝑛 .
4. 𝐴𝐴 có giá trị riêng duy nhất là 0.
5. tr(𝐴𝐴𝑚𝑚 ) = 0 với mọi 𝑚𝑚 ≥ 0.
Hệ quả
- Bậc của một ma trận lũy linh luôn luôn nhỏ hơn hoặc bằng cấp của nó. Ví dụ mọi ma trận lũy
linh cấp 2 đều bình phương bằng 0.
- Ma trận luỹ linh thì có định thức và vết luôn bằng 0 (điều ngược lại không đúng).
- Một ma trận cấp hai lũy linh khi và chỉ khi cả định thức và vết của nó bằng 0 (hai chiều).
- Ma trận đường chéo lũy linh duy nhất là ma trận không.
Các tính chất khác
a) Nếu 𝑨𝑨 − 𝜶𝜶𝜶𝜶 là ma trận luỹ linh thì mọi giá trị riêng của 𝑨𝑨 đều bằng 𝜶𝜶.
Giả sử (𝐴𝐴 − 𝛼𝛼𝛼𝛼)𝑘𝑘 = 0, 𝑘𝑘 ≤ 𝑛𝑛, khi đó (𝐴𝐴 − 𝛼𝛼𝛼𝛼)𝑛𝑛 = 0, vậy (𝜆𝜆 − 𝛼𝛼)𝑛𝑛 là đa thức đặc trưng của 𝐴𝐴 và
chỉ có giá trị riêng là 𝛼𝛼 (bội 𝑛𝑛).
b) Nếu 𝑨𝑨 là ma trận luỹ linh thì các ma trận 𝑰𝑰– 𝑨𝑨 và 𝑰𝑰 + 𝑨𝑨 khả nghịch.
Giả sử 𝐴𝐴𝑘𝑘 = 0 (𝑘𝑘 ≥ 1), ta có 𝐼𝐼 = 𝐼𝐼– 𝐴𝐴𝑘𝑘 = (𝐼𝐼 − 𝐴𝐴)(𝐼𝐼 + 𝐴𝐴 + 𝐴𝐴2 + ⋯ + 𝐴𝐴𝑘𝑘−1 )
Như vậy 𝐼𝐼– 𝐴𝐴 khả nghịch và (𝐼𝐼 − 𝐴𝐴)−1 = 𝐼𝐼 + 𝐴𝐴 + 𝐴𝐴2 + ⋯ + 𝐴𝐴𝑘𝑘−1 .
Tương tự ta cũng có 𝐼𝐼 + 𝐴𝐴 khả nghịch vì 𝐼𝐼 = 𝐼𝐼 + 𝐴𝐴2𝑘𝑘+1 = (𝐼𝐼 + 𝐴𝐴)(𝐼𝐼– 𝐴𝐴 + 𝐴𝐴2 – … + 𝐴𝐴2𝑘𝑘 )
c) Cho 𝑨𝑨 và 𝑩𝑩 là hai ma trận vuông cùng cấp và 𝑨𝑨𝑨𝑨 = 𝑩𝑩𝑩𝑩. Khi đó nếu 𝑨𝑨 và 𝑩𝑩 là
các ma trận luỹ linh thì 𝑨𝑨 + 𝑩𝑩 cũng là ma trận luỹ linh.

Đỗ Minh Triết IV-39 - MATHTASY -


Ôn thi Olympic Toán Đại số Phần IV: Ma trận lũy linh

Do 𝐴𝐴 và 𝐵𝐵 là các ma trận luỹ linh nên tồn tại các số nguyên dương 𝑝𝑝, 𝑞𝑞 sao cho
𝐴𝐴𝑝𝑝 = 0, 𝐵𝐵𝑞𝑞 = 0, giả sử 𝑝𝑝 ≥ 𝑞𝑞.
Theo giả thiết 𝐴𝐴𝐴𝐴 = 𝐵𝐵𝐵𝐵 nên ta có khai triển nhị thức Newton và dễ thấy
2𝑝𝑝
𝑖𝑖
(𝐴𝐴 + 𝐵𝐵)2𝑝𝑝 = � 𝐶𝐶2𝑝𝑝 𝐴𝐴𝑖𝑖 𝐵𝐵2𝑝𝑝−𝑖𝑖 = 0
𝑖𝑖=0

Vậy 𝐴𝐴 + 𝐵𝐵 luỹ linh.


d) Cho 𝑨𝑨 và 𝑩𝑩 là hai ma trận vuông cùng cấp và 𝑨𝑨𝑨𝑨 = 𝑩𝑩𝑩𝑩. Khi đó nếu 𝑨𝑨 và 𝑩𝑩 là
các ma trận luỹ linh đơn thì ma trận tích 𝑨𝑨𝑨𝑨 cũng là ma trận luỹ linh đơn.
Vì 𝐴𝐴 − 𝐼𝐼, 𝐵𝐵 − 𝐼𝐼 là các ma trận luỹ linh, nên tồn tại các số nguyên dương 𝑝𝑝 và 𝑞𝑞 sao cho
(𝐴𝐴– 𝐼𝐼)𝑝𝑝 = 0, (𝐵𝐵– 𝐼𝐼)𝑞𝑞 = 0
Ta có 𝐴𝐴𝐴𝐴 − 𝐼𝐼 = (𝐴𝐴 − 𝐼𝐼)𝐵𝐵 + (𝐵𝐵 − 𝐼𝐼), giả sử 𝑝𝑝 ≥ 𝑞𝑞 khi đó do 𝐴𝐴𝐴𝐴 = 𝐵𝐵𝐵𝐵 nên ta cũng có tính chất
giao hoán (𝐴𝐴 − 𝐼𝐼)𝐵𝐵(𝐵𝐵 − 𝐼𝐼) = (𝐵𝐵 − 𝐼𝐼)(𝐴𝐴 − 𝐼𝐼)𝐵𝐵. Sử dụng khai triển nhị thức Newton, ta thu được:
2𝑝𝑝
𝑖𝑖
(𝐴𝐴𝐴𝐴 − 𝐼𝐼)2𝑝𝑝 = [(𝐴𝐴 − 𝐼𝐼)𝐵𝐵 + (𝐵𝐵 − 𝐼𝐼)]2𝑝𝑝 = � 𝐶𝐶2𝑝𝑝 (𝐴𝐴 − 𝐼𝐼)𝑖𝑖 𝐵𝐵𝑖𝑖 (𝐵𝐵 − 𝐼𝐼)2𝑝𝑝−𝑖𝑖
𝑖𝑖=0

Trong 2 số 𝑖𝑖 và 2𝑝𝑝 − 𝑖𝑖 phải có một số không nhỏ hơn 𝑝𝑝 nên (𝐴𝐴 − 𝐼𝐼)𝑖𝑖 𝐵𝐵𝑖𝑖 (𝐵𝐵 − 𝐼𝐼)2𝑝𝑝−𝑖𝑖 = 0. Vậy tồn
tại số nguyên dương 2𝑝𝑝 sao cho (𝐴𝐴𝐴𝐴 − 𝐼𝐼)2𝑝𝑝 = 0, tức 𝐴𝐴𝐴𝐴– 𝐼𝐼 là ma trận luỹ linh.

Chú ý: Tương tự như khái niệm ma trận luỹ linh người ta cũng xét khái niệm tự đồng cấu luỹ
linh như sau:
Tự đồng cấu 𝑓𝑓 của 𝕂𝕂–không gian vector 𝑉𝑉 trên trường 𝕂𝕂 gọi là luỹ linh nếu có số nguyên dương
𝑞𝑞 để 𝑓𝑓 𝑞𝑞 = 0.
Thêm vào đó nếu 𝑓𝑓 𝑞𝑞−1 ≠ 0 thì 𝑞𝑞 gọi là bậc luỹ linh của 𝑓𝑓.
Tự đồng cấu 𝑓𝑓 của 𝕂𝕂–không gian vector 𝑉𝑉 trên trường 𝕂𝕂 gọi luỹ linh đơn nếu 𝑓𝑓– 𝐼𝐼𝑑𝑑𝑉𝑉 là luỹ linh
(𝐼𝐼𝑑𝑑𝑉𝑉 là tự đẳng cấu đồng nhất trên 𝑉𝑉 ).
Chứng minh tương tự như ma trận luỹ linh, ta cũng có một số tính chất của đồng cấu luỹ linh
như sau:
1. Nếu 𝑓𝑓 và 𝑔𝑔 là hai tự đồng cấu luỹ linh giao hoán được của 𝕂𝕂–không gian vector 𝑉𝑉 trên trường
𝕂𝕂 thì 𝑓𝑓 + 𝑔𝑔 cũng luỹ linh.
2. Nếu 𝑓𝑓 và 𝑔𝑔 là hai tự đồng cấu luỹ linh đơn giao hoán được của 𝕂𝕂–không gian vector 𝑉𝑉 trên
trường 𝕂𝕂 thì 𝑓𝑓. 𝑔𝑔 cũng luỹ linh đơn.
3. Nếu 𝑓𝑓 là tự đồng cấu luỹ linh của ℝ–không gian vector 𝑉𝑉 − 𝑛𝑛 chiều trên trường ℝ các số thực
thì mọi giá trị riêng của 𝑓𝑓 đều bằng 0.

Đỗ Minh Triết IV-40 - MATHTASY -


Ôn thi Olympic Toán Đại số Phần IV: Ma trận lũy linh

4. Nếu 𝑓𝑓 là tự đồng cấu luỹ linh đơn của ℝ–không gian vector 𝑉𝑉 − 𝑛𝑛 chiều trên trường ℝ các số
thực thì mọi giá trị riêng của 𝑓𝑓 đều bằng 1.

4.2. Bài tập

Bài 4.1. Cho 𝐴𝐴 là ma trận vuông cấp 𝑛𝑛 và một số nguyên 𝑚𝑚 > 𝑛𝑛. Chứng minh rằng 𝐴𝐴𝑚𝑚 =
0 nếu và chỉ nếu 𝐴𝐴𝑛𝑛 = 0.

Chứng minh.

Cách 1:

Với 𝑚𝑚 > 𝑛𝑛 nếu 𝐴𝐴𝑛𝑛 = 0 thì hiển nhiên 𝐴𝐴𝑚𝑚 = 0.


Nếu 𝐴𝐴𝑚𝑚 = 0 thì theo định nghĩa, 𝐴𝐴 luỹ linh và theo tính chất, bậc luỹ linh của nó không vượt
quá 𝑛𝑛 là cấp của 𝐴𝐴, tức là tồn tại 1 ≤ 𝑘𝑘 ≤ 𝑛𝑛 sao cho 𝐴𝐴𝑘𝑘−1 ≠ 0, 𝐴𝐴𝑘𝑘 = 0, và từ đó suy ra 𝐴𝐴𝑛𝑛 = 0.
Cách 2:

Với 𝑚𝑚 > 𝑛𝑛 nếu 𝐴𝐴𝑛𝑛 = 0 thì hiển nhiên 𝐴𝐴𝑚𝑚 = 0.


Giả sử ta có 𝐴𝐴𝑚𝑚 = 0, ta chứng minh 𝐴𝐴𝑛𝑛 = 0 với 𝑚𝑚 > 𝑛𝑛.
Ký hiệu 𝑓𝑓(𝑥𝑥) là đa thức khác 0, chuẩn tắc, bậc nhỏ nhất nhận 𝐴𝐴 làm nghiệm, tức là 𝑓𝑓(𝐴𝐴) = 0, ta
luôn tìm được đa thức 𝑓𝑓(𝑥𝑥) như thế, hoặc nó chính là đa thức đặc trưng của 𝐴𝐴 hoặc nó là một đa
thức nào đó.
Vì đa thức đặc trưng của 𝐴𝐴 có bậc 𝑛𝑛 nên 1 ≤ deg 𝑓𝑓 ≤ 𝑛𝑛.
Thực hiện phép chia 𝑥𝑥𝑚𝑚 cho 𝑓𝑓(𝑥𝑥), ta có
𝑥𝑥𝑚𝑚 = 𝑓𝑓(𝑥𝑥)𝑞𝑞(𝑥𝑥) + 𝑟𝑟(𝑥𝑥) (deg 𝑟𝑟 < deg 𝑓𝑓)
Thay 𝐴𝐴 vào, ta được 𝑟𝑟(𝐴𝐴) = 0 nhưng vì cách chọn 𝑓𝑓(𝑥𝑥) nên 𝑟𝑟(𝑥𝑥) ≡ 0. Vì vậy 𝑥𝑥𝑚𝑚 chia hết cho
𝑓𝑓(𝑥𝑥), do đó 𝑓𝑓(𝑥𝑥) có dạng 𝑓𝑓(𝑥𝑥) = 𝑎𝑎𝑥𝑥𝑘𝑘 với 𝑎𝑎 là hằng số khác 0 và 1 ≤ 𝑘𝑘 ≤ 𝑛𝑛.
Khi đó từ 𝑓𝑓(𝐴𝐴) = 0, ta có 𝐴𝐴𝑘𝑘 = 0 (1 ≤ 𝑘𝑘 ≤ 𝑛𝑛) suy ra 𝐴𝐴𝑛𝑛 = 0.

Bài 4.2. (câu A5 Putnam 1990)


a) Cho A và B là các ma trận cấp 2 thỏa mãn 𝐴𝐴𝐴𝐴𝐴𝐴𝐴𝐴 = 0. Chứng minh 𝐵𝐵𝐵𝐵𝐵𝐵𝐵𝐵 = 0.
b) Lấy ví dụ về 2 ma trận 𝐴𝐴 và 𝐵𝐵 cấp 3 thỏa mãn 𝐴𝐴𝐴𝐴𝐴𝐴𝐴𝐴 = 0 nhưng 𝐵𝐵𝐵𝐵𝐵𝐵𝐵𝐵 ≠ 0.

Giải.

a) Ta có 𝐴𝐴𝐴𝐴𝐴𝐴𝐴𝐴 = 0 suy ra 𝐵𝐵(𝐴𝐴𝐴𝐴𝐴𝐴𝐴𝐴)𝐴𝐴 = 0 hay (𝐵𝐵𝐵𝐵)3 = 0. Vậy 𝐵𝐵𝐵𝐵 luỹ linh, do đó (𝐵𝐵𝐵𝐵)2 =
𝐵𝐵𝐵𝐵𝐵𝐵𝐵𝐵 = 0.

Đỗ Minh Triết IV-41 - MATHTASY -


Ôn thi Olympic Toán Đại số Phần IV: Ma trận lũy linh

0 0 1 0 0 1 0 0 1
b) Lấy 𝐴𝐴 = �0 0 0� , 𝐵𝐵 = �1 0 0�. Khi đó 𝐴𝐴𝐴𝐴𝐴𝐴𝐴𝐴 = 0 nhưng 𝐵𝐵𝐵𝐵𝐵𝐵𝐵𝐵 = �0 0 0� ≠
0 1 0 0 0 0 0 0 0
0.

Bài 4.3. (Tính chất ma trận luỹ linh – Wikipedia)


Nếu 𝐴𝐴 là ma trận cấp 𝑛𝑛 lũy linh thì, thì ma trận 𝐼𝐼 + 𝐴𝐴 khả nghịch, đồng thời det(𝐼𝐼 + 𝐴𝐴) = 1.

Chứng minh.

𝐼𝐼 + 𝐴𝐴 khả nghịch thì đơn giản, ta chứng minh det(𝐼𝐼 + 𝐴𝐴) = 1:


Vì 𝐴𝐴 luỹ linh nên tồn tại 𝑚𝑚 ∈ ℕ∗ nhỏ nhất để 𝐴𝐴𝑚𝑚 = 0, hiển nhiên 𝑚𝑚 ≤ 𝑛𝑛. Xét đa thức 𝑝𝑝(𝑥𝑥) =
(1 − 𝑥𝑥)𝑚𝑚 , ta có 𝑝𝑝(𝐼𝐼 + 𝐴𝐴) = (−1)𝑚𝑚 𝐴𝐴𝑚𝑚 = 0 điều đó có nghĩa là đa thức tối tiểu của 𝐼𝐼 + 𝐴𝐴 là ước
của 𝑝𝑝(𝑥𝑥). Do mọi giá trị riêng của một ma trận là nghiệm của đa thức tối tiểu của nó nên các giá
trị riêng của 𝐼𝐼 + 𝐴𝐴 chỉ là 1. Vì đa thức tối tiểu và đa thức đặc trưng có cùng nhân tử chung nên
|𝐼𝐼 + 𝐴𝐴 − 𝑥𝑥𝑥𝑥| = (1 − 𝑥𝑥)𝑛𝑛
Do đó det(𝐼𝐼 + 𝐴𝐴) = 1.

Bài 4.4. (Tính chất ma trận luỹ linh – Wikipedia)


Nếu 𝐴𝐴 là một ma trận cấp 𝑛𝑛 thỏa mãn det(𝐼𝐼 + 𝑡𝑡𝑡𝑡) = 1 với mọi t, thì 𝐴𝐴 lũy linh.

Chứng minh.

Với 𝑡𝑡 = 0 thì det(𝐼𝐼 + 𝑡𝑡𝑡𝑡) = det 𝐼𝐼 = 1


Xét 𝑡𝑡 ≠ 0, ta có
1 1
det(𝐼𝐼 + 𝑡𝑡𝑡𝑡) = det �𝑡𝑡 � 𝐼𝐼 + 𝐴𝐴�� = 𝑡𝑡𝑛𝑛 det �𝐴𝐴 + 𝐼𝐼�
𝑡𝑡 𝑡𝑡
Đặt 𝜆𝜆 = 1𝑡𝑡 , ta có
1
det(𝐴𝐴 + 𝜆𝜆𝜆𝜆) = 1 ∀𝜆𝜆 ≠ 0
𝜆𝜆𝑛𝑛
Suy ra det(𝐴𝐴 + 𝜆𝜆𝜆𝜆) ≡ 𝜆𝜆𝑛𝑛 hay det(𝐴𝐴 − 𝜆𝜆𝜆𝜆) = (−𝜆𝜆)𝑛𝑛 . Mặt khác, đó chính là đa thức đặc trưng của
𝐴𝐴.
Vậy 𝐴𝐴𝑛𝑛 = 0, tức 𝐴𝐴 luỹ linh.

Bài 4.5. Câu 5 : Cho 𝐴𝐴, 𝐵𝐵 là hai ma trận vuông cấp 𝑛𝑛 và giao hoán nhau, 𝐵𝐵 luỹ linh. Chứng
minh rằng
tr(𝐴𝐴 + 𝐵𝐵) = tr 𝐴𝐴 và det(𝐴𝐴 + 𝐵𝐵) = det 𝐴𝐴.

Chứng minh.

*Theo tính chất, 𝐵𝐵 luỹ linh nên tr 𝐵𝐵 = 0 suy ra tr(𝐴𝐴 + 𝐵𝐵) = tr 𝐴𝐴 + tr 𝐵𝐵 = tr 𝐴𝐴.

Đỗ Minh Triết IV-42 - MATHTASY -


Ôn thi Olympic Toán Đại số Phần IV: Ma trận lũy linh

*Nếu det 𝐴𝐴 = 0, do 𝐵𝐵 luỹ linh nên 𝐵𝐵𝑛𝑛 = 0, từ 𝐴𝐴𝐴𝐴 = 𝐵𝐵𝐵𝐵 ta có (𝐴𝐴 + 𝐵𝐵)𝑛𝑛 = 𝐴𝐴𝐴𝐴 suy ra
𝑛𝑛
det(𝐴𝐴 + 𝐵𝐵) = det 𝐴𝐴 det 𝐶𝐶 = 0 ⇒ det(𝐴𝐴 + 𝐵𝐵) = 0 = det 𝐵𝐵.
Nếu det 𝐴𝐴 ≠ 0 tức 𝐴𝐴 khả nghịch, cũng từ 𝐴𝐴𝐴𝐴 = 𝐵𝐵𝐵𝐵, ta có 𝐵𝐵𝐴𝐴−1 = 𝐴𝐴−1 𝐵𝐵 ⇒ (𝐵𝐵𝐴𝐴−1 )𝑛𝑛 = 0 tức
𝐵𝐵𝐴𝐴−1 luỹ linh và lại theo chứng minh trên thì
det(𝐼𝐼 + 𝐵𝐵𝐴𝐴−1 ) = 1 ⇔ det(𝐴𝐴 + 𝐵𝐵) det 𝐴𝐴−1 = 1 ⇔ det(𝐴𝐴 + 𝐵𝐵) = det 𝐴𝐴.
Nhận xét: Từ 𝑑𝑑𝑑𝑑𝑑𝑑(𝐴𝐴 + 𝐵𝐵) = 𝑑𝑑𝑑𝑑𝑑𝑑 𝐴𝐴 có thể suy trực tiếp ra 𝑡𝑡𝑡𝑡(𝐴𝐴 + 𝐵𝐵) = 𝑡𝑡𝑡𝑡 𝐴𝐴.
Thật vậy vì 𝐴𝐴𝐴𝐴 = 𝐵𝐵𝐵𝐵 và 𝑑𝑑𝑑𝑑𝑑𝑑(𝐴𝐴 + 𝐵𝐵) = 𝑑𝑑𝑑𝑑𝑑𝑑 𝐴𝐴, ta cũng có 𝑑𝑑𝑑𝑑𝑑𝑑(𝐴𝐴 − 𝜆𝜆𝜆𝜆 + 𝐵𝐵) = 𝑑𝑑𝑑𝑑𝑑𝑑(𝐴𝐴 − 𝜆𝜆𝜆𝜆) (thay
vai trò của 𝐴𝐴 bởi 𝐴𝐴 − 𝜆𝜆𝜆𝜆 cũng giao hoán với 𝐵𝐵 với mọi 𝜆𝜆), tức là 𝐴𝐴 + 𝐵𝐵 và 𝐴𝐴 có cùng đa thức
đặc trưng nên cũng cùng tập giá trị riêng và do đó cũng cùng vết.

Bài 4.6. (Olympic XI) Cho 𝐴𝐴 và 𝐵𝐵 là hai ma trận vuông cùng cấp và 𝐴𝐴𝐴𝐴 = 𝐵𝐵𝐵𝐵. Khi đó nếu 𝐴𝐴 và
𝐵𝐵 là các ma trận luỹ linh thì các ma trận 𝐼𝐼 + (𝐴𝐴 + 𝐵𝐵), 𝐼𝐼– (𝐴𝐴 + 𝐵𝐵) là các ma trận khả nghịch.
Chứng minh

Bài 4.7. Cho 𝐴𝐴 và 𝐵𝐵 là hai ma trận vuông cùng cấp thoả mãn các điều kiện:
i. 𝐴𝐴𝐴𝐴 = 𝐵𝐵𝐵𝐵
ii. Tồn tại các số nguyên dương 𝑝𝑝, 𝑞𝑞 sao cho (𝐴𝐴 − 𝐼𝐼)𝑝𝑝 = (𝐵𝐵 − 𝐼𝐼)𝑞𝑞 = 0.
Chứng minh rằng ma trận tích 𝐴𝐴𝐴𝐴 có các giá trị riêng đều bằng 1.

Chứng minh.

Theo tính chất d đã chứng minh trên thì 𝐴𝐴𝐴𝐴 luỹ linh đơn, do đó đa thức đặc trưng của 𝐴𝐴𝐴𝐴 là
𝑃𝑃𝐴𝐴𝐴𝐴 (𝜆𝜆) = (1 − 𝜆𝜆)𝑛𝑛
và nó chỉ có nghiệm là 1 (bội 𝑛𝑛).

Bài 4.8. Cho 𝑇𝑇 là tự đồng cấu của không gian vector 𝑉𝑉 . Giả sử 𝑥𝑥 ∈ 𝑉𝑉 mà 𝑇𝑇 𝑚𝑚 𝑥𝑥 =
0, 𝑇𝑇 𝑚𝑚−1 𝑥𝑥 ≠ 0, 𝑚𝑚 ∈ ℕ∗ . Chứng minh rằng 𝑥𝑥, 𝑇𝑇𝑇𝑇, 𝑇𝑇 2 𝑥𝑥, … , 𝑇𝑇 𝑚𝑚−1 𝑥𝑥 độc lập tuyến tính.

Chứng minh.

Giả sử 𝑎𝑎0 𝑥𝑥 + 𝑎𝑎1 𝑇𝑇𝑇𝑇+. . . +𝑎𝑎𝑚𝑚−1 𝑇𝑇 𝑚𝑚−1 𝑥𝑥 = 0.


Nhân 𝑇𝑇 𝑚𝑚−1 vào hai vế, ta được 𝑎𝑎0 𝑇𝑇 𝑚𝑚−1 𝑥𝑥 = 0 ⇒ 𝑎𝑎0 = 0 nên 𝑎𝑎1 𝑇𝑇𝑇𝑇 + 𝑎𝑎2 𝑇𝑇 2 𝑥𝑥+. . . +𝑎𝑎𝑚𝑚−1 𝑇𝑇 𝑚𝑚−1 𝑥𝑥 = 0.
Nhân 𝑇𝑇 𝑚𝑚−2 vào hai vế, ta được 𝑎𝑎1 𝑇𝑇 𝑚𝑚−1 𝑥𝑥 = 0 ⇒ 𝑎𝑎1 = 0 nên 𝑎𝑎2 𝑇𝑇 2 𝑥𝑥 + 𝑎𝑎3 𝑇𝑇 3 𝑥𝑥+. . . +𝑎𝑎𝑚𝑚−1 𝑇𝑇 𝑚𝑚−1 𝑥𝑥 =
0.
Cứ tiếp tục như thế, cuối cùng ta có 𝑎𝑎0 = 𝑎𝑎1 =. . . = 𝑎𝑎𝑚𝑚−1 = 0.

Đỗ Minh Triết IV-43 - MATHTASY -


Ôn thi Olympic Toán Đại số Phần IV: Ma trận lũy linh

Vậy 𝑥𝑥, 𝑇𝑇𝑇𝑇, 𝑇𝑇 2 𝑥𝑥, … , 𝑇𝑇 𝑚𝑚−1 𝑥𝑥 độc lập tuyến tính.

Bài 4.9. Cho 𝐴𝐴, 𝐵𝐵 ∈ 𝑀𝑀𝑛𝑛 (ℝ), 𝐴𝐴 khả nghịch, 𝐴𝐴𝐴𝐴 = 𝐵𝐵𝐵𝐵, 𝐵𝐵𝑟𝑟 = 0 (𝑟𝑟 ≥ 1). Chứng minh 𝐴𝐴 +
𝐵𝐵𝑠𝑠 (𝑠𝑠 ≥ 1) khả nghịch

Chứng minh.

Ta có
𝐴𝐴2𝑟𝑟 = 𝐴𝐴2𝑟𝑟 − 𝐵𝐵2𝑠𝑠𝑠𝑠 = (𝐴𝐴2 )𝑟𝑟 − (𝐵𝐵2𝑠𝑠 )𝑟𝑟 = (𝐴𝐴2 − 𝐵𝐵2𝑠𝑠 )�𝐴𝐴2(𝑟𝑟−1) + 𝐴𝐴2(𝑟𝑟−2) 𝐵𝐵2𝑠𝑠 +. . . +𝐵𝐵2𝑠𝑠(𝑟𝑟−1) �
= (𝐴𝐴 + 𝐵𝐵𝑠𝑠 )(𝐴𝐴 − 𝐵𝐵𝑠𝑠 )�𝐴𝐴2(𝑟𝑟−1) + 𝐴𝐴2(𝑟𝑟−2) 𝐵𝐵2𝑠𝑠 +. . . +𝐵𝐵2𝑠𝑠(𝑟𝑟−1) �
Do 𝐴𝐴 khả nghịch nên lấy định thức hai vế, ta có điều phải chứng minh.

Bài 4.10. (Câu Olympic 2011) Cho 𝐴𝐴, 𝐵𝐵 ∈ 𝑀𝑀𝑛𝑛 (ℝ) và 𝐶𝐶 = 𝐴𝐴𝐴𝐴 − 𝐵𝐵𝐵𝐵. Chứng minh rằng
nếu ma trận 𝐶𝐶 giao hoán với cả hai ma trận 𝐴𝐴 và 𝐵𝐵 thì tồn tại số nguyên dương 𝑚𝑚 sao cho 𝐶𝐶 𝑚𝑚 = 𝑂𝑂.

Chứng minh.

*Áp dụng giả thiết 𝐴𝐴𝐴𝐴 − 𝐵𝐵𝐵𝐵 = 𝐶𝐶 và tính giao hoán của 𝐶𝐶 với 𝐴𝐴, 𝐵𝐵, ta có
𝐴𝐴𝐵𝐵𝑘𝑘 − 𝐵𝐵𝐵𝐵𝐵𝐵𝑘𝑘−1 = 𝐶𝐶𝐵𝐵𝑘𝑘−1
⇔ 𝐴𝐴𝐵𝐵𝑘𝑘 − 𝐵𝐵(𝐶𝐶 + 𝐵𝐵𝐵𝐵)𝐵𝐵𝑘𝑘−2 = 𝐶𝐶𝐵𝐵𝑘𝑘−1
⇔ 𝐴𝐴𝐵𝐵𝑘𝑘 − 𝐵𝐵2 𝐴𝐴𝐵𝐵𝑘𝑘−2 = 2𝐶𝐶𝐵𝐵𝑘𝑘−1
⇔ 𝐴𝐴𝐵𝐵𝑘𝑘 − 𝐵𝐵2 (𝐶𝐶 + 𝐵𝐵𝐵𝐵)𝐵𝐵𝑘𝑘−3 = 2𝐶𝐶𝐵𝐵𝑘𝑘−1
⇔ 𝐴𝐴𝐵𝐵𝑘𝑘 − 𝐵𝐵3 𝐴𝐴𝐵𝐵𝑘𝑘−3 = 3𝐶𝐶𝐵𝐵𝑘𝑘−1 ……
Lặp lại quá trình như thế 𝑘𝑘 lần, cuối cùng ta được
𝐴𝐴𝐵𝐵𝑘𝑘 − 𝐵𝐵𝑘𝑘 𝐴𝐴 = 𝑘𝑘𝐵𝐵𝑘𝑘−1 𝐶𝐶 ∀𝑘𝑘 ∈ ℕ (∗)
*Chọn 𝑓𝑓(𝑥𝑥) = 𝑎𝑎0 𝑥𝑥𝑛𝑛 + 𝑎𝑎1 𝑥𝑥𝑛𝑛−1 + ⋯ + 𝑎𝑎𝑛𝑛−1 𝑥𝑥 + 𝑎𝑎𝑛𝑛 (𝑎𝑎0 ≠ 0) là đa thức bậc 𝑛𝑛 bất kì.
Ta có 𝑓𝑓 ′ (𝑥𝑥) = 𝑎𝑎0 𝑛𝑛𝑥𝑥𝑛𝑛−1 + 𝑎𝑎1 (𝑛𝑛 − 1)𝑥𝑥𝑛𝑛−2 + ⋯ + 𝑎𝑎𝑛𝑛−2 2𝑥𝑥 + 𝑎𝑎𝑛𝑛−1
Từ (∗) suy ra 𝐴𝐴𝐴𝐴(𝐵𝐵) − 𝑓𝑓(𝐵𝐵)𝐴𝐴 = 𝑓𝑓 ′ (𝐵𝐵)𝐶𝐶 (∗∗)
Xét đa thức đặc trưng của 𝐵𝐵: 𝑝𝑝(𝑥𝑥) = det(𝐵𝐵 − 𝑥𝑥𝐼𝐼𝑛𝑛 ), ta có 𝑝𝑝(𝐵𝐵) = 𝑂𝑂
Theo (∗∗) ta có 𝐴𝐴𝐴𝐴(𝐵𝐵) − 𝑝𝑝(𝐵𝐵)𝐴𝐴 = 𝑝𝑝′ (𝐵𝐵)𝐶𝐶 hay 𝑂𝑂 = 𝑝𝑝′ (𝐵𝐵)𝐶𝐶
Lại chọn 𝑞𝑞(𝑥𝑥) = 𝑝𝑝′ (𝑥𝑥) và nhờ vào tính giao hoán của 𝐶𝐶, ta có 𝐴𝐴𝑝𝑝′ (𝐵𝐵)𝐶𝐶 − 𝑝𝑝′ (𝐵𝐵)𝐶𝐶𝐶𝐶 = 𝑝𝑝′′ (𝐵𝐵)𝐶𝐶 2 =
𝑂𝑂
Tiếp tục quá trình này ta được: 𝑂𝑂 = (−1)𝑛𝑛 𝑛𝑛! 𝐶𝐶 𝑛𝑛 .

Đỗ Minh Triết IV-44 - MATHTASY -


Ôn thi Olympic Toán Đại số Phần IV: Ma trận lũy linh

Bài 4.11. Cho 𝐴𝐴 ∈ 𝑀𝑀2 (ℝ). Không dùng định nghĩa ma trận luỹ linh và công cụ đa thức,
chứng minh rằng ∀𝑘𝑘 ∈ ℕ, 𝑘𝑘 > 2: 𝐴𝐴𝑘𝑘 = 0 ⇔ 𝐴𝐴2 = 0.

Chứng minh.

Cách 1:

- Nếu 𝐴𝐴2 = 0 thì 𝐴𝐴𝑘𝑘 = 0 (𝑘𝑘 > 2).


𝑎𝑎 𝑏𝑏
- Nếu 𝐴𝐴𝑘𝑘 = 0 thì |𝐴𝐴| = 0, tức là nếu ký hiệu 𝐴𝐴 = � � thì 𝑎𝑎𝑎𝑎 − 𝑏𝑏𝑏𝑏 = 0.
𝑐𝑐 𝑑𝑑
Vì |𝐴𝐴 − 𝜆𝜆𝜆𝜆| = 𝜆𝜆2 − (𝑎𝑎 + 𝑑𝑑)𝜆𝜆 + (𝑎𝑎𝑎𝑎 − 𝑏𝑏𝑏𝑏) nên theo định lý Cayley – Hamilton, ta có
𝐴𝐴2 − (𝑎𝑎 + 𝑑𝑑)𝐴𝐴 + 𝑎𝑎𝑎𝑎 − 𝑏𝑏𝑏𝑏 = 0 ⇒ 𝐴𝐴2 = (𝑎𝑎 + 𝑑𝑑)𝐴𝐴 ⇒ 𝐴𝐴3 = (𝑎𝑎 + 𝑑𝑑)𝐴𝐴2 = (𝑎𝑎 + 𝑑𝑑)2 𝐴𝐴
⇒ ⋯ ⇒ 𝐴𝐴𝑘𝑘 = (𝑎𝑎 + 𝑑𝑑)𝑘𝑘−1 𝐴𝐴 ⇒ (𝑎𝑎 + 𝑑𝑑)𝑘𝑘−1 𝐴𝐴 = 0
Lúc này, nếu 𝑎𝑎 + 𝑑𝑑 ≠ 0 thì 𝐴𝐴 = 0 ⇒ 𝐴𝐴2 = 0. Còn nếu 𝑎𝑎 + 𝑑𝑑 = 0 thì do 𝐴𝐴2 = (𝑎𝑎 + 𝑑𝑑)𝐴𝐴 nên 𝐴𝐴2 =
0.
Điều phải chứng minh.

Cách 2:

 Phần đảo: 𝐴𝐴2 = 0 thì 𝐴𝐴𝑘𝑘 = 0 ∀𝑘𝑘 > 2


 Phần thuận:
- Nếu 𝐴𝐴 = 0 thì hiển nhiên.
𝑎𝑎1 𝑎𝑎2
- Nếu 𝐴𝐴 ≠ 0 thì do 𝐴𝐴𝑘𝑘 = 0 (𝑘𝑘 > 2) nên det 𝐴𝐴 = 0 hay 𝐴𝐴 có dạng 𝐴𝐴 = �𝜆𝜆𝑎𝑎 ∗ 2
𝜆𝜆𝑎𝑎2 � , 𝜆𝜆 ∈ ℝ , 𝑎𝑎1 +
1
𝑎𝑎22 ≠ 0.
Ta có
𝑎𝑎1 𝑎𝑎2 𝑎𝑎1 𝑎𝑎2 𝑎𝑎1 (𝑎𝑎1 + 𝜆𝜆𝑎𝑎2 ) 𝑎𝑎2 (𝑎𝑎1 + 𝜆𝜆𝑎𝑎2 )
𝐴𝐴2 = �𝜆𝜆𝑎𝑎 � �
𝜆𝜆𝑎𝑎2 𝜆𝜆𝑎𝑎1 𝜆𝜆𝑎𝑎2 � = �𝜆𝜆𝑎𝑎1 (𝑎𝑎1 + 𝜆𝜆𝑎𝑎2 ) 𝜆𝜆𝑎𝑎2 (𝑎𝑎1 + 𝜆𝜆𝑎𝑎2 )� = (𝑎𝑎1 + 𝜆𝜆𝑎𝑎2 )𝐴𝐴
1

Suy ra 𝐴𝐴𝑘𝑘 = (𝑎𝑎1 + 𝜆𝜆𝑎𝑎2 )𝑘𝑘−1 𝐴𝐴.


Do 𝐴𝐴𝑘𝑘 = 0, 𝐴𝐴 ≠ 0 nên (𝑎𝑎1 + 𝜆𝜆𝑎𝑎2 )𝑘𝑘−1 = 0 hay 𝑎𝑎1 + 𝜆𝜆𝑎𝑎2 = 0. Vậy 𝐴𝐴2 = 0.

Đỗ Minh Triết IV-45 - MATHTASY -


PHẦN V. VẾT CỦA MA TRẬN (Trace of the Matrix)

5.1. Định nghĩa và tính chất

Định nghĩa: Vết của một ma trận vuông 𝐴𝐴 là tổng tất cả các phần tử trên đường chéo chính
của nó. Ký hiệu: trace 𝐴𝐴 , tr 𝐴𝐴 , V(𝐴𝐴), thường dùng tr 𝐴𝐴.
Tính chất:
- tr 𝐼𝐼 = 𝑛𝑛
- tr(𝑘𝑘𝑘𝑘) = 𝑘𝑘 tr 𝐴𝐴
- tr(𝐴𝐴 + 𝐵𝐵) = tr 𝐴𝐴 + tr 𝐵𝐵
- tr 𝐴𝐴 = tr(𝐴𝐴𝑇𝑇 )
- tr(𝐴𝐴𝐴𝐴) = tr(𝐵𝐵𝐵𝐵) (𝐴𝐴, 𝐵𝐵 vuông cùng cấp).
- Vết 𝐴𝐴 bằng tổng các giá trị riêng của 𝐴𝐴. Từ đó, hai ma trận đồng dạng vì có cùng tập giá trị
riêng nên cũng cùng vết.

5.2. Bài tập

Bài 5.1. Cho 𝐴𝐴, 𝐵𝐵 là hai ma trận vuông thực cấp 𝑛𝑛 thoả 𝐴𝐴2 𝐵𝐵 + 𝐵𝐵𝐴𝐴2 = 2𝐴𝐴𝐴𝐴𝐴𝐴. Tính
tr[(𝐴𝐴𝐴𝐴 − 𝐵𝐵𝐵𝐵)𝑘𝑘 ] , 𝑘𝑘 ∈ ℕ∗

Giải.

Đặt 𝑋𝑋 = 𝐴𝐴𝐴𝐴 − 𝐵𝐵𝐵𝐵 thì


𝑋𝑋𝑋𝑋 = (𝐴𝐴𝐴𝐴 − 𝐵𝐵𝐵𝐵)𝐴𝐴 = 𝐴𝐴𝐴𝐴𝐴𝐴 − 𝐵𝐵𝐴𝐴2 = 𝐴𝐴2 𝐵𝐵 − 𝐴𝐴𝐴𝐴𝐴𝐴 = 𝐴𝐴(𝐴𝐴𝐴𝐴 − 𝐵𝐵𝐵𝐵) = 𝐴𝐴𝐴𝐴
Do đó, ta có 𝑋𝑋 𝑘𝑘 𝐴𝐴 = 𝐴𝐴𝑋𝑋 𝑘𝑘 ∀𝑘𝑘 và
𝑋𝑋 𝑘𝑘 = 𝑋𝑋 𝑘𝑘−1 𝑋𝑋 = 𝑋𝑋 𝑘𝑘−1 (𝐴𝐴𝐴𝐴 − 𝐵𝐵𝐵𝐵) = 𝑋𝑋 𝑘𝑘−1 𝐴𝐴𝐴𝐴 − 𝑋𝑋 𝑘𝑘−1 𝐵𝐵𝐵𝐵 = 𝐴𝐴𝑋𝑋 𝑘𝑘−1 𝐵𝐵 − 𝑋𝑋 𝑘𝑘−1 𝐵𝐵𝐵𝐵
= 𝐴𝐴(𝑋𝑋 𝑘𝑘−1 𝐵𝐵) − (𝑋𝑋 𝑘𝑘−1 𝐵𝐵)𝐴𝐴
Suy ra tr(𝑋𝑋 𝑘𝑘 ) = tr[(𝐴𝐴𝐴𝐴 − 𝐵𝐵𝐵𝐵)𝑘𝑘 ] = 0.

Bài 5.2. (Câu 2 đề chọn đội tuyển vòng 2 ĐH An Giang 2010)


a) Cho 𝐴𝐴 là ma trận vuông cấp 𝑛𝑛 sao cho 𝐴𝐴𝐴𝐴𝑇𝑇 = 𝐴𝐴2 . Chứng minh rằng tr[(𝐴𝐴 − 𝐴𝐴𝑇𝑇 )𝑇𝑇 (𝐴𝐴 − 𝐴𝐴𝑇𝑇 )] = 0.
b) Cho 𝐴𝐴, 𝐵𝐵, 𝐶𝐶 là các ma trận vuông cấp 𝑛𝑛, 𝐶𝐶 khả nghịch. Giả sử 𝐷𝐷 là ma trận nghiệm đúng phương
trình 𝐴𝐴𝐴𝐴 + 𝐵𝐵 = 𝐶𝐶 và 𝐵𝐵𝐶𝐶 −1 + 𝐶𝐶 −1 𝐵𝐵 = 0. Chứng minh rằng tr(𝐴𝐴𝐷𝐷𝐶𝐶 −1 ) + tr(𝐶𝐶 −1 𝐴𝐴𝐴𝐴) = 2𝑛𝑛.

Đỗ Minh Triết V-46 - MATHTASY -


Ôn thi Olympic Toán Đại số Phần V: Vết của Ma trận

Chứng minh.

a) tr[(𝐴𝐴 − 𝐴𝐴𝑇𝑇 )𝑇𝑇 (𝐴𝐴 − 𝐴𝐴𝑇𝑇 )] = tr[(𝐴𝐴𝑇𝑇 − 𝐴𝐴)(𝐴𝐴 − 𝐴𝐴𝑇𝑇 )] = tr[𝐴𝐴𝑇𝑇 𝐴𝐴 − (𝐴𝐴𝑇𝑇 )2 − 𝐴𝐴2 + 𝐴𝐴𝐴𝐴𝑇𝑇 ]
= tr(𝐴𝐴𝑇𝑇 𝐴𝐴) − tr[(𝐴𝐴2 )𝑇𝑇 ] − tr(𝐴𝐴2 ) + tr(𝐴𝐴𝐴𝐴𝑇𝑇 ) = 0
b) Theo giả thiết, ta có 𝐴𝐴𝐴𝐴 + 𝐵𝐵 = 𝐶𝐶 suy ra 𝐴𝐴𝐴𝐴𝐶𝐶 −1 + 𝐵𝐵𝐶𝐶 −1 = 𝐼𝐼 và 𝐶𝐶 −1 𝐴𝐴𝐴𝐴 + 𝐶𝐶 −1 𝐵𝐵 = 𝐼𝐼 từ đó
ta có 𝐴𝐴𝐴𝐴𝐶𝐶 −1 + 𝐶𝐶 −1 𝐴𝐴𝐴𝐴 = 2𝐼𝐼. Suy ra điều phải chứng minh.

Bài 5.3. Cho 𝐴𝐴 là ma trận vuông cấp 𝑛𝑛. Chứng minh rằng không tồn tại ma trận 𝐼𝐼 + 𝐵𝐵 khả
nghịch sao cho 𝐴𝐴𝐴𝐴 − 𝐵𝐵𝐵𝐵 = 𝐼𝐼 + 𝐵𝐵.

Chứng minh.

Từ giả thiết, ta có
𝐴𝐴𝐴𝐴 − 𝐵𝐵𝐵𝐵 = 𝐴𝐴(𝐼𝐼 + 𝐵𝐵) − (𝐼𝐼 + 𝐵𝐵)𝐴𝐴 = 𝐼𝐼 + 𝐵𝐵
Đặt 𝐶𝐶 = 𝐼𝐼 + 𝐵𝐵, giả sử 𝐶𝐶 khả nghịch, khi đó
𝐴𝐴 − 𝐶𝐶𝐶𝐶𝐶𝐶 −1 = 𝐼𝐼
Mặt khác, do 𝐴𝐴 đồng dạng với ma trận 𝐶𝐶𝐶𝐶𝐶𝐶 −1 nên tr(𝐴𝐴 − 𝐶𝐶𝐶𝐶𝐶𝐶 −1 ) = 0 ≠ 1 = tr 𝐼𝐼, mâu thuẫn.
Vậy không tồn tại ma trận 𝐶𝐶 = 𝐼𝐼 + 𝐵𝐵 khả nghịch sao cho 𝐴𝐴𝐴𝐴 − 𝐵𝐵𝐵𝐵 = 𝐼𝐼 + 𝐵𝐵.

Bài 5.4. (Đề chọn đội tuyển vòng 2 ĐH Kinh tế Quốc dân 2012)
Cho 𝑃𝑃 , 𝑄𝑄 là 2 ma trận vuông cấp 𝑛𝑛. Gọi 𝑈𝑈 , 𝑉𝑉 là 2 ma trận khác nhau là nghiệm của phương trình
𝑋𝑋 2 − 𝑃𝑃𝑃𝑃 + 𝑄𝑄 = 0. Chứng minh rằng tr(𝑈𝑈 + 𝑉𝑉 ) = tr 𝑃𝑃 và det(𝑈𝑈𝑈𝑈 ) = det 𝑄𝑄 là đúng nếu 𝑈𝑈 − 𝑉𝑉 khả
nghịch. Cho phản ví dụ với 𝑛𝑛 = 2 nếu 𝑈𝑈 − 𝑉𝑉 suy biến.

Giải.

*Chứng minh tr(𝑈𝑈 + 𝑉𝑉 ) = tr 𝑃𝑃


Theo giả thiết, ta có 𝑈𝑈 2 − 𝑃𝑃𝑃𝑃 + 𝑄𝑄 = 0 và 𝑉𝑉 2 − 𝑃𝑃𝑃𝑃 + 𝑄𝑄 = 0
Trừ từng vế, ta có 𝑈𝑈 2 − 𝑉𝑉 2 = 𝑃𝑃 (𝑈𝑈 − 𝑉𝑉 ) ⇔ (𝑈𝑈 + 𝑉𝑉 )(𝑈𝑈 − 𝑉𝑉 ) − (𝑉𝑉𝑉𝑉 − 𝑈𝑈𝑈𝑈 ) = 𝑃𝑃 (𝑈𝑈 − 𝑉𝑉 )
Đặt 𝑇𝑇 = 𝑈𝑈 − 𝑉𝑉
Nếu 𝑇𝑇 khả nghịch thì (𝑈𝑈 + 𝑉𝑉 ) − (𝑉𝑉𝑉𝑉 − 𝑈𝑈𝑈𝑈 )𝑇𝑇 −1 = 𝑃𝑃 (1)
Mặt khác (𝑉𝑉𝑉𝑉 − 𝑈𝑈𝑈𝑈 ) = 𝑉𝑉 (𝑈𝑈 − 𝑉𝑉 ) − (𝑈𝑈 − 𝑉𝑉 )𝑉𝑉 = 𝑉𝑉𝑉𝑉 − 𝑇𝑇𝑇𝑇 ⇒ (𝑉𝑉𝑉𝑉 − 𝑈𝑈𝑈𝑈 )𝑇𝑇 −1 = 𝑉𝑉 − 𝑇𝑇𝑇𝑇 𝑇𝑇 −1
Vì 𝑉𝑉 đồng dạng với 𝑇𝑇𝑇𝑇 𝑇𝑇 −1 nên tr[(𝑉𝑉𝑉𝑉 − 𝑈𝑈𝑈𝑈 )𝑇𝑇 −1 ] = tr(𝑉𝑉 − 𝑇𝑇𝑇𝑇 𝑇𝑇 −1 ) = 0
Kết hợp với (1), ta có điều phải chứng minh.
*Chứng minh det(𝑈𝑈𝑈𝑈 ) = det 𝑄𝑄
Ta có 𝑄𝑄 = 𝑃𝑃𝑃𝑃 − 𝑈𝑈 2 = (𝑃𝑃 − 𝑈𝑈 )𝑈𝑈 nên det 𝑄𝑄 = det[(𝑃𝑃 − 𝑈𝑈 )𝑈𝑈 ] = det(𝑃𝑃 − 𝑈𝑈 ) det 𝑈𝑈 .

Đỗ Minh Triết V-47 - MATHTASY -


Ôn thi Olympic Toán Đại số Phần V: Vết của Ma trận

Ta chứng minh det(𝑃𝑃 − 𝑈𝑈 ) = det 𝑉𝑉 , tiếp tục sử dụng tính đồng dạng, ta có
(𝑃𝑃 − 𝑈𝑈 )(𝑈𝑈 − 𝑉𝑉 ) = 𝑃𝑃𝑃𝑃 − 𝑈𝑈 2 + 𝑈𝑈𝑈𝑈 = 𝑈𝑈 2 − 𝑉𝑉 2 − 𝑈𝑈 2 + 𝑈𝑈𝑈𝑈 = 𝑈𝑈𝑈𝑈 − 𝑉𝑉 2 = 𝑇𝑇𝑇𝑇
Suy ra (𝑃𝑃 − 𝑈𝑈 ) = 𝑇𝑇𝑇𝑇 𝑇𝑇 −1
Tức là (𝑃𝑃 − 𝑈𝑈 ) đồng dạng với 𝑉𝑉 nên det(𝑃𝑃 − 𝑈𝑈 ) = det 𝑉𝑉 , ta có điều phải chứng minh.
*Phản ví dụ, chọn
1 0 0 0 1 1 0 2
𝑈𝑈 = � � ; 𝑉𝑉 = � � ; 𝑃𝑃 = � � ; 𝑄𝑄 = � �
0 2 0 2 0 2 0 0
1 0
thì khi đó 𝑈𝑈, 𝑉𝑉 khác nhau là nghiệm của phương trình 𝑋𝑋 2 − 𝑃𝑃𝑃𝑃 + 𝑄𝑄 = 0, (𝑈𝑈 − 𝑉𝑉 ) = � �
0 0
0 0 0 2
suy biến nhưng det(𝑈𝑈𝑈𝑈 ) = det � � = 0 = det � � = det 𝑄𝑄.
0 4 0 0

Đỗ Minh Triết V-48 - MATHTASY -


PHẦN VI. HẠNG CỦA MA TRẬN (Rank of the matrix)

6.1. Định nghĩa và tính chất

Định nghĩa: Hạng của ma trận 𝐴𝐴 là cấp cao nhất của định thức con khác 0 của 𝐴𝐴, ký hiệu
rank 𝐴𝐴.
Định lý: Cho 𝐴𝐴 là ma trận cỡ 𝑚𝑚 × 𝑛𝑛 . Các số sau đây là bằng nhau và bằng rank 𝐴𝐴:
1. Số tối đại các cột độc lập tuyến tính của 𝐴𝐴, nói cách khác, số chiều của không gian sinh bởi
các vector cột của 𝐴𝐴 (trong ℝ𝑛𝑛 ).
2. Số tối đại các hàng độc lập tuyến tính của 𝐴𝐴, nói cách khác, số chiều của không gian sinh bởi
các vector hàng của 𝐴𝐴 (trong ℝ𝑚𝑚 ).
3. Số chiều của không gian ảnh của ánh xạ tuyến tính nhận 𝐴𝐴 làm ma trận biểu diễn.
Dễ thấy, rank𝐴𝐴 = rank(𝐴𝐴𝑇𝑇 ) và rank𝐴𝐴 ≤ min{𝑚𝑚, 𝑛𝑛}. Đôi khi để nghiên cứu tính chất của hạng
ma trận, người ta thường dùng ánh xạ tuyến tính nhận nó làm ma trận biểu diễn.
Một số tính chất và bất đẳng thức quan trọng
Với mọi ma trận 𝐴𝐴
rank 𝐴𝐴 = dim(Im𝐴𝐴)
∀𝑥𝑥 ∈ ℝ𝑛𝑛 : 𝐴𝐴𝐴𝐴 ∈ Im 𝐴𝐴 ; 𝐴𝐴𝐴𝐴 = 0 → 𝑥𝑥 ∈ Ker 𝐴𝐴
Nói riêng, 𝐴𝐴2 = 0 thì Im 𝐴𝐴 ⊂ Ker 𝐴𝐴 (do 𝐴𝐴2 𝑥𝑥 = 𝐴𝐴(𝐴𝐴𝐴𝐴) = 0)
Với 𝐴𝐴𝑛𝑛 , 𝐵𝐵𝑛𝑛 thì
dim(Im𝐴𝐴) + dim(Ker𝐴𝐴) = 𝑛𝑛
rank(𝐴𝐴𝐴𝐴) ≤ rank 𝐴𝐴 (rank(𝐴𝐴𝐴𝐴) ≤ rank 𝐵𝐵), dấu “=” xảy ra khi 𝐵𝐵 (𝐴𝐴) khả nghịch
rank(𝐴𝐴 + 𝐵𝐵) ≤ rank 𝐴𝐴 + rank 𝐵𝐵
Với 𝐴𝐴𝑚𝑚×𝑛𝑛 , 𝐵𝐵𝑛𝑛×𝑝𝑝 thì ta có bất đẳng thức Sylvester
rank 𝐴𝐴 + rank 𝐵𝐵 − 𝑛𝑛 ≤ rank(𝐴𝐴𝐴𝐴) ≤ min{rank 𝐴𝐴 , rank 𝐵𝐵}

6.2. Bài tập

Bài 6.1. Cho các ma trận 𝐴𝐴𝑚𝑚×𝑛𝑛 , 𝐵𝐵𝑚𝑚 , 𝐶𝐶𝑛𝑛 trong đó 𝐵𝐵, 𝐶𝐶 khả nghịch. Chứng minh rằng
rank(𝐵𝐵𝐵𝐵) = rank(𝐴𝐴𝐴𝐴) = rank 𝐴𝐴.

Chứng minh.

Đỗ Minh Triết VI-49 - MATHTASY -


Ôn thi Olympic Toán Đại số Phần VI: Hạng của Ma trận

Xem 𝐴𝐴, 𝐵𝐵 tương ứng là các ánh xạ tuyến tính 𝐴𝐴: ℝ𝑛𝑛 → ℝ𝑚𝑚 , 𝐵𝐵: ℝ𝑚𝑚 → ℝ𝑚𝑚 . Do 𝐵𝐵 khả nghịch nên
toán tử 𝐵𝐵 là một đẳng cấu của ℝ𝑚𝑚 . Do đó
rank(𝐵𝐵𝐵𝐵) = dim�𝐵𝐵𝐵𝐵(ℝ𝑚𝑚 )� = dim �𝐵𝐵�𝐴𝐴(ℝ𝑚𝑚 )�� = dim�𝐴𝐴(ℝ𝑚𝑚 )� = rank 𝐴𝐴

Tương tự rank(𝐴𝐴𝐴𝐴) = rank 𝐴𝐴.

Bài 6.2. Chứng minh rằng mọi ma trận 𝐴𝐴𝑚𝑚×𝑛𝑛 , 𝐵𝐵𝑚𝑚×𝑛𝑛 , ta có


rank(𝐴𝐴 ± 𝐵𝐵) ≤ rank 𝐴𝐴 + rank 𝐵𝐵
|rank 𝐴𝐴 − rank 𝐵𝐵| ≤ rank(𝐴𝐴 ± 𝐵𝐵)

Chứng minh.

*Xem 𝐴𝐴, 𝐵𝐵 là các ánh xạ tuyến tính từ ℝ𝑛𝑛 vào ℝ𝑚𝑚 , khi đó ta có
rank(𝐴𝐴 + 𝐵𝐵) = dim(Im(𝐴𝐴 + 𝐵𝐵))
Do ∀𝑥𝑥 ∈ ℝ𝑛𝑛 thì (𝐴𝐴 ± 𝐵𝐵)𝑥𝑥 = (𝐴𝐴𝐴𝐴 ± 𝐵𝐵𝐵𝐵) ∈ (Im 𝐴𝐴 + Im 𝐵𝐵) nên Im(𝐴𝐴 + 𝐵𝐵) ⊂ (Im 𝐴𝐴 + Im 𝐵𝐵).
Bởi vậy mà
rank(𝐴𝐴 + 𝐵𝐵) = dim(Im(𝐴𝐴 + 𝐵𝐵)) ≤ dim(Im 𝐴𝐴 + Im 𝐵𝐵) ≤ dim(Im 𝐴𝐴) + dim(Im 𝐵𝐵)
= rank 𝐴𝐴 + rank 𝐵𝐵
Từ đó
rank(𝐴𝐴 ± 𝐵𝐵) ≤ rank 𝐴𝐴 + rank(±𝐵𝐵) = rank 𝐴𝐴 + rank 𝐵𝐵
*Giả sử rank 𝐴𝐴 ≥ rank 𝐵𝐵, ta có
rank 𝐴𝐴 = rank(𝐴𝐴 − 𝐵𝐵 + 𝐵𝐵) ≤ rank(𝐴𝐴 − 𝐵𝐵) + rank 𝐵𝐵 ⇒ rank 𝐴𝐴 − rank 𝐵𝐵 ≤ rank(𝐴𝐴 − 𝐵𝐵)
Tương tự nếu rank 𝐵𝐵 ≥ rank 𝐴𝐴, ta có
rank 𝐵𝐵 − rank 𝐴𝐴 ≤ rank(𝐵𝐵 − 𝐴𝐴) = rank(𝐴𝐴 − 𝐵𝐵)
Suy ra |rank 𝐴𝐴 − rank 𝐵𝐵| ≤ rank(𝐴𝐴 − 𝐵𝐵)
Chứng minh tương tự, ta cũng có |rank 𝐴𝐴 − rank 𝐵𝐵| ≤ rank(𝐴𝐴 + 𝐵𝐵).

Bài 6.3. (Câu 4 đề dự tuyển ĐH Thuỷ Lợi 2009) Cho 𝐴𝐴 là ma trận thực hạng 𝑟𝑟. Chứng
minh rằng các ma trận 𝐴𝐴𝑇𝑇 𝐴𝐴 và 𝐴𝐴𝐴𝐴𝑇𝑇 cũng có hạng bằng 𝑟𝑟.

Chứng minh.

Giả sử 𝐴𝐴 là ma trận cỡ 𝑚𝑚 × 𝑛𝑛 (𝑚𝑚 có thể bằng 𝑛𝑛). Khi ấy 𝐴𝐴𝑇𝑇 𝐴𝐴 có cỡ 𝑛𝑛 × 𝑛𝑛. Ta xét hai hệ phương
trình đại số tuyến tính 𝑛𝑛 ẩn
𝐴𝐴𝐴𝐴 = 𝜃𝜃 (1) và 𝐴𝐴𝑇𝑇 𝐴𝐴𝐴𝐴 = 𝜃𝜃 (2)

Đỗ Minh Triết VI-50 - MATHTASY -


Ôn thi Olympic Toán Đại số Phần VI: Hạng của Ma trận

𝑥𝑥1 0
⎛ 𝑥𝑥2 ⎞ ⎛ 0⎞
với 𝑥𝑥 = ⎜
⎜ ⋮ ⎟ ⎟ ; 𝜃𝜃 = ⎜
⎜⋮⎟ ⎟
⎝𝑥𝑥𝑛𝑛 ⎠ ⎝0⎠n×1
Ta chứng minh hai hệ này có chung tập nghiệm.
Rõ ràng tập nghiệm của (1) là con tập nghiệm của hệ (2).
Ngược lại, nếu 𝑥𝑥0 là nghiệm của (2) tức là 𝐴𝐴𝑇𝑇 𝐴𝐴𝑥𝑥0 = 𝜃𝜃 thì 𝑥𝑥𝑇𝑇0 𝐴𝐴𝑇𝑇 𝐴𝐴𝑥𝑥0 = 0 nên (𝐴𝐴𝑥𝑥0 )𝑇𝑇 (𝐴𝐴𝑥𝑥0 ) = 0 ⇒
𝐴𝐴𝑥𝑥0 = 𝜃𝜃 tức 𝑥𝑥0 cũng là nghiệm của (1).
Chứng minh tương tự với 𝐴𝐴𝐴𝐴𝑇𝑇 .

Bài 6.4. Cho 𝑃𝑃 , 𝑄𝑄, 𝑅𝑅 là các ma trận vuông cùng cấp. Chứng minh rằng
rank(𝑃𝑃𝑃𝑃) + rank(𝑄𝑄𝑄𝑄) ≤ rank 𝑄𝑄 + rank(𝑃𝑃𝑃𝑃𝑃𝑃)

Chứng minh.
𝑃𝑃𝑃𝑃 0 𝑃𝑃𝑃𝑃 −𝑃𝑃𝑃𝑃𝑃𝑃 0 −𝑃𝑃𝑃𝑃𝑃𝑃
rank(𝑃𝑃𝑃𝑃) + rank(𝑄𝑄𝑄𝑄) ≤ rank � � = rank � � = rank � �
𝑄𝑄 𝑄𝑄𝑄𝑄 𝑄𝑄 0 𝑄𝑄 0
= rank 𝑄𝑄 + rank(𝑃𝑃𝑃𝑃𝑃𝑃)

Bài 6.5. (Câu 6 đề chọn đội tuyển vòng 2 ĐH Kinh tế Quốc dân 2010 – Câu 3 đề chọn đội tuyển
ĐHKHTN-ĐHQGHN 2010)
Cho 𝐴𝐴, 𝐵𝐵 là các ma trận vuông cùng cấp sao cho 𝐴𝐴𝑇𝑇 𝐵𝐵 = 𝐴𝐴𝐵𝐵𝑇𝑇 = 0. Chứng minh rằng
rank(𝐴𝐴 + 𝐵𝐵) = rank 𝐴𝐴 + rank 𝐵𝐵
Nếu chỉ có 𝐴𝐴𝑇𝑇 𝐵𝐵 = 0 thì bài Toán còn đúng không? Cho một ví dụ chứng tỏ.

Chứng minh.

Áp dụng (𝑋𝑋𝑋𝑋 )𝑇𝑇 = 𝑌𝑌 𝑇𝑇 𝑋𝑋 𝑇𝑇 , từ giả thiết dễ dàng ta có 𝐵𝐵𝑇𝑇 𝐴𝐴 = 𝐵𝐵𝐴𝐴𝑇𝑇 = 0, kết hợp hai chứng minh
trên, từ đó
rank 𝐴𝐴 + rank 𝐵𝐵 = rank(𝐴𝐴𝐴𝐴𝑇𝑇 ) + rank(𝐵𝐵𝑇𝑇 𝐵𝐵) = rank(𝐴𝐴𝐴𝐴𝑇𝑇 + 𝐵𝐵𝐴𝐴𝑇𝑇 ) + rank(𝐵𝐵𝑇𝑇 𝐵𝐵 + 𝐵𝐵𝑇𝑇 𝐴𝐴)
= rank[(𝐴𝐴 + 𝐵𝐵)𝐴𝐴𝑇𝑇 ] + rank[𝐵𝐵𝑇𝑇 (𝐵𝐵 + 𝐴𝐴)] = rank[𝐵𝐵𝑇𝑇 (𝐴𝐴 + 𝐵𝐵)] + rank[(𝐴𝐴 + 𝐵𝐵)𝐴𝐴𝑇𝑇 ]
≤ rank(𝐴𝐴 + 𝐵𝐵) + rank[𝐵𝐵𝑇𝑇 (𝐴𝐴 + 𝐵𝐵)𝐴𝐴𝑇𝑇 ] = rank(𝐴𝐴 + 𝐵𝐵) + rank(𝐵𝐵𝑇𝑇 𝐴𝐴𝐴𝐴𝑇𝑇 + 𝐵𝐵𝑇𝑇 𝐵𝐵𝐴𝐴𝑇𝑇 )
= rank(𝐴𝐴 + 𝐵𝐵)
Mặt khác rank(𝐴𝐴 + 𝐵𝐵) ≤ rank 𝐴𝐴 + rank 𝐵𝐵.
Ta có điều phải chứng minh.
* Nếu chỉ có 𝐴𝐴𝑇𝑇 𝐵𝐵 = 0 thì bài Toán không còn đúng. Ví dụ, chọn
0 0 1 0
𝐴𝐴 = � � ; 𝐵𝐵 = � �
1 0 0 0
thoả mãn

Đỗ Minh Triết VI-51 - MATHTASY -


Ôn thi Olympic Toán Đại số Phần VI: Hạng của Ma trận

0 1 1 0 0 0
𝐴𝐴𝑇𝑇 𝐵𝐵 = � �� �=� �
0 0 0 0 0 0
nhưng
1 0
rank(𝐴𝐴 + 𝐵𝐵) = rank � � = 1 ≠ rank 𝐴𝐴 + rank 𝐵𝐵 = 1 + 1 = 2
1 0

Bài 6.6. (Tổng quát câu 5 Olympic 2003)


Cho 𝐴𝐴 là một ma trận vuông luỹ linh. Chứng minh rằng với mọi số nguyên dương 𝑛𝑛, ta luôn có
rank 𝐴𝐴 = rank(𝐴𝐴 + 𝐴𝐴2 +. . . 𝐴𝐴𝑛𝑛 )

Chứng minh.

Cách 1:

Giả sử 𝐴𝐴𝑘𝑘 = 0, 𝑘𝑘 ∈ ℕ∗ . Đặt 𝐵𝐵 = 𝐴𝐴 + 𝐴𝐴2 +. . . 𝐴𝐴𝑛𝑛 . Để chứng minh 𝐴𝐴, 𝐵𝐵 cùng hạng, ta chứng minh
hai hệ 𝐴𝐴𝐴𝐴 = 0 và 𝐵𝐵𝐵𝐵 = 0 có cùng tập nghiệm, với 𝑥𝑥 ∈ ℝ𝑛𝑛 .
Giả sử 𝑥𝑥0 là nghiệm của hệ 𝐴𝐴𝐴𝐴 = 0, khi đó
𝐵𝐵𝑥𝑥0 = (𝐴𝐴 + 𝐴𝐴2 +. . . 𝐴𝐴𝑛𝑛 )𝑥𝑥0 = 𝐴𝐴𝑥𝑥0 + 𝐴𝐴(𝐴𝐴𝑥𝑥0 )+. . . +𝐴𝐴𝑛𝑛−1 (𝐴𝐴𝑥𝑥0 ) = 0
tức là 𝑥𝑥0 cũng là nghiệm của hệ 𝐵𝐵𝐵𝐵 = 0.
Bây giờ, giả sử 𝑥𝑥0 là nghiệm của hệ 𝐵𝐵𝐵𝐵 = 0
𝐵𝐵𝑥𝑥0 = (𝐴𝐴 + 𝐴𝐴2 +. . . 𝐴𝐴𝑛𝑛 )𝑥𝑥0 = 0 ⇒ 𝐴𝐴𝑥𝑥0 = −𝐴𝐴2 (𝐼𝐼 + 𝐴𝐴+. . . +𝐴𝐴𝑛𝑛−2 )𝑥𝑥0 ∀𝑛𝑛 ≥ 2
Đặt 𝐶𝐶 = −(𝐼𝐼 + 𝐴𝐴+. . . +𝐴𝐴𝑛𝑛−2 ), ta có 𝐴𝐴𝐴𝐴 = 𝐶𝐶𝐶𝐶 và
𝐴𝐴𝑥𝑥0 = 𝐴𝐴2 𝐶𝐶𝑥𝑥0 = 𝐶𝐶𝐶𝐶(𝐴𝐴𝑥𝑥0 ) = 𝐶𝐶𝐶𝐶(𝐴𝐴2 𝐶𝐶𝑥𝑥0 ) = 𝐶𝐶 2 𝐴𝐴2 (𝐴𝐴𝑥𝑥0 ) =. . . = 𝐶𝐶 𝑙𝑙 𝐴𝐴𝑙𝑙 (𝐴𝐴𝑥𝑥0 ) = 0 ∀ 𝑙𝑙 ≥ 𝑘𝑘
điều đó chứng tỏ 𝑥𝑥0 cũng là nghiệm của hệ 𝐴𝐴𝐴𝐴 = 0.

Cách 2:

Do 𝐴𝐴 luỹ linh nên giả sử 𝐴𝐴𝑛𝑛 = 0, khi đó


𝐼𝐼 = 𝐼𝐼 − 𝐴𝐴𝑛𝑛 = (𝐼𝐼 − 𝐴𝐴)(𝐼𝐼 + 𝐴𝐴+. . . +𝐴𝐴𝑛𝑛−1 )
Suy ra 𝐼𝐼 − 𝐴𝐴 khả nghịch và (𝐼𝐼 − 𝐴𝐴)−1 = (𝐼𝐼 + 𝐴𝐴+. . . +𝐴𝐴𝑛𝑛−1 ).
Áp dụng đẳng thức ∀𝑋𝑋 ∈ 𝑀𝑀𝑛𝑛 (𝕂𝕂): rank 𝑋𝑋 = rank 𝑋𝑋𝑋𝑋 với mọi 𝑌𝑌 khả nghịch, ta có
rank 𝐴𝐴 = rank[𝐴𝐴(𝐼𝐼 − 𝐴𝐴)−1 ] = rank(𝐴𝐴 + 𝐴𝐴2 +. . . 𝐴𝐴𝑛𝑛 )
Điều phải chứng minh.

Bài 6.7. (Câu 3 Olympic 1997)


Chứng minh rằng với 𝐴𝐴 ∈ 𝑀𝑀𝑛𝑛 (ℝ), ta đều tìm được số nguyên dương 𝑛𝑛 sao cho rank(𝐴𝐴𝑘𝑘 ) =
rank(𝐴𝐴𝑘𝑘+1 ) ∀ 𝑘𝑘 ≥ 𝑛𝑛.

Chứng minh.

Đỗ Minh Triết VI-52 - MATHTASY -


Ôn thi Olympic Toán Đại số Phần VI: Hạng của Ma trận

Từ bất đẳng thức rank 𝐴𝐴𝐴𝐴 ≤ rank 𝐴𝐴 �𝐵𝐵 ∈ 𝑀𝑀𝑛𝑛 (ℝ)�, dấu “=” xảy ra khi 𝐵𝐵 khả nghịch, ta có
𝑛𝑛 ≥ rank 𝐴𝐴 ≥ rank(𝐴𝐴2 ) ≥. . . ≥ rank(𝐴𝐴𝑛𝑛 ) ≥. . . ≥ 0
Vì rank(𝐴𝐴𝑘𝑘 ) ∈ ℕ nên kể từ một số 𝑘𝑘 nào đó trở đi, ta sẽ có các đẳng thức
rank(𝐴𝐴𝑘𝑘 ) = rank(𝐴𝐴𝑘𝑘+1 ) = rank(𝐴𝐴𝑘𝑘+2 ) =. ..

Bài 6.8. (Câu 3b đề chọn đội tuyển ĐH Giao thông vận tải 2011)
Cho 𝑝𝑝 là số nguyên dương, 𝐴𝐴 là ma trận vuông cấp 𝑛𝑛 sao cho 𝐴𝐴𝑝𝑝+1 = 𝐴𝐴. Chứng minh rằng
a) rank 𝐴𝐴 + rank(𝐼𝐼 − 𝐴𝐴𝑝𝑝 ) = 𝑛𝑛.
b) Nếu 𝑝𝑝 nguyên tố thì
rank(𝐼𝐼 − 𝐴𝐴) = rank(𝐼𝐼 − 𝐴𝐴2 ) = ⋯ = rank(𝐼𝐼 − 𝐴𝐴𝑝𝑝−1 )

Chứng minh.

a) Theo giả thiết 𝐴𝐴(𝐼𝐼 − 𝐴𝐴𝑝𝑝 ) = 0 nên Im(𝐼𝐼 − 𝐴𝐴𝑝𝑝 ) ⊂ Ker 𝐴𝐴.
Ngoài ra, nếu 𝑥𝑥 ∈ Ker 𝐴𝐴 thì (𝐼𝐼 − 𝐴𝐴𝑝𝑝 )𝑥𝑥 = 𝑥𝑥, tức 𝑥𝑥 ∈ Im(𝐼𝐼 − 𝐴𝐴𝑝𝑝 ) do đó Ker 𝐴𝐴 ⊂ Im(𝐼𝐼 − 𝐴𝐴𝑝𝑝 ).
Vậy Im(𝐼𝐼 − 𝐴𝐴𝑝𝑝 ) = Ker 𝐴𝐴, suy ra
rank(𝐼𝐼 − 𝐴𝐴𝑝𝑝 ) = dim[Im(𝐼𝐼 − 𝐴𝐴𝑝𝑝 )] = dim[Ker 𝐴𝐴] = 𝑛𝑛 − rank 𝐴𝐴
hay rank 𝐴𝐴 + rank(𝐼𝐼 − 𝐴𝐴𝑝𝑝 ) = 𝑛𝑛
b) Xét hệ thặng dư đầy đủ modun 𝑘𝑘 với 1 ≤ 𝑘𝑘 ≤ 𝑝𝑝 − 1 sau {1,2, … , 𝑘𝑘 − 1, 𝑘𝑘}. Khi đó
𝑆𝑆 = {𝑝𝑝 + 1,2𝑝𝑝 + 1, … , (𝑘𝑘 − 1)𝑝𝑝 + 1, 𝑘𝑘𝑘𝑘 + 1}
cũng là một hệ thặng dư đầy đủ modun 𝑘𝑘. Do vậy tồn tại 𝑞𝑞 ∈ 𝑆𝑆 sao cho 𝑞𝑞𝑞𝑞 + 1 chia hết cho 𝑘𝑘. Vì
vậy
rank(𝐼𝐼 − 𝐴𝐴) ≥ rank(𝐼𝐼 − 𝐴𝐴𝑘𝑘 ) ≥ rank(𝐼𝐼 − 𝐴𝐴𝑞𝑞𝑞𝑞+1 ) = rank(𝐼𝐼 − 𝐴𝐴)
𝑞𝑞 ∈ 𝑆𝑆 → 𝑞𝑞 = 𝑎𝑎𝑎𝑎 + 1(1 ≤ 𝑎𝑎 ≤ 𝑘𝑘) → 𝑞𝑞𝑞𝑞 + 1 = (𝑎𝑎𝑎𝑎 + 1)𝑝𝑝 + 1 = 𝑎𝑎𝑝𝑝2 + 𝑝𝑝 + 1 ⋮ 𝑘𝑘 ? ? ?

Bài 6.9. (Câu 5 Olympic 1995)


Tìm hạng của ma trận phụ hợp của ma trận 𝐴𝐴 cấp 𝑛𝑛 theo rank 𝐴𝐴.

Giải.

Gọi 𝐶𝐶 = �𝑐𝑐𝑖𝑖𝑖𝑖 � là ma trận phụ hợp của 𝐴𝐴, trong đó 𝑐𝑐𝑖𝑖𝑖𝑖 = (−1)𝑖𝑖+𝑗𝑗 �𝑀𝑀𝑖𝑖𝑖𝑖 � trong đó 𝑀𝑀𝑖𝑖𝑖𝑖 là ma trận con
bù cấp 𝑛𝑛 − 1 của phần tử 𝑎𝑎𝑖𝑖𝑖𝑖 . Xét các trường hợp sau:
- Nếu 𝐴𝐴 khả nghịch thì 𝐴𝐴−1 = |𝐴𝐴|
1
𝐶𝐶 𝑇𝑇 nên rank 𝐶𝐶 = rank 𝐶𝐶 𝑇𝑇 = rank�|𝐴𝐴|
1
𝐶𝐶 𝑇𝑇 � = rank(𝐴𝐴−1 ) = 𝑛𝑛.

- Nếu rank 𝐴𝐴 ≤ 𝑛𝑛 − 2 thì mọi định thức con cấp 𝑛𝑛 − 1 của nó đều bằng 0, nghĩa là 𝐶𝐶 = 0 nên
rank 𝐶𝐶 = 0.
- Nếu rank 𝐴𝐴 = 𝑛𝑛 − 1 thì |𝐴𝐴| = 0 và trong 𝐶𝐶 có ít nhất một phần tử khác 0 nên rank 𝐶𝐶 ≠ 0.

Đỗ Minh Triết VI-53 - MATHTASY -


Ôn thi Olympic Toán Đại số Phần VI: Hạng của Ma trận

Ta có 𝐴𝐴𝐶𝐶 𝑇𝑇 = |𝐴𝐴|𝐼𝐼 = 0. Xem 𝐴𝐴, 𝐶𝐶 𝑇𝑇 như là các toán tử tuyến tính trên ℝ𝑛𝑛 , ta có
𝐴𝐴𝐶𝐶 𝑇𝑇 𝑥𝑥 = 𝐴𝐴(𝐶𝐶 𝑇𝑇 𝑥𝑥) = 0 ⇒ 𝐶𝐶 𝑇𝑇 𝑥𝑥 ∈ Ker 𝐴𝐴 hay Im(𝐶𝐶 𝑇𝑇 ) ⊂ Ker 𝐴𝐴
⇒ dim[Im(𝐶𝐶 𝑇𝑇 )] ≤ dim(Ker 𝐴𝐴) = 𝑛𝑛 − dim(Im𝐴𝐴) = 𝑛𝑛 − rank 𝐴𝐴 = 1
Ta có rank 𝐶𝐶 = rank 𝐶𝐶 𝑇𝑇 = dim[Im(𝐶𝐶 𝑇𝑇 )] ≤ 1 . Nếu rank 𝐶𝐶 = 0 thì 𝐶𝐶 = 0 , mâu thuẫn. Vậy
rank 𝐶𝐶 = 1.

Bài 6.10. Cho 𝐴𝐴 là một ma trận vuông cấp 𝑛𝑛 thoả 𝐴𝐴2 = 0 và tồn tại ma trận 𝐵𝐵 sao cho
𝐴𝐴𝐴𝐴 + 𝐵𝐵𝐵𝐵 = 𝐼𝐼. Chứng minh rằng 𝑛𝑛 là một số chẵn.

Chứng minh.

Ta có 2 rank 𝐴𝐴 ≤ rank(𝐴𝐴2 ) + 𝑛𝑛 hay rank 𝐴𝐴 ≤ 𝑛𝑛2 . Mặt khác


𝑛𝑛 = rank 𝐼𝐼 = rank(𝐴𝐴𝐴𝐴 + 𝐵𝐵𝐵𝐵) ≤ rank(𝐴𝐴𝐴𝐴) + rank(𝐵𝐵𝐵𝐵) ≤ 2 min{rank 𝐴𝐴 , rank 𝐵𝐵} ≤ 2 rank 𝐴𝐴 ≤ 𝑛𝑛
Như vậy, ta phải có rank 𝐴𝐴 = 𝑛𝑛2 nên 𝑛𝑛 phải là số chẵn.

Bài 6.11. Cho 𝐴𝐴, 𝐵𝐵 là các ma trận cấp 𝑛𝑛 thoả 𝐼𝐼 − 𝐴𝐴𝐴𝐴 khả nghịch. Chứng minh rằng 𝐼𝐼 −
𝐵𝐵𝐵𝐵 cũng khả nghịch.

Chứng minh.

Cách 1: Phản chứng

Giả sử ngược lại, |𝐼𝐼 − 𝐵𝐵𝐵𝐵| = 0 khi đó hệ (𝐼𝐼 − 𝐵𝐵𝐵𝐵)𝑥𝑥 = 0 có nghiệm không tầm thường, tức là
nghiệm 𝑥𝑥 ≠ 0. Ta có 𝐵𝐵𝐵𝐵𝐵𝐵 = 𝑥𝑥, đặt 𝑦𝑦 = 𝐴𝐴𝐴𝐴 thì 𝑦𝑦 ≠ 0. Từ đó
(𝐼𝐼 − 𝐴𝐴𝐴𝐴)𝑦𝑦 = 𝑦𝑦 − 𝐴𝐴𝐴𝐴𝐴𝐴 = 𝑦𝑦 − 𝐴𝐴𝐴𝐴𝐴𝐴𝐴𝐴 = 𝑦𝑦 − 𝐴𝐴𝐴𝐴 = 𝑦𝑦 − 𝑦𝑦 = 0
Điều đó chứng tỏ 𝑦𝑦 ≠ 0 là nghiệm không tầm thường của hệ (𝐼𝐼 − 𝐴𝐴𝐴𝐴)𝑥𝑥 = 0, tức 𝐼𝐼 − 𝐴𝐴𝐴𝐴 suy biến,
mâu thuẫn giả thiết.

Cách 2: Chứng minh |I − AB| = |I − BA|

Cách 2a:

|𝐼𝐼 − 𝐴𝐴𝐴𝐴| = �𝐼𝐼 − 𝐴𝐴𝐴𝐴 0� = ��𝐼𝐼 −𝐴𝐴� � 𝐼𝐼 𝐴𝐴�� = �𝐼𝐼 −𝐴𝐴� � 𝐼𝐼 𝐴𝐴� = � 𝐼𝐼 𝐴𝐴�
𝐵𝐵 𝐼𝐼 0 𝐼𝐼 𝐵𝐵 𝐼𝐼 0 𝐼𝐼 𝐵𝐵 𝐼𝐼 𝐵𝐵 𝐼𝐼
𝐼𝐼 0
= � 𝐼𝐼 𝐴𝐴� �𝐼𝐼 −𝐴𝐴� = �� 𝐼𝐼 𝐴𝐴� �𝐼𝐼 −𝐴𝐴�� = � � = |𝐼𝐼 − 𝐵𝐵𝐵𝐵|
𝐵𝐵 𝐼𝐼 0 𝐼𝐼 𝐵𝐵 𝐼𝐼 0 𝐼𝐼 𝐵𝐵 𝐼𝐼 − 𝐵𝐵𝐵𝐵
Cách 2b:
 Nếu 𝐵𝐵 khả nghịch thì
det(𝐼𝐼 − 𝐴𝐴𝐴𝐴) = det(𝐵𝐵 −1 𝐵𝐵 − 𝐵𝐵−1 𝐵𝐵𝐵𝐵𝐵𝐵) = det 𝐵𝐵−1 . det(𝐼𝐼 − 𝐵𝐵𝐵𝐵) . det 𝐵𝐵 = det(𝐼𝐼 − 𝐵𝐵𝐵𝐵)
 Nếu 𝐵𝐵 không khả nghịch, ta xét ma trận 𝐵𝐵𝑥𝑥 = 𝑥𝑥𝑥𝑥 + 𝐵𝐵, 𝑥𝑥 ∈ ℝ.

Đỗ Minh Triết VI-54 - MATHTASY -


Ôn thi Olympic Toán Đại số Phần VI: Hạng của Ma trận

Rõ ràng det 𝐵𝐵𝑥𝑥 là một đa thức bậc 𝑛𝑛 của 𝑥𝑥 và chỉ có nhiều nhất là 𝑛𝑛 nghiệm thực, do vậy mà có
vô số 𝑥𝑥 ∈ ℝ để det 𝐵𝐵𝑥𝑥 ≠ 0, tức là 𝐵𝐵𝑥𝑥 khả nghịch.
Lúc đó theo chứng minh trên thì
det(𝐼𝐼 − 𝐴𝐴𝐵𝐵𝑥𝑥 ) = det(𝐼𝐼 − 𝐵𝐵𝑥𝑥 𝐴𝐴) (𝑥𝑥 ∈ ℝ: det 𝐵𝐵𝑥𝑥 ≠ 0)
Ta có det(𝐼𝐼 − 𝐴𝐴𝐵𝐵𝑥𝑥 ) và det(𝐼𝐼 − 𝐵𝐵𝑥𝑥 𝐴𝐴) là hai đa thức bậc nhỏ thua 𝑛𝑛 của 𝑥𝑥, mà giá trị của chúng
bằng nhau tại vô số những điểm 𝑥𝑥 nói trên mà det 𝐵𝐵𝑥𝑥 ≠ 0. Bởi vậy hai đa thức này phải trùng
nhau (tức là các hệ số tương ứng phải bằng nhau).
Vậy det(𝐼𝐼 − 𝐴𝐴𝐵𝐵𝑥𝑥 ) ≡ det(𝐼𝐼 − 𝐵𝐵𝑥𝑥 𝐴𝐴).
Nói riêng, với 𝑥𝑥 = 0 thì 𝐵𝐵𝑥𝑥 = 𝐵𝐵0 = 𝐵𝐵, ta có det(𝐼𝐼 − 𝐴𝐴𝐴𝐴) = det(𝐼𝐼 − 𝐵𝐵𝐵𝐵).

Bài 6.12. (Câu 1b Olympic Quốc tế 1994) Cho ma trận vuông 𝐴𝐴 cấp 𝑛𝑛 khả nghịch sau
1 1 1 1 … 1

⎜1 2 2 2 … 2⎞⎟

⎜1 2 1 1 … 1⎟⎟
𝐴𝐴 = ⎜
⎜ ⎟
⎜1 2 1 2 … 2⎟⎟
⎜⋮ ⋮ ⋮ ⋮ … …⎟
⎝1 2 1 2 … …⎠
Hỏi có bao nhiêu phần tử 0 của ma trận 𝐴𝐴−1 .

Giải.
𝑥𝑥1 𝑦𝑦1
Ta tìm 𝐴𝐴 −1
= �𝑏𝑏𝑖𝑖𝑖𝑖 �, giả sử 𝐴𝐴𝐴𝐴 = 𝑦𝑦 với 𝑥𝑥 = � � , 𝑦𝑦 = � ⋮ � ∈ ℝ𝑛𝑛 , ta tìm ma trận 𝐵𝐵 sao cho

𝑥𝑥𝑛𝑛 𝑦𝑦𝑛𝑛
𝑥𝑥 = 𝐵𝐵𝐵𝐵, khi đó 𝐵𝐵 = 𝐴𝐴−1
⎧𝑥𝑥1 + 𝑥𝑥2 + 𝑥𝑥3 + 𝑥𝑥4 + 𝑥𝑥5 +. . . + 𝑥𝑥𝑛𝑛−1 + 𝑥𝑥𝑛𝑛 = 𝑦𝑦1
� 𝑥𝑥1 + 2𝑥𝑥2 + 2𝑥𝑥3 + 2𝑥𝑥4 + 2𝑥𝑥5 +. . . +2𝑥𝑥𝑛𝑛−1 + 2𝑥𝑥𝑛𝑛 = 𝑦𝑦2

� 𝑥𝑥1 + 2𝑥𝑥2 + 𝑥𝑥3 + 𝑥𝑥4 + 𝑥𝑥5 +. . . + 𝑥𝑥𝑛𝑛−1 + 𝑥𝑥𝑛𝑛 = 𝑦𝑦3

� 𝑥𝑥 + 2𝑥𝑥 + 𝑥𝑥 + 2𝑥𝑥 + 2𝑥𝑥 +. . . +2𝑥𝑥
� 1 2 3 4 5 𝑛𝑛−1 + 2𝑥𝑥𝑛𝑛 = 𝑦𝑦4
𝐴𝐴𝐴𝐴 = 𝑦𝑦 ⇔ 𝑥𝑥1 + 2𝑥𝑥2 + 𝑥𝑥3 + 2𝑥𝑥4 + 𝑥𝑥5 +. . . + 𝑥𝑥𝑛𝑛−1 + 𝑥𝑥𝑛𝑛 = 𝑦𝑦5

� ⋮

� 𝑥𝑥1 + 2𝑥𝑥2 + 𝑥𝑥3 + 2𝑥𝑥4 + 𝑥𝑥5 +. . . + 𝑥𝑥𝑛𝑛−1 + 𝑥𝑥𝑛𝑛 = 𝑦𝑦𝑛𝑛−2

� 𝑥𝑥1 + 2𝑥𝑥2 + 𝑥𝑥3 + 2𝑥𝑥4 + 𝑥𝑥5 +. . . + 𝑥𝑥𝑛𝑛−1 + 𝑥𝑥𝑛𝑛 = 𝑦𝑦𝑛𝑛−1

⎩ 𝑥𝑥1 + 2𝑥𝑥2 + 𝑥𝑥3 + 2𝑥𝑥4 + 𝑥𝑥5 +. . . + 𝑥𝑥𝑛𝑛−1 + 𝑥𝑥𝑛𝑛 = 𝑦𝑦𝑛𝑛
Lấy dòng 2 trừ dòng 1, ta được 𝑥𝑥1 = 2𝑦𝑦1 − 𝑦𝑦2
Lấy dòng 3 trừ dòng 1, ta được 𝑥𝑥2 = −𝑦𝑦1 + 𝑦𝑦3
Lấy dòng 4 trừ dòng 2, ta được 𝑥𝑥3 = 𝑦𝑦2 − 𝑦𝑦4
Lấy dòng 5 trừ dòng 3, ta được 𝑥𝑥4 = −𝑦𝑦3 + 𝑦𝑦5
Lấy dòng 𝑘𝑘 trừ dòng 𝑘𝑘 − 2, ta được 𝑥𝑥𝑘𝑘 = (−1)𝑘𝑘 𝑦𝑦𝑘𝑘−1 + (−1)𝑘𝑘−1 𝑦𝑦𝑘𝑘+1
Từ đó

Đỗ Minh Triết VI-55 - MATHTASY -


Ôn thi Olympic Toán Đại số Phần VI: Hạng của Ma trận

2 −1 0 … 0 0

⎜ −1 0 1 … 0 0 ⎞


⎜ 0 1 0 −1 0 0 ⎟

𝐴𝐴−1 =⎜
⎜ … ⋮ ⋮


⎜ ⋮ ⋮ ⋮ ⎟

⎜ 0 (−1) 𝑛𝑛−2
0 (−1)𝑛𝑛−1 ⎟

0 0
⎝ 0 0 0 … (−1)𝑛𝑛−1 (−1)𝑛𝑛 ⎠
𝐴𝐴−1 có phần tử 𝑏𝑏11 = 2, 𝑏𝑏𝑛𝑛𝑛𝑛 = (−1)𝑛𝑛 , 𝑏𝑏𝑖𝑖,𝑖𝑖+1 = 𝑏𝑏𝑖𝑖+1,𝑖𝑖 = (−1)𝑖𝑖 , các phần tử còn lại bằng 0. Vậy số
các phần tử bằng 0 là 𝑛𝑛2 − 2 − 2(𝑛𝑛 − 1) = 𝑛𝑛2 − 2𝑛𝑛.

Bài 6.13. (Câu 6 Olympic 2002) Cho 𝐴𝐴 ∈ 𝑀𝑀𝑛𝑛 (ℝ) có hạng 1. Chứng minh tồn tại duy nhất
số thực 𝑘𝑘 sao cho 𝐴𝐴2 = 𝑘𝑘𝑘𝑘.

Chứng minh.

Vì rank 𝐴𝐴 = 1 nên 𝐴𝐴 có dạng


𝑘𝑘1 𝑎𝑎1 𝑘𝑘1 𝑎𝑎2 … 𝑘𝑘1 𝑎𝑎𝑛𝑛−1 𝑘𝑘1 𝑎𝑎𝑛𝑛

⎜ 𝑘𝑘 𝑎𝑎 𝑘𝑘2 𝑎𝑎2 … 𝑘𝑘2 𝑎𝑎𝑛𝑛−1 𝑘𝑘2 𝑎𝑎𝑛𝑛 ⎞⎟
⎜ 2 1 ⎟
𝐴𝐴 = ⎜
⎜ ⋮ ⋮ ⋱ ⋮ ⋮ ⎟ ⎟ ≠0

⎜𝑘𝑘𝑛𝑛−1 𝑎𝑎1 ⎟
𝑘𝑘𝑛𝑛−1 𝑎𝑎2 … 𝑘𝑘𝑛𝑛−1 𝑎𝑎𝑛𝑛−1 𝑘𝑘𝑛𝑛−1 𝑎𝑎𝑛𝑛 ⎟
⎝ 𝑘𝑘𝑛𝑛 𝑎𝑎1 𝑘𝑘𝑛𝑛 𝑎𝑎2 … 𝑘𝑘𝑛𝑛 𝑎𝑎𝑛𝑛−1 𝑘𝑘𝑛𝑛 𝑎𝑎𝑛𝑛 ⎠
���������
không mất tính tổng quát, giả sử hàng 1 khác 0 và 𝑘𝑘𝑖𝑖 ≠ 0 �𝑖𝑖 = 1, 𝑛𝑛�.
𝑘𝑘1
⎛ 𝑘𝑘2 ⎞
Đặt 𝑉𝑉 = ⎜

⎜ ⋮ ⎟
⎟ ≠ 0 và 𝑈𝑈 = (𝑎𝑎1

𝑎𝑎2 … 𝑎𝑎𝑛𝑛 ) ≠ 0, khi đó

⎝𝑘𝑘𝑛𝑛 ⎠
𝑉𝑉𝑉𝑉 = 𝐴𝐴 , 𝑈𝑈𝑈𝑈 = tr 𝐴𝐴
Từ đó suy ra
𝐴𝐴2 = (𝑉𝑉𝑉𝑉)(𝑉𝑉𝑉𝑉) = 𝑉𝑉 (𝑈𝑈𝑈𝑈 )𝑈𝑈 = (tr 𝐴𝐴)𝑉𝑉𝑉𝑉 = (tr 𝐴𝐴)𝐴𝐴 = 𝑘𝑘𝑘𝑘
Nếu tồn tại 𝑘𝑘′ thoả mãn 𝐴𝐴2 = 𝑘𝑘′ 𝐴𝐴 thì từ
0 = 𝐴𝐴2 − 𝐴𝐴2 = (𝑘𝑘 − 𝑘𝑘′ )𝐴𝐴 và 𝐴𝐴 ≠ 0
Suy ra 𝑘𝑘 = 𝑘𝑘′ .
Vậy tồn tại số thực 𝑘𝑘 duy nhất và hoàn toàn xác định (chính là vết 𝐴𝐴) thoả mãn 𝐴𝐴2 = 𝑘𝑘𝑘𝑘.

Bài 6.14. (Câu 3a đề chọn đội tuyển ĐH GTVT 2011)


Chứng minh rằng với mọi ma trận vuông cấp 2 𝐴𝐴, 𝐵𝐵, ta luôn có
rank(𝐴𝐴𝐴𝐴) − rank(𝐵𝐵𝐵𝐵) ≤ 1
Lấy ví dụ chứng tỏ đẳng thức xảy ra.

Chứng minh.

Đỗ Minh Triết VI-56 - MATHTASY -


Ôn thi Olympic Toán Đại số Phần VI: Hạng của Ma trận

- Nếu 𝐴𝐴 = 𝐵𝐵 = 0 thì rank(𝐴𝐴𝐴𝐴) − rank(𝐵𝐵𝐵𝐵) = 0 ≤ 1.


- Nếu 𝐴𝐴, 𝐵𝐵 cùng khả nghịch thì rank(𝐴𝐴𝐴𝐴) − rank(𝐵𝐵𝐵𝐵) = 2 − 2 = 0 ≤ 1.
- Nếu một trong hai ma trận 𝐴𝐴, 𝐵𝐵 khả nghịch, giả sử đó là 𝐴𝐴 thì
rank(𝐴𝐴𝐴𝐴) − rank(𝐵𝐵𝐵𝐵) = rank 𝐵𝐵 − rank 𝐵𝐵 = 0 ≤ 1.
- Nếu cả 𝐴𝐴, 𝐵𝐵 ≠ 0 và cùng suy biến, tức là rank 𝐴𝐴 = rank 𝐵𝐵 = 1 thì khi đó rank 𝐴𝐴𝐴𝐴 ≤ rank 𝐴𝐴 =
1, rank 𝐵𝐵𝐵𝐵 ≤ rank 𝐵𝐵 = 1 (do 𝐴𝐴, 𝐵𝐵 suy biến). Vậy trong trường hợp này:
+) Nếu 𝐴𝐴𝐴𝐴, 𝐵𝐵𝐵𝐵 cùng hạng (0 hoặc 1) thì rank(𝐴𝐴𝐴𝐴) − rank(𝐵𝐵𝐵𝐵) = 0 ≤ 1.
+) Nếu rank 𝐴𝐴𝐴𝐴 = 1, rank 𝐵𝐵𝐵𝐵 = 0 hay ngược lại, ta có rank(𝐴𝐴𝐴𝐴) − rank(𝐵𝐵𝐵𝐵) = ±1 ≤ 1.
Điều phải chứng minh.
Ví dụ chứng tỏ đẳng thức xảy ra:
1 1 1 1 2 2 0 0
Chọn 𝐴𝐴 = � � , 𝐵𝐵 = � � thì 𝐴𝐴𝐴𝐴 = � � , 𝐵𝐵𝐵𝐵 = � � ta có
−1 −1 1 1 −2 −2 0 0
rank 𝐴𝐴𝐴𝐴 − rank 𝐵𝐵𝐵𝐵 = 1 − 0 = 1.

Bài 6.15. (Câu 1 Olympic Quốc tế 1995)


Cho 𝑋𝑋 là một ma trận cấp 𝑛𝑛 không suy biến có dạng 𝑋𝑋 = (𝑋𝑋1 𝑋𝑋2 … 𝑋𝑋𝑛𝑛 ), 𝑌𝑌 là ma trận có dạng
𝑌𝑌 = (𝑋𝑋2 𝑋𝑋3 … 𝑋𝑋𝑛𝑛 0). Chứng minh rằng ma trận 𝐴𝐴 = 𝑌𝑌 𝑋𝑋 −1 và 𝐵𝐵 = 𝑋𝑋 −1 𝑌𝑌 có hạng là 𝑛𝑛 − 1 và
tìm giá trị riêng của 𝐴𝐴 và 𝐵𝐵.

Chứng minh.

*Chứng minh rank 𝐴𝐴 = rank 𝐵𝐵 = 𝑛𝑛 − 1:

Cách 1:

Đặt 𝐽𝐽 = �𝑎𝑎𝑖𝑖𝑖𝑖 � là ma trận cấp 𝑛𝑛 trong đó 𝑎𝑎𝑖𝑖𝑖𝑖 = 1 nếu 𝑖𝑖 = 𝑗𝑗 + 1, các phần tử còn lại bằng 0:
0 0 0 … 0 0

⎜ 1 0 0 … 0 0⎞ ⎟
⎜ 0 1 0 … 0 0⎟
𝐽𝐽 = ⎜




⎜ ⋮ ⋮ ⋮ … 0 0⎟ ⎟
⎜0 0 0 … 0 0⎟
⎝0 0 0 … 1 0⎠
Rõ ràng rank 𝐽𝐽 = 𝑛𝑛 − 1. Hơn nữa 𝑌𝑌 = 𝑋𝑋𝑋𝑋 nên 𝐴𝐴 = 𝑌𝑌 𝑋𝑋 −1 = 𝑋𝑋𝑋𝑋𝑋𝑋 −1 và 𝐵𝐵 = 𝑋𝑋 −1 𝑌𝑌 = 𝑋𝑋 −1 𝑋𝑋𝑋𝑋 =
𝐽𝐽 .
Suy ra rank 𝐴𝐴 = rank(𝑋𝑋𝑋𝑋𝑋𝑋 −1 ) = rank(𝑋𝑋𝑋𝑋𝑋𝑋 −1 ) = rank(𝑋𝑋𝑋𝑋) = rank 𝐵𝐵 = rank 𝐽𝐽 = 𝑛𝑛 − 1.

Cách 2:

Do 𝑋𝑋 không suy biến nên rank 𝑌𝑌 = rank(𝑌𝑌 𝑋𝑋 −1 ) = rank(𝑋𝑋 −1 𝑌𝑌 ) hay rank 𝑌𝑌 = rank 𝐴𝐴 = rank 𝐵𝐵.

Đỗ Minh Triết VI-57 - MATHTASY -


Ôn thi Olympic Toán Đại số Phần VI: Hạng của Ma trận

Cũng do 𝑋𝑋 không suy biến nên các vector 𝑋𝑋1 , … , 𝑋𝑋𝑛𝑛 độc lập tuyến tính suy ra 𝑌𝑌 có các cột
𝑋𝑋2 , … , 𝑋𝑋𝑛𝑛 độc lập tuyến tính nên rank 𝑌𝑌 = 𝑛𝑛 − 1.
*Tìm giá trị riêng của 𝐴𝐴 và 𝐵𝐵:
Do rank 𝐴𝐴 = rank 𝐵𝐵 = 𝑛𝑛 − 1 nên số chiều không gian nghiệm của hai hệ 𝐴𝐴𝐴𝐴 = 0 và 𝐵𝐵𝑥𝑥 = 0 (ứng
với giá trị riêng 0) là 1. Do đó 𝐴𝐴, 𝐵𝐵 chỉ có giá trị riêng là 0 và 1 (giá trị riêng 1 có bội 𝑛𝑛 − 1).

Bài 6.16. Cho 𝐴𝐴 = �𝑎𝑎𝑖𝑖𝑖𝑖 � = (sin(𝑖𝑖 + 𝑗𝑗)) ∈ 𝑀𝑀𝑛𝑛 (ℝ), 𝑛𝑛 ∈ ℕ∗ . Tìm rank 𝐴𝐴.

Giải.
sin 2 sin 3 sin 4 … sin(𝑛𝑛 + 1)

⎜ sin 3 sin 4 sin 5 … sin(𝑛𝑛 + 2) ⎞


⎜ ⎟
𝐴𝐴 = ⎜ sin 4 sin 5 sin 6 … sin(𝑛𝑛 + 3) ⎟


⎜ ⎟

⋮ ⋮ ⋮ … ⋮
⎝sin(𝑛𝑛 + 1) sin(𝑛𝑛 + 2) sin(𝑛𝑛 + 3) … sin(𝑛𝑛 + 𝑛𝑛)⎠
Nếu 𝑛𝑛 = 1 thì rank 𝐴𝐴 = 1.
Nếu 𝑛𝑛 ≥ 2, ký hiệu 𝑑𝑑𝑖𝑖 là dòng thứ 𝑖𝑖 của 𝐴𝐴.
Chú ý kĩ rằng 𝑑𝑑𝑖𝑖 + 𝑑𝑑𝑖𝑖+2 = 2 cos 1 𝑑𝑑𝑖𝑖+1 hay 𝑑𝑑𝑖𝑖 = 2 cos 1 𝑑𝑑𝑖𝑖+1 − 𝑑𝑑𝑖𝑖+2 , tức là từ dòng thứ 3 trở đi, mỗi
dòng biểu thị tuyến tính được qua hai dòng ngay trước nó.
Mặt khác, xét định thức con cấp 2 nằm ở góc trái trên cùng của 𝐴𝐴
sin 2 sin 3
� �≠0
sin 3 sin 4
Vậy rank 𝐴𝐴 = 2.

Bài 6.17. Tìm hạng của ma trận 𝐴𝐴 = �𝑎𝑎𝑖𝑖𝑖𝑖 � cấp 𝑛𝑛 xác định bởi 𝑎𝑎𝑖𝑖𝑖𝑖 = 𝑖𝑖 − 2010𝑗𝑗.

Giải.

Nếu 𝑛𝑛 = 1 thì rank 𝐴𝐴 = 1.


Nếu 𝑛𝑛 ≥ 2 , tách 𝐴𝐴 = 𝐵𝐵 + 𝐶𝐶 , trong đó 𝐵𝐵 = �𝑏𝑏𝑖𝑖𝑖𝑖 �, 𝐶𝐶 = �𝑐𝑐𝑖𝑖𝑖𝑖 � , được xác định bởi 𝑏𝑏𝑖𝑖𝑖𝑖 = 𝑖𝑖, 𝑐𝑐𝑖𝑖𝑖𝑖 =
−2010𝑗𝑗. Khi đó
rank 𝐴𝐴 = rank(𝐵𝐵 + 𝐶𝐶) ≤ rank 𝐵𝐵 + rank 𝐶𝐶 = 1 + 1 = 2
Mặt khác, xét định thức con cấp 2 góc trên bên trái của 𝐴𝐴
1 − 1.2010 1 − 2.2010
� � = 2010 ≠ 0
2 − 1.2010 2 − 2.2010
Vậy rank 𝐴𝐴 = 2.

Một số bài Toán về tính giao hoán và phản giao hoán

Đỗ Minh Triết VI-58 - MATHTASY -


Ôn thi Olympic Toán Đại số Phần VI: Hạng của Ma trận

Cho 𝐴𝐴, 𝐵𝐵 ∈ 𝑀𝑀𝑛𝑛 (𝕂𝕂), 𝐴𝐴𝐴𝐴 = 𝐵𝐵𝐵𝐵. Khi đó dễ dàng chứng minh được
-) 𝐴𝐴(𝐵𝐵 − 𝑘𝑘𝑘𝑘)𝑛𝑛 = (𝐵𝐵 − 𝑘𝑘𝑘𝑘)𝑛𝑛 𝐴𝐴 ∀𝑘𝑘 ∈ 𝕂𝕂, ∀𝑛𝑛 ∈ ℕ.
-) (𝐴𝐴 − 𝑘𝑘𝑘𝑘)𝑛𝑛 𝐵𝐵 = 𝐵𝐵(𝐴𝐴 − 𝑘𝑘𝑘𝑘)𝑛𝑛 ∀𝑘𝑘 ∈ 𝕂𝕂, ∀𝑛𝑛 ∈ ℕ.
-) Nếu 𝐴𝐴 khả nghịch thì 𝐴𝐴−1 𝐵𝐵 = 𝐵𝐵𝐴𝐴−1 .
-) Nếu 𝐵𝐵 khả nghịch thì 𝐴𝐴𝐵𝐵−1 = 𝐵𝐵−1 𝐴𝐴.

Bài 6.18. Cho 𝐴𝐴, 𝐵𝐵 là ma trận vuông cấp 𝑛𝑛, luỹ đẳng (idempotent). Chứng minh nếu 𝐴𝐴 + 𝐵𝐵
luỹ đẳng thì 𝐴𝐴𝐴𝐴 = 𝐵𝐵𝐵𝐵 = 0.

Chứng minh.

Nếu 𝐴𝐴 + 𝐵𝐵 luỹ đẳng


𝐴𝐴 + 𝐵𝐵 = (𝐴𝐴 + 𝐵𝐵)2 = 𝐴𝐴2 + 𝐵𝐵2 + 𝐴𝐴𝐴𝐴 + 𝐵𝐵𝐵𝐵 = 𝐴𝐴 + 𝐵𝐵 + 𝐴𝐴𝐴𝐴 + 𝐵𝐵𝐵𝐵
thì 𝐴𝐴𝐴𝐴 + 𝐵𝐵𝐵𝐵 = 0 suy ra 0 = 𝐴𝐴𝐴𝐴𝐴𝐴 + 𝐴𝐴𝐴𝐴𝐴𝐴 = 𝐴𝐴𝐴𝐴 + 𝐴𝐴𝐴𝐴𝐴𝐴 (1)
Mặt khác 0 = 𝐴𝐴𝐴𝐴𝐴𝐴𝐴𝐴 + 𝐴𝐴𝐴𝐴𝐴𝐴𝐴𝐴 = 𝐴𝐴𝐴𝐴 + 𝐴𝐴𝐴𝐴𝐴𝐴𝐴𝐴 và vì 𝐴𝐴𝐴𝐴 = −𝐵𝐵𝐵𝐵 nên
0 = 𝐴𝐴𝐴𝐴 + 𝐴𝐴𝐴𝐴𝐴𝐴𝐴𝐴 = 𝐴𝐴𝐴𝐴 − 𝐴𝐴𝐴𝐴𝐴𝐴𝐴𝐴 = 𝐴𝐴𝐴𝐴 − 𝐴𝐴𝐴𝐴𝐴𝐴 (2)
Từ (1) và (2) suy ra 𝐴𝐴𝐴𝐴 = 0 = 𝐵𝐵𝐵𝐵.

Bài 6.19. (Câu 5b Olympic 2010) Cho 𝐴𝐴, 𝐵𝐵, 𝐶𝐶 là các ma trận thực, vuông cấp 𝑛𝑛, trong đó
𝐴𝐴 khả nghịch và đồng thời giao hoán 𝐵𝐵, 𝐶𝐶. Giả sử 𝐶𝐶(𝐴𝐴 + 𝐵𝐵) = 𝐵𝐵. Chứng minh 𝐵𝐵, 𝐶𝐶 giao hoán với
nhau.

Chứng minh.

Từ giả thiết, suy ra 𝐴𝐴 + 𝐶𝐶(𝐴𝐴 + 𝐵𝐵) = 𝐴𝐴 + 𝐵𝐵 hay 𝐴𝐴 = (𝐼𝐼 − 𝐶𝐶)(𝐴𝐴 + 𝐵𝐵)


Do 𝐴𝐴 khả nghịch và đồng thời giao hoán cả 𝐵𝐵, 𝐶𝐶 nên 𝐼𝐼 = (𝐼𝐼 − 𝐶𝐶)(𝐴𝐴 + 𝐵𝐵)𝐴𝐴−1 = (𝐼𝐼 − 𝐶𝐶)𝐴𝐴−1 (𝐴𝐴 +
𝐵𝐵)
Suy ra (𝐼𝐼 − 𝐶𝐶) và 𝐴𝐴−1 (𝐴𝐴 + 𝐵𝐵) là nghịch đảo của nhau nên
𝐼𝐼 = (𝐼𝐼 − 𝐶𝐶)𝐴𝐴−1 (𝐴𝐴 + 𝐵𝐵) = 𝐴𝐴−1 (𝐴𝐴 + 𝐵𝐵)(𝐼𝐼 − 𝐶𝐶)
⇔ 𝐴𝐴 = (𝐴𝐴 + 𝐵𝐵)(𝐼𝐼 − 𝐶𝐶)
⇔ 𝐵𝐵 = (𝐴𝐴 + 𝐵𝐵)𝐶𝐶
Vậy (𝐴𝐴 + 𝐵𝐵)𝐶𝐶 = 𝐶𝐶(𝐴𝐴 + 𝐵𝐵) tức 𝐵𝐵𝐵𝐵 = 𝐶𝐶𝐶𝐶.

Bài 6.20. Cho 𝐴𝐴, 𝐵𝐵 là hai ma trận vuông cùng cấp thoả mãn 𝑎𝑎𝑎𝑎𝑎𝑎 + 𝑏𝑏𝑏𝑏 + 𝑐𝑐𝑐𝑐 =
𝑑𝑑𝑑𝑑 (𝑎𝑎𝑎𝑎𝑎𝑎𝑎𝑎 ≠ 0). Chứng minh 𝐴𝐴𝐴𝐴 = 𝐵𝐵𝐵𝐵.

Chứng minh.

Đỗ Minh Triết VI-59 - MATHTASY -


Ôn thi Olympic Toán Đại số Phần VI: Hạng của Ma trận

Ta có 𝑎𝑎𝑎𝑎𝑎𝑎 + 𝑏𝑏𝑏𝑏 + 𝑐𝑐𝑐𝑐 = 𝑑𝑑𝑑𝑑


⇔ 𝐴𝐴(𝑎𝑎𝑎𝑎 + 𝑏𝑏𝑏𝑏) + 𝑐𝑐𝑐𝑐 = 𝑑𝑑𝐼𝐼 ⇔ 𝑎𝑎𝑎𝑎(𝑎𝑎𝑎𝑎 + 𝑏𝑏𝑏𝑏) + 𝑎𝑎𝑎𝑎𝑎𝑎 = 𝑎𝑎𝑎𝑎𝑎𝑎
⇔ 𝑎𝑎𝑎𝑎(𝑎𝑎𝑎𝑎 + 𝑏𝑏𝑏𝑏) + 𝑎𝑎𝑎𝑎𝑎𝑎 + 𝑏𝑏𝑏𝑏𝑏𝑏 = (𝑎𝑎𝑎𝑎 + 𝑏𝑏𝑏𝑏)𝐼𝐼 ⇔ 𝑎𝑎𝑎𝑎(𝑎𝑎𝑎𝑎 + 𝑏𝑏𝑏𝑏) + 𝑐𝑐(𝑎𝑎𝑎𝑎 + 𝑏𝑏𝑏𝑏) = (𝑎𝑎𝑎𝑎 + 𝑏𝑏𝑏𝑏)𝐼𝐼
1
⇔ (𝑎𝑎𝑎𝑎 + 𝑐𝑐𝑐𝑐)(𝑎𝑎𝑎𝑎 + 𝑏𝑏𝑏𝑏) = (𝑎𝑎𝑎𝑎 + 𝑏𝑏𝑏𝑏)𝐼𝐼 ⇔ 𝑎𝑎𝑎𝑎+𝑏𝑏𝑏𝑏
(𝑎𝑎𝑎𝑎 + 𝑐𝑐𝑐𝑐)(𝑎𝑎𝑎𝑎 + 𝑏𝑏𝑏𝑏) = 𝐼𝐼
Suy ra 1
𝑎𝑎𝑎𝑎+𝑏𝑏𝑏𝑏
(𝑎𝑎𝑎𝑎 + 𝑐𝑐𝑐𝑐), (𝑎𝑎𝑎𝑎 + 𝑏𝑏𝑏𝑏) khả nghịch và là nghịch đảo của nhau nên
1
(𝑎𝑎𝑎𝑎 + 𝑏𝑏𝑏𝑏)(𝑎𝑎𝑎𝑎 + 𝑐𝑐𝑐𝑐) = 𝐼𝐼 ⇔. . . ⇔ 𝑎𝑎𝑎𝑎𝑎𝑎 + 𝑏𝑏𝑏𝑏 + 𝑐𝑐𝑐𝑐 = 𝑑𝑑𝑑𝑑
𝑎𝑎𝑎𝑎 + 𝑏𝑏𝑏𝑏
Vậy 𝐴𝐴𝐴𝐴 = 𝐵𝐵𝐵𝐵.

Bài 6.21. Cho 𝐴𝐴, 𝐵𝐵 là hai ma trận vuông cùng cấp thoả 𝑎𝑎𝑎𝑎𝑎𝑎 + 𝑏𝑏𝑏𝑏 + 𝑐𝑐𝑐𝑐 = 0 (𝑎𝑎𝑎𝑎𝑎𝑎 ≠ 0).
Chứng minh rằng
a) 𝐴𝐴𝐴𝐴 = 𝐵𝐵𝐵𝐵.
b) rank 𝐴𝐴 = rank 𝐵𝐵.

Chứng minh.

a) Ta có 𝑎𝑎𝑎𝑎𝑎𝑎 + 𝑏𝑏𝑏𝑏 + 𝑐𝑐𝑐𝑐 = 0


⇔ 𝐴𝐴(𝑎𝑎𝑎𝑎 + 𝑏𝑏𝑏𝑏) + 𝑐𝑐𝑐𝑐 = 0 ⇔ 𝑎𝑎𝑎𝑎(𝑎𝑎𝑎𝑎 + 𝑏𝑏𝑏𝑏) + 𝑎𝑎𝑎𝑎𝑎𝑎 = 0
⇔ 𝑎𝑎𝑎𝑎(𝑎𝑎𝑎𝑎 + 𝑏𝑏𝑏𝑏) + 𝑎𝑎𝑎𝑎𝑎𝑎 + 𝑏𝑏𝑏𝑏𝑏𝑏 = 𝑏𝑏𝑏𝑏𝑏𝑏 ⇔ 𝑎𝑎𝑎𝑎(𝑎𝑎𝑎𝑎 + 𝑏𝑏𝑏𝑏) + 𝑐𝑐(𝑎𝑎𝑎𝑎 + 𝑏𝑏𝑏𝑏) = 𝑏𝑏𝑏𝑏𝑏𝑏
⇔ (𝑎𝑎𝑎𝑎 + 𝑐𝑐𝑐𝑐)(𝑎𝑎𝑎𝑎 + 𝑏𝑏𝑏𝑏) = 𝑏𝑏𝑏𝑏𝑏𝑏 ⇔ (𝑎𝑎𝑐𝑐 𝐴𝐴 + 𝐼𝐼)(𝑎𝑎𝑏𝑏 𝐵𝐵 + 𝐼𝐼) = 𝐼𝐼
Suy ra (𝑎𝑎𝑐𝑐 𝐴𝐴 + 𝐼𝐼), (𝑎𝑎𝑏𝑏 𝐵𝐵 + 𝐼𝐼) khả nghịch và là nghịch đảo của nhau nên
𝑎𝑎 𝑎𝑎
� 𝐵𝐵 + 𝐼𝐼� � 𝐴𝐴 + 𝐼𝐼� = 𝐼𝐼 ⇔. . . ⇔ 𝑎𝑎𝑎𝑎𝑎𝑎 + 𝑏𝑏𝑏𝑏 + 𝑐𝑐𝑐𝑐 = 0
𝑏𝑏 𝑐𝑐
Vậy 𝐴𝐴𝐴𝐴 = 𝐵𝐵𝐵𝐵.
b) Ta có
𝑎𝑎
rank 𝐴𝐴 = rank(−𝑏𝑏𝑏𝑏) = rank(𝑎𝑎𝑎𝑎𝑎𝑎 + 𝑐𝑐𝑐𝑐) = rank[(𝑎𝑎𝑎𝑎 + 𝑐𝑐𝑐𝑐)𝐵𝐵] = rank �� 𝐴𝐴 + 𝐼𝐼� 𝐵𝐵� = rank 𝐵𝐵
𝑐𝑐

Bài 6.22. (Câu 3 Olympic 2009) Cho 𝐴𝐴, 𝐵𝐵, 𝐶𝐶 là các ma trận vuông cấp 𝑛𝑛 sao cho 𝐶𝐶 giao
hoán với 𝐴𝐴 và 𝐵𝐵, 𝐶𝐶 = 𝐼𝐼 và 𝐴𝐴𝐴𝐴 = 2(𝐴𝐴 + 𝐵𝐵)𝐶𝐶.
2

Chứng minh 𝐴𝐴𝐴𝐴 = 𝐵𝐵𝐵𝐵.


Nếu có thêm điều kiện 𝐴𝐴 + 𝐵𝐵 + 𝐶𝐶 = 0 hãy chứng tỏ rank(𝐴𝐴 − 𝐶𝐶) + rank(𝐵𝐵 − 𝐶𝐶) = 𝑛𝑛

Chứng minh.

a) Từ giả thiết, ta có:


𝐴𝐴𝐴𝐴 − 2𝐴𝐴𝐴𝐴 − 2𝐵𝐵𝐵𝐵 = 0 ⇔ 𝐴𝐴(𝐵𝐵 − 2𝐶𝐶) − 2𝐵𝐵𝐵𝐵 = 0 ⇔ 𝐴𝐴(𝐵𝐵 − 2𝐶𝐶) − 2𝐵𝐵𝐵𝐵 + 4𝐶𝐶 2 = 4𝐶𝐶 2

Đỗ Minh Triết VI-60 - MATHTASY -


Ôn thi Olympic Toán Đại số Phần VI: Hạng của Ma trận

⇔ 𝐴𝐴(𝐵𝐵 − 2𝐶𝐶) − 2𝐶𝐶(𝐵𝐵 − 2𝐶𝐶) = 4𝐼𝐼 ⇔ (𝐴𝐴 − 2𝐶𝐶)(𝐵𝐵 − 2𝐶𝐶) = 4𝐼𝐼 ⇔ �12 (𝐴𝐴 − 2𝐶𝐶)��12 (𝐵𝐵 − 2𝐶𝐶)� =
𝐼𝐼
Suy ra �12 (𝐴𝐴 − 2𝐶𝐶)� và �12 (𝐵𝐵 − 2𝐶𝐶)� là nghịch đảo của nhau nên chúng giao hoán
1 1 1 1
� (𝐴𝐴 − 2𝐶𝐶)� � (𝐵𝐵 − 2𝐶𝐶)� = � (𝐵𝐵 − 2𝐶𝐶)� � (𝐴𝐴 − 2𝐶𝐶)� = 𝐼𝐼
2 2 2 2
Nhân phân phối lại, ta được 𝐴𝐴𝐴𝐴 = 𝐵𝐵𝐵𝐵.
b) Nếu có thêm điều kiện 𝐴𝐴 + 𝐵𝐵 + 𝐶𝐶 = 0 thì ta có
𝐴𝐴𝐴𝐴 + 𝐵𝐵𝐵𝐵 + 𝐶𝐶 2 = 0
Do đó
(𝐴𝐴 − 𝐶𝐶)(𝐵𝐵 − 𝐶𝐶) = 𝐴𝐴𝐴𝐴 − 𝐴𝐴𝐴𝐴 − 𝐶𝐶𝐶𝐶 + 𝐶𝐶 2 = 𝐴𝐴𝐴𝐴 − (𝐴𝐴 + 𝐵𝐵)𝐶𝐶 + 𝐶𝐶 2
= 2(𝐴𝐴 + 𝐵𝐵)𝐶𝐶 − (𝐴𝐴 + 𝐵𝐵)𝐶𝐶 + 𝐶𝐶 2 = (𝐴𝐴 + 𝐵𝐵)𝐶𝐶 + 𝐶𝐶 2 = 𝐴𝐴𝐴𝐴 + 𝐵𝐵𝐵𝐵 + 𝐶𝐶 2 = 0
Suy ra
𝑛𝑛 = 𝑛𝑛 + rank[(𝐴𝐴 − 𝐶𝐶)(𝐵𝐵 − 𝐶𝐶)] ≥ rank(𝐴𝐴 − 𝐶𝐶) + rank(𝐵𝐵 − 𝐶𝐶) ≥ rank[(𝐴𝐴 − 𝐶𝐶) + (𝐵𝐵 − 𝐶𝐶)]
= rank(𝐴𝐴 + 𝐵𝐵 − 2𝐶𝐶) = rank(−3𝐶𝐶) = rank 𝐶𝐶 = 𝑛𝑛.

Bài 6.23. (Câu 2 Olympic Quốc tế ngày thứ nhất 2009)


Cho 𝐴𝐴, 𝐵𝐵, 𝐶𝐶 là các ma trận vuông cùng cấp, 𝐴𝐴 khả nghịch. Chứng minh rằng nếu (𝐴𝐴 − 𝐵𝐵)𝐶𝐶 = 𝐵𝐵𝐴𝐴−1
thì 𝐶𝐶(𝐴𝐴 − 𝐵𝐵) = 𝐴𝐴−1 𝐵𝐵.

Chứng minh.

Từ (𝐴𝐴 − 𝐵𝐵)𝐶𝐶 = 𝐵𝐵𝐴𝐴−1 , ta có (𝐴𝐴 − 𝐵𝐵)𝐶𝐶 − 𝐵𝐵𝐴𝐴−1 + 𝐴𝐴𝐴𝐴−1 = 𝐼𝐼 hay (𝐴𝐴 − 𝐵𝐵)(𝐶𝐶 + 𝐴𝐴−1 ) = 𝐼𝐼.
Suy ra (𝐴𝐴 − 𝐵𝐵) và (𝐶𝐶 + 𝐴𝐴−1 ) là nghịch đảo của nhau nên ta có (𝐶𝐶 + 𝐴𝐴−1 )(𝐴𝐴 − 𝐵𝐵) = 𝐼𝐼.
Vậy 𝐶𝐶(𝐴𝐴 − 𝐵𝐵) = 𝐴𝐴−1 𝐵𝐵 .

Đỗ Minh Triết VI-61 - MATHTASY -


PHẦN VII. GIÁ TRỊ RIÊNG, VECTOR RIÊNG (Eigenvalue,
Eigenvector)

7.1. Cơ sở lý thuyết

Số khuyết của một ma trận 𝐴𝐴 ∈ 𝑀𝑀𝑛𝑛 (𝕂𝕂) là 𝑛𝑛 − rank 𝐴𝐴. Nói cách khác, số khuyết của 𝐴𝐴 chính là
số chiều không gian nghiệm của hệ 𝐴𝐴𝐴𝐴 = 0 trong 𝕂𝕂𝑛𝑛 .
Giả sử 𝜆𝜆1 , … , 𝜆𝜆𝑚𝑚 là 𝑚𝑚 giá trị riêng phân biệt của 𝐴𝐴 và 𝜆𝜆𝑖𝑖 là nghiệm bội 𝑠𝑠𝑖𝑖 , 𝑠𝑠𝑖𝑖 còn gọi là bội đại
số (Algebraic Multiplicity) của 𝜆𝜆𝑖𝑖 , còn số khuyết 𝑘𝑘𝑖𝑖 của ma trận (𝐴𝐴 − 𝜆𝜆𝑖𝑖 𝐼𝐼) gọi là bội hình học
(Geometric Multiplicity) của 𝜆𝜆𝑖𝑖 , cũng chính là số chiều của không gian con riêng của 𝐴𝐴 ứng với
trị riêng 𝜆𝜆𝑖𝑖 (hay số chiều không gian nghiệm của hệ (𝐴𝐴 − 𝜆𝜆𝑖𝑖 𝐼𝐼)𝑥𝑥 = 0).
Ta luôn có ∀ 𝜆𝜆𝑖𝑖 thì 𝑘𝑘𝑖𝑖 ≤ 𝑠𝑠𝑖𝑖 tức là với mỗi trị riêng của 𝐴𝐴 thì bội hình học của nó không lớn hơn
bội đại số và hiển nhiên 𝑠𝑠1 +. . . 𝑠𝑠𝑛𝑛 = 𝑛𝑛.
Định lí: Điều kiện cần và đủ để 𝐴𝐴 chéo hoá (diagonalize, diagonalization) được là ∀ 𝑖𝑖 = 1, … , 𝑚𝑚
thì 𝑠𝑠𝑖𝑖 = 𝑘𝑘𝑖𝑖 nghĩa là mọi trị riêng của nó có bội hình học bằng bội đại số. Nói cách khác, 𝐴𝐴 chéo
hoá được nếu và chỉ nếu nó có đủ 𝑛𝑛 (bằng cấp của 𝐴𝐴) vector riêng độc lập tuyến tính.
Hệ quả: 𝐴𝐴 có đủ 𝑛𝑛 giá trị riêng phân biệt thì 𝐴𝐴 chéo hoá được.
Định lý: Giá trị riêng của một phép biến đổi tuyến tính của không gian vector 𝑛𝑛 chiều V trên
trường 𝕂𝕂 không phụ thuộc vào cơ sở.

Chứng minh.

Giả sử 𝐴𝐴 là ma trận của phép biến đổi tuyến tính 𝑓𝑓 đối với cơ sở 𝛼𝛼1 , … , 𝛼𝛼𝑛𝑛 (1) và với cơ sở mới
𝛽𝛽1 , … , 𝛽𝛽𝑛𝑛 (2), 𝑓𝑓 có ma trận là 𝐵𝐵. Khi đó 𝐵𝐵 = 𝑆𝑆 −1 𝐴𝐴𝐴𝐴 trong đó 𝑆𝑆 là ma trận chuyển từ cơ sở (1)
sang cơ sở (2). Ta có

|𝐵𝐵 − 𝜆𝜆𝜆𝜆| = |𝑆𝑆 −1 𝐴𝐴𝐴𝐴 − 𝜆𝜆𝑆𝑆 −1 𝑆𝑆| = |𝑆𝑆 −1 (𝐴𝐴 − 𝜆𝜆𝜆𝜆)𝑆𝑆| = |𝑆𝑆 −1 ||𝐴𝐴 − 𝜆𝜆𝜆𝜆||𝑆𝑆| = |𝐴𝐴 − 𝜆𝜆𝜆𝜆| (đpcm)

Hệ quả: Nếu hai ma trận 𝐴𝐴 và 𝐵𝐵 đồng dạng thì chúng có cùng đa thức đặc trưng.

Nhận xét: Điều ngược lại không đúng (𝑛𝑛 ≥ 2).


0 1
Ví dụ: Hai ma trận 𝐴𝐴 = 0, 𝐵𝐵 = � � không đồng dạng nhưng đa thức đặc trưng của chúng trùng nhau
0 0
|𝐴𝐴 − 𝜆𝜆𝜆𝜆| = |𝐵𝐵 − 𝜆𝜆𝜆𝜆| = 𝜆𝜆2

Dạng toàn phương (Quadratic form)


1. Định nghĩa: Dạng toàn phương 𝑛𝑛 biến 𝑥𝑥1 , … , 𝑥𝑥𝑛𝑛 là một hàm có dạng

Đỗ Minh Triết VII-62 - MATHTASY -


Ôn thi Olympic Toán Đại số Phần VII: Giá trị riêng, vector riêng

𝑛𝑛
𝑓𝑓(𝑥𝑥) = 𝑓𝑓(𝑥𝑥1 , … , 𝑥𝑥𝑛𝑛 ) = � 𝑎𝑎𝑖𝑖𝑖𝑖 𝑥𝑥𝑖𝑖 𝑥𝑥𝑗𝑗
𝑖𝑖,𝑗𝑗=1

= 𝑎𝑎11 𝑥𝑥21
+ 𝑎𝑎12 𝑥𝑥1 𝑥𝑥2 +. . . 𝑎𝑎1𝑛𝑛 𝑥𝑥1 𝑥𝑥𝑛𝑛 + 𝑎𝑎21 𝑥𝑥2 𝑥𝑥1 +. . . +𝑎𝑎2𝑛𝑛 𝑥𝑥2 𝑥𝑥𝑛𝑛 + 𝑎𝑎22 𝑥𝑥22 . . . +𝑎𝑎𝑛𝑛1 𝑥𝑥𝑛𝑛 𝑥𝑥1 +. . . +𝑎𝑎𝑛𝑛𝑛𝑛 𝑥𝑥2𝑛𝑛
𝑎𝑎11 … 𝑎𝑎1𝑛𝑛 𝑥𝑥1
trong đó 𝑎𝑎𝑖𝑖𝑖𝑖 = 𝑎𝑎𝑗𝑗𝑗𝑗 , vậy 𝑓𝑓(𝑥𝑥) = ( 1 𝑥𝑥 … 𝑥𝑥 𝑛𝑛 ) � ⋮ ⋱ ⋮ � � ⋮ � = 𝑥𝑥𝑇𝑇 𝐴𝐴𝐴𝐴 (𝐴𝐴 = 𝐴𝐴𝑇𝑇 )
𝑎𝑎𝑛𝑛1 … 𝑎𝑎𝑛𝑛𝑛𝑛 𝑥𝑥𝑛𝑛
Dạng chính tắc: là dạng không chứa các tích chéo 𝑥𝑥𝑖𝑖 𝑥𝑥𝑗𝑗 (𝑖𝑖 ≠ 𝑗𝑗), chỉ chứa 𝑥𝑥2𝑖𝑖 .

Dạng chuẩn: là dạng chính tắc mà hệ số của 𝑥𝑥2𝑖𝑖 là −1,0,1.

2. Đưa dạng toàn phương về dạng chính tắc (Canonic form, canonical form):
Phương pháp Lagrange:
- Nếu dạng toàn phương vừa có tích chéo vừa có bình phương.
Ví dụ:
𝑓𝑓(𝑥𝑥) = 𝑓𝑓(𝑥𝑥1 , 𝑥𝑥2 , 𝑥𝑥3 ) = 𝑥𝑥21 + 2𝑥𝑥22 − 7𝑥𝑥23 − 4𝑥𝑥1 𝑥𝑥2 + 8𝑥𝑥1 𝑥𝑥3
= 𝑥𝑥21 − 4𝑥𝑥1 𝑥𝑥2 + 8𝑥𝑥1 𝑥𝑥3 + 2𝑥𝑥22 − 7𝑥𝑥23 = (𝑥𝑥1 − 2𝑥𝑥2 + 4𝑥𝑥3 )2 − 2𝑥𝑥22 − 23𝑥𝑥23 + 16𝑥𝑥2 𝑥𝑥3
= (𝑥𝑥1 − 2𝑥𝑥2 + 4𝑥𝑥3 )2 − (2𝑥𝑥22 + 23𝑥𝑥23 − 16𝑥𝑥2 𝑥𝑥3 ) = (𝑥𝑥1 − 2𝑥𝑥2 + 4𝑥𝑥3 )2 − [2(𝑥𝑥2 − 4𝑥𝑥3 )2 − 9𝑥𝑥23 ]
= (𝑥𝑥1 − 2𝑥𝑥2 + 4𝑥𝑥3 )2 − 2(𝑥𝑥2 − 4𝑥𝑥3 )2 + 9𝑥𝑥23 = 𝑦𝑦12 − 2𝑦𝑦22 + 9𝑦𝑦32 = 𝑓𝑓(𝑦𝑦)
- Nếu dạng toàn phương chỉ chứa tích chéo, chẳng hạn 𝑎𝑎12 thì ta đổi biến
𝑥𝑥1 + 𝑥𝑥2 𝑥𝑥 − 𝑥𝑥2
𝑦𝑦1 = , 𝑦𝑦2 = 1 , 𝑦𝑦3 = 𝑥𝑥3 , … , 𝑦𝑦𝑛𝑛 = 𝑥𝑥𝑛𝑛
2 2
Ví dụ:
𝑓𝑓(𝑥𝑥) = 𝑓𝑓(𝑥𝑥1 , 𝑥𝑥2 , 𝑥𝑥3 ) = 𝑥𝑥1 𝑥𝑥2 + 𝑥𝑥2 𝑥𝑥3 + 𝑥𝑥3 𝑥𝑥1
Đổi biến
𝑥𝑥1 + 𝑥𝑥2 𝑥𝑥 − 𝑥𝑥2
𝑦𝑦1 = , 𝑦𝑦2 = 1 , 𝑦𝑦3 = 𝑥𝑥3 ⇒ 𝑥𝑥1 = 𝑦𝑦1 + 𝑦𝑦2 , 𝑥𝑥2 = 𝑦𝑦1 − 𝑦𝑦2
2 2
𝑓𝑓(𝑦𝑦) = 𝑓𝑓(𝑦𝑦1 , 𝑦𝑦2 , 𝑦𝑦3 ) = (𝑦𝑦1 + 𝑦𝑦2 )(𝑦𝑦1 − 𝑦𝑦2 ) + 𝑦𝑦3 (𝑦𝑦1 − 𝑦𝑦2 ) + 𝑦𝑦3 (𝑦𝑦1 + 𝑦𝑦2 )
= 𝑦𝑦12 − 𝑦𝑦22 + 2𝑦𝑦1 𝑦𝑦3 = (𝑦𝑦1 + 𝑦𝑦3 )2 − 𝑦𝑦22 − 𝑦𝑦32 = 𝑧𝑧12 − 𝑧𝑧22 − 𝑧𝑧32 = 𝑓𝑓(𝑧𝑧)
Phương pháp chéo hoá trực giao (Orthogonal Diagonalization):
Đổi biến 𝑥𝑥 = 𝑈𝑈𝑈𝑈, trong đó 𝑈𝑈 là ma trận Unita làm chéo hoá 𝐴𝐴, khi đó
𝑓𝑓(𝑥𝑥) = 𝑥𝑥𝑇𝑇 𝐴𝐴𝐴𝐴 = (𝑈𝑈𝑈𝑈)𝑇𝑇 𝐴𝐴(𝑈𝑈𝑈𝑈) = 𝑦𝑦 𝑇𝑇 𝑈𝑈 𝑇𝑇 𝐴𝐴𝐴𝐴𝐴𝐴 = 𝑦𝑦 𝑇𝑇 𝐵𝐵𝐵𝐵
trong đó 𝐵𝐵 = 𝑈𝑈 𝑇𝑇 𝐴𝐴𝐴𝐴 là ma trận chéo.
Ví dụ:
𝑓𝑓(𝑥𝑥) = 𝑓𝑓(𝑥𝑥1 , 𝑥𝑥2 , 𝑥𝑥3 ) = 2𝑥𝑥21 + 2𝑥𝑥22 + 2𝑥𝑥23 + 2𝑥𝑥1 𝑥𝑥2 + 2𝑥𝑥2 𝑥𝑥3 + 2𝑥𝑥3 𝑥𝑥1
2 1 1
𝐴𝐴 = �1 2 1� có đa thức đặc trưng 𝑃𝑃𝐴𝐴 (𝜆𝜆) = (4 − 𝜆𝜆)(1 − 𝜆𝜆)2
1 1 2

Đỗ Minh Triết VII-63 - MATHTASY -


Ôn thi Olympic Toán Đại số Phần VII: Giá trị riêng, vector riêng

*Cơ sở của không gian riêng 𝐸𝐸4 ứng với trị riêng 𝜆𝜆 = 4 là 𝐵𝐵 = {𝑋𝑋1 = (1,1,1)}
Cơ sở của không gian riêng 𝐸𝐸1 ứng với trị riêng 𝜆𝜆 = 1 là 𝐶𝐶 = {𝑋𝑋2 = (−1,1,0), 𝑋𝑋3 = (−1,0,1)}
Đặt 𝐴𝐴 = 𝐵𝐵 ∪ 𝐶𝐶 là một cơ sở của ℝ.
*Trực giao 𝐴𝐴:
𝐶𝐶 = {𝑋𝑋2 = (−1,1,0), 𝑋𝑋3 = (−1,0,1)}

𝑌𝑌2 = 𝑋𝑋2 = (−1,1,0) ⇒ �|𝑌𝑌2 |� = 2
𝑋𝑋3 𝑌𝑌2 −1 −1 3
𝑌𝑌3 = 𝑋𝑋3 − 2 𝑌𝑌2 = � , , 1� ⇒ �|𝑌𝑌3 |� = �
𝑌𝑌2 2 2 2
*Trực chuẩn 𝐴𝐴:
𝑌𝑌1 1 1 1
𝑍𝑍1 = = �√ , √ , √ �
�|𝑌𝑌1 |� 3 3 3
𝑌𝑌2 −1 1
𝑍𝑍2 = = �√ , √ , 0�
�|𝑌𝑌2 |� 2 2
𝑌𝑌3 −1 −1 2
𝑍𝑍3 = = �√ , √ , √ �
�|𝑌𝑌3 |� 6 6 6
√1 −1
√ −1

3 2 6

⎜ ⎞
Ma trận trực giao Unita 𝑈𝑈 làm chéo hoá 𝐴𝐴: 𝑈𝑈 = ⎜ 1 1 √ ⎟
−1
6⎟
√ √
⎜ 3 2 ⎟
1 √2
⎝√3 0 6⎠

4 0 0
𝑥𝑥 = 𝑈𝑈𝑈𝑈 ⇒ 𝑓𝑓(𝑦𝑦) = 𝑓𝑓(𝑦𝑦1 , 𝑦𝑦2 , 𝑦𝑦3 ) = 𝑦𝑦 𝑇𝑇 𝐵𝐵𝐵𝐵 = 𝑦𝑦 𝑇𝑇 �0 1 0� 𝑦𝑦 = 4𝑦𝑦12 + 𝑦𝑦22 + 𝑦𝑦32
0 0 1
3. Dạng toàn phương xác định dấu
Luật quán tính: Trong 2 dạng chính tắc bất kì của cùng một dạng toàn phương, số các hệ số
dương bằng nhau, số các hệ số âm bằng nhau.
Xác định dấu của dạng toàn phương 𝒇𝒇(𝒙𝒙):
Ta xét sau khi đã đưa dạng toàn phương 𝑓𝑓(𝑥𝑥) về dạng chính tắc 𝑓𝑓(𝑦𝑦):
- Xác định dương nếu 𝑓𝑓(𝑦𝑦) > 0 ∀𝑦𝑦 ≠ 0.
- Xác định âm (negative definite) nếu 𝑓𝑓(𝑦𝑦) < 0 ∀𝑦𝑦 ≠ 0.
- Bán xác định dương (semi positive definite) nếu 𝑓𝑓(𝑦𝑦) ≥ 0 ∀𝑦𝑦 ≠ 0, ∃𝑦𝑦0 ≠ 0: 𝑓𝑓(𝑦𝑦0 ) = 0.
- Bán xác định âm (semi negative definite) nếu 𝑓𝑓(𝑦𝑦) ≤ 0 ∀𝑦𝑦 ≠ 0, ∃𝑦𝑦0 ≠ 0: 𝑓𝑓(𝑦𝑦0 ) = 0.
- Không xác định dấu (indefinite) nếu ∃𝑦𝑦1 , 𝑦𝑦2 : 𝑓𝑓(𝑦𝑦1 ) > 0, 𝑓𝑓(𝑦𝑦2 ) < 0.
Ví dụ:
Xét các dạng toàn phương sau:
𝑓𝑓(𝑥𝑥) = 𝑥𝑥21 + 2𝑥𝑥22 + 4𝑥𝑥23 > 0 ∀𝑥𝑥 ≠ 0 ⇒ 𝑓𝑓(𝑥𝑥) xác định dương.

Đỗ Minh Triết VII-64 - MATHTASY -


Ôn thi Olympic Toán Đại số Phần VII: Giá trị riêng, vector riêng

𝑔𝑔(𝑥𝑥) = −𝑥𝑥21 − 2𝑥𝑥22 − 4𝑥𝑥23 < 0 ∀𝑥𝑥 ≠ 0 ⇒ 𝑔𝑔(𝑥𝑥) xác định âm.
ℎ(𝑥𝑥) = 𝑥𝑥21 + 4𝑥𝑥23 , ta thấy 𝑓𝑓(𝑥𝑥) ≥ 0 ∀𝑥𝑥 ≠ 0 và ∃𝑥𝑥 = (0,1,0) ≠ 0 , 𝑓𝑓(0,1,0) = 0 ⇒ ℎ(𝑥𝑥) bán xác
định dương.
𝑘𝑘(𝑥𝑥) = −𝑥𝑥21 − 4𝑥𝑥23 , ta thấy 𝑓𝑓(𝑥𝑥) ≤ 0 ∀𝑥𝑥 ≠ 0 và ∃𝑥𝑥 = (0,1,0) ≠ 0 , 𝑓𝑓(0,1,0) = 0 ⇒ 𝑘𝑘(𝑥𝑥) bán xác
định âm.
𝑙𝑙(𝑥𝑥) = 𝑥𝑥21 − 2𝑥𝑥22 + 4𝑥𝑥23 , ta có
(1,0,0) ≠ 0, (0,1,0) ≠ 0, 𝑓𝑓(1,0,0) = 1 > 0, 𝑓𝑓(0,1,0) = −2 < 0
⇒ 𝑙𝑙(𝑥𝑥) không xác định dấu.
Định lý:
Ngôn ngữ ánh xạ: Mọi dạng toàn phương hệ số thực, xác định dương đều có các giá trị riêng
𝜆𝜆𝑖𝑖 (𝑖𝑖 = 1, … , 𝑛𝑛) là số thực dương.
Ngôn ngữ ma trận: Mọi ma trận 𝐴𝐴 thực, đối xứng, xác định dương đều có các giá trị riêng
𝜆𝜆𝑖𝑖 (𝑖𝑖 = 1, … , 𝑛𝑛) là số thực dương. Khi đó hiển nhiên det 𝐴𝐴 = ∏𝑛𝑛𝑖𝑖=1 𝜆𝜆𝑖𝑖 > 0.

Định lý Sylvester: 𝐴𝐴 là ma trận đối xứng, xác định dương nếu và chỉ nếu mọi định thức con
chính của 𝐴𝐴 đều dương, xác định âm nếu các định thức con chính của nó đan dấu.
Ví dụ:
9 6 −5
1. 𝐴𝐴 = � 6 6 −1 � có các định thức con chính
−5 −1 6
9 6 −5
9 6
∆1 = 9 > 0, ∆2 = � � = 18 > 0, ∆3 = � 6 6 −1 � = 9 > 0
6 6
−5 −1 6
Vậy 𝐴𝐴 xác định dương.
1 2 0
2. 𝐴𝐴 = �2 −5 1� có các định thức con chính
0 1 2
1 2 0
1 2
∆1 = 1 > 0, ∆2 = � � = −9 < 0, ∆3 = �2 −5 1� = 19 > 0
2 −5
0 1 2
Vậy 𝐴𝐴 xác định âm.

Một số bài Toán cơ sở


Câu 1: Chứng minh rằng nếu 𝜆𝜆 là một giá trị riêng của 𝐴𝐴 thì
a) 𝜆𝜆𝑘𝑘 là giá trị riêng của 𝐴𝐴𝑘𝑘 .
b) 𝜆𝜆−1 là giá trị riêng của 𝐴𝐴−1 (nếu 𝐴𝐴 khả nghịch).

Đỗ Minh Triết VII-65 - MATHTASY -


Ôn thi Olympic Toán Đại số Phần VII: Giá trị riêng, vector riêng

c) Nếu 𝑓𝑓(𝑥𝑥) ∈ ℝ[𝑥𝑥] thì mọi giá trị riêng của 𝑓𝑓(𝐴𝐴) đều có dạng 𝑓𝑓(𝜆𝜆) với 𝜆𝜆 là giá trị riêng của 𝐴𝐴.
Hệ quả: Nếu 𝑓𝑓(𝑥𝑥) ∈ ℝ[𝑥𝑥] và 𝑓𝑓(𝐴𝐴) = 0 thì các trị riêng của 𝐴𝐴 đều là nghiệm của 𝑓𝑓(𝑥𝑥).

Câu 2: Cho 𝐴𝐴 là ma trận vuông cấp 𝑛𝑛


a) |𝐴𝐴| là tích tất cả các giá trị riêng của nó.
b) Nếu 𝐴𝐴 ∈ 𝑀𝑀𝑛𝑛 (ℤ) có trị riêng 𝜆𝜆 ∈ ℤ thì 𝜆𝜆 là ước của |𝐴𝐴|.
c) tr(𝐴𝐴) là tổng các trị riêng của 𝐴𝐴.

Chứng minh.

Giả sử đa thức đặc trưng của 𝐴𝐴 có dạng


𝑃𝑃𝐴𝐴 (𝜆𝜆) = |𝐴𝐴 − 𝜆𝜆𝜆𝜆| = (−1)𝑛𝑛 𝜆𝜆𝑛𝑛 + (−1)𝑛𝑛−1 𝑐𝑐𝑛𝑛−1 𝜆𝜆𝑛𝑛−1 +. . . −𝑐𝑐1 𝜆𝜆 + 𝑐𝑐0
a) Các giá trị riêng là nghiệm của đa thức đặc trưng nên theo định lý Viete, ta có
(−1)𝑛𝑛
𝜆𝜆1 … 𝜆𝜆𝑛𝑛 = 𝑐𝑐 = 𝑐𝑐0 = 𝑃𝑃𝐴𝐴 (0) = |𝐴𝐴|
(−1)𝑛𝑛 0
b) Nếu 𝐴𝐴 ∈ 𝑀𝑀𝑛𝑛 (ℤ) có trị riêng 𝜆𝜆 ∈ ℤ và theo trên, ta có
(−1)𝑛𝑛 𝜆𝜆𝑛𝑛 + (−1)𝑛𝑛−1 𝑐𝑐𝑛𝑛−1 𝜆𝜆𝑛𝑛−1 +. . . −𝑐𝑐1 𝜆𝜆 + det 𝐴𝐴 = 0
Suy ra 𝜆𝜆| det 𝐴𝐴.
c) Theo định lý Viete, ta có
(−1)𝑛𝑛−1 𝑐𝑐𝑛𝑛−1
𝜆𝜆1 + ⋯ + 𝜆𝜆𝑛𝑛 = − = 𝑐𝑐𝑛𝑛−1
(−1)𝑛𝑛
Mặt khác, trong khai triển |𝐴𝐴 − 𝜆𝜆𝜆𝜆| có chứa số hạng (𝑎𝑎11 − 𝜆𝜆) … (𝑎𝑎𝑛𝑛𝑛𝑛 − 𝜆𝜆), nhân phân phối ra thì
hệ số của 𝜆𝜆𝑛𝑛−1 là
(−1)𝑛𝑛−1 𝑐𝑐𝑛𝑛−1 = (−1)𝑛𝑛−1 (𝑎𝑎11 +. . . +𝑎𝑎𝑛𝑛𝑛𝑛 ) = (−1)𝑛𝑛−1 tr 𝐴𝐴
Từ đó 𝑐𝑐𝑛𝑛−1 = 𝜆𝜆1 + ⋯ + 𝜆𝜆𝑛𝑛 = tr 𝐴𝐴

Câu 3 (Định lý Hadamard) Cho 𝐴𝐴 = �𝑎𝑎𝑖𝑖𝑖𝑖 � ∈ 𝑀𝑀𝑛𝑛 (𝕂𝕂). Chứng minh 𝐴𝐴 khả nghịch
nếu một trong hai điều sau xảy ra:
𝑛𝑛
𝑎𝑎) |𝑎𝑎𝑖𝑖𝑖𝑖 | > ��𝑎𝑎𝑖𝑖𝑖𝑖 � ∀𝑖𝑖
𝑗𝑗=1
𝑗𝑗≠𝑖𝑖
= 1, … , 𝑛𝑛. Ta nói ma trận A có các phần tử chéo là trội trong cùng một hàng.
𝑛𝑛
𝑏𝑏) �𝑎𝑎𝑗𝑗𝑗𝑗 � > ��𝑎𝑎𝑖𝑖𝑖𝑖 � ∀𝑗𝑗
𝑖𝑖=1
𝑖𝑖≠𝑗𝑗
= 1, … , 𝑛𝑛. Ta nói ma trận A có các phần tử chéo là trội trong cùng một cột.

Đỗ Minh Triết VII-66 - MATHTASY -


Ôn thi Olympic Toán Đại số Phần VII: Giá trị riêng, vector riêng

Chứng minh.

Giả sử 𝐴𝐴 suy biến, tức là các cột của 𝐴𝐴, ký hiệu là 𝐶𝐶1 , … , 𝐶𝐶𝑛𝑛 , phụ thuộc tuyến tính, hay là tồn
tại một tổ hợp tuyến tính không tầm thường thoả mãn

𝑘𝑘1 𝐶𝐶1 +. . . +𝑘𝑘𝑛𝑛 𝐶𝐶𝑛𝑛 = 0 (𝑘𝑘𝑖𝑖 ∈ ℝ, 𝑖𝑖 = 1, … , 𝑛𝑛)


Đặt |𝑘𝑘𝑠𝑠 | = max{|𝑘𝑘1 |, … , |𝑘𝑘𝑛𝑛 |}, hiển nhiên |𝑘𝑘𝑠𝑠 | > 0. Xét dòng 𝑠𝑠 của ma trận 𝐴𝐴:

� 𝑛𝑛 � 𝑛𝑛 𝑛𝑛
|𝑘𝑘𝑠𝑠 ||𝑎𝑎𝑠𝑠𝑠𝑠 | = �� 𝑘𝑘𝑗𝑗 𝑎𝑎𝑠𝑠𝑠𝑠 � ≤ ��𝑘𝑘𝑗𝑗 ��𝑎𝑎𝑠𝑠𝑠𝑠 � ≤ �|𝑘𝑘𝑠𝑠 |�𝑎𝑎𝑠𝑠𝑠𝑠 �
� 𝑗𝑗=1 � 𝑗𝑗=1 𝑗𝑗=1
� 𝑗𝑗≠𝑠𝑠 � 𝑗𝑗≠𝑠𝑠 𝑗𝑗≠𝑠𝑠

Suy ra
𝑛𝑛
|𝑎𝑎𝑠𝑠𝑠𝑠 | ≤ ��𝑎𝑎𝑠𝑠𝑠𝑠 �
𝑗𝑗=1
𝑗𝑗≠𝑠𝑠

Mâu thuẫn giả thiết, do đó 𝐴𝐴 khả nghịch.

Chứng minh b) hoàn toàn tương tự với giả sử các hàng của 𝐴𝐴 phụ thuộc tuyến tính (hoặc lập
luận tương tự như trên cho ma trận 𝐴𝐴𝑇𝑇 ).

Câu 4 (Từ forum.mathscope.org) Cho 𝐴𝐴 = �𝑎𝑎𝑖𝑖𝑖𝑖 � ∈ 𝑀𝑀𝑛𝑛 (𝕂𝕂) và 𝜆𝜆 là giá trị riêng
bất kì của 𝐴𝐴. Chứng minh rằng luôn tồn tại một hàng 𝑠𝑠 (hoặc một cột 𝑚𝑚) (1 ≤ 𝑠𝑠, 𝑚𝑚 ≤ 𝑛𝑛) thoả
mãn

𝑛𝑛 ⎛ 𝑛𝑛 ⎞
|𝑎𝑎𝑠𝑠𝑠𝑠 − 𝜆𝜆| ≤ ��𝑎𝑎𝑠𝑠𝑠𝑠 � ⎜

⎜ hay |𝑎𝑎𝑚𝑚𝑚𝑚 − 𝜆𝜆| ≤ �|𝑎𝑎 𝑖𝑖𝑖𝑖 | ⎟


𝑗𝑗=1 𝑖𝑖=1
𝑗𝑗≠𝑠𝑠 ⎝ 𝑖𝑖≠𝑚𝑚 ⎠

Chứng minh.
𝑥𝑥1
Gọi 𝑥𝑥 = � ⋮ � ≠ 0 là một vector riêng ứng với giá trị riêng 𝜆𝜆 của 𝐴𝐴.
𝑥𝑥𝑛𝑛
Đặt |𝑥𝑥𝑠𝑠 | = max{|𝑥𝑥1 |, … , |𝑥𝑥𝑛𝑛 |}, hiển nhiên |𝑥𝑥𝑠𝑠 | > 0. Ta có 𝐴𝐴𝐴𝐴 = 𝜆𝜆𝜆𝜆, so sánh hàng 𝑠𝑠 hai vế

𝑎𝑎𝑠𝑠1 𝑥𝑥1 +. . . +𝑎𝑎𝑠𝑠,𝑠𝑠−1 𝑥𝑥𝑠𝑠−1 + 𝑎𝑎𝑠𝑠𝑠𝑠 𝑥𝑥𝑠𝑠 + 𝑎𝑎𝑠𝑠,𝑠𝑠+1 𝑥𝑥𝑠𝑠+1 . . . +𝑎𝑎𝑠𝑠𝑠𝑠 𝑥𝑥𝑛𝑛 = 𝜆𝜆𝑥𝑥𝑠𝑠

Suy ra

� 𝑛𝑛 � 𝑛𝑛 𝑛𝑛
|𝑎𝑎𝑠𝑠𝑠𝑠 − 𝜆𝜆||𝑥𝑥𝑠𝑠 | = �� 𝑎𝑎𝑠𝑠𝑠𝑠 𝑥𝑥𝑗𝑗 � ≤ ��𝑎𝑎𝑠𝑠𝑠𝑠 ��𝑥𝑥𝑗𝑗 � ≤ ��𝑎𝑎𝑠𝑠𝑠𝑠 �|𝑥𝑥𝑠𝑠 |
� 𝑗𝑗=1 � 𝑗𝑗=1 𝑗𝑗=1
� 𝑗𝑗≠𝑠𝑠 � 𝑗𝑗≠𝑠𝑠 𝑗𝑗≠𝑠𝑠

Đỗ Minh Triết VII-67 - MATHTASY -


Ôn thi Olympic Toán Đại số Phần VII: Giá trị riêng, vector riêng

Do đó
𝑛𝑛
|𝑎𝑎𝑠𝑠𝑠𝑠 − 𝜆𝜆| ≤ ��𝑎𝑎𝑠𝑠𝑠𝑠 �|𝑥𝑥𝑠𝑠 |
𝑗𝑗=1
𝑗𝑗≠𝑠𝑠

Trên đây là cách chứng minh trực tiếp cho bài Toán hoặc có thể áp dụng ngay định lý
Hadamard cho ma trận 𝐴𝐴 − 𝜆𝜆𝜆𝜆 suy biến, khi đó tồn tại một hàng 𝑠𝑠 (hoặc một cột 𝑚𝑚) thoả mãn
điều phải chứng minh.

CHÚ Ý: |𝑘𝑘| ở đây chỉ giá trị tuyệt đối khi 𝑘𝑘 ∈ ℝ, và là modun của 𝑘𝑘 khi 𝑘𝑘 ∈ ℂ.

Ta phát biểu lại định lý Hadamard bằng lời như sau:

Cho 𝐴𝐴 là ma trận vuông cấp 𝑛𝑛, nếu modun của mỗi phần tử ở đường chéo chính lớn hơn tổng
modun các phần tử còn lại trên cùng một dòng (hay cùng một cột) với phần tử chéo đó thì 𝐴𝐴
khả nghịch.

Ngược lại, nếu 𝐴𝐴 suy biến thì tồn tại một hàng (hay một cột) của 𝐴𝐴 sao cho modun của phần tử
chéo trên đó không lớn hơn tổng modun các phần tử còn lại trên cùng một dòng (hay cùng một
cột) đó.

7.2. Bài tập

Bài 7.1. (Câu 4 Olympic 2006) Cho 𝐴𝐴 là ma trận vuông cấp 𝑛𝑛 sao cho mỗi dòng của nó
chứa đúng hai phần tử khác 0, trong đó phần tử nằm ở đường chéo chính bằng 2006, phần tử còn lại
bằng 1. Chứng minh det 𝐴𝐴 > 0.

Chứng minh.

(Nếu chỉ chứng minh 𝐴𝐴 khả nghịch, tham khảo bài tập phần Một số kiến thức cơ bản về ma trận).
Giả sử 𝜆𝜆 là một giá trị riêng bất kì của 𝐴𝐴, khi đó 𝐴𝐴 − 𝜆𝜆𝜆𝜆 suy biến và theo định lý Hadamard, tồn
tại một hàng 𝑠𝑠 của 𝐴𝐴 sao cho
𝑛𝑛
|𝑎𝑎𝑠𝑠𝑠𝑠 − 𝜆𝜆| ≤ ��𝑎𝑎𝑠𝑠𝑠𝑠 � hay |𝜆𝜆 − 2006| ≤ 1
𝑗𝑗=1
𝑗𝑗≠𝑠𝑠

Suy ra 2005 ≤ 𝜆𝜆 ≤ 2007. Vậy mọi giá trị riêng của 𝐴𝐴 đều dương nên det 𝐴𝐴 > 0.

Bài 7.2. Cho 𝐴𝐴 = �𝑎𝑎𝑖𝑖𝑖𝑖 � ∈ 𝑀𝑀𝑛𝑛 (ℝ) có các phần tử không âm thoả mãn

Đỗ Minh Triết VII-68 - MATHTASY -


Ôn thi Olympic Toán Đại số Phần VII: Giá trị riêng, vector riêng

𝑛𝑛
� 𝑎𝑎𝑖𝑖𝑖𝑖 = 1 ∀𝑖𝑖 = 1, … , 𝑛𝑛
𝑗𝑗=1

Chứng minh mọi giá trị riêng của 𝐴𝐴 đều có giá trị tuyệt đối không lớn hơn 1.

Chứng minh.

Giả sử 𝜆𝜆 là một giá trị riêng bất kì của 𝐴𝐴, khi đó 𝐴𝐴 − 𝜆𝜆𝜆𝜆 suy biến và theo định lý Hadamard, tồn
tại một hàng 𝑠𝑠 của 𝐴𝐴 sao cho
𝑛𝑛
|𝑎𝑎𝑠𝑠𝑠𝑠 − 𝜆𝜆| ≤ ��𝑎𝑎𝑠𝑠𝑠𝑠 �
𝑗𝑗=1
𝑗𝑗≠𝑠𝑠

Do đó
𝑛𝑛 𝑛𝑛
|𝜆𝜆| = |𝜆𝜆 − 𝑎𝑎𝑠𝑠𝑠𝑠 + 𝑎𝑎𝑠𝑠𝑠𝑠 | ≤ |𝜆𝜆 − 𝑎𝑎𝑠𝑠𝑠𝑠 | + |𝑎𝑎𝑠𝑠𝑠𝑠 | ≤ ��𝑎𝑎𝑠𝑠𝑠𝑠 � + |𝑎𝑎𝑠𝑠𝑠𝑠 | = ��𝑎𝑎𝑠𝑠𝑠𝑠 � = 1
𝑗𝑗=1 𝑗𝑗=1
𝑗𝑗≠𝑠𝑠

Điều phải chứng minh.

Bài 7.3. Cho 𝐴𝐴 = �𝑎𝑎𝑖𝑖𝑖𝑖 � ∈ 𝑀𝑀𝑛𝑛 (ℝ). Chứng minh rằng giá trị tuyệt đối của một giá trị riêng bất
kì của 𝐴𝐴 không vượt quá tổng các giá trị tuyệt đối của các phần tử của 𝐴𝐴.

Chứng minh.

Giả sử 𝜆𝜆 là một giá trị riêng của 𝐴𝐴, vì |𝐴𝐴 − 𝜆𝜆𝜆𝜆| = 0 nên tồn tại 𝑐𝑐 = (𝑐𝑐1 … 𝑐𝑐𝑛𝑛 )𝑇𝑇 ∈ ℝ𝑛𝑛 , 𝑐𝑐 ≠ 0 sao
cho (𝐴𝐴 − 𝜆𝜆𝜆𝜆)𝑐𝑐 = 0. Giả sử |𝑐𝑐𝑘𝑘 | = max{|𝑐𝑐1 |, … , |𝑐𝑐𝑛𝑛 |}, hiển nhiên |𝑐𝑐𝑘𝑘 | > 0, ta có

𝑐𝑐1 𝑎𝑎𝑘𝑘1 +. . . +𝑐𝑐𝑘𝑘−1 𝑎𝑎𝑘𝑘,𝑘𝑘−1 + 𝑐𝑐𝑘𝑘 (𝑎𝑎𝑘𝑘𝑘𝑘 − 𝜆𝜆) + 𝑐𝑐𝑘𝑘+1 𝑎𝑎𝑘𝑘,𝑘𝑘+1 +. . . +𝑐𝑐𝑛𝑛 𝑎𝑎𝑘𝑘𝑘𝑘 = 0
𝑐𝑐1 𝑐𝑐 𝑐𝑐 𝑐𝑐
⇒ 𝑎𝑎𝑘𝑘1 +. . . + 𝑘𝑘−1 𝑎𝑎𝑘𝑘,𝑘𝑘−1 + (𝑎𝑎𝑘𝑘𝑘𝑘 − 𝜆𝜆) + 𝑘𝑘+1 𝑎𝑎𝑘𝑘,𝑘𝑘+1 +. . . + 𝑛𝑛 𝑎𝑎𝑘𝑘𝑘𝑘 = 0
𝑐𝑐𝑘𝑘 𝑐𝑐𝑘𝑘 𝑐𝑐𝑘𝑘 𝑐𝑐𝑘𝑘

Suy ra
𝑛𝑛
𝑐𝑐𝑗𝑗
𝜆𝜆 = 𝑎𝑎𝑘𝑘𝑘𝑘 + � 𝑎𝑎
𝑐𝑐 𝑘𝑘𝑘𝑘
𝑗𝑗=1 𝑘𝑘
𝑗𝑗≠𝑘𝑘

Từ đó

� 𝑛𝑛
𝑐𝑐𝑗𝑗 � 𝑛𝑛
𝑐𝑐𝑗𝑗 𝑛𝑛 𝑛𝑛
|𝜆𝜆| = �𝑎𝑎𝑘𝑘𝑘𝑘 + � 𝑎𝑎𝑘𝑘𝑘𝑘 � ≤ |𝑎𝑎𝑘𝑘𝑘𝑘 | + � � � �𝑎𝑎𝑘𝑘𝑘𝑘 � ≤ |𝑎𝑎𝑘𝑘𝑘𝑘 | + ��𝑎𝑎𝑘𝑘𝑘𝑘 � = ��𝑎𝑎𝑘𝑘𝑘𝑘 �
� 𝑐𝑐
𝑗𝑗=1 𝑘𝑘
� 𝑐𝑐
𝑗𝑗=1 𝑘𝑘 𝑗𝑗=1 𝑗𝑗=1
� 𝑗𝑗≠𝑘𝑘 � 𝑗𝑗≠𝑘𝑘 𝑗𝑗≠𝑘𝑘

Đỗ Minh Triết VII-69 - MATHTASY -


Ôn thi Olympic Toán Đại số Phần VII: Giá trị riêng, vector riêng

Bài 7.4. Cho 𝐴𝐴 = �𝑎𝑎𝑖𝑖𝑖𝑖 � ∈ 𝑀𝑀𝑛𝑛 (ℝ) với các phần tử đều dương sao cho tổng các phần tử trên
cùng một hàng bất kì của nó đều bằng 1. Chứng minh rằng 𝐴𝐴 không có bất kì giá trị riêng nào có trị
tuyệt đối lớn hơn 1.

Chứng minh.

Giả sử 𝜆𝜆 là một giá trị riêng của 𝐴𝐴 và 𝑥𝑥 = (𝑥𝑥1 … 𝑥𝑥𝑛𝑛 )𝑇𝑇 là một vector riêng tương ứng.

Gọi 𝑥𝑥𝑖𝑖 là phần tử của 𝑥𝑥 có trị tuyệt đối lớn nhất, ta có


𝑛𝑛
𝜆𝜆𝑥𝑥𝑖𝑖 = � 𝑎𝑎𝑖𝑖𝑖𝑖 𝑥𝑥𝑗𝑗
𝑗𝑗=1

Do đó
𝑛𝑛 𝑛𝑛
|𝜆𝜆||𝑥𝑥𝑖𝑖 | ≤ � 𝑎𝑎𝑖𝑖𝑖𝑖 �𝑥𝑥𝑗𝑗 � ≤ |𝑥𝑥𝑖𝑖 | � 𝑎𝑎𝑖𝑖𝑖𝑖 = |𝑥𝑥𝑖𝑖 |
𝑗𝑗=1 𝑗𝑗=1

Suy ra |𝜆𝜆| ≤ 1.

Bài 7.5. (Đề chọn đội tuyển ĐH Ngân hàng 2012)


Cho 𝐴𝐴 là ma trận vuông cấp 𝑛𝑛 có tổng các phần tử của cùng một cột bằng 1, 𝑥𝑥 = (𝑥𝑥1 … 𝑥𝑥𝑛𝑛 )𝑇𝑇 là
vector riêng của 𝐴𝐴 có tổng các phần tử khác 0. Chứng minh rằng giá trị riêng của 𝐴𝐴 ứng với 𝑥𝑥 bằng 1.
Chứng minh
Giả sử 𝜆𝜆 là giá trị riêng của 𝐴𝐴 ứng với vector 𝑥𝑥, tức là 𝐴𝐴𝐴𝐴 = 𝜆𝜆𝜆𝜆 hay (𝐴𝐴 − 𝜆𝜆𝜆𝜆)𝑥𝑥 = 0, ta có

𝑎𝑎11 − 𝜆𝜆 … 𝑎𝑎1𝑛𝑛 𝑥𝑥1 ⎧ (𝑎𝑎11 − 𝜆𝜆)𝑥𝑥1 + 𝑎𝑎12 𝑥𝑥2 +. . . +𝑎𝑎1𝑛𝑛 𝑥𝑥𝑛𝑛 = 0


� 𝑎𝑎 𝑥𝑥 + (𝑎𝑎 − 𝜆𝜆)𝑥𝑥 +. . . +𝑎𝑎 𝑥𝑥 = 0
� ⋮ ⋱ ⋮ � � ⋮ � = 0 ⇔ ⎨ 21 1 22 2 𝑛𝑛2 𝑛𝑛
𝑥𝑥𝑛𝑛 ⋮
𝑎𝑎𝑛𝑛1 … 𝑎𝑎𝑛𝑛𝑛𝑛 − 𝜆𝜆 �
⎩𝑎𝑎𝑛𝑛1 𝑥𝑥1 + 𝑎𝑎𝑛𝑛2 𝑥𝑥2 +. . . +(𝑎𝑎𝑛𝑛𝑛𝑛 − 𝜆𝜆)𝑥𝑥𝑛𝑛 = 0
Cộng theo vế tất cả các phương trình, ta được
(1 − 𝜆𝜆)(𝑥𝑥11 +. . . +𝑥𝑥1𝑛𝑛 ) = 0
Do 𝑥𝑥11 +. . . +𝑥𝑥1𝑛𝑛 ≠ 0 nên 𝜆𝜆 = 1.

Bài 7.6. Cho ma trận 𝐴𝐴 cấp 𝑛𝑛 ≥ 1 sau


2 −1 … 0 0

⎜ −1 2 … 0 0⎞⎟
𝐴𝐴 = �𝑎𝑎𝑖𝑖𝑖𝑖 � = ⎜

⎜ ⋮ ⋮ ⋱ ⋮ ⋮⎟


⎜ 0 0 … 2 −1 ⎟
⎝ 0 0 … −1 2⎠
Chứng minh mọi giá trị riêng của 𝐴𝐴 đều là các số thực dương.

Chứng minh.

Cách 1:

Đỗ Minh Triết VII-70 - MATHTASY -


Ôn thi Olympic Toán Đại số Phần VII: Giá trị riêng, vector riêng

Do 𝐴𝐴 là ma trận đối xứng nên 𝐴𝐴 có các giá trị riêng đều là số thực.
Giả sử 𝜆𝜆 là một giá trị riêng của 𝐴𝐴 và 𝑥𝑥 = (𝑥𝑥1 … 𝑥𝑥𝑛𝑛 ) là một vector riêng của 𝐴𝐴 ứng với 𝜆𝜆. Do
𝑥𝑥 ≠ 0 nên (|𝑥𝑥1 | … |𝑥𝑥𝑛𝑛 |) > 0. Gọi 1 ≤ 𝑘𝑘 ≤ 𝑛𝑛 ∈ là chỉ số nhỏ nhất sao cho |𝑥𝑥𝑘𝑘 | đạt cực đại, nghĩa
là 𝑘𝑘 = min�1, … , 𝑛𝑛�|𝑥𝑥𝑘𝑘 | ≥ �𝑥𝑥𝑗𝑗 � ∀ 𝑗𝑗 = 1, … , 𝑛𝑛�. Để ý rằng có thể giả thiết 𝑥𝑥𝑘𝑘 > 0 vì nếu ngược lại,
ta thay 𝑥𝑥 bởi – 𝑥𝑥. Xét phần tử ở hàng thứ 𝑘𝑘 của 𝐴𝐴𝐴𝐴, ta có
𝑎𝑎𝑘𝑘,𝑘𝑘−1 𝑥𝑥𝑘𝑘−1 + 𝑎𝑎𝑘𝑘𝑘𝑘 𝑥𝑥𝑘𝑘 + 𝑎𝑎𝑘𝑘,𝑘𝑘+1 𝑥𝑥𝑘𝑘+1 = −𝑥𝑥𝑘𝑘−1 + 2𝑥𝑥𝑘𝑘 − 𝑥𝑥𝑘𝑘+1 = 𝜆𝜆𝑥𝑥𝑘𝑘
Theo cách đã đặt ở trên thì 𝑥𝑥𝑘𝑘−1 < 𝑥𝑥𝑘𝑘 và 𝑥𝑥𝑘𝑘+1 ≤ 𝑥𝑥𝑘𝑘 nên 𝜆𝜆𝑥𝑥𝑘𝑘 > 0 và do đó 𝜆𝜆 > 0.

Cách 2:

𝐴𝐴 là ma trận đối xứng, để chứng minh mọi giá trị riêng của 𝐴𝐴 đều là các số thực dương, ta chứng
minh 𝐴𝐴 xác định dương, tức là ∀𝑥𝑥 ≠ 0: 𝑥𝑥𝑇𝑇 𝐴𝐴𝐴𝐴 > 0. Thật vậy
2 −1 … 0 0 𝑥𝑥1

⎜ −1 2 … 0 0⎞⎟⎛ 𝑥𝑥2 ⎞
𝑥𝑥𝑛𝑛 ) ⎜ ⎜ ⎟
𝑥𝑥𝑇𝑇 𝐴𝐴𝐴𝐴 = (𝑥𝑥1 𝑥𝑥2 … 𝑥𝑥𝑛𝑛−1 ⎜
⎜ ⋮ ⋮ ⋱ ⋮ ⋮⎟



⎜ ⋮ ⎟⎟
⎜ 0 ⎜ ⎟
0 … 2 −1 ⎟ 𝑥𝑥𝑛𝑛−1
⎝ 0 0 … −1 2⎠ ⎝ 𝑥𝑥𝑛𝑛 ⎠
𝑥𝑥1
⎛ 𝑥𝑥
⎜ 2 ⎞ ⎟
= (2𝑥𝑥1 − 𝑥𝑥2 −𝑥𝑥1 + 2𝑥𝑥2 − 𝑥𝑥3 … −𝑥𝑥𝑛𝑛−2 + 2𝑥𝑥𝑛𝑛−1 − 𝑥𝑥𝑛𝑛 −𝑥𝑥𝑛𝑛−1 + 2𝑥𝑥𝑛𝑛 ) ⎜
⎜ ⋮ ⎟
⎜𝑥𝑥𝑛𝑛−1 ⎟

⎝ 𝑥𝑥𝑛𝑛 ⎠
= (2𝑥𝑥1 − 𝑥𝑥2 )𝑥𝑥1 + (−𝑥𝑥1 + 2𝑥𝑥2 − 𝑥𝑥3 )𝑥𝑥2 +. . . +(−𝑥𝑥𝑛𝑛−2 + 2𝑥𝑥𝑛𝑛−1 − 𝑥𝑥𝑛𝑛 )𝑥𝑥𝑛𝑛−1 + (−𝑥𝑥𝑛𝑛−1 + 2𝑥𝑥𝑛𝑛 )𝑥𝑥𝑛𝑛
= 2(𝑥𝑥21 +. . . +𝑥𝑥2𝑛𝑛 ) − 2(𝑥𝑥1 𝑥𝑥2 + 𝑥𝑥2 𝑥𝑥3 +. . . +𝑥𝑥𝑛𝑛−1 𝑥𝑥𝑛𝑛 ) = (𝑥𝑥21 +. . . +𝑥𝑥2𝑛𝑛 ) + (𝑥𝑥1 −. . . −𝑥𝑥𝑛𝑛 )2 + 2𝑥𝑥𝑛𝑛 𝑥𝑥1
= (𝑥𝑥1 + 𝑥𝑥𝑛𝑛 )2 + (𝑥𝑥22 +. . . +𝑥𝑥2𝑛𝑛−1 ) + (𝑥𝑥1 −. . . −𝑥𝑥𝑛𝑛 )2 > 0 ∀𝑥𝑥 ≠ 0

Bài 7.7. Cho 𝐴𝐴 ∈ 𝑀𝑀𝑛𝑛 (ℂ) và 𝑓𝑓(𝑥𝑥) là một đa thức bậc 𝑚𝑚. Chứng minh rằng nếu 𝜆𝜆1 , … , 𝜆𝜆𝑛𝑛 là
các giá trị riêng của 𝐴𝐴 thì
a) |𝑓𝑓(𝐴𝐴)| = 𝑓𝑓(𝜆𝜆1 ) … 𝑓𝑓(𝜆𝜆𝑛𝑛 )
b) 𝑓𝑓(𝜆𝜆1 ), … , 𝑓𝑓(𝜆𝜆𝑛𝑛 ) là các giá trị riêng của 𝑓𝑓(𝐴𝐴).

Chứng minh.

a) Gọi 𝑝𝑝𝐴𝐴 (𝜆𝜆) = |𝐴𝐴 − 𝜆𝜆𝜆𝜆| là đa thức đặc trưng của 𝐴𝐴 và 𝑓𝑓(𝑥𝑥) là đa thức bậc 𝑚𝑚 có các nghiệm
𝑥𝑥1 , … , 𝑥𝑥𝑚𝑚 (thực, phức kể cả bội). Ta có

𝑝𝑝𝐴𝐴 (𝜆𝜆) = |𝐴𝐴 − 𝜆𝜆𝜆𝜆| = (−1)𝑛𝑛 (𝜆𝜆 − 𝜆𝜆1 ) … (𝜆𝜆 − 𝜆𝜆𝑛𝑛 )


𝑓𝑓(𝑥𝑥) = 𝑐𝑐(𝑥𝑥 − 𝑥𝑥1 ) … (𝑥𝑥 − 𝑥𝑥𝑚𝑚 )

Do đó

𝑓𝑓(𝐴𝐴) = 𝑐𝑐(𝐴𝐴 − 𝑥𝑥1 𝐼𝐼) … (𝐴𝐴 − 𝑥𝑥𝑚𝑚 𝐼𝐼)

Đỗ Minh Triết VII-71 - MATHTASY -


Ôn thi Olympic Toán Đại số Phần VII: Giá trị riêng, vector riêng

𝑚𝑚
⇒ |𝑓𝑓(𝐴𝐴)| = 𝑐𝑐𝑛𝑛 |𝐴𝐴 − 𝑥𝑥1 𝐼𝐼| … |𝐴𝐴 − 𝑥𝑥𝑚𝑚 𝐼𝐼| = 𝑐𝑐𝑛𝑛 � 𝑝𝑝𝐴𝐴 (𝑥𝑥𝑖𝑖 )
𝑖𝑖=1

Mặt khác
𝑛𝑛
𝑝𝑝𝐴𝐴 (𝑥𝑥𝑖𝑖 ) = (−1)𝑛𝑛 (𝑥𝑥𝑖𝑖 − 𝜆𝜆1 ) … (𝑥𝑥𝑖𝑖 − 𝜆𝜆𝑛𝑛 ) = ��𝜆𝜆𝑗𝑗 − 𝑥𝑥𝑖𝑖 �
𝑗𝑗=1

Vì vậy
𝑚𝑚 𝑚𝑚 𝑛𝑛 𝑛𝑛 𝑚𝑚 𝑛𝑛
|𝑓𝑓(𝐴𝐴)| = 𝑐𝑐𝑛𝑛 � 𝑝𝑝𝐴𝐴 (𝑥𝑥𝑖𝑖 ) = 𝑐𝑐𝑛𝑛 � ��𝜆𝜆𝑗𝑗 − 𝑥𝑥𝑖𝑖 � = � 𝑐𝑐 ��𝜆𝜆𝑗𝑗 − 𝑥𝑥𝑖𝑖 � = � 𝑓𝑓�𝜆𝜆𝑗𝑗 � = 𝑓𝑓(𝜆𝜆1 ) … 𝑓𝑓(𝜆𝜆𝑛𝑛 )
𝑖𝑖=1 𝑖𝑖=1 𝑗𝑗=1 𝑗𝑗=1 𝑖𝑖=1 𝑗𝑗=1

b) Đặt 𝑞𝑞(𝑥𝑥) = 𝑓𝑓(𝑥𝑥) − 𝜆𝜆 và áp dụng kết quả trên, ta có


𝑛𝑛
|𝑞𝑞(𝐴𝐴)| = |𝑓𝑓(𝐴𝐴) − 𝜆𝜆𝜆𝜆| = ��𝑓𝑓�𝜆𝜆𝑗𝑗 � − 𝜆𝜆𝜆𝜆� = [𝑓𝑓(𝜆𝜆1 ) − 𝜆𝜆] … [𝑓𝑓(𝜆𝜆𝑛𝑛 ) − 𝜆𝜆] = 𝑞𝑞(𝜆𝜆1 ) … 𝑞𝑞(𝜆𝜆𝑛𝑛 )
𝑗𝑗=1

hay

|𝑓𝑓(𝐴𝐴) − 𝜆𝜆𝜆𝜆| = (−1)𝑛𝑛 [𝜆𝜆 − 𝑓𝑓(𝜆𝜆1 )] … [𝜆𝜆 − 𝑓𝑓(𝜆𝜆𝑛𝑛 )]

Vậy các giá trị riêng của 𝑓𝑓(𝐴𝐴) là 𝑓𝑓(𝜆𝜆1 ), … , 𝑓𝑓(𝜆𝜆𝑛𝑛 ).

Bài 7.8. Cho 𝐴𝐴, 𝐵𝐵 ∈ 𝑀𝑀𝑛𝑛 (ℝ). Các khẳng định sau đúng hay sai, giải thích
a) Nếu 𝑛𝑛 lẻ thì 𝐴𝐴 luôn có vector riêng.
b) Mọi giá trị riêng của 𝐴𝐴 đều là của 𝐴𝐴𝑇𝑇 , tuy nhiên, các vector riêng thì không nhất thiết như vậy.
c) (Câu 2a Olympic 1999) Mọi giá trị riêng của 𝐴𝐴𝐴𝐴 cũng là giá trị riêng của 𝐵𝐵𝐵𝐵.
d) Mọi vector riêng của 𝐴𝐴𝐴𝐴 cũng là vector riêng của 𝐵𝐵𝐵𝐵.

Chứng minh.

a) Đúng. Do |𝐴𝐴 − 𝜆𝜆𝜆𝜆| là đa thức bậc lẻ nên có ít nhất một nghiệm thực nên 𝐴𝐴 luôn có vector
riêng.
1 1
b) Đúng. Do |𝐴𝐴𝑇𝑇 − 𝜆𝜆𝜆𝜆| = |(𝐴𝐴𝑇𝑇 − 𝜆𝜆𝜆𝜆)𝑇𝑇 | = |𝐴𝐴 − 𝜆𝜆𝜆𝜆|, có thể lấy 𝐴𝐴 = � �.
0 1
c) Đúng. Giả sử 𝑥𝑥 ≠ 0 là vector riêng của 𝐴𝐴𝐴𝐴 ứng với giá trị riêng 𝜆𝜆. Khi đó nếu 𝐵𝐵𝐵𝐵 ≠ 0 thì
𝐵𝐵𝐵𝐵(𝐵𝐵𝐵𝐵) = 𝐵𝐵(𝐴𝐴𝐴𝐴𝐴𝐴) = 𝐵𝐵(𝜆𝜆𝜆𝜆) = 𝜆𝜆(𝐵𝐵𝐵𝐵)
nghĩa là 𝜆𝜆 cũng là giá trị riêng của 𝐵𝐵𝐵𝐵 (ứng với vector riêng 𝐵𝐵𝐵𝐵).
Còn nếu 𝐵𝐵𝐵𝐵 = 0 thì từ 𝐴𝐴𝐴𝐴𝐴𝐴 = 𝜆𝜆𝜆𝜆 suy ra 𝜆𝜆 = 0 (vì 𝑥𝑥 ≠ 0). Do đó |𝐴𝐴𝐴𝐴| = 0 = |𝐵𝐵𝐵𝐵| tức là 𝐵𝐵𝐵𝐵
cũng nhận 𝜆𝜆 = 0 làm giá trị riêng.
1 1 1 1 1 2 2 2
d) Sai. Chẳng hạn lấy 𝐴𝐴 = � � , 𝐵𝐵 = � � thì 𝐴𝐴𝐴𝐴 = � � và 𝐵𝐵𝐵𝐵 = � �.
1 1 0 1 1 2 1 1

Đỗ Minh Triết VII-72 - MATHTASY -


Ôn thi Olympic Toán Đại số Phần VII: Giá trị riêng, vector riêng

1 1 1 2
Dễ thấy 𝐴𝐴𝐴𝐴 � � = 3 � � nhưng 𝐵𝐵𝐵𝐵 � � = 2 � �.
1 1 1 1

Bài 7.9. Cho ma trận 𝐴𝐴 thoả 𝐴𝐴2 = 𝐼𝐼. Chứng minh rằng
a) rank(𝐴𝐴 + 𝐼𝐼) + rank(𝐴𝐴 − 𝐼𝐼) = 𝑛𝑛
b) 𝐴𝐴 đồng dạng với một ma trận đường chéo gồm các phần tử là 1 hoặc -1.

Chứng minh.

a) 𝑛𝑛 = rank(2𝐼𝐼) = rank[(𝐴𝐴 + 𝐼𝐼) − (𝐴𝐴 − 𝐼𝐼)] ≤ rank(𝐴𝐴 + 𝐼𝐼) + rank(𝐴𝐴 − 𝐼𝐼) ≤ rank(𝐴𝐴2 − 𝐼𝐼 2 ) + 𝑛𝑛 =
𝑛𝑛
b) Do rank([ 𝐼𝐼 + 𝐴𝐴 𝐼𝐼 − 𝐴𝐴 ]) = rank([ 𝐼𝐼 𝐼𝐼 − 𝐴𝐴 ]) = rank 𝐼𝐼 = 𝑛𝑛 nên ta có thể lấy trong ma trận
[ 𝐼𝐼 + 𝐴𝐴 𝐼𝐼 − 𝐴𝐴 ] 𝑛𝑛 cột độc lập tuyến tính lập thành một ma trận không suy biến, ta đặt là 𝑃𝑃 .
Giả sử 𝑃𝑃 có 𝑟𝑟 cột đầu thuộc 𝐼𝐼 + 𝐴𝐴 và 𝑛𝑛 − 𝑟𝑟 cột sau thuộc 𝐼𝐼 − 𝐴𝐴. Do (𝐼𝐼 + 𝐴𝐴)(𝐼𝐼 − 𝐴𝐴) = 0 nên
(𝐼𝐼 + 𝐴𝐴)𝑃𝑃 có 𝑛𝑛 − 𝑟𝑟 cột sau bằng 0 và (𝐼𝐼 − 𝐴𝐴)𝑃𝑃 có 𝑟𝑟 cột đầu bằng 0.
𝑐𝑐11 … 𝑐𝑐1𝑟𝑟 0 … 0 0 … 0 𝑐𝑐1,𝑟𝑟+1 … 𝑐𝑐1𝑛𝑛
(𝐼𝐼 + 𝐴𝐴)𝑃𝑃 = � ⋮ ⋱ ⋮ ⋮ ⋱ ⋮ � và (𝐼𝐼 − 𝐴𝐴)𝑃𝑃 = � ⋮ ⋱ ⋮ ⋮ ⋱ ⋮ �
𝑐𝑐𝑛𝑛1 … 𝑐𝑐𝑛𝑛𝑛𝑛 0 … 0 0 … 0 𝑐𝑐𝑛𝑛,𝑟𝑟+1 … 𝑐𝑐𝑛𝑛𝑛𝑛
Cộng theo vế, ta được
𝑐𝑐11 … 𝑐𝑐1𝑟𝑟 𝑐𝑐1,𝑟𝑟+1 … 𝑐𝑐1𝑛𝑛
2𝑃𝑃 = � ⋮ ⋱ ⋮ ⋮ ⋱ ⋮ �
𝑐𝑐𝑛𝑛1 … 𝑐𝑐𝑛𝑛𝑛𝑛 𝑐𝑐𝑛𝑛,𝑟𝑟+1 … 𝑐𝑐𝑛𝑛𝑛𝑛
Trừ theo vế, ta được
𝑐𝑐11 … 𝑐𝑐1𝑟𝑟 −𝑐𝑐1,𝑟𝑟+1 … −𝑐𝑐1𝑛𝑛
2𝐴𝐴𝐴𝐴 = � ⋮ ⋱ ⋮ ⋮ ⋱ ⋮ �
𝑐𝑐𝑛𝑛1 … 𝑐𝑐𝑛𝑛𝑛𝑛 −𝑐𝑐𝑛𝑛,𝑟𝑟+1 … −𝑐𝑐𝑛𝑛𝑛𝑛
1 0 … 0 0
𝑐𝑐11 … 𝑐𝑐1𝑟𝑟 𝑐𝑐1,𝑟𝑟+1 … 𝑐𝑐1𝑛𝑛 ⎡0 1 … 0 0⎤ 𝐼𝐼 0
=� ⋮ ⋱ ⋮ ⋮ ⋱ ⋮ �⎢⎢⋮ ⋮ ⋱ 0 0⎥⎥ = 2𝑃𝑃 � 𝑟𝑟 �
… 𝑐𝑐𝑛𝑛𝑛𝑛 ⎢0 0 𝐼𝐼𝑛𝑛−𝑟𝑟
𝑐𝑐𝑛𝑛1 … 𝑐𝑐𝑛𝑛𝑛𝑛 𝑐𝑐𝑛𝑛,𝑟𝑟+1 0 … −1 0⎥
⎣0 0 … 0 −1 ⎦
𝐼𝐼 0
Từ đó ta có 𝑃𝑃 −1 𝐴𝐴𝐴𝐴 = � 𝑟𝑟 �.
0 𝐼𝐼𝑛𝑛−𝑟𝑟

Bài 7.10. Cho 𝑉𝑉 là không gian vector hữu hạn chiều trên trường số hữu tỉ ℚ, 𝑀𝑀 là một tự
đồng cấu của 𝑉𝑉 , 𝑀𝑀(𝑥𝑥) ≠ 𝑥𝑥 ∀𝑥𝑥 ∈ 𝑉𝑉 \{0}. Giả sử 𝑀𝑀 𝑝𝑝 = 𝐼𝐼𝑑𝑑𝑉𝑉 , với 𝑝𝑝 là số nguyên tố. Chứng minh rằng
số chiều của 𝑉𝑉 chia hết cho 𝑝𝑝 − 1.

Chứng minh.

Do 𝑀𝑀 𝑝𝑝 = 𝐼𝐼 nên nên đa thức tối tiểu 𝑞𝑞(𝑥𝑥) của 𝑀𝑀 phải là ước của

Đỗ Minh Triết VII-73 - MATHTASY -


Ôn thi Olympic Toán Đại số Phần VII: Giá trị riêng, vector riêng

𝑥𝑥𝑝𝑝 − 1 = (𝑥𝑥 − 1)(𝑥𝑥𝑝𝑝−1 +. . . +𝑥𝑥 + 1)


Do 𝑀𝑀 (𝑥𝑥) ≠ 𝑥𝑥 ∀𝑥𝑥 ∈ 𝑉𝑉 \{0} nên 1 không là giá trị riêng của 𝑀𝑀 suy ra 𝑞𝑞(𝑥𝑥) là ước của
(𝑥𝑥𝑝𝑝−1 +. . . +𝑥𝑥 + 1) . Nhưng (𝑥𝑥𝑝𝑝−1 +. . . +𝑥𝑥 + 1) là đa thức khả quy trên ℚ nên 𝑞𝑞(𝑥𝑥) =
𝑥𝑥𝑝𝑝−1 +. . . +𝑥𝑥 + 1.
Mặt khác, đa thức đặc trưng 𝑝𝑝(𝑥𝑥) và đa thức tối tiểu 𝑞𝑞(𝑥𝑥) có chung nhân tử bất khả quy. Do đó
𝑘𝑘
𝑝𝑝(𝑥𝑥) = �𝑞𝑞(𝑥𝑥)� , 𝑘𝑘 ≥ 1. Vậy dim 𝑉𝑉 = rank 𝑀𝑀 = deg 𝑝𝑝(𝑥𝑥) = 𝑘𝑘(𝑝𝑝 − 1).

Bài 7.11. Chứng minh rằng ma trận


1 1 + 𝑚𝑚 1
𝐴𝐴 = �1 + 𝑚𝑚 1 1 + 𝑚𝑚� (𝑚𝑚 > 0)
1 1 + 𝑚𝑚 1
có một giá trị riêng dương và một giá trị riêng âm.

Chứng minh.
1 − 𝜆𝜆 1 + 𝑚𝑚 1 −𝜆𝜆 0 𝜆𝜆
𝑃𝑃𝐴𝐴 (𝜆𝜆) = |𝐴𝐴 − 𝜆𝜆𝜆𝜆| = �1 + 𝑚𝑚 1 − 𝜆𝜆 1 + 𝑚𝑚� = � 𝑚𝑚 −𝜆𝜆 − 𝑚𝑚 𝑚𝑚 + 𝜆𝜆�
1 1 + 𝑚𝑚 1 − 𝜆𝜆 1 1 + 𝑚𝑚 1 − 𝜆𝜆
−1 0 1 0 0 1
= 𝜆𝜆 � 𝑚𝑚 −𝜆𝜆 − 𝑚𝑚 𝑚𝑚 + 𝜆𝜆� = 𝜆𝜆 �2𝑚𝑚 + 𝜆𝜆 −𝜆𝜆 − 𝑚𝑚 𝑚𝑚 + 𝜆𝜆�
1 1 + 𝑚𝑚 1 − 𝜆𝜆 2 − 𝜆𝜆 1 + 𝑚𝑚 1 − 𝜆𝜆
2𝑚𝑚 + 𝜆𝜆 −𝜆𝜆 − 𝑚𝑚 2(𝑚𝑚 + 1) 1 − 𝜆𝜆
= 𝜆𝜆 � � = 𝜆𝜆 � � = 𝜆𝜆[2(𝑚𝑚 + 1)2 − (𝜆𝜆 − 1)(𝜆𝜆 − 2)
2 − 𝜆𝜆 1 + 𝑚𝑚 2 − 𝜆𝜆 1 + 𝑚𝑚
= −𝜆𝜆[𝜆𝜆2 − 3𝜆𝜆 − 2𝑚𝑚(𝑚𝑚 + 2)]
Tam thức bậc hai 𝜆𝜆2 − 3𝜆𝜆 − 2𝑚𝑚(𝑚𝑚 + 2) có biệt thức ∆= 9 + 8𝑚𝑚(𝑚𝑚 + 2) > 0 do 𝑚𝑚 > 0 nên nó có
2 nghiệm phân biệt 𝜆𝜆1 , 𝜆𝜆2 .
Mặt khác, theo định lý Viete, ta có
𝜆𝜆1 𝜆𝜆2 = −2𝑚𝑚(𝑚𝑚 + 2) < 0 (do 𝑚𝑚 > 0)
Vậy 𝐴𝐴 có một giá trị riêng dương và một giá trị riêng âm.

𝑎𝑎 𝑏𝑏
Bài 7.12. Cho 𝐴𝐴 = � � là một ma trận thực với 𝑎𝑎, 𝑏𝑏, 𝑐𝑐, 𝑑𝑑 > 0. Chứng minh rằng 𝐴𝐴 có
𝑐𝑐 𝑑𝑑
𝑥𝑥
một vector riêng �𝑦𝑦 � ∈ ℝ2 với 𝑥𝑥, 𝑦𝑦 > 0.

Chứng minh.

Đa thức đặc trưng của 𝐴𝐴: 𝑃𝑃𝐴𝐴 (𝜆𝜆) = 𝜆𝜆2 − (𝑎𝑎 + 𝑑𝑑)𝜆𝜆 + (𝑎𝑎𝑎𝑎 − 𝑏𝑏𝑏𝑏) có nghiệm
1
𝜆𝜆1,2 = �𝑎𝑎 + 𝑑𝑑 ± �(𝑎𝑎 − 𝑑𝑑)2 + 4𝑏𝑏𝑏𝑏�
2
𝑥𝑥
Giả sử 𝑣𝑣 = �𝑦𝑦 � ≠ 0 là một vector riêng ứng với giá trị riêng 𝜆𝜆1 .

Đỗ Minh Triết VII-74 - MATHTASY -


Ôn thi Olympic Toán Đại số Phần VII: Giá trị riêng, vector riêng

- Trường hợp 𝑥𝑥 > 0, từ 𝐴𝐴𝐴𝐴 = 𝜆𝜆1 𝑣𝑣, ta có


1
𝑎𝑎𝑎𝑎 + 𝑏𝑏𝑏𝑏 = �𝑎𝑎 + 𝑑𝑑 + �(𝑎𝑎 − 𝑑𝑑)2 + 4𝑏𝑏𝑏𝑏�𝑥𝑥
2
⇔ 2𝑏𝑏𝑏𝑏 = �(𝑑𝑑 − 𝑎𝑎) + �(𝑎𝑎 − 𝑑𝑑)2 + 4𝑏𝑏𝑏𝑏�𝑥𝑥
Do 𝑏𝑏 > 0, 𝑥𝑥 > 0, (𝑑𝑑 − 𝑎𝑎) + �(𝑎𝑎 − 𝑑𝑑)2 + 4𝑏𝑏𝑏𝑏 > 0 nên 𝑦𝑦 > 0.
- Trường hợp 𝑥𝑥 < 0 thì (−𝑣𝑣) là cũng vector riêng ứng với giá trị riêng (−𝜆𝜆1 ), từ 𝐴𝐴(−𝑣𝑣) = (−𝜆𝜆1 )𝑣𝑣,
−𝑥𝑥 𝑥𝑥1
làm tương tự như trên, ta suy ra 𝑦𝑦 < 0. Vậy 𝐴𝐴 có vector riêng 𝑢𝑢 = −𝑣𝑣 = � −𝑦𝑦 � = �𝑥𝑥 � với
2
𝑥𝑥1 , 𝑦𝑦1 > 0.
- Dễ thấy, trường hợp 𝑥𝑥 = 0 dẫn đến 𝑦𝑦 = 0, điều này không thể xảy ra vì 𝑣𝑣 ≠ 0.
Điều phải chứng minh.

Bài 7.13. Cho 𝐴𝐴 là ma trận vuông cấp 𝑛𝑛. Chứng minh rằng 𝐴𝐴 = 𝑘𝑘𝑘𝑘 với 𝑘𝑘 là số nào đó nếu
và chỉ nếu mọi vector 𝑥𝑥 ≠ 0 đều là vector riêng của 𝐴𝐴.

Chứng minh.

Điều kiện cần: 𝐴𝐴𝐴𝐴 = 𝑘𝑘𝑘𝑘𝑘𝑘 = 𝑘𝑘𝑘𝑘 ∀ 𝑥𝑥 ≠ 0.


Điều kiện đủ: Lần lượt chọn 𝑥𝑥 = 𝑒𝑒𝑖𝑖 = (0, … ,0,1,0, … 0) là các phần tử trong cơ sở chính tắc của
ℝ𝑛𝑛 . Từ 𝐴𝐴𝑒𝑒𝑖𝑖 = 𝜆𝜆𝑖𝑖 𝑒𝑒𝑖𝑖 suy ra 𝐴𝐴 có phải dạng diag(𝜆𝜆1 , … , 𝜆𝜆𝑛𝑛 ). Hơn nữa 𝜆𝜆𝑖𝑖 𝑒𝑒𝑖𝑖 + 𝜆𝜆𝑗𝑗 𝑒𝑒𝑗𝑗 = 𝐴𝐴�𝑒𝑒𝑖𝑖 + 𝑒𝑒𝑗𝑗 � =
𝜆𝜆′ �𝑒𝑒𝑖𝑖 + 𝑒𝑒𝑗𝑗 � nên 𝜆𝜆𝑖𝑖 = 𝜆𝜆𝑗𝑗 = 𝜆𝜆′ ∀𝑖𝑖 ≠ 𝑗𝑗. Vậy 𝐴𝐴 = 𝜆𝜆𝜆𝜆.

Bài 7.14. Cho ma trận 𝐴𝐴 cấp 𝑛𝑛 có dạng


0 1 0 ⋯ 0

⎜0 0 1 ⋱ ⋮⎞

𝐴𝐴 = ⎜

⎜⋮ ⋱ ⋱ ⋱ 0⎟


⎜0 ⋯ ⋱ 0 1⎟
⎝𝑎𝑎 0 ⋯ 0 0⎠
Tìm các giá trị riêng của 𝐴𝐴.

Giải.

Đa thức đặc trưng của 𝐴𝐴


−𝜆𝜆 1 0 … 0
� 0 −𝜆𝜆 1 ⋱ ⋮�
|𝐴𝐴 − 𝜆𝜆𝜆𝜆| = �� ⋮ ⋱ ⋱ ⋱ 0�

� 0 ⋯ ⋱ −𝜆𝜆 1�
� 𝑎𝑎 0 ⋯ 0 −𝜆𝜆 �
Khai triển định thức theo cột thứ nhất, ta có

Đỗ Minh Triết VII-75 - MATHTASY -


Ôn thi Olympic Toán Đại số Phần VII: Giá trị riêng, vector riêng

−𝜆𝜆 1 0 … 0 1 0 0 … 0
� 0 −𝜆𝜆 1 ⋱ ⋮� �−𝜆𝜆 1 0 ⋱ ⋮�
−𝜆𝜆 �� ⋮ ⋱ ⋱ ⋱

0 � + (−1)𝑛𝑛+1 𝑎𝑎 �� ⋮ ⋱ ⋱ ⋱ 0�� = (−𝜆𝜆)𝑛𝑛 + (−1)𝑛𝑛+1 𝑎𝑎
� 0 ⋯ ⋱ −𝜆𝜆 1� � 0 ⋯ ⋱ 1 0�
� 0 0 ⋯ 0 −𝜆𝜆 � � 0 0 ⋯ −𝜆𝜆 1�
= (−1)𝑛𝑛 [𝜆𝜆𝑛𝑛 − 𝑎𝑎]
Suy ra |𝐴𝐴 − 𝜆𝜆𝜆𝜆| = 0 ⇔ 𝜆𝜆𝑛𝑛 = 𝑎𝑎.

*) Nếu 𝑛𝑛 lẻ thì 𝐴𝐴 có giá trị riêng duy nhất là 𝜆𝜆 = 𝑛𝑛
𝑎𝑎
*) Nếu 𝑛𝑛 chẵn và 𝑎𝑎 = 0 thì 𝐴𝐴 có giá trị riêng duy nhất là 𝜆𝜆 = 0

*) Nếu 𝑛𝑛 chẵn và 𝑎𝑎 > 0 thì 𝐴𝐴 có 2 giá trị riêng thực là 𝜆𝜆 = ± 𝑛𝑛 𝑎𝑎.

*) Nếu 𝑛𝑛 chẵn và 𝑎𝑎 < 0 thì 𝐴𝐴 có các giá trị riêng phức là 𝜆𝜆 = 𝑛𝑛 𝑎𝑎.

Bài 7.15. : Cho ma trận


0 0 … 0 1

⎜0 0 … 1 0⎞⎟
𝐴𝐴 = ⎜

⎜ ⋮ ⋮ ⋰ ⋮ ⋮⎟

⎜0 1 … 0 0⎟⎟
⎝1 0 … 0 0⎠
là ma trận có các phần tử trên đường chéo phụ bằng 1, các phần tử khác bằng 0.
Chứng minh rằng 𝐴𝐴 chéo hoá được và tìm ma trận khả nghịch 𝑃𝑃 thoả 𝑃𝑃 −1 𝐴𝐴𝐴𝐴 là ma trận đường chéo.

Chứng minh.
−𝜆𝜆 0 … 0 1
� 0 −𝜆𝜆 … 1 0�
Ta có |𝐴𝐴 − 𝜆𝜆𝜆𝜆| = �� ⋮ ⋮ ⋰ ⋮ ⋮ ��
� 0 1 … −𝜆𝜆 0�
� 1 0 … 0 −𝜆𝜆 �
Khai triển Laplace theo cột đầu và cột cuối, ta được
(𝜆𝜆 − 1)𝑘𝑘 . (𝜆𝜆 + 1)𝑘𝑘 𝑛𝑛 = 2𝑘𝑘
|𝐴𝐴 − 𝜆𝜆𝜆𝜆| = �
−(𝜆𝜆 − 1)𝑘𝑘+1 (𝜆𝜆 + 1)𝑘𝑘 𝑛𝑛 = 2𝑘𝑘 + 1
Như vậy 𝐴𝐴 chỉ có hai giá trị riêng là 1 và -1.
Với 𝜆𝜆 = 1, ta có hệ (𝐴𝐴 − 𝐼𝐼)𝑥𝑥 = 0
- Nếu 𝑛𝑛 = 2𝑘𝑘 thì
⎧−𝑥𝑥1 + 𝑥𝑥𝑛𝑛 =0
� −𝑥𝑥 + 𝑥𝑥 =0
2 𝑛𝑛−1
(𝐴𝐴 − 𝐼𝐼)𝑥𝑥 = 0 ⇔ ⇔ (𝑥𝑥1 , 𝑥𝑥2 , … , 𝑥𝑥𝑘𝑘 , 𝑥𝑥𝑘𝑘 , … , 𝑥𝑥2 , 𝑥𝑥1 )
⎨ ⋮

⎩ −𝑥𝑥𝑘𝑘 + 𝑥𝑥𝑛𝑛−𝑘𝑘+1 = 0
Ta được 𝑘𝑘 vector riêng tương ứng là
(1,0, … ,0,1), … , (0, … 1,1, … ,0)

Đỗ Minh Triết VII-76 - MATHTASY -


Ôn thi Olympic Toán Đại số Phần VII: Giá trị riêng, vector riêng

- Nếu 𝑛𝑛 = 2𝑘𝑘 + 1 thì


⎧ −𝑥𝑥1 + 𝑥𝑥𝑛𝑛 =0
� −𝑥𝑥2 + 𝑥𝑥𝑛𝑛−1 =0

(𝐴𝐴 − 𝐼𝐼)𝑥𝑥 = 0 ⇔ ⋮ ⇔ (𝑥𝑥1 , 𝑥𝑥2 , … , 𝑥𝑥𝑘𝑘 , 𝑥𝑥𝑘𝑘+1 , 𝑥𝑥𝑘𝑘 , … , 𝑥𝑥2 , 𝑥𝑥1 )

� −𝑥𝑥𝑘𝑘 + 𝑥𝑥𝑛𝑛−𝑘𝑘+1 = 0

⎩0𝑥𝑥𝑘𝑘+1 =0
Ta được 𝑘𝑘 + 1 vector riêng tương ứng là
(1,0, … ,0,1), … , (0, … 1,0,1, … ,0), (0, … ,0,1,0, … ,0)
Với 𝜆𝜆 = −1, ta có hệ (𝐴𝐴 + 𝐼𝐼)𝑥𝑥 = 0
- Nếu 𝑛𝑛 = 2𝑘𝑘 thì
⎧𝑥𝑥1 + 𝑥𝑥𝑛𝑛 =0
� 𝑥𝑥 + 𝑥𝑥 =0
(𝐴𝐴 + 𝐼𝐼)𝑥𝑥 = 0 ⇔ ⎨ 2 𝑛𝑛−1
⇔ (𝑥𝑥1 , 𝑥𝑥2 , … , 𝑥𝑥𝑘𝑘 , −𝑥𝑥𝑘𝑘 , … , −𝑥𝑥2 , −𝑥𝑥1 )


⎩ 𝑥𝑥𝑘𝑘 + 𝑥𝑥𝑛𝑛−𝑘𝑘+1 = 0
Ta được 𝑘𝑘 vector riêng tương ứng là
(1,0, … ,0, −1), … , (0, … 1, −1, … ,0)
- Nếu 𝑛𝑛 = 2𝑘𝑘 + 1 thì
⎧ 𝑥𝑥1 + 𝑥𝑥𝑛𝑛 =0
� 𝑥𝑥2 + 𝑥𝑥𝑛𝑛−1 =0

(𝐴𝐴 + 𝐼𝐼)𝑥𝑥 = 0 ⇔ ⋮ ⇔ (𝑥𝑥1 , 𝑥𝑥2 , … , 𝑥𝑥𝑘𝑘 , 0, −𝑥𝑥𝑘𝑘 , … , −𝑥𝑥2 , −𝑥𝑥1 )

� 𝑥𝑥𝑘𝑘 + 𝑥𝑥𝑛𝑛−𝑘𝑘+1 =0

⎩2𝑥𝑥𝑘𝑘+1 =0
Ta được 𝑘𝑘 vector riêng tương ứng là
(1,0, … ,0, −1), … , (0, … 1,0, −1, … ,0)
Vậy là ∀𝑛𝑛, ta đều tìm được đủ 𝑛𝑛 vector riêng độc lập tuyến tính ứng với 2 giá trị riêng là 1 và -
1. Theo tính Toán trên, 𝐴𝐴 sẽ đồng dạng với ma trận đường chéo gồm 𝑘𝑘 số -1 ở phía trên, còn lại
là 1. Ma trận 𝑃𝑃 cần tìm là
1 0 … 0 1

⎜ 0 1 … 1 0⎞

𝑃𝑃 = ⎜

⎜ ⋮ ⋮ ⋰ ⋮ ⋮⎟


⎜ 0 −1 … 1 0⎟
⎝ −1 0 … 0 1⎠
với chỉ 𝑘𝑘 số −1.

Bài 7.16. Tìm điều kiện cần và đủ để ma trận 𝐴𝐴 có các phần tử trên đường chéo phụ là 𝑎𝑎1 , … , 𝑎𝑎𝑛𝑛 , các
phần tử còn lại là 0 chéo hoá được.

Giải.

Đỗ Minh Triết VII-77 - MATHTASY -


Ôn thi Olympic Toán Đại số Phần VII: Giá trị riêng, vector riêng

−𝜆𝜆 0 … 0 𝑎𝑎1
� 0 −𝜆𝜆 … �
𝑎𝑎2 0
� �
Ta có |𝐴𝐴 − 𝜆𝜆𝜆𝜆| = � ⋮ ⋮ ⋰ ⋮ ⋮�
� 0 𝑎𝑎𝑛𝑛−1 … −𝜆𝜆 0�
� 𝑎𝑎𝑛𝑛 0 … 0 −𝜆𝜆 �
Khai triển Laplace theo cột đầu và cột cuối, ta được
𝑘𝑘

� �(𝜆𝜆2 − 𝑎𝑎𝑖𝑖 𝑎𝑎2𝑘𝑘+1−𝑖𝑖 ) vớ i 𝑛𝑛 = 2𝑘𝑘

𝑖𝑖=1
|𝐴𝐴 − 𝜆𝜆𝜆𝜆| =
⎨ 𝑘𝑘

�−(𝜆𝜆 − 𝑎𝑎𝑘𝑘+1 ) �(𝜆𝜆 − 𝑎𝑎𝑖𝑖 𝑎𝑎2𝑘𝑘+2−𝑖𝑖 ) vớ i 𝑛𝑛 = 2𝑘𝑘 + 1
2
⎩ 𝑖𝑖=1

Bài 7.17. Cho ma trận


2 −1 0
𝐴𝐴 = � −1 2 −1 �
0 −1 2
Chứng minh rằng mọi ma trận 𝐵𝐵 thực, giao hoán với 𝐴𝐴 đều có dạng
𝐵𝐵 = 𝑎𝑎𝑎𝑎 + 𝑏𝑏𝑏𝑏 + 𝑐𝑐𝐴𝐴2

Chứng minh.

Đa thức đặc trưng của 𝐴𝐴 là


√ √
𝑃𝑃𝐴𝐴 (𝑥𝑥) = (2 − 𝑥𝑥)�2 + 2 − 𝑥𝑥��2 − 2 − 𝑥𝑥�
𝐴𝐴 có 3 giá trị riêng phân biệt nên 𝐴𝐴 chéo hoá được. Giả sử 𝑃𝑃 là ma trận khả nghịch sao cho 𝐴𝐴 =
√ √
𝑃𝑃𝑃𝑃𝑃𝑃 −1 với 𝐷𝐷 = diag�2,2 + 2, 2 − 2�. Lại do 𝐵𝐵 giao hoán với 𝐴𝐴 nên 𝐵𝐵 cũng chéo hoá được với
cùng cơ sở, thật vậy
𝐵𝐵𝐵𝐵 = 𝐴𝐴𝐴𝐴 ⇔ 𝐵𝐵𝐵𝐵𝐵𝐵𝑃𝑃 −1 = 𝑃𝑃𝑃𝑃𝑃𝑃 −1 𝐵𝐵 ⇔ (𝑃𝑃 −1 𝐵𝐵𝐵𝐵 )𝐷𝐷 = 𝐷𝐷(𝑃𝑃 −1 𝐵𝐵𝐵𝐵 )
Đặt 𝑃𝑃 −1 𝐵𝐵𝑃𝑃 = diag(𝛼𝛼, 𝛽𝛽, 𝛾𝛾) = 𝐹𝐹 là ma trận đường chéo.

Cách 1:

Xét hệ phương trình tuyến tính


2
1 2√ 2√ 𝑎𝑎 𝛼𝛼
⎛ 2 + 2 �2 + 2�2 ⎞
𝑎𝑎𝑎𝑎 + 𝑏𝑏𝑏𝑏 + 𝑐𝑐𝐷𝐷 = 𝐹𝐹 ⇔ 1
2 ⎜ ⎟ � 𝑏𝑏 � = �𝛽𝛽 �
√ √ 𝛾𝛾
⎝1 2 − 2 �2 − 2�2 ⎠ 𝑐𝑐
Đây là hệ Crammer với ma trận hệ số là ma trận Vandermonde có định thức khác 0 nên hệ luôn
có nghiệm.
Nhân 𝑃𝑃 vào bên trái, 𝑃𝑃 −1 vào bên phải hai vế của hệ, ta có điều phải chứng minh.

Cách 2:
√ √
Xét đa thức bậc hai 𝑓𝑓(𝑥𝑥) = 𝑎𝑎 + 𝑏𝑏𝑏𝑏 + 𝑐𝑐𝑥𝑥2 , đặt 𝜆𝜆1 = 2, 𝜆𝜆2 = 2 + 2, 𝜆𝜆3 = 2 − 2, ta có

Đỗ Minh Triết VII-78 - MATHTASY -


Ôn thi Olympic Toán Đại số Phần VII: Giá trị riêng, vector riêng

𝑓𝑓(𝐴𝐴) = 𝑎𝑎𝑎𝑎 + 𝑏𝑏𝑏𝑏 + 𝑐𝑐𝐴𝐴2 = 𝑃𝑃 (𝑎𝑎𝑎𝑎 + 𝑏𝑏𝑏𝑏 + 𝑐𝑐𝐷𝐷2 )𝑃𝑃 −1


𝑎𝑎 + 𝑏𝑏𝜆𝜆1 + 𝑐𝑐𝜆𝜆21 0 0 𝑓𝑓(𝜆𝜆1 ) 0 0

= 𝑃𝑃 ⎜ ⎞
⎟ −1 ⎛ 0 ⎞
0 𝑎𝑎 + 𝑏𝑏𝜆𝜆2 + 𝑐𝑐𝜆𝜆22 0 𝑃𝑃 = 𝑃𝑃 ⎜ 0 𝑓𝑓(𝜆𝜆2 ) ⎟ 𝑃𝑃 −1
⎝ 0 0 𝑎𝑎 + 𝑏𝑏𝜆𝜆3 + 𝑐𝑐𝜆𝜆23 ⎠ ⎝ 0 0 𝑓𝑓(𝜆𝜆3 )⎠
Như vậy, ta cần phải tìm 𝑓𝑓(𝑥𝑥) thoả mãn 𝑓𝑓(𝜆𝜆1 ) = 𝛼𝛼, 𝑓𝑓(𝜆𝜆2 ) = 𝛽𝛽, 𝑓𝑓(𝜆𝜆3 ) = 𝛾𝛾, chẳng hạn chọn đa thức
nội suy Lagrange bậc hai 𝐿𝐿2 (𝑥𝑥):
(𝑥𝑥 − 𝜆𝜆2 )(𝑥𝑥 − 𝜆𝜆3 ) (𝑥𝑥 − 𝜆𝜆1 )(𝑥𝑥 − 𝜆𝜆3 ) (𝑥𝑥 − 𝜆𝜆1 )(𝑥𝑥 − 𝜆𝜆2 )
𝑓𝑓(𝑥𝑥) = 𝐿𝐿2 (𝑥𝑥) = 𝛼𝛼 + 𝛽𝛽 + 𝛾𝛾
(𝜆𝜆1 − 𝜆𝜆2 )(𝜆𝜆1 − 𝜆𝜆3 ) (𝜆𝜆2 − 𝜆𝜆1 )(𝜆𝜆2 − 𝜆𝜆3 ) (𝜆𝜆3 − 𝜆𝜆1 )(𝜆𝜆3 − 𝜆𝜆2 )
Khi đó
𝛼𝛼 0 0
𝑎𝑎𝑎𝑎 + 𝑏𝑏𝑏𝑏 + 𝑐𝑐𝐴𝐴2 = 𝑓𝑓(𝐴𝐴) = 𝑃𝑃 � 0 𝛽𝛽 0 � 𝑃𝑃 −1 = 𝑃𝑃𝑃𝑃 𝑃𝑃 −1 = 𝐵𝐵
0 0 𝛾𝛾

Cách 3:
𝛼𝛼 𝛽𝛽 𝛾𝛾
Đặt 𝐵𝐵 = ⎛
⎜ 𝛿𝛿 𝜖𝜖 𝜉𝜉 ⎞

⎝ 𝜂𝜂 𝜃𝜃 𝜄𝜄 ⎠
Từ 𝐴𝐴𝐴𝐴 = 𝐵𝐵𝐵𝐵 ta thu được hệ 9 phương trình 9 ẩn, giải ra ta có
𝛼𝛼 𝛽𝛽 𝛾𝛾

𝐵𝐵 = ⎜𝛽𝛽 𝛼𝛼 + 𝛾𝛾 𝛽𝛽 ⎞

⎝𝛾𝛾 𝛽𝛽 𝛼𝛼⎠
Tiếp tục giải hệ 𝐵𝐵 = 𝑎𝑎𝑎𝑎 + 𝑏𝑏𝑏𝑏 + 𝑐𝑐𝐴𝐴2 , ta có
𝑎𝑎 = 𝛼𝛼 + 2𝛽𝛽 + 3𝛾𝛾
� 𝑏𝑏 = −𝛽𝛽 − 4𝛾𝛾
𝑐𝑐 = 𝛾𝛾
Vậy 𝐵𝐵 = (𝛼𝛼 + 2𝛽𝛽 + 3𝛾𝛾)𝐼𝐼 + (−𝛽𝛽 − 4𝛾𝛾)𝐴𝐴 + 𝛾𝛾𝐴𝐴2 .

Bài 7.18. Cho 𝐴𝐴 ∈ 𝑀𝑀2 (ℂ). Với mọi số nguyên dương 𝑛𝑛, ký hiệu 𝑥𝑥𝑛𝑛 = det(𝐴𝐴𝑛𝑛 + 𝐼𝐼). Chứng
minh rằng nếu 𝑥𝑥1 = 𝑥𝑥2 = 1 thì 𝑥𝑥𝑛𝑛 hoặc bằng 1 hoặc bằng 4 ∀𝑛𝑛.

Chứng minh.

Đặt 𝑃𝑃𝐴𝐴 (𝜆𝜆) = det(𝐴𝐴 − 𝜆𝜆𝜆𝜆) = 𝜆𝜆2 − 𝑎𝑎𝑎𝑎 + 𝑏𝑏 là đa thức đặc trưng của 𝐴𝐴, trong đó 𝑎𝑎 = tr(𝐴𝐴), 𝑏𝑏 =
det 𝐴𝐴. Ta biết rằng 𝑃𝑃𝐴𝐴 (𝐴𝐴) = 𝑂𝑂.
Theo giả thiết, ta có 𝑥𝑥1 = det(𝐴𝐴 + 𝐼𝐼) = 𝑃𝑃𝐴𝐴 (−1) = 1 hay 𝑎𝑎 + 𝑏𝑏 = 0 và
1 = 𝑥𝑥2 = det(𝐴𝐴2 + 𝐼𝐼) = det(𝐴𝐴 + 𝑖𝑖𝑖𝑖) det(𝐴𝐴 − 𝑖𝑖𝑖𝑖) = 𝑃𝑃 (−𝑖𝑖)𝑃𝑃 (𝑖𝑖) = (−1 + 𝑎𝑎𝑎𝑎 + 𝑏𝑏)(−1 − 𝑎𝑎𝑎𝑎 + 𝑏𝑏)
= (𝑏𝑏 − 1)2 + 𝑎𝑎2
𝑎𝑎 = −𝑏𝑏
Hệ quả là � .
𝑎𝑎2 + 𝑏𝑏2 − 2𝑏𝑏 = 0

Đỗ Minh Triết VII-79 - MATHTASY -


Ôn thi Olympic Toán Đại số Phần VII: Giá trị riêng, vector riêng

Trường hợp 𝑎𝑎 = 𝑏𝑏 = 0 dẫn đến 𝐴𝐴2 = 0 nên 𝐴𝐴𝑛𝑛 = 0 ∀𝑛𝑛 ≥ 2 và do đó 𝑥𝑥𝑛𝑛 = 1 ∀𝑛𝑛.
Còn nếu 𝑎𝑎 = −1, 𝑏𝑏 = 1 thì khi đó 𝑃𝑃𝐴𝐴 (𝜆𝜆) = 𝜆𝜆2 + 𝜆𝜆 + 1 dẫn đến 𝐴𝐴2 + 𝐴𝐴 + 𝐼𝐼 = 0 nên 𝐴𝐴3 = 𝐼𝐼. Theo
quy nạp, ta có
+) 𝐴𝐴3𝑘𝑘 = 𝐼𝐼 và 𝑥𝑥3𝑘𝑘 = det(2𝐼𝐼) = 4.
+) 𝐴𝐴3𝑘𝑘+1 = 𝐴𝐴3𝑘𝑘 𝐴𝐴 = 𝐴𝐴 và 𝑥𝑥3𝑘𝑘+1 = det(𝐴𝐴 + 𝐼𝐼) = 𝑥𝑥1 = 1.
+) 𝐴𝐴3𝑘𝑘+2 = 𝐴𝐴3𝑘𝑘 𝐴𝐴2 = 𝐴𝐴2 và 𝑥𝑥3𝑘𝑘+2 = det(𝐴𝐴2 + 𝐼𝐼) = 𝑥𝑥2 = 1.
Tóm lại, 𝑥𝑥𝑛𝑛 ∈ {1,4} ∀𝑛𝑛.

Bài 7.19. Với 𝑥𝑥 ∈ ℝ, đặt


𝑥𝑥 1 1 1
⎡1 𝑥𝑥 1 1 ⎤
𝐴𝐴𝑥𝑥 = ⎢
1 1 𝑥𝑥 1 ⎥
⎣ 1 1 1 𝑥𝑥⎦
a) Chứng minh det 𝐴𝐴𝑥𝑥 = (𝑥𝑥 − 1)3 (𝑥𝑥 + 3).
b) Chứng minh nếu 𝑥𝑥 ≠ 1, 3 thì 𝐴𝐴−1
𝑥𝑥 = −(𝑥𝑥 − 1) (𝑥𝑥 + 3) 𝐴𝐴−𝑥𝑥−2 .
−1 −1

Chứng minh.

a) Tính toán trực tiếp cho ra kết quả.


b) Theo trên thì det(𝐴𝐴𝑥𝑥 − 𝜆𝜆𝜆𝜆) = det 𝐴𝐴𝑥𝑥−𝜆𝜆 = (𝑥𝑥 − 𝜆𝜆 − 1)3 (𝑥𝑥 − 𝜆𝜆 + 3).
Suy ra đa thức tối tiểu của 𝐴𝐴𝑥𝑥 là (𝑥𝑥 − 𝜆𝜆 − 1)(𝑥𝑥 − 𝜆𝜆 + 3)
Nếu 𝑥𝑥 ≠ 1, 3 thì 𝐴𝐴𝑥𝑥 khả nghịch và (𝑥𝑥 − 1)(𝑥𝑥 + 3) ≠ 0, do đó
0 = [(𝑥𝑥 − 1)𝐼𝐼 − 𝐴𝐴𝑥𝑥 ][(𝑥𝑥 + 3)𝐼𝐼 − 𝐴𝐴𝑥𝑥 ] = (𝑥𝑥 − 1)(𝑥𝑥 + 3)𝐼𝐼 − 2(𝑥𝑥 + 1)𝐴𝐴𝑥𝑥 + 𝐴𝐴2𝑥𝑥
⇔ (𝑥𝑥 − 1)(𝑥𝑥 + 3)𝐴𝐴−1
𝑥𝑥 − 2(𝑥𝑥 + 1)𝐼𝐼 + 𝐴𝐴𝑥𝑥 = 0

⇔ (𝑥𝑥 − 1)(𝑥𝑥 + 3)𝐴𝐴−1


𝑥𝑥 + [𝐴𝐴𝑥𝑥 − 2(𝑥𝑥 + 1)𝐼𝐼] = 0

⇔ (𝑥𝑥 − 1)(𝑥𝑥 + 3)𝐴𝐴−1


𝑥𝑥 + 𝐴𝐴−𝑥𝑥−2 = 0

𝑥𝑥 = −(𝑥𝑥 − 1) (𝑥𝑥 + 3) 𝐴𝐴−𝑥𝑥−2 .


⇔ 𝐴𝐴−1 −1 −1

Bài 7.20. Trong tất cả các ma trận đối xứng cấp hai 𝐴𝐴 = �𝑎𝑎𝑖𝑖𝑖𝑖 � có các giá trị riêng là 𝜆𝜆1 , 𝜆𝜆2 ,
tìm ma trận có phần tử 𝑎𝑎12 lớn nhất (nhỏ nhất).

Giải.
cos 𝑡𝑡 sin 𝑡𝑡
Từ giả thiết, 𝐴𝐴 có thể chéo hoá trực giao được bởi ma trận trực giao 𝑃𝑃 = � �, ta có
− sin 𝑡𝑡 cos 𝑡𝑡
cos 𝑡𝑡 sin 𝑡𝑡 𝜆𝜆1 0 cos 𝑡𝑡 − sin 𝑡𝑡
𝐴𝐴 = � �� �� �
− sin 𝑡𝑡 cos 𝑡𝑡 0 𝜆𝜆2 sin 𝑡𝑡 cos 𝑡𝑡

Đỗ Minh Triết VII-80 - MATHTASY -


Ôn thi Olympic Toán Đại số Phần VII: Giá trị riêng, vector riêng

Từ đó suy ra 𝑎𝑎12 = (𝜆𝜆2 − 𝜆𝜆1 ) cos 𝑡𝑡 sin 𝑡𝑡 = 12 (𝜆𝜆2 − 𝜆𝜆1 ) sin 2𝑡𝑡, vì vậy giá trị lớn nhất và nhỏ nhất mà
nó đạt được là
|𝜆𝜆2 − 𝜆𝜆1 | −|𝜆𝜆2 − 𝜆𝜆1 |
(𝑎𝑎12 )𝑚𝑚𝑚𝑚𝑚𝑚 = ; (𝑎𝑎12 )𝑚𝑚𝑚𝑚𝑚𝑚 =
2 2

Bài 7.21. Cho 𝐴𝐴 là một ma trận phức và 𝑘𝑘 là số tự nhiên. Chứng minh rằng tồn tại ma trận 𝑋𝑋 sao cho
𝑋𝑋 𝑘𝑘 = 𝐴𝐴.

Chứng minh.

Ta đã biết rằng mọi ma trận luôn chéo hoá được trên trường số phức. Chéo hoá 𝐴𝐴 trên ℂ, ta được
𝐴𝐴 = 𝑃𝑃𝑃𝑃𝑃𝑃 −1 , trong đó 𝐵𝐵 là ma trận đường chéo có các phần tử chéo là các giá trị riêng phức của
𝐴𝐴.
Chọn ma trận 𝑋𝑋 = 𝑃𝑃𝑃𝑃𝑃𝑃 −1 , trong đó 𝐶𝐶 là ma trận đường chéo có các phần tử chéo tương ứng là
một căn bậc 𝑘𝑘 của các phần tử chéo của ma trận 𝐵𝐵. Hiển nhiên khi đó 𝑃𝑃 𝐶𝐶 𝑘𝑘 𝑃𝑃 −1 = 𝑃𝑃𝑃𝑃𝑃𝑃 −1 = 𝐴𝐴.

Bài 7.22. (Câu 6 Olympic 1995) Cho 𝐵𝐵 là ma trận vuông cấp 𝑛𝑛 và 𝜆𝜆 là giá trị riêng thực của
𝐵𝐵. Chứng minh rằng với mọi 𝑎𝑎0 , 𝑎𝑎1 , … , 𝑎𝑎𝑛𝑛 ∈ ℝ, 𝑛𝑛 ∈ ℕ, ta đều có
𝑛𝑛 𝑛𝑛
det �� 𝑎𝑎𝑘𝑘 𝐵𝐵𝑘𝑘 − � 𝑎𝑎𝑘𝑘 𝜆𝜆𝑘𝑘 𝐸𝐸 � = 0
𝑘𝑘=0 𝑘𝑘=0

Chứng minh.

Ta có
𝑛𝑛 𝑛𝑛 𝑛𝑛
� 𝑎𝑎𝑘𝑘 𝐵𝐵𝑘𝑘 − � 𝑎𝑎𝑘𝑘 𝜆𝜆𝑘𝑘 𝐸𝐸 = � 𝑎𝑎𝑘𝑘 (𝐵𝐵𝑘𝑘 − 𝜆𝜆𝑘𝑘 𝐸𝐸)
𝑘𝑘=0 𝑘𝑘=0 𝑘𝑘=0
𝑛𝑛
= � 𝑎𝑎𝑘𝑘 (𝐵𝐵 − 𝜆𝜆𝐼𝐼)(𝐵𝐵𝑘𝑘−1 + 𝜆𝜆𝐵𝐵𝑘𝑘−2 +. . . +𝜆𝜆𝑘𝑘−2 𝐵𝐵 + 𝜆𝜆𝑘𝑘−1 𝐼𝐼)
𝑘𝑘=0
𝑛𝑛
= (𝐵𝐵 − 𝜆𝜆𝐼𝐼) � 𝑎𝑎𝑘𝑘 (𝐵𝐵𝑘𝑘−1 + 𝜆𝜆𝐵𝐵𝑘𝑘−2 +. . . +𝜆𝜆𝑘𝑘−2 𝐵𝐵 + 𝜆𝜆𝑘𝑘−1 𝐼𝐼) = (𝐵𝐵 − 𝜆𝜆𝐼𝐼)𝑀𝑀
𝑘𝑘=0

Suy ra
𝑛𝑛 𝑛𝑛
det �� 𝑎𝑎𝑘𝑘 𝐵𝐵𝑘𝑘 − � 𝑎𝑎𝑘𝑘 𝜆𝜆𝑘𝑘 𝐸𝐸 � = det(𝐵𝐵 − 𝜆𝜆𝐼𝐼) det 𝑀𝑀 = 0
𝑘𝑘=0 𝑘𝑘=0

Bài 7.23. (Câu 5 Olympic 2001) Cho 𝐴𝐴 là ma trận vuông cấp 𝑛𝑛 có các phần tử là những số
nguyên chẵn. Chứng minh 𝐴𝐴 không có giá trị riêng là số nguyên lẻ.

Chứng minh.

Đỗ Minh Triết VII-81 - MATHTASY -


Ôn thi Olympic Toán Đại số Phần VII: Giá trị riêng, vector riêng

Giả sử 𝐴𝐴 có giá trị riêng 𝜆𝜆 là số nguyên lẻ, khi đó |𝐴𝐴 − 𝜆𝜆𝜆𝜆| là một định thức mà các phần tử trên
đường chéo chính là các số nguyên lẻ, các phần tử còn lại là số nguyên chẵn. Khi đó |𝐴𝐴 − 𝜆𝜆𝜆𝜆| ≡
1 (mod 2) nên |𝐴𝐴 − 𝜆𝜆𝜆𝜆| ≠ 0, tức 𝜆𝜆 không phải là giá trị riêng của 𝐴𝐴, mâu thuẫn.
Vậy 𝐴𝐴 không có giá trị riêng là số nguyên lẻ.

Bài 7.24. (Câu 2 đề chọn đội tuyển vòng 1 ĐH An Giang 2009)


a) Cho 𝐴𝐴 ∈ 𝑀𝑀𝑛𝑛 (𝕂𝕂). Chứng minh rằng có không quá 𝑛𝑛 phần tử 𝛼𝛼 khác nhau trong 𝕂𝕂 sao cho
det(𝛼𝛼𝛼𝛼 − 𝐴𝐴) = 0.
b) Cho 𝐴𝐴, 𝐵𝐵 ∈ 𝑀𝑀𝑛𝑛 (𝕂𝕂). Chứng minh rằng nếu 𝐴𝐴 khả nghịch thì có không quá 𝑛𝑛 phần tử 𝛼𝛼 khác nhau
trong 𝕂𝕂 sao cho ma trận 𝛼𝛼𝛼𝛼 + 𝐵𝐵 suy biến.

Chứng minh.

a) Do đa thức det(𝑥𝑥𝑥𝑥 − 𝐴𝐴) khác 0 bậc không quá 𝑛𝑛 nên nó có không quá 𝑛𝑛 nghiệm trong 𝕂𝕂. Suy
ra điều phải chứng minh.
b) Do 𝐴𝐴 khả nghịch nên ta có
det(𝑥𝑥𝑥𝑥 + 𝐵𝐵) = 0 ⇔ det(𝑥𝑥𝑥𝑥 + 𝐵𝐵𝐴𝐴−1 ) det 𝐴𝐴 = 0 ⇔ det(𝑥𝑥𝑥𝑥 + 𝐵𝐵𝐴𝐴−1 ) = 0 ⇔ det[𝑥𝑥𝑥𝑥 − (−𝐵𝐵𝐴𝐴−1 )] =
0
Áp dụng câu a), suy ra điều phải chứng minh.

Bài 7.25. (Câu 3 Olympic 2008) Cho 𝐴𝐴 ∈ 𝑀𝑀2 (ℝ) thoả det 𝐴𝐴 < 0. Chứng minh tồn tại hai số
thực phân biệt 𝜆𝜆1 , 𝜆𝜆2 và hai ma trận 𝐴𝐴1 , 𝐴𝐴2 sao cho
𝐴𝐴𝑛𝑛 = 𝜆𝜆𝑛𝑛1 𝐴𝐴1 + 𝜆𝜆𝑛𝑛2 𝐴𝐴2 ∀𝑛𝑛 = 1,2, …

Chứng minh.

Đa thức đặc trưng của 𝐴𝐴 có dạng


𝑃𝑃𝐴𝐴 (𝜆𝜆) = det(𝐴𝐴 − 𝜆𝜆𝜆𝜆) = 𝜆𝜆2 − (tr 𝐴𝐴)𝜆𝜆 + det 𝐴𝐴

Cách 1:

Do det 𝐴𝐴 < 0 nên 𝑃𝑃𝐴𝐴 (𝜆𝜆) có biệt thức ∆= (tr 𝐴𝐴)2 − 4 det 𝐴𝐴 > 0 suy ra 𝑃𝑃𝐴𝐴 (𝜆𝜆) có hai nghiệm phân
biệt 𝜆𝜆1 , 𝜆𝜆2 , tức là 𝐴𝐴 chéo hoá được, và tồn tại ma trận 𝑃𝑃 khả nghịch sao cho
𝜆𝜆1 0
𝐴𝐴 = 𝑃𝑃 � � 𝑃𝑃 −1
0 𝜆𝜆2
Suy ra
𝜆𝜆𝑛𝑛 0 𝜆𝜆𝑛𝑛 0 0 0 𝜆𝜆𝑛𝑛 0 0 0
𝐴𝐴𝑛𝑛 = 𝑃𝑃 � 1 𝑛𝑛 � 𝑃𝑃
−1
= 𝑃𝑃 �� 1 �+� 𝑛𝑛 �� 𝑃𝑃
−1
= 𝑃𝑃 � 1 � 𝑃𝑃 −1 + 𝑃𝑃 � � 𝑃𝑃 −1
0 𝜆𝜆2 0 0 0 𝜆𝜆2 0 0 0 𝜆𝜆𝑛𝑛2
1 0 0 0
= 𝜆𝜆𝑛𝑛1 𝑃𝑃 � � 𝑃𝑃 −1 + 𝜆𝜆𝑛𝑛2 𝑃𝑃 � � 𝑃𝑃 −1
0 0 0 1

Đỗ Minh Triết VII-82 - MATHTASY -


Ôn thi Olympic Toán Đại số Phần VII: Giá trị riêng, vector riêng

1 0 0 0
Đặt 𝐴𝐴1 = 𝑃𝑃 � � 𝑃𝑃 −1 , 𝐴𝐴2 = 𝑃𝑃 � � 𝑃𝑃 −1 , ta có điều phải chứng minh.
0 0 0 1
Cách 2:

Do det 𝐴𝐴 < 0 nên Δ > 0 suy ra phương trình có hai nghiệm thực phân biệt 𝜆𝜆1 , 𝜆𝜆2 . Khi đó, đặt
1 1
𝐵𝐵1 = (𝐴𝐴 − 𝜆𝜆2 𝐼𝐼), 𝐵𝐵2 = (𝐴𝐴 − 𝜆𝜆1 𝐼𝐼)
𝜆𝜆1 − 𝜆𝜆2 𝜆𝜆2 − 𝜆𝜆1
thì ta có 𝐵𝐵1 + 𝐵𝐵2 = 𝐼𝐼, 𝜆𝜆1 𝐵𝐵1 + 𝜆𝜆2 𝐵𝐵2 = 𝐴𝐴, 𝐵𝐵1 𝐵𝐵2 = 𝐵𝐵2 𝐵𝐵1 = 0
Vậy 𝐴𝐴𝑛𝑛 = 𝜆𝜆𝑛𝑛1 𝐵𝐵1𝑛𝑛 + 𝜆𝜆𝑛𝑛2 𝐵𝐵2𝑛𝑛 = 𝜆𝜆𝑛𝑛1 𝐴𝐴1 + 𝜆𝜆𝑛𝑛2 𝐴𝐴2 , ∀𝑛𝑛 = 1,2, …
Nhận xét:
Thực ra, nếu đặt ngay
1 1
𝐴𝐴1 = (𝐴𝐴 − 𝜆𝜆2 𝐼𝐼), 𝐴𝐴2 = (𝐴𝐴 − 𝜆𝜆1 𝐼𝐼)
𝜆𝜆1 − 𝜆𝜆2 𝜆𝜆2 − 𝜆𝜆1
thì ta có 𝐴𝐴𝑛𝑛1 = 𝐴𝐴1 , 𝐴𝐴𝑛𝑛2 = 𝐴𝐴2
Thật vậy
⎧det𝐴𝐴1 = 0

𝑎𝑎 − 𝜆𝜆2 𝑑𝑑 − 𝜆𝜆2 𝑎𝑎 + 𝑑𝑑 − 2𝜆𝜆2 𝜆𝜆1 + 𝜆𝜆2 − 2𝜆𝜆2

� tr 𝐴𝐴1 = + = = =1
⎩ 𝜆𝜆1 − 𝜆𝜆2 𝜆𝜆1 − 𝜆𝜆2 𝜆𝜆1 − 𝜆𝜆2 𝜆𝜆1 − 𝜆𝜆2
Áp dụng định lý Cayley – Hamilton cho ma trận 𝐴𝐴1 , ta có 𝐴𝐴21 = 𝐴𝐴1 , suy ra 𝐴𝐴𝑛𝑛1 = 𝐴𝐴1 .
Cách khác: 𝐴𝐴21 − 𝐴𝐴1 = 𝐴𝐴1 (𝐴𝐴1 − 𝐼𝐼) = 𝐴𝐴1 (−𝐴𝐴2 ) = 0, suy ra 𝐴𝐴𝑛𝑛1 = 𝐴𝐴1
Cách khác:
𝐴𝐴𝑛𝑛1 − 𝐴𝐴1 = 𝐴𝐴1 (𝐴𝐴𝑛𝑛−1
1 − 𝐼𝐼) = (𝐼𝐼 − 𝐴𝐴2 )(𝐴𝐴𝑛𝑛−1
1 − 𝐼𝐼) = 𝐴𝐴𝑛𝑛−1
1 − 𝐼𝐼 − 𝐴𝐴2 𝐴𝐴𝑛𝑛−1
1 + 𝐴𝐴2 = 𝐴𝐴𝑛𝑛−1
1 − (𝐼𝐼 − 𝐴𝐴2 )
𝑛𝑛−1 2
= 𝐴𝐴1 − 𝐴𝐴1 =. . . = 𝐴𝐴1 − 𝐴𝐴1 = 𝐴𝐴1 (𝐴𝐴1 − 𝐼𝐼) = 𝐴𝐴1 (−𝐴𝐴2 ) = 0
Lý luận tương tự ta được 𝐴𝐴𝑛𝑛2 = 𝐴𝐴2 .
Vậy 𝐴𝐴𝑛𝑛 = 𝜆𝜆𝑛𝑛1 𝐴𝐴1 + 𝜆𝜆𝑛𝑛2 𝐴𝐴2 ∀𝑛𝑛 = 1,2, …

Bài 7.26. Cho ma trận 𝐴𝐴 có 𝑛𝑛 giá trị riêng dương phân biệt 𝜆𝜆1 , … , 𝜆𝜆𝑛𝑛 . Chứng minh rằng
det(𝐴𝐴 + 𝐴𝐴−1 ) ≥ 2𝑛𝑛 .

Chứng minh.

𝐴𝐴 có 𝑛𝑛 giá trị riêng dương phân biệt, đặt là 𝜆𝜆1 , … , 𝜆𝜆𝑛𝑛 , nên 𝐴𝐴 chéo hoá được, tức là tồn tại ma
trận 𝑇𝑇 khả nghịch sao cho
𝐴𝐴 = 𝑇𝑇 diag(𝜆𝜆1 , … , 𝜆𝜆𝑛𝑛 )𝑇𝑇 −1
Dễ dàng suy ra
1 1
𝐴𝐴−1 = 𝑇𝑇 diag � , … , � 𝑇𝑇 −1
𝜆𝜆1 𝜆𝜆𝑛𝑛

Đỗ Minh Triết VII-83 - MATHTASY -


Ôn thi Olympic Toán Đại số Phần VII: Giá trị riêng, vector riêng

Từ đó và áp dụng bất đẳng thức Cauchy, ta được


1 1
det(𝐴𝐴 + 𝐴𝐴−1 ) = det �𝑇𝑇 diag(𝜆𝜆1 , … , 𝜆𝜆𝑛𝑛 )𝑇𝑇 −1 + 𝑇𝑇diag � , … , � 𝑇𝑇 −1 �
𝜆𝜆1 𝜆𝜆𝑛𝑛
1 1
= det 𝑇𝑇 det �diag(𝜆𝜆1 , … , 𝜆𝜆𝑛𝑛 ) + diag � , … , �� det(𝑇𝑇 −1 )
𝜆𝜆1 𝜆𝜆𝑛𝑛
𝑛𝑛
1 1 1
= det �diag �𝜆𝜆1 + , … , 𝜆𝜆𝑛𝑛 + �� = � �𝜆𝜆𝑖𝑖 + � ≥ 2𝑛𝑛
𝜆𝜆1 𝜆𝜆𝑛𝑛 𝑖𝑖=1
𝜆𝜆 𝑖𝑖

Bài 7.27. (Câu 5 Olympic 2008)


Cho 𝐴𝐴 là ma trận vuông cấp 𝑛𝑛 khả nghịch. Mọi phần tử của các ma trận 𝐴𝐴, 𝐴𝐴−1 là số nguyên. Chứng
minh rằng nếu 𝐴𝐴 có 𝑛𝑛 giá trị riêng đều là các số thực thì
|det(𝐴𝐴 + 𝐴𝐴−1 )| ≥ 2𝑛𝑛

Chứng minh.

Do các phần tử của 𝐴𝐴, 𝐴𝐴−1 đều là số nguyên nên det 𝐴𝐴 , det 𝐴𝐴−1 cũng là số nguyên. Mặt khác
|det 𝐴𝐴||det 𝐴𝐴−1 | = |det 𝐴𝐴 det 𝐴𝐴−1 | = 1
Suy ra |det 𝐴𝐴| = |det 𝐴𝐴−1 | = 1
Với mỗi giá trị riêng 𝜆𝜆 của 𝐴𝐴, ứng với vector riêng 𝑣𝑣 ≠ 0, tức là 𝐴𝐴𝐴𝐴 = 𝜆𝜆𝜆𝜆 thì 1 + 𝜆𝜆2 là giá trị riêng
của 𝐼𝐼 + 𝐴𝐴2 , thât vậy
(𝐼𝐼 + 𝐴𝐴2 )𝑣𝑣 = 𝐼𝐼𝐼𝐼 + 𝐴𝐴2 𝑣𝑣 = 𝐼𝐼𝐼𝐼 + 𝐴𝐴𝐴𝐴𝐴𝐴 = 𝐼𝐼𝐼𝐼 + 𝜆𝜆𝜆𝜆𝜆𝜆 = (1 + 𝜆𝜆2 )𝑣𝑣
Gọi 𝑎𝑎1 , 𝑎𝑎2 , … , 𝑎𝑎𝑛𝑛 là tất cả các giá trị riêng thực của 𝐴𝐴, khi đó 𝐴𝐴 chéo hoá được, tức là tồn tại ma
trận 𝑃𝑃 khả nghịch sao cho
𝐴𝐴 = 𝑃𝑃 diag(𝑎𝑎1 , … , 𝑎𝑎𝑛𝑛 )𝑃𝑃 −1 ⇒ 𝐼𝐼 + 𝐴𝐴2 = 𝑃𝑃 diag(1 + 𝑎𝑎21 , … ,1 + 𝑎𝑎2𝑛𝑛 )𝑃𝑃 −1
Từ đó và áp dụng bất đẳng thức Cauchy, ta được
|det(𝐴𝐴 + 𝐴𝐴−1 )| = |det[𝐴𝐴−1 (𝐼𝐼 + 𝐴𝐴2 )]| = |det 𝐴𝐴−1 ||det(𝐼𝐼 + 𝐴𝐴2 )|
= 1. |det 𝑃𝑃 det[diag(1 + 𝑎𝑎21 , … ,1 + 𝑎𝑎2𝑛𝑛 )] det(𝑃𝑃 −1 )|
= 1. (1 + 𝑎𝑎21 )(1 + 𝑎𝑎22 ) … (1 + 𝑎𝑎2𝑛𝑛 ) ≥ 2𝑛𝑛 |𝑎𝑎1 𝑎𝑎2 … 𝑎𝑎𝑛𝑛 | = 2𝑛𝑛 |det 𝐴𝐴| = 2𝑛𝑛 .

Bài 7.28. Cho 𝐴𝐴 ∈ 𝑀𝑀𝑛𝑛 (ℝ) thoả tr[(𝐴𝐴 − 𝐴𝐴𝑇𝑇 )2𝑛𝑛 ] = 0. Chứng minh 𝐴𝐴 = 𝐴𝐴𝑇𝑇 .

Chứng minh.

Cách 1:

Ký hiệu 𝐴𝐴 = �𝑎𝑎𝑖𝑖𝑖𝑖 �, dễ thấy ngay 𝐴𝐴 − 𝐴𝐴𝑇𝑇 là một ma trận phản đối xứng nên 𝑖𝑖(𝐴𝐴 − 𝐴𝐴𝑇𝑇 ) là ma trận
Hermite. Do đó có thể chéo hoá nó bởi ma trận Unita, tức là tồn tại ma trận Unita 𝑈𝑈 sao cho
𝑖𝑖(𝐴𝐴 − 𝐴𝐴𝑇𝑇 ) = 𝑈𝑈 diag(𝑎𝑎1 , … , 𝑎𝑎𝑛𝑛 ) 𝑈𝑈 −1

Đỗ Minh Triết VII-84 - MATHTASY -


Ôn thi Olympic Toán Đại số Phần VII: Giá trị riêng, vector riêng

trong đó 𝑎𝑎𝑖𝑖 , 𝑖𝑖 = ���������


1, 𝑛𝑛 là các giá trị riêng thực của 𝑖𝑖(𝐴𝐴 − 𝐴𝐴𝑇𝑇 ). Từ đó và kết hợp với giả thiết, ta có
2𝑛𝑛
tr ��𝑖𝑖(𝐴𝐴 − 𝐴𝐴𝑇𝑇 )� � = − tr[(𝐴𝐴 − 𝐴𝐴𝑇𝑇 )2𝑛𝑛 ] = −(𝑎𝑎2𝑛𝑛 2𝑛𝑛
1 +. . . +𝑎𝑎𝑛𝑛 ) = 0

suy ra 𝑎𝑎1 =. . . = 𝑎𝑎𝑛𝑛 = 0, từ đó 𝑖𝑖(𝐴𝐴 − 𝐴𝐴𝑇𝑇 ) = 0.


Vậy 𝐴𝐴 = 𝐴𝐴𝑇𝑇 .

Cách 2:

Đặt 𝐵𝐵 = �𝑏𝑏𝑖𝑖𝑖𝑖 � = 𝐴𝐴 − 𝐴𝐴𝑇𝑇 , dễ thấy 𝐵𝐵 là ma trận phản đối xứng nên 𝐵𝐵 = −𝐵𝐵𝑇𝑇 , suy ra 𝐵𝐵2 =
(𝐵𝐵𝑇𝑇 )2 = (𝐵𝐵2 )𝑇𝑇 tức 𝐵𝐵2 là ma trận đối xứng, do đó nó chéo hoá được và các giá trị riêng, đặt là
𝑏𝑏1 , … , 𝑏𝑏𝑛𝑛 , đều là các số thực
𝐵𝐵2 = 𝑃𝑃 diag(𝑏𝑏1 , … , 𝑏𝑏𝑛𝑛 )𝑃𝑃 −1
Ngoài ra ∀𝑥𝑥 ≠ 0, ta có
𝑥𝑥𝑇𝑇 𝐵𝐵2 𝑥𝑥 = −𝑥𝑥𝑇𝑇 𝐵𝐵𝑇𝑇 𝐵𝐵𝐵𝐵 = −(𝐵𝐵𝐵𝐵)𝑇𝑇 (𝐵𝐵𝐵𝐵) ≤ 0 (𝐵𝐵𝐵𝐵 có thể bằng 0)
Suy ra 𝐵𝐵2 là bán xác định âm nên các giá trị riêng của nó không dương. Mặt khác, 𝑏𝑏1𝑛𝑛 , … , 𝑏𝑏𝑛𝑛𝑛𝑛 là
giá trị riêng của 𝐵𝐵2𝑛𝑛 , từ giả thiết, ta có
tr(𝐵𝐵2𝑛𝑛 ) = 𝑏𝑏1𝑛𝑛 +. . . +𝑏𝑏𝑛𝑛𝑛𝑛 = 0
Nếu 𝑛𝑛 chẵn thì 𝑏𝑏1 =. . . = 𝑏𝑏𝑛𝑛 = 0, nếu 𝑛𝑛 lẻ thì do 𝑏𝑏1𝑛𝑛 , … , 𝑏𝑏𝑛𝑛𝑛𝑛 là những số thực không dương, ta cũng
có 𝑏𝑏1 =. . . = 𝑏𝑏𝑛𝑛 = 0. Như vậy 𝐵𝐵2 chỉ có giá trị riêng là 0, nên 𝐵𝐵2 = 𝑃𝑃 diag(𝑏𝑏1 , … , 𝑏𝑏𝑛𝑛 )𝑃𝑃 −1 = 0.
Mặt khác 0 = 𝐵𝐵2 = −𝐵𝐵𝐵𝐵𝑇𝑇 , so sánh đường chéo hai bên
𝑛𝑛
− � 𝑏𝑏𝑖𝑖𝑖𝑖 = 0 ∀ 𝑖𝑖 = 1, … , 𝑛𝑛
𝑗𝑗=1

Vậy 0 = 𝐵𝐵 = 𝐴𝐴 − 𝐴𝐴𝑇𝑇 hay 𝐴𝐴 = 𝐴𝐴𝑇𝑇 .


Nhận xét: hai cách làm khá tương tự nhau, từ giả thiết suy ra ma trận chéo đều là ma trận 0 suy
ra điều phải chứng minh. Tuy nhiên, cách 1 ngắn gọn, các bước đi đến kết quả ít hơn.

Bài 7.29. Cho 𝐴𝐴, 𝑀𝑀 ∈ 𝑀𝑀𝑛𝑛 (ℝ), 𝑀𝑀 chéo hoá được. Chứng minh nếu 𝑀𝑀 𝑘𝑘 𝐴𝐴 = 0 khi và chỉ khi
𝑀𝑀𝑀𝑀 = 0 với mọi 𝑘𝑘 ∈ ℕ∗ .

Chứng minh.

Vì 𝑀𝑀 chéo hoá được nên tồn tại ma trận 𝑃𝑃 khả nghịch và ma trận chéo 𝐷𝐷 sao cho 𝑀𝑀 =
𝑃𝑃𝑃𝑃𝑃𝑃 −1 , ta có
𝑀𝑀 𝑘𝑘 𝐴𝐴 = 0 ⇔ 𝑃𝑃 𝐷𝐷𝑘𝑘 𝑃𝑃 −1 𝐴𝐴 = 0 ⇔ 𝐷𝐷𝑘𝑘 𝑃𝑃 −1 𝐴𝐴 = 0
𝜆𝜆1 … 0
Đặt 𝐷𝐷 = � ⋮ ⋱ ⋮ � và 𝑃𝑃 −1 𝐴𝐴 = �𝑎𝑎𝑖𝑖𝑖𝑖 �, khi đó
0 … 𝜆𝜆1

Đỗ Minh Triết VII-85 - MATHTASY -


Ôn thi Olympic Toán Đại số Phần VII: Giá trị riêng, vector riêng

𝜆𝜆𝑘𝑘1 𝑎𝑎11 … 𝜆𝜆𝑘𝑘1 𝑎𝑎1𝑛𝑛


𝐷𝐷𝑘𝑘 𝑃𝑃 −1 𝐴𝐴 = ⎛
⎜ ⋮ ⋱ ⋮ ⎞ ⎟ = 0 ⇔ 𝜆𝜆𝑘𝑘𝑖𝑖 𝑎𝑎𝑖𝑖𝑖𝑖 = 0 ⇔ 𝜆𝜆𝑖𝑖 𝑎𝑎𝑖𝑖𝑖𝑖 = 0 ⇔ 𝐷𝐷𝑃𝑃 −1 𝐴𝐴 = 0
⎝𝜆𝜆𝑘𝑘𝑛𝑛 𝑎𝑎𝑛𝑛1 … 𝑘𝑘
𝜆𝜆𝑛𝑛 𝑎𝑎𝑛𝑛𝑛𝑛 ⎠
⇔ 𝑃𝑃𝑃𝑃𝑃𝑃 −1 𝐴𝐴 = 𝑀𝑀𝑀𝑀 = 0
Điều phải chứng minh.

Đỗ Minh Triết VII-86 - MATHTASY -


PHẦN VIII. ĐA THỨC ĐẶC TRƯNG, ĐA THỨC TỐI TIỂU

8.1. Cơ sở lý thuyết

Giả sử 𝐴𝐴 ∈ 𝑀𝑀𝑛𝑛 (𝕂𝕂), ta có định nghĩa, định lý và một số tính chất sau:
Đa thức đặc trưng:
det(𝐴𝐴 − 𝑥𝑥𝑥𝑥) = (−1)𝑛𝑛 𝑥𝑥𝑛𝑛 + (−1)𝑛𝑛−1 (tr 𝐴𝐴)𝑥𝑥𝑛𝑛−1 +. . . −(𝐴𝐴11 +. . . +𝐴𝐴𝑛𝑛𝑛𝑛 )𝑥𝑥 + det 𝐴𝐴
Đa thức đặc trưng thường được ký hiệu là 𝑃𝑃𝐴𝐴 (𝑥𝑥), 𝑝𝑝𝐴𝐴 (𝑥𝑥), 𝜒𝜒𝐴𝐴 (𝑥𝑥).
Định lý Cayley – Hamilton: 𝑃𝑃𝐴𝐴 (𝐴𝐴) = 0 tức là đa thức đặc trưng nhận 𝐴𝐴 làm nghiệm.
Đa thức tối tiểu: là đa thức khác 0, chuẩn tắc, bậc nhỏ nhất nhận 𝐴𝐴 làm nghiệm. Đa thức tối
tiểu thường được ký hiệu là Π𝐴𝐴 (𝑥𝑥)
Tính chất:
- Đa thức tối tiểu tồn tại và duy nhất.
- Một đa thức bất kì nhận 𝐴𝐴 làm nghiệm khi và chỉ khi nó chia hết cho đa thức tối tiểu.
- Mọi giá trị riêng của 𝐴𝐴 đều là nghiệm của đa thức tối tiểu. Như vậy, nếu 𝐴𝐴 có 𝑛𝑛 giá trị riêng
phân biệt thì đa thức tối tiểu của nó trùng với đa thức đặc trưng. Nói cách khác, 𝐴𝐴 chéo hoá
được khi và chỉ khi đa thức tối tiểu của nó không có nghiệm bội.
- Nghiệm của đa thức tối tiểu là nghiệm của đa thức đặc trưng và ngược lại.

8.2. Bài tập

0 1 0
Bài 8.1. Cho ma trận 𝐴𝐴 = � −4 4 0� và đa thức
−2 1 2
𝑓𝑓(𝑥𝑥) = −𝑥𝑥 + 6𝑥𝑥 − 12𝑥𝑥 + 8𝑥𝑥 − 𝑥𝑥4 + 6𝑥𝑥3 − 12𝑥𝑥2 + 10𝑥𝑥 + 1. Tính 𝑓𝑓(𝐴𝐴)
8 7 6 5

Giải.

Đa thức đặc trưng của ma trận 𝐴𝐴 là 𝑃𝑃𝐴𝐴 (𝑥𝑥) = −𝑥𝑥3 + 6𝑥𝑥2 − 12𝑥𝑥 + 8
Chia đa thức 𝑓𝑓(𝑥𝑥) cho đa thức 𝑃𝑃𝐴𝐴 (𝑥𝑥) được thương 𝑞𝑞(𝑥𝑥) = 𝑥𝑥5 + 𝑥𝑥 và dư 𝑟𝑟(𝑥𝑥) = 2𝑥𝑥 + 1. Do đó
1 2 0
𝑓𝑓(𝐴𝐴) = 𝑟𝑟(𝐴𝐴) = 2𝐴𝐴 + 𝐸𝐸 = � −8 9 0�
−4 2 5

Bài 8.2. Cho 𝑀𝑀 là ma trận vuông thực cấp 3 thoả 𝑀𝑀 3 = 𝐼𝐼, 𝑀𝑀 ≠ 𝐼𝐼.
a) Tìm các giá trị riêng của 𝑀𝑀.

Đỗ Minh Triết VIII-87 - MATHTASY -


Ôn thi Olympic Toán Đại số Phần VIII: Đa thức đặc trưng, đa thức tối tiểu

b) Tìm một ma trận có tính chất như thế

Giải.

a) Do 𝑀𝑀 là nghiệm của đa thức 𝑥𝑥3 − 1 nên đa thức tối tiểu của 𝑀𝑀 phải là ước của 𝑥𝑥3 − 1. Mặt
khác, 𝑀𝑀 có ít nhất một giá trị riêng thực nên đa thức tối tiểu của nó có chứa nhân tử 𝑥𝑥 − 1.
Nhưng vì 𝑀𝑀 ≠ 𝐼𝐼 nên 𝑥𝑥 − 1 không thể là đa thức tối tiểu của 𝑀𝑀 được. Do đó 𝑥𝑥3 − 1 cũng là đa
thức tối tiểu của 𝑀𝑀. Vậy 𝑀𝑀 chỉ có giá trị riêng thực là 1.
1 0 √
0

⎜0
1 3⎞
b) 𝑀𝑀 = ⎜ √
2 ⎟
2 ⎟
− 3 1
⎝0 2 2 ⎠

Bài 8.3. Cho đa thức bậc hai 𝑝𝑝(𝑥𝑥) = 𝑎𝑎𝑥𝑥2 + 𝑏𝑏𝑏𝑏 + 𝑐𝑐 không có nghiệm thực. Chứng minh rằng
tồn tại ma trận vuông 𝐴𝐴 cấp 𝑛𝑛 thoả mãn phương trình 𝑝𝑝(𝐴𝐴) = 0 nếu và chỉ nếu 𝑛𝑛 chẵn.

Chứng minh.

Nếu 𝐴𝐴 tồn tại thì nó có đa thức tối tiểu là ước của 𝑝𝑝(𝑥𝑥). Do đa thức này không có nghiệm thực
nên 𝑝𝑝(𝑥𝑥) cũng chính là đa thức tối tiểu của 𝐴𝐴, vì vậy đa thức đặc trưng của 𝐴𝐴 phải có dạng
(𝑎𝑎𝑥𝑥2 + 𝑏𝑏𝑏𝑏 + 𝑐𝑐)𝑘𝑘 . Vậy 𝑛𝑛 phải chẵn. (Cách khác: do 𝑝𝑝(𝑥𝑥) không có nghiệm thực nên ta phân tích
𝑎𝑎𝐴𝐴2 + 𝑏𝑏𝑏𝑏 + 𝑐𝑐𝑐𝑐 = (𝛼𝛼𝛼𝛼 + 𝛽𝛽𝛽𝛽)2 + 𝛾𝛾𝛾𝛾 = 0 suy ra |(𝛼𝛼𝛼𝛼 + 𝛽𝛽𝛽𝛽)2 | = |−𝛾𝛾𝛾𝛾| = (−1)𝑛𝑛 𝛾𝛾 𝑛𝑛 . Vậy 𝑛𝑛 chẵn).
Ngược lại, ta chọn một ma trận cấp hai 𝐴𝐴0 sao cho đa thức đặc trưng của nó là 𝑝𝑝(𝑥𝑥). Lập ma
trận khối chéo 𝐴𝐴 gồm khối 𝐴𝐴0 trên đường chéo chính, đây chính là ma trận cấp 𝑛𝑛 cần tìm
𝑛𝑛
2
−2 5
(chẳng hạn với 𝑝𝑝(𝑥𝑥) = 𝑥𝑥2 + 2𝑥𝑥 + 5, ta chọn 𝐴𝐴0 = � �).
−1 0

Bài 8.4. Cho 𝐴𝐴 là ma trận vuông cấp 𝑛𝑛 thoả 𝐴𝐴2 = 𝐴𝐴. Tìm đa thức đặc trưng của 𝐴𝐴.

Giải.

Từ điều kiện 𝐴𝐴2 = 𝐴𝐴 chứng tỏ đa thức tối tiểu của 𝐴𝐴 là 𝑥𝑥2 − 𝑥𝑥 = 𝑥𝑥(𝑥𝑥 − 1), do đó đa thức đặc
trưng của 𝐴𝐴 phải có dạng (−1)𝑛𝑛 𝑥𝑥𝑛𝑛−𝑟𝑟 (𝑥𝑥 − 1)𝑟𝑟 .
Không gian con riêng của 𝐴𝐴 ứng với 𝑥𝑥 = 0 là 𝑁𝑁𝐴𝐴 (0) = {𝑥𝑥|𝐴𝐴𝐴𝐴 = 0} có số chiều 𝑛𝑛 − rank 𝐴𝐴. Do
đó 𝑟𝑟 = rank 𝐴𝐴.

Bài 8.5. (Câu 2 Olympic 1997) Cho 𝐴𝐴 = �𝑎𝑎𝑖𝑖𝑖𝑖 � ∈ 𝑀𝑀𝑛𝑛 (ℝ), rank 𝐴𝐴 = 𝑟𝑟 ≤ 𝑛𝑛, 𝐴𝐴2 = 𝐴𝐴. Tính
các giá trị có thể có của tr 𝐴𝐴.

Giải.

Cách 1:

Đỗ Minh Triết VIII-88 - MATHTASY -


Ôn thi Olympic Toán Đại số Phần VIII: Đa thức đặc trưng, đa thức tối tiểu

Theo trên, đa thức đặc trưng của 𝐴𝐴 có dạng (−1)𝑛𝑛 𝑥𝑥𝑛𝑛−𝑟𝑟 (𝑥𝑥 − 1)𝑟𝑟 , trong đó 𝑟𝑟 = rank 𝐴𝐴. Vậy 𝐴𝐴 chỉ
có giá trị riêng là 0 và 1 mà vết 𝐴𝐴 lại là tổng các giá trị riêng của nó, và vì 1 là nghiệm bội 𝑟𝑟 của
đa thức đặc trưng nên tr 𝐴𝐴 = 𝑟𝑟.

Cách 2:

Xét phép biến đổi tuyến tính 𝑓𝑓 trong ℝ𝑛𝑛 nhận 𝐴𝐴 làm ma trận biểu diễn trong hệ cơ sở 𝑒𝑒1 , … , 𝑒𝑒𝑛𝑛 .
Ta đã biết giá trị của tr 𝐴𝐴 không phụ thuộc vào cách chọn cơ sở trong ℝ𝑛𝑛 .
Giả sử 𝑢𝑢 ∈ Ker 𝑓𝑓 ∩ Im 𝑓𝑓 thì ta có 𝐴𝐴𝐴𝐴 = 0 do 𝑢𝑢 ∈ Ker 𝑓𝑓, mặt khác, 𝑢𝑢 ∈ Im 𝑓𝑓 và 𝐴𝐴2 = 𝐴𝐴 nên 𝑢𝑢 =
𝐴𝐴𝐴𝐴 ⇒ 𝐴𝐴𝐴𝐴 = 𝐴𝐴2 𝑥𝑥 = 𝐴𝐴𝐴𝐴 = 0, tức là 𝑥𝑥 ∈ Ker 𝑓𝑓, do vậy mà 𝑢𝑢 = 𝐴𝐴𝐴𝐴 = 0.
Suy ra ℝ𝑛𝑛 là tổng trực tiếp của hai không gian con Ker 𝑓𝑓 và Im 𝑓𝑓
ℝ𝑛𝑛 = Ker 𝑓𝑓 ⨁ Im 𝑓𝑓
theo giả thết, hai không gian này lần lượt có số chiều là 𝑛𝑛 − 𝑟𝑟 và 𝑟𝑟.
Chọn trong Ker 𝑓𝑓 một cơ sở 𝑒𝑒̅1 , … , 𝑒𝑒̅𝑛𝑛−𝑟𝑟 và trong Im 𝑓𝑓 một cơ sở 𝑒𝑒̅𝑛𝑛−𝑟𝑟+1 , … , 𝑒𝑒̅𝑛𝑛 thì khi đó 𝑒𝑒̅1 , … , 𝑒𝑒̅𝑛𝑛
là một cơ sở trong ℝ𝑛𝑛 , ta có
0 , �������������������
𝑘𝑘 = 1, 𝑛𝑛 − 𝑟𝑟
𝑓𝑓(𝑒𝑒̅𝑘𝑘 ) = �
𝑒𝑒̅𝑘𝑘 , ������������������������������
𝑘𝑘 = 𝑛𝑛 − 𝑟𝑟 + 1, 𝑛𝑛
Như vậy, trong cơ sở mới, ta có ma trận
0 0
𝐴𝐴 ̅ = � �
0 𝐼𝐼𝑟𝑟
Suy ra tr 𝐴𝐴 = tr 𝐴𝐴 ̅ = 𝑟𝑟.

Bài 8.6. Cho 𝐴𝐴, 𝐵𝐵 là hai ma trận vuông cùng cấp, luỹ đẳng. Chứng minh 𝐴𝐴, 𝐵𝐵 đồng dạng
khi và chỉ khi chúng cùng hạng.

Chứng minh.

Điều kiện cần là hiển nhiên, ta chứng minh điều kiện đủ.
Theo trên, đa thức đặc trưng của 𝐴𝐴 và 𝐵𝐵 lần lượt là
(−1)𝑛𝑛 𝑥𝑥𝑛𝑛−𝑟𝑟1 (𝑥𝑥 − 1)𝑟𝑟1 và (−1)𝑛𝑛 𝑥𝑥𝑛𝑛−𝑟𝑟2 (𝑥𝑥 − 1)𝑟𝑟2
trong đó 𝑟𝑟1 = rank 𝐴𝐴 , 𝑟𝑟2 = rank 𝐵𝐵.
Khi đó nếu 𝑟𝑟1 = 𝑟𝑟2 = 𝑟𝑟 thì cả 𝐴𝐴 và 𝐵𝐵 đều đồng dạng với ma trận chéo với 𝑟𝑟 số 1 và 𝑛𝑛 − 𝑟𝑟 số 0
trên đường chéo và do đó chúng đồng dạng nhau.

Bài 8.7. (Câu 5a Olympic 2010) Cho 𝐴𝐴 ∈ 𝑀𝑀𝑛𝑛 (ℝ), 𝑛𝑛 ≥ 2, tr 𝐴𝐴 = 10 và rank 𝐴𝐴 = 1. Tìm đa
thức đặc trưng và đa thức tối tiểu của 𝐴𝐴.

Giải:

Đỗ Minh Triết VIII-89 - MATHTASY -


Ôn thi Olympic Toán Đại số Phần VIII: Đa thức đặc trưng, đa thức tối tiểu

Cách 1: Tính trực tiếp

Vì rank 𝐴𝐴 = 1 nên tồn tại vector dòng khác 0 trong 𝐴𝐴 và các vector dòng còn lại đều biểu diễn
tuyến tính được qua dòng này. Do đó ma trận 𝐴𝐴 có dạng sau:
𝜆𝜆1 𝑥𝑥1 𝜆𝜆1 𝑥𝑥2 … 𝜆𝜆1 𝑥𝑥𝑛𝑛

⎜ 𝜆𝜆2 𝑥𝑥1 𝜆𝜆2 𝑥𝑥2 … 𝜆𝜆𝑛𝑛 𝑥𝑥𝑛𝑛 ⎞⎟

⎜ ⎟
𝐴𝐴 = ⎜ … … … … ⎟ ⎟
⎜ 𝜆𝜆 𝜆𝜆𝑖𝑖 𝑥𝑥2 … 𝜆𝜆𝑖𝑖 𝑥𝑥𝑛𝑛 ⎟

⎜ 𝑖𝑖 𝑥𝑥1 ⎟

⎜ … … … … ⎟
⎝𝜆𝜆𝑛𝑛 𝑥𝑥1 𝜆𝜆𝑛𝑛 𝑥𝑥2 … 𝜆𝜆𝑛𝑛 𝑥𝑥𝑛𝑛 ⎠
𝜆𝜆1 𝑥𝑥1

⎜ 𝜆𝜆2 ⎞⎟ ⎛ 𝑥𝑥2 ⎞
⎜ ⎟ ⎜
⎜ ⎟
⎜ … ⎟ ⎜ …⎟ ⎟ ≠ 0.
Đặt 𝑈𝑈 = ⎜
⎜ ⎟ ≠ 0, 𝑉𝑉 = ⎜ 𝑥𝑥𝑖𝑖 ⎟
⎜ 𝜆𝜆𝑖𝑖 ⎟
⎟ ⎜
⎜ ⎟

⎜…⎟ ⎟ ⎜…⎟ ⎟
𝜆𝜆
⎝ 𝑛𝑛 ⎠ ⎝𝑥𝑥𝑛𝑛 ⎠

Khi đó 𝐴𝐴 = 𝑈𝑈 𝑇𝑇 𝑉𝑉 và 𝑉𝑉 𝑈𝑈 𝑇𝑇 = 𝜆𝜆1 𝑥𝑥1 + 𝜆𝜆2 𝑥𝑥2 +. . . +𝜆𝜆𝑖𝑖 𝑥𝑥𝑖𝑖 +. . . +𝜆𝜆𝑛𝑛 𝑥𝑥𝑛𝑛 = tr 𝐴𝐴 = 10


Ta có 𝐴𝐴2 = (𝑈𝑈 𝑇𝑇 𝑉𝑉 )(𝑈𝑈 𝑇𝑇 𝑉𝑉 ) = 𝑈𝑈 𝑇𝑇 (𝑉𝑉 𝑈𝑈 𝑇𝑇 )𝑉𝑉 = 𝑈𝑈 𝑇𝑇 (tr 𝐴𝐴)𝑉𝑉 = (tr 𝐴𝐴)𝑈𝑈 𝑇𝑇 𝑉𝑉 = (tr 𝐴𝐴)𝐴𝐴 = 10𝐴𝐴
Vậy đa thức tối tiểu của 𝐴𝐴 là 𝑃𝑃 (𝑡𝑡) = 𝑡𝑡2 − 10𝑡𝑡.
Tính định thức 𝐷𝐷𝑛𝑛 = det(𝐴𝐴 + 𝑡𝑡𝑡𝑡)
𝜆𝜆 𝑥𝑥 + 𝑡𝑡 𝜆𝜆1 𝑥𝑥2 … 𝜆𝜆1 𝑥𝑥𝑛𝑛
� 1 1 �
𝜆𝜆2 𝑥𝑥1 𝜆𝜆2 𝑥𝑥2 + 𝑡𝑡 … 𝜆𝜆2 𝑥𝑥𝑛𝑛
� �
⋮ ⋮ … ⋮
𝐷𝐷𝑛𝑛 = � �
𝜆𝜆
� 𝑖𝑖 1 𝑥𝑥 𝜆𝜆 𝑖𝑖 𝑥𝑥2 … 𝜆𝜆𝑖𝑖 𝑥𝑥𝑛𝑛 �
� ⋮ ⋮ … ⋮ �
� 𝜆𝜆𝑛𝑛 𝑥𝑥1 𝜆𝜆𝑛𝑛 𝑥𝑥2 … 𝜆𝜆𝑛𝑛 𝑥𝑥𝑛𝑛 + 𝑡𝑡�
𝜆𝜆 𝑥𝑥 + 𝑡𝑡 𝜆𝜆1 𝑥𝑥2 … 𝜆𝜆1 𝑥𝑥𝑛𝑛 𝜆𝜆 𝑥𝑥 + 𝑡𝑡 𝜆𝜆1 𝑥𝑥2 … 𝜆𝜆1 𝑥𝑥𝑛𝑛
� 1 1 � � 1 1 �
𝜆𝜆2 𝑥𝑥1 𝜆𝜆2 𝑥𝑥2 + 𝑡𝑡 … 𝜆𝜆2 𝑥𝑥𝑛𝑛 𝜆𝜆2 𝑥𝑥1 𝜆𝜆2 𝑥𝑥2 + 𝑡𝑡 … 𝜆𝜆2 𝑥𝑥𝑛𝑛
� �
⋮ ⋮ … ⋮ � �� ⋮ ⋮ … ⋮ �

=� +
𝜆𝜆
� 𝑖𝑖 1 𝑥𝑥 𝜆𝜆 𝑥𝑥
𝑖𝑖 2 … 𝜆𝜆𝑖𝑖 𝑥𝑥𝑛𝑛 � � 𝜆𝜆𝑖𝑖 𝑥𝑥1 𝜆𝜆𝑖𝑖 𝑥𝑥2 … 𝜆𝜆𝑖𝑖 𝑥𝑥𝑛𝑛 �
� ⋮ ⋮ … ⋮ � � ⋮ ⋮ … ⋮ �
� 𝜆𝜆𝑛𝑛 𝑥𝑥1 𝜆𝜆𝑛𝑛 𝑥𝑥2 … 𝜆𝜆𝑛𝑛 𝑥𝑥𝑛𝑛 � � 0 0 … 𝑡𝑡 �
𝜆𝜆 𝑥𝑥 + 𝑡𝑡 𝜆𝜆1 𝑥𝑥2 … 𝜆𝜆1 𝑡𝑡 0 … 0
� 1 1 � �0
𝜆𝜆2 𝑥𝑥1 𝜆𝜆2 𝑥𝑥2 + 𝑡𝑡 … 𝜆𝜆2 𝑡𝑡 … 0�
� � �⋮
⋮ ⋮ … ⋮ � ⋮ … ⋮�
= 𝜆𝜆𝑛𝑛 𝑥𝑥𝑛𝑛 � + 𝑡𝑡𝐷𝐷𝑛𝑛−1 = 𝜆𝜆𝑛𝑛 𝑥𝑥𝑛𝑛 � � + 𝑡𝑡𝐷𝐷𝑛𝑛−1
𝜆𝜆
� 𝑖𝑖 1 𝑥𝑥 𝜆𝜆 𝑖𝑖 𝑥𝑥2 … 𝜆𝜆𝑖𝑖 � �0 0 … 0�
� ⋮ ⋮ … ⋮ � �⋮ ⋮ … ⋮�
� 𝑥𝑥1 𝑥𝑥2 … 1� �𝑥𝑥1 𝑥𝑥2 … 1�
= 𝜆𝜆𝑛𝑛 𝑥𝑥𝑛𝑛 𝑡𝑡𝑛𝑛−1 + 𝑡𝑡𝐷𝐷𝑛𝑛−1 = 𝜆𝜆𝑛𝑛 𝑥𝑥𝑛𝑛 𝑡𝑡𝑛𝑛−1 + 𝑡𝑡(𝜆𝜆𝑛𝑛−1 𝑥𝑥𝑛𝑛−1 𝑡𝑡𝑛𝑛−2 + 𝑡𝑡𝐷𝐷𝑛𝑛−2 )
= (𝜆𝜆𝑛𝑛 𝑥𝑥𝑛𝑛 + 𝜆𝜆𝑛𝑛−1 𝑥𝑥𝑛𝑛−1 )𝑡𝑡𝑛𝑛−1 + 𝑡𝑡2 𝐷𝐷𝑛𝑛−2
= (𝜆𝜆𝑛𝑛 𝑥𝑥𝑛𝑛 + 𝜆𝜆𝑛𝑛−1 𝑥𝑥𝑛𝑛−1 )𝑡𝑡𝑛𝑛−1 + 𝑡𝑡2 (𝜆𝜆𝑛𝑛−2 𝑥𝑥𝑛𝑛−2 𝑡𝑡𝑛𝑛−3 + 𝑡𝑡𝐷𝐷𝑛𝑛−3 )
= (𝜆𝜆𝑛𝑛 𝑥𝑥𝑛𝑛 + 𝜆𝜆𝑛𝑛−1 𝑥𝑥𝑛𝑛−1 + 𝜆𝜆𝑛𝑛−2 𝑥𝑥𝑛𝑛−2 )𝑡𝑡𝑛𝑛−1 + 𝑡𝑡3 𝐷𝐷𝑛𝑛−3 =. ..

Đỗ Minh Triết VIII-90 - MATHTASY -


Ôn thi Olympic Toán Đại số Phần VIII: Đa thức đặc trưng, đa thức tối tiểu

= 𝑡𝑡𝑛𝑛−1 (𝜆𝜆𝑛𝑛 𝑥𝑥𝑛𝑛 + 𝜆𝜆𝑛𝑛−1 𝑥𝑥𝑛𝑛−1 +. . . +𝜆𝜆2 𝑥𝑥2 + 𝐷𝐷1 )


= 𝑡𝑡𝑛𝑛−1 (tr 𝐴𝐴 + 𝑡𝑡) = 𝑡𝑡𝑛𝑛−1 (𝑡𝑡 + 10)
(có thể sử dụng tính đa tuyến tính tính định thức này)
Vậy đa thức đặc trưng của 𝐴𝐴 là (−1)𝑛𝑛 𝑡𝑡𝑛𝑛−1 (𝑡𝑡 − 10).

Cách 2:

Vì rank 𝐴𝐴 = 1 hay dim Ker 𝐴𝐴 = 𝑛𝑛 − 1 nên 𝐴𝐴 có đúng 𝑛𝑛 − 1 vector riêng ứng với 0. Bởi vậy mà
một giá trị riêng còn lại là số thực. Từ đó 𝐴𝐴 chéo hóa được và trên đường chéo chỉ có một phần
tử khác 0 chính là 10. Suy ra ngay đa thức đặc trưng là (−1)𝑛𝑛 𝑡𝑡𝑛𝑛−1 (𝑡𝑡 − 10) và đa thức tối tiểu.

0 0 𝑎𝑎
Bài 8.8. Cho ma trận 𝐴𝐴 = �1 0 𝑏𝑏 � ∈ 𝑀𝑀𝑛𝑛 (𝕂𝕂). Tìm đa thức tối tiểu của 𝐴𝐴.
0 1 𝑐𝑐
Giải.

Đa thức đặc trưng của 𝐴𝐴


𝑃𝑃𝐴𝐴 (𝜆𝜆) = −𝜆𝜆3 + 𝑐𝑐𝜆𝜆2 + 𝑏𝑏𝑏𝑏 + 𝑎𝑎
Ta sẽ chứng tỏ đây cũng là đa thức tối tiểu của 𝐴𝐴. Thật vậy, với 𝑥𝑥0 = (1,0,0) khi đó
𝑥𝑥0 = (1,0,0) ; 𝐴𝐴𝑥𝑥0 = (0,1,0) ; 𝐴𝐴2 𝑥𝑥0 = (0,0,1)
là 3 vector độc lập tuyến tính.
Giả sử 𝐴𝐴 là nghiệm của một đa thức bậc hai, tức là 𝑘𝑘1 𝐴𝐴2 + 𝑘𝑘2 𝐴𝐴 + 𝑘𝑘3 𝐼𝐼 = 0 (𝑘𝑘1 ≠ 0) suy ra
𝑘𝑘1 𝐴𝐴2 𝑥𝑥0 + 𝑘𝑘2 𝐴𝐴𝑥𝑥0 + 𝑘𝑘3 𝑥𝑥0 = 0
từ đây cho ta 𝑘𝑘1 = 𝑘𝑘2 = 𝑘𝑘3 = 0, mâu thuẫn.
Tương tự, giả sử 𝐴𝐴 là nghiệm của một đa thức bậc nhất, tức là 𝑚𝑚1 𝐴𝐴 + 𝑚𝑚2 𝐼𝐼 = 0 (𝑚𝑚1 ≠ 0) suy ra
𝑚𝑚1 𝐴𝐴𝑥𝑥0 + 𝑚𝑚2 𝑥𝑥0 = 0
từ đây cho ta 𝑚𝑚1 = 𝑚𝑚2 = 0, mâu thuẫn.
Vậy đa thức tối tiểu phải có bậc 3 và đó cũng chính là đa thức đặc trưng.

Bài 8.9. (Câu 1 Olympic Quốc tế 1999)


a) Chứng tỏ rằng với mọi số tự nhiên 𝑛𝑛 ≠ 0, luôn tồn tại ma trận 𝐴𝐴 thực cấp 𝑛𝑛 thoả mãn 𝐴𝐴3 = 𝐴𝐴 +
𝐼𝐼.
b) Chứng minh với mỗi ma trận 𝐴𝐴 thực cấp 𝑛𝑛 thoả mãn 𝐴𝐴3 = 𝐴𝐴 + 𝐼𝐼 thì det 𝐴𝐴 > 0.

Chứng minh.

Đỗ Minh Triết VIII-91 - MATHTASY -


Ôn thi Olympic Toán Đại số Phần VIII: Đa thức đặc trưng, đa thức tối tiểu

a) Xét ma trận 𝐴𝐴 = 𝑥𝑥𝑥𝑥. Khi đó 𝐴𝐴3 = 𝐴𝐴 + 𝐼𝐼 xảy ra khi và chỉ khi 𝑥𝑥3 = 𝑥𝑥 + 1 vì 𝐴𝐴3 − 𝐴𝐴 − 𝐼𝐼 =
(𝑥𝑥3 − 𝑥𝑥 − 1)𝐼𝐼. Do đa thức 𝑥𝑥3 − 𝑥𝑥 − 1 bậc lẻ nên nó luôn có nghiệm thực. Điều này chứng tỏ luôn
có một ma trận 𝐴𝐴 thực, cấp 𝑛𝑛 thoả mãn 𝐴𝐴3 = 𝐴𝐴 + 𝐼𝐼, đó chính là 𝑥𝑥𝑥𝑥.
b) Dễ dàng kiểm tra đa thức 𝑥𝑥3 − 𝑥𝑥 − 1 có một nghiệm thực dương 𝑥𝑥1 và hai nghiệm phức liên
hợp 𝑥𝑥2 , 𝑥𝑥3 . Nếu ma trận 𝐴𝐴 thoả mãn 𝐴𝐴3 = 𝐴𝐴 + 𝐼𝐼 thì giá trị riêng của nó chỉ có thể là 𝑥𝑥1 , 𝑥𝑥2 , 𝑥𝑥3
trên. Do đó, đa thức đặc trưng của 𝐴𝐴 là (−1)𝑛𝑛 (𝑥𝑥3 − 𝑥𝑥 − 1)𝑛𝑛 = (−1)𝑛𝑛 (𝑥𝑥 − 𝑥𝑥1 )𝛼𝛼 (𝑥𝑥2 + 𝑎𝑎𝑥𝑥 + 𝑏𝑏)𝛽𝛽 (𝛼𝛼 +
2𝛽𝛽 = 𝑛𝑛) với 𝑥𝑥2 , 𝑥𝑥3 là hai nghiệm phức liên hợp của phương trình 𝑥𝑥2 + 𝑎𝑎𝑎𝑎 + 𝑏𝑏 = 0.
Mặt khác, định thức của 𝐴𝐴 bằng tích các nghiệm (kể cả bội) của phương trình đặc trưng
𝛽𝛽 𝛽𝛽
det 𝐴𝐴 = 𝑥𝑥𝛼𝛼 𝛼𝛼
1 𝑥𝑥2 𝑥𝑥3 = 𝑥𝑥1 (𝑥𝑥2 𝑥𝑥3 )
𝛽𝛽

Vì 𝑥𝑥1 , 𝑥𝑥2 𝑥𝑥3 = |𝑥𝑥2 |2 là các số thực dương nên det 𝐴𝐴 > 0.

Bài 8.10. Cho 𝐴𝐴 ∈ 𝑀𝑀𝑛𝑛 (𝕂𝕂), 𝐵𝐵 ∈ 𝑀𝑀𝑚𝑚 (𝕂𝕂), 𝐴𝐴, 𝐵𝐵 không có giá trị riêng chung. Chứng minh
rằng
a) Nếu ma trận 𝑋𝑋 cỡ 𝑛𝑛 × 𝑚𝑚 thoả 𝐴𝐴𝐴𝐴 − 𝑋𝑋𝑋𝑋 = 0 thì 𝑋𝑋 = 0.
b) Phương trình 𝐴𝐴𝐴𝐴 − 𝑋𝑋𝑋𝑋 = 𝐶𝐶 với ma trận 𝐶𝐶 cỡ 𝑚𝑚 × 𝑛𝑛 có không quá một nghiệm là ma trận 𝑋𝑋 cỡ
𝑚𝑚 × 𝑛𝑛

Chứng minh.

a) Gọi 𝑞𝑞(𝑥𝑥) là đa thức tối tiểu của 𝐵𝐵. Giả sử


𝑘𝑘 𝑘𝑘
𝑞𝑞(𝑥𝑥) = �(𝑥𝑥 − 𝜆𝜆𝑖𝑖 )𝜇𝜇𝑖𝑖 ⇒ 𝑞𝑞(𝐵𝐵) = �(𝐵𝐵 − 𝜆𝜆𝑖𝑖 𝐼𝐼𝑚𝑚 )𝜇𝜇𝑖𝑖 = 0
𝑖𝑖=1 𝑖𝑖=1

Bằng quy nạp, dễ dàng chứng minh được (𝐴𝐴 − 𝜆𝜆𝐼𝐼𝑛𝑛 ) 𝑋𝑋 = 𝑋𝑋(𝐵𝐵 − 𝜆𝜆𝐼𝐼𝑚𝑚 )𝑘𝑘 ∀ 𝑘𝑘 ∈ ℕ, ∀𝜆𝜆.
𝑘𝑘

Từ đó suy ra
𝑘𝑘 𝑘𝑘
�(𝐴𝐴 − 𝜆𝜆𝑖𝑖 𝐼𝐼𝑛𝑛 )𝜇𝜇𝑖𝑖 𝑋𝑋 = 𝑋𝑋 �(𝐵𝐵 − 𝜆𝜆𝑖𝑖 𝐼𝐼𝑚𝑚 )𝜇𝜇𝑖𝑖 = 0
𝑖𝑖=1 𝑖𝑖=1

Vì các giá tị riêng của 𝐵𝐵 không là giá trị riêng của 𝐴𝐴 nên các ma trận (𝐴𝐴 − 𝜆𝜆𝑖𝑖 𝐼𝐼𝑛𝑛 ) đều khả nghịch.
Vậy 𝑋𝑋 = 0.
b) Suy ra từ a), cụ thể:
- Nếu 𝐶𝐶 = 0 thì phương trình chỉ có một nghiệm 𝑋𝑋 = 0.
- Nếu 𝐶𝐶 ≠ 0 thì phương trình vô nghiệm.

Đỗ Minh Triết VIII-92 - MATHTASY -


PHẦN IX. ĐỊNH THỨC (Determinant)

9.1. Cơ sở lý thuyết

9.1.1. Tính định thức

Phương pháp 1: Khai triển Laplace

Giả sử 𝐴𝐴 = �𝑎𝑎𝑖𝑖𝑖𝑖 � ∈ 𝑀𝑀𝑛𝑛 (𝕂𝕂) và 𝑘𝑘 ∈ ℕ thoả 1 ≤ 𝑘𝑘 < 𝑛𝑛. Xét hai bộ chỉ số
1 ≤ 𝑖𝑖1 <. . . < 𝑖𝑖𝑘𝑘 ≤ 𝑛𝑛 và 1 ≤ 𝑗𝑗1 <. . . < 𝑗𝑗𝑘𝑘 ≤ 𝑛𝑛
Ma trận cấp 𝑘𝑘 gồm các phần tử nằm trên giao của 𝑘𝑘 hàng 𝑖𝑖1 , … , 𝑖𝑖𝑘𝑘 và 𝑘𝑘 cột 𝑗𝑗1 , … , 𝑗𝑗𝑘𝑘 của ma trận
𝑗𝑗 ,…,𝑗𝑗
𝐴𝐴 gọi là một ma trận con cấp 𝑘𝑘, kí hiệu 𝐴𝐴𝑖𝑖 1,…,𝑖𝑖 𝑘𝑘 hay 𝐴𝐴(𝑖𝑖1 , … , 𝑖𝑖𝑘𝑘 ; 𝑗𝑗1 , … , 𝑗𝑗𝑘𝑘 ), định thức của ma trận
1 𝑘𝑘

này gọi là định thức con cấp 𝑘𝑘. Phần ma trận còn lại (cấp 𝑛𝑛 − 𝑘𝑘) gọi là ma trận con bù của
̅ , … , 𝑖𝑖 ; 𝑗𝑗 , … , 𝑗𝑗 ), định thức của nó gọi là định thức
𝐴𝐴(𝑖𝑖1 , … , 𝑖𝑖𝑘𝑘 ; 𝑗𝑗1 , … , 𝑗𝑗𝑘𝑘 ) và được kí hiệu là 𝐴𝐴(𝑖𝑖 1 𝑘𝑘 1 𝑘𝑘

con bù của |𝐴𝐴(𝑖𝑖1 , … , 𝑖𝑖𝑘𝑘 ; 𝑗𝑗1 , … , 𝑗𝑗𝑘𝑘 )| trong 𝐴𝐴, còn


̅ , … , 𝑖𝑖 ; 𝑗𝑗 , … , 𝑗𝑗 )�
(−1)𝑠𝑠(𝐼𝐼,𝐽𝐽) �𝐴𝐴(𝑖𝑖 1 𝑘𝑘 1 𝑘𝑘

được gọi là phần bù đại số của 𝐴𝐴(𝑖𝑖1 , … , 𝑖𝑖𝑘𝑘 ; 𝑗𝑗1 , … , 𝑗𝑗𝑘𝑘 ), trong đó 𝑠𝑠(𝐼𝐼, 𝐽𝐽 ) = 𝑖𝑖1 +. . . +𝑖𝑖𝑘𝑘 + 𝑗𝑗1 +. . . +𝑗𝑗𝑘𝑘 .
Định lý (Khai triển Laplace): Giả sử ta chọn 𝑘𝑘 dòng (tương ứng cột) trong một định thức
cấp 𝑛𝑛 (1 ≤ 𝑘𝑘 < 𝑛𝑛). Khi đó định thức đã cho bằng tổng của các tích của định thức con cấp 𝑘𝑘 lấy
ra từ 𝑘𝑘 dòng (tương ứng cột) với phần bù đại số của chúng
|𝐴𝐴| = � ̅ , … , 𝑖𝑖 ; 𝑗𝑗 , … , 𝑗𝑗 )�
𝐴𝐴(𝑖𝑖1 , … , 𝑖𝑖𝑘𝑘 ; 𝑗𝑗1 , … , 𝑗𝑗𝑘𝑘 )(−1)𝑠𝑠(𝐼𝐼,𝐽𝐽) �𝐴𝐴(𝑖𝑖1 𝑘𝑘 1 𝑘𝑘
1≤𝑗𝑗1 <...<𝑗𝑗𝑘𝑘 ≤𝑛𝑛

̅ 𝑗𝑗)� là phần bù đại số của phần tử 𝑎𝑎 . Khi đó


Hệ quả: Ký hiệu 𝑐𝑐𝑖𝑖𝑖𝑖 = (−1)𝑖𝑖+𝑗𝑗 �𝐴𝐴(𝑖𝑖, 𝑖𝑖𝑖𝑖
𝑛𝑛 𝑛𝑛
|𝐴𝐴| = � 𝑎𝑎𝑖𝑖𝑖𝑖 𝑐𝑐𝑖𝑖𝑖𝑖 = � 𝑎𝑎𝑖𝑖𝑖𝑖 𝑐𝑐𝑖𝑖𝑖𝑖
𝑖𝑖=1 𝑗𝑗=1

Hệ quả: Cho 𝑀𝑀 là ma trận khối chéo gồm các ma trận vuông 𝐴𝐴1 , … 𝐴𝐴𝑘𝑘 , khi đó
𝐴𝐴1 … 0
|𝑀𝑀| = � ⋮ ⋱ ⋮ � = |𝐴𝐴1 | … |𝐴𝐴𝑘𝑘 |
0 … 𝐴𝐴𝑘𝑘
Công thức cũng đúng trong trường hợp các phần tử bên trên hay dưới đường chéo chính khác 0.

Ví dụ 1: Tính

Đỗ Minh Triết IX-93 - MATHTASY -


Ôn thi Olympic Toán Đại số Phần IX: Định thức

1 1 1 0 0
�1 2 3 0 0�
𝐷𝐷 = ��0 1 1 1 1 ��
�0 𝑎𝑎 𝑏𝑏 𝑐𝑐 𝑑𝑑 �
�0 𝑎𝑎2 𝑏𝑏2 𝑐𝑐2 𝑑𝑑2 �
Giải.

Khai triển Laplace theo hai dòng đầu, ta được


1 1 1 1 1 1
1 1 1 1
𝐷𝐷 = (−1)1+2+1+2 � � � 𝑏𝑏 𝑐𝑐 𝑑𝑑 � + (−1)1+2+1+3 � � � 𝑎𝑎 𝑐𝑐 𝑑𝑑 �
1 2 2 1 3 2
𝑏𝑏 𝑐𝑐2 𝑑𝑑2 𝑎𝑎 𝑐𝑐2 𝑑𝑑2
= (𝑐𝑐 − 𝑏𝑏)(𝑑𝑑 − 𝑏𝑏)(𝑑𝑑 − 𝑐𝑐) − 2(𝑐𝑐 − 𝑎𝑎)(𝑑𝑑 − 𝑎𝑎)(𝑑𝑑 − 𝑐𝑐)

Ví dụ 2 : Tính định thức cấp 2𝑛𝑛 có các phần tử trên đường chéo chính là 𝑎𝑎, các phần
tử trên đường chéo phụ là 𝑏𝑏, các phần tử còn lại là 0.

Giải.

Khai triển Laplace theo cột đầu và cột cuối, ta được


𝑎𝑎 0 … 0 𝑏𝑏
�0 𝑎𝑎 … 𝑏𝑏 0 �
𝑎𝑎 𝑏𝑏
𝐷𝐷2𝑛𝑛 = �� ⋮ ⋮ ⋱ ⋮ ⋮ �� = � � 𝐷𝐷2𝑛𝑛−2 = (𝑎𝑎2 − 𝑏𝑏2 )𝐷𝐷2𝑛𝑛−2 = (𝑎𝑎2 − 𝑏𝑏2 )2 𝐷𝐷2𝑛𝑛−4 =. . . = (𝑎𝑎2 − 𝑏𝑏2 )𝑛𝑛
� 0 𝑏𝑏 … 𝑏𝑏 𝑎𝑎
𝑎𝑎 0�
� 𝑏𝑏 0 … 0 𝑎𝑎�

Ví dụ 3 : Tính
𝑎𝑎11 𝑎𝑎12 𝑎𝑎13 … 𝑎𝑎1,2𝑛𝑛−2 𝑎𝑎1,2𝑛𝑛−1 𝑎𝑎1,2𝑛𝑛
� 0 𝑎𝑎22 𝑎𝑎23 … 𝑎𝑎2,2𝑛𝑛−2 𝑎𝑎2,2𝑛𝑛−1 0 �
� … �
0 0 𝑎𝑎33 𝑎𝑎3,2𝑛𝑛−2 0 0
� �
𝐷𝐷2𝑛𝑛 =�⋮ ⋮ ⋮ ⋱ ⋮ ⋮ ⋮ �
� 0 0 𝑎𝑎2𝑛𝑛−2,3 … 𝑎𝑎2𝑛𝑛−2,2𝑛𝑛−2 0 0 �
� 0 𝑎𝑎2𝑛𝑛−1,2 𝑎𝑎2𝑛𝑛−1,3 … 𝑎𝑎2𝑛𝑛−1,2𝑛𝑛−2 𝑎𝑎2𝑛𝑛−1,2𝑛𝑛−1 0 �
� 𝑎𝑎2𝑛𝑛,1 𝑎𝑎2𝑛𝑛,2 𝑎𝑎2𝑛𝑛,3 … 𝑎𝑎2𝑛𝑛,2𝑛𝑛−2 𝑎𝑎2𝑛𝑛,2𝑛𝑛−1 𝑎𝑎2𝑛𝑛,2𝑛𝑛 �

Giải.

Làm tương tự ví dụ trên


𝑛𝑛
𝐷𝐷2𝑛𝑛 = �𝑎𝑎11 𝑎𝑎2𝑛𝑛,2𝑛𝑛 − 𝑎𝑎1,2𝑛𝑛 𝑎𝑎2𝑛𝑛,1 �𝐷𝐷2𝑛𝑛−2 =. . . = ��𝑎𝑎𝑖𝑖𝑖𝑖 𝑎𝑎2𝑛𝑛−𝑖𝑖+1,2𝑛𝑛−𝑖𝑖+1 − 𝑎𝑎𝑖𝑖,2𝑛𝑛−𝑖𝑖+1 𝑎𝑎2𝑛𝑛−𝑖𝑖+1,𝑖𝑖 �
𝑖𝑖=1

Phương pháp 2: Biến đổi sơ cấp - Gauss

Tính chất:

Đỗ Minh Triết IX-94 - MATHTASY -


Ôn thi Olympic Toán Đại số Phần IX: Định thức

Trong một định thức:


1. Nếu đổi chỗ hai hàng thì định thức đổi dấu.
2. Nếu mỗi phần tử của một hàng có nhân tử chung thì có thể đưa nhân tử chung đó ra ngoài
định thức.
3. Nếu cộng vào một hàng một bội của một hàng khác thì định thức không đổi.
Hệ quả:
1. Nếu định thức có một hàng 0 thì định thức bằng 0.
2. Nếu hai hàng tỉ lệ thì định thức bằng 0.
3. Nếu một hàng là một tổ hợp tuyến tính của các hàng khác thì định thức bằng 0.
4. Nếu thêm vào một hàng một tổ hợp tuyến tính của các hàng khác thì đinh thức không đổi.
(Tính chất và hệ quả trên vẫn đúng nếu ta thay “hàng” bởi “cột”).
Sử dụng các tính chất và hệ quả trên, ta có thể đưa định thức về dạng tam giác, khi đó định thức
sẽ bằng tích các phần tử trên đường chéo chính. Phương pháp này gọi là phương pháp Gauss, nó
giải quyết được khá nhiều bài Toán tính định thức.

Ví dụ : Cho 𝐴𝐴 là ma trận vuông cấp 𝑛𝑛 có dạng 𝐴𝐴 = �𝑎𝑎𝑖𝑖𝑖𝑖 �, trong đó 𝑎𝑎𝑖𝑖𝑖𝑖 = min{𝑖𝑖, 𝑗𝑗}.
Tính det 𝐴𝐴.

Giải.
1 1 1 … 1
�1 2 2 … 2�
Ta có det 𝐴𝐴 = ��1 2 3 … 3 ��
�⋮ ⋮ ⋮ ⋱ ⋮�
�1 2 3 … 𝑛𝑛�
1 −1 −2 … −(𝑛𝑛 − 1)
� �
1 0 −1 … −(𝑛𝑛 − 2)
� �
Nhân cột thứ nhất với – 𝑖𝑖 rồi cộng vào cột thứ 𝑖𝑖, ta được �1 0 0 … −(𝑛𝑛 − 3)�
�⋮ ⋮ ⋮ ⋱ ⋮ �
�1 0 0 … 0 �
Khai triển theo dòng thứ 𝑛𝑛, ta có (−1)𝑛𝑛+1 . (−1)𝑛𝑛−1 = 1

Ví dụ 1 : Tính
1 2 3 … 𝑛𝑛 − 2 𝑛𝑛 − 1 𝑛𝑛
�2 3 4 … 𝑛𝑛 − 1 𝑛𝑛 𝑛𝑛 �
𝐷𝐷 = �� 3 4 5 … 𝑛𝑛 𝑛𝑛 𝑛𝑛 ��
�⋮ ⋮ ⋮ … ⋮ ⋮ ⋮�
�𝑛𝑛 𝑛𝑛 𝑛𝑛 … 𝑛𝑛 𝑛𝑛 𝑛𝑛�

Đỗ Minh Triết IX-95 - MATHTASY -


Ôn thi Olympic Toán Đại số Phần IX: Định thức

Giải.

Lần lượt nhân dòng 𝑖𝑖 với -1 rồi cộng và dòng 𝑖𝑖 + 1 với 𝑖𝑖 = 𝑛𝑛 − 1, … ,1, ta được
1 2 3 … 𝑛𝑛 − 2 𝑛𝑛 − 1 𝑛𝑛
�1 1 1 … 1 1 0 �

𝐷𝐷 = �1 1 1 … 1 0 0 ��
�⋮ ⋮ ⋮ … ⋮ ⋮ ⋮�
�1 0 0 … 0 0 0�
(𝑛𝑛+1)(𝑛𝑛+2) 𝑛𝑛(𝑛𝑛−1)
−3
= (−1)𝑛𝑛+1 (−1)𝑛𝑛 … (−1)3 𝑛𝑛 = (−1) 2 𝑛𝑛 = (−1) 2 𝑛𝑛

Ví dụ : Tính
1 + 𝑎𝑎1 𝑎𝑎2 𝑎𝑎3 … 𝑎𝑎𝑛𝑛
� �
𝑎𝑎 1 + 𝑎𝑎2 𝑎𝑎3 … 𝑎𝑎𝑛𝑛
� 1 �
𝐷𝐷 = � 𝑎𝑎1 𝑎𝑎2 1 + 𝑎𝑎3 … 𝑎𝑎𝑛𝑛 �
� ⋮ ⋮ ⋮ ⋱ ⋮ �
� 𝑎𝑎1 𝑎𝑎2 𝑎𝑎3 … 1 + 𝑎𝑎𝑛𝑛 �

Giải.

Cộng mọi cột vào cột 1, ta được


1 + 𝑎𝑎1 +. . . +𝑎𝑎𝑛𝑛 𝑎𝑎2 𝑎𝑎3 … 𝑎𝑎𝑛𝑛
� �
1 + 𝑎𝑎1 +. . . +𝑎𝑎𝑛𝑛 1 + 𝑎𝑎2 𝑎𝑎3 … 𝑎𝑎𝑛𝑛
� �
�1 + 𝑎𝑎1 +. . . +𝑎𝑎𝑛𝑛 𝑎𝑎2 1 + 𝑎𝑎3 … 𝑎𝑎𝑛𝑛 �
� ⋮ ⋮ ⋮ ⋱ ⋮ �
�1 + 𝑎𝑎1 +. . . +𝑎𝑎𝑛𝑛 𝑎𝑎2 𝑎𝑎3 … 1 + 𝑎𝑎𝑛𝑛 �
Lấy hàng 1 nhân -1 rồi cộng vào mọi hàng khác, ta được
1 + 𝑎𝑎1 +. . . +𝑎𝑎𝑛𝑛 𝑎𝑎2 𝑎𝑎3 … 𝑎𝑎𝑛𝑛
� �
0 1 0 … 0
� �
� 0 0 1 … 0 � = 1 + 𝑎𝑎1 +. . . +𝑎𝑎𝑛𝑛
� ⋮ ⋮ ⋮ ⋱ ⋮ �
� 0 0 0 … 1�

Ví dụ (Câu I.2 đề chọn đội tuyển CĐ SP BR-VT 2012) : Tính


3 1 1 … 1
�1 4 1 … 1 �
𝐷𝐷 = ��1 1 5 … 1 �

�⋮ ⋮ ⋮ ⋱ ⋮ �
�1 1 1 … 𝑛𝑛 + 1 �

Giải.

Lấy dòng 1 nhân -1 rồi cộng vào mọi dòng khác, ta có

Đỗ Minh Triết IX-96 - MATHTASY -


Ôn thi Olympic Toán Đại số Phần IX: Định thức

3 1 1 … 1
� −2 3 0 … 0�
𝐷𝐷 = �� −2 0 4 … 0 ��
� ⋮ ⋮ ⋮ ⋱ ⋮�
� −2 0 0 … 𝑛𝑛�
Với 𝑖𝑖 = 2, … , 𝑛𝑛 − 1, ta nhân cột 𝑖𝑖 với 2
𝑖𝑖+1
rồi cộng vào cột đầu tiên
2 2
� 3 + +. . . + 1 1 … 1�
3 𝑛𝑛 … 0�
� 0 3 0 2 2 𝑛𝑛! 3 1 1
𝐷𝐷 = � … 0 � = �3 + 3 +. . . + 𝑛𝑛� 2 = 𝑛𝑛! �2 + 3 +. . . + 𝑛𝑛�
� 0 0 4 �
� ⋮ ⋮ ⋮ ⋱ ⋮�
� 0 0 0 … 𝑛𝑛�
𝑛𝑛
1 1 1 1
= 𝑛𝑛! �1 + + +. . . � = 𝑛𝑛! �
2 3 𝑛𝑛 𝑖𝑖=1
𝑖𝑖

Ví dụ 2 : Tính
1 … 1 1 1
� 𝑎𝑎1 … 𝑎𝑎1 𝑎𝑎1 �
𝑎𝑎1 − 𝑏𝑏1
� �
𝐷𝐷 = � 𝑎𝑎2 … 𝑎𝑎2 − 𝑏𝑏2 𝑎𝑎2 𝑎𝑎2 �
� ⋮ ⋰ ⋮ ⋮ ⋮ �
� 𝑎𝑎𝑛𝑛 − 𝑏𝑏𝑛𝑛 … 𝑎𝑎𝑛𝑛 𝑎𝑎𝑛𝑛 𝑎𝑎𝑛𝑛 �

Giải.

Lấy cột cuối nhân -1 rồi lần lược cộng vào tất cả các cột khác, ta có
0 … 0 0 1
� 0 … 0 −𝑏𝑏1 𝑎𝑎1 �
� �
𝐷𝐷 = � 0 … −𝑏𝑏2 0 𝑎𝑎2 � = (−1)𝑛𝑛+2 (−𝑏𝑏𝑛𝑛 )(−1)𝑛𝑛+1 (−𝑏𝑏𝑛𝑛−1 ) … (−1)4 (−𝑏𝑏2 )(−1)3 (−𝑏𝑏1 )
� ⋮ ⋰ ⋮ ⋮ ⋮ �
�−𝑏𝑏𝑛𝑛 … 0 0 𝑎𝑎𝑛𝑛 �
(𝑛𝑛+2)(𝑛𝑛+3) 𝑛𝑛(𝑛𝑛−1)
−3
= (−1) 2 (−1)𝑛𝑛 𝑏𝑏𝑛𝑛 … 𝑏𝑏1 = (−1) 2 𝑏𝑏1 … 𝑏𝑏𝑛𝑛

Ví dụ 3 : Tính
𝑎𝑎 𝑎𝑎1 𝑎𝑎2 … 𝑎𝑎𝑛𝑛
�𝑎𝑎1 𝑎𝑎 𝑎𝑎2 … 𝑎𝑎𝑛𝑛 �
𝐷𝐷 = ��𝑎𝑎1 𝑎𝑎2 𝑎𝑎 … 𝑎𝑎𝑛𝑛 ��
�⋮ ⋮ ⋮ ⋱ ⋮ �
�𝑎𝑎1 𝑎𝑎2 𝑎𝑎3 … 𝑎𝑎 �
Giải.

Cộng các cột vào cột đầu rồi rút nhân tử chung (𝑎𝑎 + 𝑎𝑎1 +. . . +𝑎𝑎𝑛𝑛 ) ra ngoài

Đỗ Minh Triết IX-97 - MATHTASY -


Ôn thi Olympic Toán Đại số Phần IX: Định thức

1 𝑎𝑎1 𝑎𝑎2 … 𝑎𝑎𝑛𝑛


� … 𝑎𝑎𝑛𝑛 �
1 𝑎𝑎 𝑎𝑎2
� … 𝑎𝑎𝑛𝑛 �
𝐷𝐷 = (𝑎𝑎 + 𝑎𝑎1 +. . . +𝑎𝑎𝑛𝑛 ) �1 𝑎𝑎2 𝑎𝑎 �
�⋮ ⋮ ⋮ ⋱ ⋮ �
�1 𝑎𝑎2 𝑎𝑎3 … 𝑎𝑎 �
Lấy hàng đầu nhân -1 rồi lần lượt cộng vào các dòng khác rồi khai triển theo cột đầu, ta được
1 𝑎𝑎1 𝑎𝑎2 … 𝑎𝑎𝑛𝑛
� �
0 𝑎𝑎 − 𝑎𝑎1 0 … 0
� �
𝐷𝐷 = (𝑎𝑎 + 𝑎𝑎1 +. . . +𝑎𝑎𝑛𝑛 ) �0 𝑎𝑎2 − 𝑎𝑎1 𝑎𝑎 − 𝑎𝑎2 … 0 �
�⋮ ⋮ ⋮ ⋱ ⋮ �
�0 𝑎𝑎2 − 𝑎𝑎1 𝑎𝑎3 − 𝑎𝑎2 … 𝑎𝑎 − 𝑎𝑎𝑛𝑛 �
= (𝑎𝑎 + 𝑎𝑎1 +. . . +𝑎𝑎𝑛𝑛 )(𝑎𝑎 − 𝑎𝑎1 )(𝑎𝑎 − 𝑎𝑎2 ) … (𝑎𝑎 − 𝑎𝑎𝑛𝑛 )

Ví dụ 4 Tính định thức Vandermonde


1 1 … 1
�𝑥𝑥 𝑥𝑥2 … 𝑥𝑥𝑛𝑛 �
� 1 �
𝐷𝐷𝑛𝑛 = �𝑥𝑥21 𝑥𝑥22 … 𝑥𝑥2𝑛𝑛 �
� ⋮ ⋮ … ⋮ �
�𝑥𝑥𝑛𝑛−1
1
𝑛𝑛−1
𝑥𝑥2 𝑛𝑛−1
… 𝑥𝑥𝑛𝑛 �

Giải.

Lần lượt nhân dòng 𝑖𝑖 với −𝑥𝑥1 rồi cộng và dòng 𝑖𝑖 + 1 với 𝑖𝑖 = 𝑛𝑛 − 1, … ,1, ta được
1 1 … 1
�0 𝑥𝑥2 − 𝑥𝑥1 … 𝑥𝑥𝑛𝑛 − 𝑥𝑥1 �
� �
𝐷𝐷 = 𝐷𝐷(𝑥𝑥1 , … , 𝑥𝑥𝑛𝑛 ) = �0 𝑥𝑥2 (𝑥𝑥2 − 𝑥𝑥1 ) … 𝑥𝑥𝑛𝑛 (𝑥𝑥𝑛𝑛 − 𝑥𝑥1 ) �
� ⋮ ⋮ … ⋮ �
𝑛𝑛−2 𝑛𝑛−2
� 0 𝑥𝑥2 (𝑥𝑥2 − 𝑥𝑥1 ) … 𝑥𝑥𝑛𝑛 (𝑥𝑥𝑛𝑛 − 𝑥𝑥1 ) �
𝑥𝑥2 − 𝑥𝑥1 … 𝑥𝑥𝑛𝑛 − 𝑥𝑥1 1 … 1
� 𝑥𝑥 (𝑥𝑥 − 𝑥𝑥 ) … 𝑥𝑥 (𝑥𝑥 − 𝑥𝑥 ) � �𝑥𝑥 … 𝑥𝑥 �
2 2 1 𝑛𝑛 𝑛𝑛 1 2 𝑛𝑛
=� = (𝑥𝑥 − 𝑥𝑥 ) … (𝑥𝑥 − 𝑥𝑥 ) �
� ⋮ … ⋮ �
� 2 1 𝑛𝑛 1
�⋮ … ⋮ ��
�𝑥𝑥𝑛𝑛−2
2 (𝑥𝑥2 − 𝑥𝑥1 ) … 𝑥𝑥𝑛𝑛−2 𝑛𝑛 (𝑥𝑥𝑛𝑛 − 𝑥𝑥1 )� �𝑥𝑥𝑛𝑛−2
2 … 𝑥𝑥𝑛𝑛−2
𝑛𝑛 �

= (𝑥𝑥2 − 𝑥𝑥1 ) … (𝑥𝑥𝑛𝑛 − 𝑥𝑥1 )𝐷𝐷(𝑥𝑥2 , … , 𝑥𝑥𝑛𝑛 )


Cứ tiếp tục tương tự, ta được
𝐷𝐷 = � �𝑥𝑥𝑖𝑖 − 𝑥𝑥𝑗𝑗 �
1≤𝑗𝑗<𝑖𝑖≤𝑛𝑛

Ví dụ 5 Tính định thức của ma trận vuông 𝐴𝐴 = �𝑎𝑎𝑖𝑖𝑖𝑖 � cấp 2010 với
(−1)|𝑖𝑖−𝑗𝑗| 2010 , 𝑖𝑖 ≠ 𝑗𝑗
𝑎𝑎𝑖𝑖𝑖𝑖 = �
2009, 𝑖𝑖 = 𝑗𝑗

Giải.

Đỗ Minh Triết IX-98 - MATHTASY -


Ôn thi Olympic Toán Đại số Phần IX: Định thức

Ta có
2009 −2010 2010 … −2010 2010
� −2010 2009 −2010 … 2010 −2010 �
|𝐴𝐴| = �� ⋮ ⋮ ⋮ … ⋮ ⋮ ��
� 2010 −2010 2010 … 2009 −2010 �
� −2010 2010 −2010 … −2010 2009�
Cộng hàng thứ 𝑖𝑖 vào hàng thứ 𝑖𝑖 − 1 với 𝑖𝑖 = 2, … ,2010, ta được
−1 −1 0 … 0 0
� 0 −1 −1 … 0 0 �
|𝐴𝐴| = �� ⋮ ⋮ ⋮ … ⋮ ⋮ �

� 0 0 0 … −1 −1 �
�−2010 2010 −2010 … −2010 2009 �
Lấy cột 𝑖𝑖 nhân -1 rồi cộng vào cột 𝑖𝑖 + 1 với 𝑖𝑖 = 1, … ,2009, ta có
−1 0 0 … 0 0
� 0 −1 0 … 0 0 �
|𝐴𝐴| = �� ⋮ ⋮ ⋮ … ⋮ ⋮ �

� 0 0 0 … −1 0 �
� −2010 2.2010 3. (−2010) … 2009. (−2010) 2009.2011 �
Vậy |𝐴𝐴| = −2009.2011.

Ví dụ 5 : Tính định thức của ma trận vuông 𝐴𝐴 = �𝑎𝑎𝑖𝑖𝑖𝑖 � cấp 𝑛𝑛 với


|𝑖𝑖−𝑗𝑗| 𝑖𝑖 ≠ 𝑗𝑗
𝑎𝑎𝑖𝑖𝑖𝑖 = �(−1)
2 𝑖𝑖 = 𝑗𝑗

Giải.

Ta có
2 −1 +1 … ∓1 ±1
� −1 2 −1 … ±1 ∓1 �
� ∓1 ±1 ��
|𝐴𝐴| = � +1 −1 2 …
� ⋮ ⋮ ⋮ ⋱ ⋮ ⋮�
� ∓1 ±1 ∓1 … 2 −1 �
� ±1 ∓1 ±1 … −1 2�
Cộng dòng 2 vào dòng 1, dòng 3 vào dòng 2,…, dòng 𝑛𝑛 vào dòng 𝑛𝑛 − 1, ta được
1 1 0 … 0 0
� 0 1 1 … 0 0�
� 0��
|𝐴𝐴| = � 0 0 1 … 0
� ⋮ ⋮ ⋮ ⋱ ⋮ ⋮�
� 0 0 0 … 1 1�
� ±1 ∓1 ±1 … −1 2�
Nhân cột 1 với -1 rồi cộng vào cột 2, nhân cột 2 với -1 rồi cộng vào cột 3,… ta được

Đỗ Minh Triết IX-99 - MATHTASY -


Ôn thi Olympic Toán Đại số Phần IX: Định thức

1 1 0 … 0 0
� 0 1 1 … 0 0 �
� �
|𝐴𝐴| = � 0 0 1 … 0 0 � = 𝑛𝑛 + 1
� ⋮ ⋮ ⋮ ⋱ ⋮ ⋮ �
� 0 0 0 … 1 1 �
� ±1 ∓2 ±3 … −𝑛𝑛 + 1 𝑛𝑛 + 1 �

Ví dụ 6 (Câu 1 Olympic Quốc tế 1996 – Câu 1 Olympic Việt Nam 2008) :


Cho 𝑎𝑎0 , 𝑑𝑑 ∈ ℝ, đặt 𝑎𝑎𝑗𝑗 = 𝑎𝑎0 + 𝑗𝑗𝑗𝑗 với 𝑗𝑗 = 1, … , 𝑛𝑛. Tính |𝐴𝐴| biết 𝐴𝐴 là ma trận cấp 𝑛𝑛 + 1 cho bởi
𝑎𝑎0 𝑎𝑎1 𝑎𝑎2 … 𝑎𝑎𝑛𝑛−1 𝑎𝑎𝑛𝑛
�𝑎𝑎1 𝑎𝑎0 𝑎𝑎1 … 𝑎𝑎𝑛𝑛−2 𝑎𝑎𝑛𝑛−1 �
�𝑎𝑎2 𝑎𝑎1 𝑎𝑎0 … 𝑎𝑎𝑛𝑛−3 𝑎𝑎𝑛𝑛−2 �
|𝐴𝐴| = � ⋮ ⋮ ⋮ ⋱ ⋮

⋮ �

�𝑎𝑎𝑛𝑛−1 𝑎𝑎𝑛𝑛−2 𝑎𝑎𝑛𝑛−3 … 𝑎𝑎0 𝑎𝑎1 �
� 𝑎𝑎𝑛𝑛 𝑎𝑎𝑛𝑛−1 𝑎𝑎𝑛𝑛−2 … 𝑎𝑎1 𝑎𝑎0 �

Giải.

Cộng cột đầu vào cột cuối, ta được


𝑎𝑎0 𝑎𝑎1 𝑎𝑎2 … 𝑎𝑎𝑛𝑛−1 1
�𝑎𝑎 𝑎𝑎0 𝑎𝑎1 �
1 … 𝑎𝑎𝑛𝑛−2 1
�𝑎𝑎 𝑎𝑎1 𝑎𝑎0 �
2 … 𝑎𝑎𝑛𝑛−3 1�
|𝐴𝐴| = (𝑎𝑎0 + 𝑎𝑎𝑛𝑛 ) �
� ⋮ ⋮ ⋮ ⋱ ⋮ ⋮�
�𝑎𝑎𝑛𝑛−1 𝑎𝑎𝑛𝑛−2 𝑎𝑎𝑛𝑛−3 … 𝑎𝑎0 1�
� 𝑎𝑎𝑛𝑛 𝑎𝑎𝑛𝑛−1 𝑎𝑎𝑛𝑛−2 … 𝑎𝑎1 1�
Lấy dòng 𝑖𝑖 nhân -1 rồi cộng vào dòng 𝑖𝑖 + 1 với 𝑖𝑖 = 𝑛𝑛 − 1, … ,1, ta được
𝑎𝑎0 𝑎𝑎1 𝑎𝑎2 … 𝑎𝑎𝑛𝑛−1 1
� 𝑑𝑑 −𝑑𝑑 −𝑑𝑑 �
… −𝑑𝑑 0
� �
|𝐴𝐴| = (𝑎𝑎0 + 𝑎𝑎𝑛𝑛 ) � 𝑑𝑑 𝑑𝑑 −𝑑𝑑 … −𝑑𝑑 0�
� ⋮ ⋮ ⋮ ⋱ ⋮ ⋮�
�𝑑𝑑 𝑑𝑑 𝑑𝑑 … −𝑑𝑑 0�
�𝑑𝑑 𝑑𝑑 𝑑𝑑 … 𝑑𝑑 0�
𝑑𝑑 −𝑑𝑑 −𝑑𝑑 … −𝑑𝑑 −𝑑𝑑
� 𝑑𝑑 𝑑𝑑 −𝑑𝑑 … −𝑑𝑑 −𝑑𝑑 �
� �
= (−1)𝑛𝑛+2 (𝑎𝑎0 + 𝑎𝑎𝑛𝑛 ) � 𝑑𝑑 𝑑𝑑 𝑑𝑑 … −𝑑𝑑 −𝑑𝑑 �
�⋮ ⋮ ⋮ ⋱ ⋮ ⋮�
�𝑑𝑑 𝑑𝑑 𝑑𝑑 … 𝑑𝑑 −𝑑𝑑 �
�𝑑𝑑 𝑑𝑑 𝑑𝑑 … 𝑑𝑑 𝑑𝑑�
Cộng dòng cuối vào tất cả các dòng còn lại

2𝑑𝑑 0 0 … 0 0
�2𝑑𝑑 2𝑑𝑑 0 … 0 0�
� … 0 0�� = (−1)𝑛𝑛 2𝑛𝑛−1 (2𝑎𝑎 + 𝑛𝑛𝑛𝑛)𝑑𝑑𝑛𝑛
|𝐴𝐴| = (−1)𝑛𝑛+2 (𝑎𝑎0 + 𝑎𝑎𝑛𝑛 ) �2𝑑𝑑 2𝑑𝑑 2𝑑𝑑
0
� ⋮ ⋮ ⋮ ⋱ ⋮ ⋮�
�2𝑑𝑑 2𝑑𝑑 2𝑑𝑑 … 2𝑑𝑑 0�
� 𝑑𝑑 𝑑𝑑 𝑑𝑑 … 𝑑𝑑 𝑑𝑑 �

Đỗ Minh Triết IX-100 - MATHTASY -


Ôn thi Olympic Toán Đại số Phần IX: Định thức

Phương pháp 3: Sử dụng tính đa tuyến tính (Tách thành tổng các định thức)

Tính chất: Ký hiệu 𝑐𝑐𝑖𝑖 (tương ứng ℎ𝑖𝑖 ) là cột (tương ứng hàng) thứ 𝑖𝑖 của ma trận 𝐴𝐴, khi đó
|𝐴𝐴| = |𝑐𝑐1 … 𝜆𝜆𝑐𝑐𝑖𝑖 … 𝑐𝑐𝑛𝑛 | = 𝜆𝜆|𝑐𝑐1 … 𝑐𝑐𝑖𝑖 … 𝑐𝑐𝑛𝑛 | ∀ 𝜆𝜆 ∈ 𝕂𝕂
|𝐴𝐴| = |𝑐𝑐1 … 𝑐𝑐𝑘𝑘 + 𝑐𝑐𝑘𝑘′ … 𝑐𝑐𝑛𝑛 | = |𝑐𝑐1 … 𝑐𝑐𝑘𝑘 … 𝑐𝑐𝑛𝑛 | + |𝑐𝑐1 … 𝑐𝑐𝑘𝑘′ … 𝑐𝑐𝑛𝑛 |

Ví dụ 1 (Câu 1b Olympic 1993) : Tính


1 + 𝑎𝑎1 𝑏𝑏1 𝑎𝑎1 𝑏𝑏2 … 𝑎𝑎1 𝑏𝑏𝑛𝑛
� �
𝑎𝑎 𝑏𝑏 1 + 𝑎𝑎2 𝑏𝑏2 … 𝑎𝑎2 𝑏𝑏𝑛𝑛
𝐷𝐷 = � 2 1 �
� ⋮ ⋮ ⋱ ⋮ �
� 𝑎𝑎𝑛𝑛 𝑏𝑏1 𝑎𝑎𝑛𝑛 𝑏𝑏2 … 1 + 𝑎𝑎𝑛𝑛 𝑏𝑏𝑛𝑛 �

Giải.

Ta viết mỗi phần tử không nằm trên đường chéo chính dưới dạng 0 + 𝑎𝑎𝑖𝑖 𝑏𝑏𝑖𝑖 , 𝑖𝑖 = 1, … , 𝑛𝑛 để mỗi cột
của định thức là tổng của hai loại cột sau: cột loại (1) là các cột của ma trận đơn vị 𝐼𝐼𝑛𝑛 , cột loại
(2) có dạng 𝑏𝑏𝑗𝑗 . [𝑎𝑎1 𝑎𝑎2 … 𝑎𝑎𝑛𝑛 ]𝑇𝑇 , 𝑗𝑗 = 1, … , 𝑛𝑛.
Sử dụng tính đa tuyến tính để tách 𝐷𝐷 thành tổng các định thức có các cột là một trong hai loại
trên, có 3 loại định thức sau
- Loại 1: chỉ gồm các cột loại (1), đó chính là |𝐼𝐼𝑛𝑛 | = 1 và chỉ có nó là định thức loại này.
- Loại 2: chỉ có đúng một cột loại (2), các cột còn lại loại 1, có 𝑛𝑛 định thức loại này. Dễ thấy định
thức có cột loại 2 ở vị trí cột 𝑗𝑗 bằng 𝑎𝑎𝑗𝑗 𝑏𝑏𝑗𝑗 . Do đó, tổng 𝑛𝑛 định thức loại này là 𝑎𝑎1 𝑏𝑏1 +. . . +𝑎𝑎𝑛𝑛 𝑏𝑏𝑛𝑛 .
- Loại 3: là loại có từ 2 cột loại 2 trở lên, do chúng tỉ lệ nên định thức loại này bằng 0.
Vậy 𝐷𝐷 = 1 + 𝑎𝑎1 𝑏𝑏1 +. . . +𝑎𝑎𝑛𝑛 𝑏𝑏𝑛𝑛 .

Ví dụ 2 : Tính
1 + 𝑎𝑎1 𝑏𝑏1 1 + 𝑎𝑎1 𝑏𝑏2 … 1 + 𝑎𝑎1 𝑏𝑏𝑛𝑛
� �
1 + 𝑎𝑎2 𝑏𝑏1 1 + 𝑎𝑎2 𝑏𝑏2 … 1 + 𝑎𝑎2 𝑏𝑏𝑛𝑛
𝐷𝐷𝑛𝑛 = � �
� ⋮ ⋮ ⋱ ⋮ �
�1 + 𝑎𝑎 𝑛𝑛 𝑏𝑏1 1 + 𝑎𝑎𝑛𝑛 𝑏𝑏2 … 1 + 𝑎𝑎𝑛𝑛 𝑏𝑏𝑛𝑛 �

Giải.

Thực hiện tương tự ví dụ trên, dễ dàng tính được 𝐷𝐷𝑛𝑛 = 0.

Ví dụ 3* : Tính

Đỗ Minh Triết IX-101 - MATHTASY -


Ôn thi Olympic Toán Đại số Phần IX: Định thức

1 + 𝑎𝑎1 + 𝑏𝑏1 𝑎𝑎1 + 𝑏𝑏2 … 𝑎𝑎1 + 𝑏𝑏𝑛𝑛


� �
𝑎𝑎 + 𝑏𝑏1 1 + 𝑎𝑎2 + 𝑏𝑏2 … 𝑎𝑎2 + 𝑏𝑏𝑛𝑛
𝐷𝐷𝑛𝑛 = � 2 �
� ⋮ ⋮ ⋱ ⋮ �
� 𝑎𝑎𝑛𝑛 + 𝑏𝑏1 𝑎𝑎𝑛𝑛 + 𝑏𝑏2 … 1 + 𝑎𝑎𝑛𝑛 + 𝑏𝑏𝑛𝑛 �

Giải.

Tách 𝐷𝐷 thành tổng các định thức mà mỗi cột của chúng là một trong ba loại: cột loại (1) của ma
trận đơn vị, cột loại (2) [𝑎𝑎1 𝑎𝑎2 … 𝑎𝑎𝑛𝑛 ]𝑇𝑇 , cột loại (3) [𝑏𝑏1 𝑏𝑏2 … 𝑏𝑏𝑛𝑛 ]𝑇𝑇 . Có các định thức loại
sau:
- Loại 1: chỉ gồm các cột loại (1), đó chính là |𝐼𝐼𝑛𝑛 | = 1 và chỉ có nó là định thức loại này.
- Loại 2: có từ 2 cột loại (2) hay (3) trở lên, chúng đều bằng 0.
- Loại 3: chỉ có một cột loại (2), còn lại là cột loại (1), có 𝑛𝑛 định thức loại này, tổng của chúng là
𝑎𝑎1 +. . . +𝑎𝑎𝑛𝑛 .
- Loại 4: chỉ có một cột loại (3), còn lại là cột loại (1), có 𝑛𝑛 định thức loại này, tổng của chúng là
𝑏𝑏1 +. . . +𝑏𝑏𝑛𝑛 .
- Loại 5: có đúng 1 cột loại (2), một cột loại (3), còn lại là cột loại (1). Đối với một cặp chỉ số
𝑖𝑖 > 𝑗𝑗, ta có
+ Nếu cột loại (2) ở vị trí 𝑖𝑖, cột loại (3) ở vị trí 𝑗𝑗 thì định thức bằng 𝑎𝑎𝑖𝑖 𝑏𝑏𝑗𝑗 − 𝑎𝑎𝑗𝑗 𝑏𝑏𝑗𝑗 .
+ Nếu cột loại (3) ở vị trí 𝑖𝑖, cột loại (2) ở vị trí 𝑗𝑗 thì định thức bằng 𝑏𝑏𝑖𝑖 𝑎𝑎𝑗𝑗 − 𝑎𝑎𝑖𝑖 𝑏𝑏𝑖𝑖 .
Từ đó
𝐷𝐷 = 1 + � (𝑎𝑎𝑖𝑖 + 𝑏𝑏𝑖𝑖 ) + � �𝑎𝑎𝑖𝑖 − 𝑎𝑎𝑗𝑗 ��𝑏𝑏𝑖𝑖 − 𝑏𝑏𝑗𝑗 �
1≤𝑖𝑖≤𝑛𝑛 1≤𝑖𝑖<𝑗𝑗≤𝑛𝑛

Ví dụ 4 (Câu 6 đề dự tuyển ĐH Khoa học Huế 2010):


1) Tìm mối liên hệ giữa hai định thức sau
𝑎𝑎11 … 𝑎𝑎1𝑛𝑛 𝑎𝑎11 + 𝑥𝑥 … 𝑎𝑎1𝑛𝑛 + 𝑥𝑥
𝐷𝐷 = � ⋮ ⋱ ⋮ ∗
� và 𝐷𝐷 (𝑥𝑥) = � ⋮ ⋱ ⋮ �
𝑎𝑎𝑛𝑛1 … 𝑎𝑎𝑛𝑛𝑛𝑛 𝑎𝑎𝑛𝑛1 + 𝑥𝑥 … 𝑎𝑎𝑛𝑛𝑛𝑛 + 𝑥𝑥
2) Chứng minh rằng tổng các phần bù đại số của các phần tử trong một định thức không
đổi, nếu ta cộng thêm vào tất cả các phần tử cùng một số.

Giải.

1) Ta tách 𝐷𝐷∗ (𝑥𝑥) thành tổng của các định thức: định thức nào chứa hai cột toàn 𝑥𝑥 trở lên
bằng 0, chỉ có một định thức không chứa 𝑥𝑥, chính là 𝐷𝐷. Ngoài ra, còn có 𝑛𝑛 định thức mới
có đúng một cột toàn 𝑥𝑥, các cột còn lại tương ứng là các cột của 𝐷𝐷. Khai triển theo cột
toàn 𝑥𝑥 đó, ta được:

Đỗ Minh Triết IX-102 - MATHTASY -


Ôn thi Olympic Toán Đại số Phần IX: Định thức

𝑛𝑛
𝐷𝐷∗ (𝑥𝑥) = 𝐷𝐷 + 𝑥𝑥 � 𝐴𝐴𝑖𝑖𝑖𝑖
𝑖𝑖,𝑗𝑗=1

trong đó, 𝐴𝐴𝑖𝑖𝑖𝑖 là phần bù đại số của phần tử 𝑎𝑎𝑖𝑖𝑖𝑖 .


2) Kí hiệu 𝑆𝑆𝐴𝐴 , 𝑆𝑆𝐵𝐵 là tổng các phần bù đại số của các phần tử trong ma trận 𝐴𝐴 = �𝑎𝑎𝑖𝑖𝑖𝑖 � và
𝐵𝐵 = �𝑎𝑎𝑖𝑖𝑖𝑖 + 𝑐𝑐� = �𝑏𝑏𝑖𝑖𝑖𝑖 �. Theo trên, ta được:
det 𝐴𝐴 = det�𝑏𝑏𝑖𝑖𝑖𝑖 − 𝑐𝑐� = det 𝐵𝐵 − 𝑐𝑐𝑆𝑆𝐵𝐵 = det�𝑎𝑎𝑖𝑖𝑖𝑖 + 𝑐𝑐� − 𝑐𝑐𝑆𝑆𝐵𝐵 = det 𝐴𝐴 + 𝑐𝑐𝑆𝑆𝐴𝐴 − 𝑐𝑐𝑆𝑆𝐵𝐵
Suy ra 𝑆𝑆𝐴𝐴 = 𝑆𝑆𝐵𝐵 .

Ví dụ (Câu I.2 đề chọn đội tuyển CĐ SP BR-VT 2012) : Tính


3 1 1 … 1
�1 4 1 … 1 �
𝐷𝐷 = ��1 1 5 … 1 �

�⋮ ⋮ ⋮ ⋱ ⋮ �
�1 1 1 … 𝑛𝑛 + 1 �

Giải.

Ký hiệu
2 0 0 … 0
�0 3 0 … 0�
𝐷𝐷∗ = ��0 0 4 … 0 ��
�⋮ ⋮ ⋮ ⋱ ⋮�
�0 0 0 … 𝑛𝑛�
và 𝐷𝐷∗ (𝑥𝑥) là định thức mà các phần tử của nó tương ứng là các phần tử của 𝐷𝐷∗ được cộng thêm
một số 𝑥𝑥, 𝑆𝑆𝐷𝐷∗ là tổng tất cả các phần bù đại số ứng với mọi phần tử của 𝐷𝐷∗ . Theo trên, ta có
𝑛𝑛! 𝑛𝑛! 1 1
𝐷𝐷 = 𝐷𝐷∗ (1) = 𝐷𝐷∗ + 𝑆𝑆𝐷𝐷∗ = 𝑛𝑛! + � +. . . + � = 𝑛𝑛! �1 + +. . . + �
2 𝑛𝑛 2 𝑛𝑛

Ví dụ 5 : Tính
𝑎𝑎1 𝑥𝑥 … 𝑥𝑥
� 𝑥𝑥 𝑎𝑎2 … 𝑥𝑥 �
𝐷𝐷𝑛𝑛 = � ⋮ ⋮ ⋱ ⋮ �� , 𝑥𝑥 ∉ {0, 𝑎𝑎1 , … , 𝑎𝑎𝑛𝑛 }

� 𝑥𝑥 𝑥𝑥 … 𝑎𝑎𝑛𝑛 �

Giải.

Ký hiệu
𝑎𝑎 − 𝑥𝑥 0 … 0
� 1 �
0 𝑎𝑎 − 𝑥𝑥 … 0
𝐷𝐷∗𝑛𝑛 = � 2

� ⋮ ⋮ ⋱ ⋮ �
� 0 0 … 𝑎𝑎𝑛𝑛 − 𝑥𝑥�

Đỗ Minh Triết IX-103 - MATHTASY -


Ôn thi Olympic Toán Đại số Phần IX: Định thức

và 𝐷𝐷∗𝑛𝑛 (𝑐𝑐) là định thức mà các phần tử của nó tương ứng là các phần tử của 𝐷𝐷∗𝑛𝑛 được cộng thêm
một số 𝑐𝑐, 𝑆𝑆𝐷𝐷∗𝑛𝑛 là tổng tất cả các phần bù đại số ứng với mọi phần tử của 𝐷𝐷∗𝑛𝑛 . Theo trên, ta có
𝑛𝑛
𝐷𝐷𝑛𝑛 = 𝐷𝐷∗𝑛𝑛 (𝑥𝑥) + 𝑥𝑥𝑆𝑆𝐷𝐷∗𝑛𝑛 = (𝑎𝑎1 − 𝑥𝑥) … (𝑎𝑎𝑛𝑛 − 𝑥𝑥) + 𝑥𝑥 �(𝑎𝑎1 − 𝑥𝑥) … (𝑎𝑎𝑖𝑖−1 − 𝑥𝑥)(𝑎𝑎𝑖𝑖+1 − 𝑥𝑥) … (𝑎𝑎𝑛𝑛 − 𝑥𝑥)
𝑖𝑖=1
1 1 1
= 𝑥𝑥(𝑎𝑎1 − 𝑥𝑥) … (𝑎𝑎𝑛𝑛 − 𝑥𝑥) � + +. . . + �
𝑥𝑥 𝑎𝑎1 − 𝑥𝑥 𝑎𝑎𝑛𝑛 − 𝑥𝑥

Ví dụ 6 : Tính
𝑎𝑎1 𝑥𝑥 𝑥𝑥 … 𝑥𝑥 𝑥𝑥 𝑥𝑥 𝑥𝑥 𝑥𝑥 𝑎𝑎1
� 𝑦𝑦 𝑎𝑎2 𝑥𝑥 … 𝑥𝑥 � � 𝑥𝑥 𝑥𝑥 𝑥𝑥 𝑎𝑎2 𝑦𝑦 �
𝑛𝑛(𝑛𝑛−1)
𝐷𝐷 = �� 𝑦𝑦 𝑦𝑦 𝑎𝑎3 … 𝑥𝑥 �� = (−1) 2 �� 𝑥𝑥 𝑥𝑥 𝑎𝑎3 𝑦𝑦 𝑦𝑦 ��
�⋮ ⋮ ⋮ ⋱ ⋮ � � ⋮ ⋰ ⋮ ⋮ ⋮�
� 𝑦𝑦 𝑦𝑦 𝑦𝑦 𝑦𝑦 𝑎𝑎𝑛𝑛 � �𝑎𝑎𝑛𝑛 𝑦𝑦 𝑦𝑦 𝑦𝑦 𝑦𝑦 �

Giải.

Gọi 𝑆𝑆𝐷𝐷 là tổng tất cả các phần bù đại số ứng với mọi phần tử của 𝐷𝐷 và 𝐷𝐷∗ (𝑐𝑐) là định thức mà
mọi phần tử của nó tương ứng là phần tử của 𝐷𝐷 được cộng thêm một số 𝑐𝑐. Khi đó, ta có
𝐷𝐷 − 𝑥𝑥𝑆𝑆𝐷𝐷 = 𝐷𝐷∗ (−𝑥𝑥) = (𝑎𝑎1 − 𝑥𝑥) … (𝑎𝑎𝑛𝑛 − 𝑥𝑥) 𝑦𝑦𝑦𝑦 − 𝑥𝑥𝑥𝑥𝑆𝑆𝐷𝐷 = 𝑦𝑦(𝑎𝑎1 − 𝑥𝑥) … (𝑎𝑎𝑛𝑛 − 𝑥𝑥)
� ∗ ⇒�
𝐷𝐷 − 𝑦𝑦𝑆𝑆𝐷𝐷 = 𝐷𝐷 (−𝑦𝑦) = (𝑎𝑎1 − 𝑦𝑦) … (𝑎𝑎𝑛𝑛 − 𝑦𝑦) 𝑥𝑥𝑥𝑥 − 𝑥𝑥𝑥𝑥𝑆𝑆𝐷𝐷 = 𝑥𝑥(𝑎𝑎1 − 𝑦𝑦) … (𝑎𝑎𝑛𝑛 − 𝑦𝑦)
Suy ra
𝑥𝑥(𝑎𝑎1 − 𝑦𝑦) … (𝑎𝑎𝑛𝑛 − 𝑦𝑦) − 𝑦𝑦(𝑎𝑎1 − 𝑥𝑥) … (𝑎𝑎𝑛𝑛 − 𝑥𝑥)
𝐷𝐷 =
𝑥𝑥 − 𝑦𝑦

Ví dụ 7* (Tổng quát câu 6a Olympic 2011) : Tính


𝑥𝑥 + 1 𝑥𝑥2 + 1 … 𝑥𝑥𝑛𝑛 + 1
� 12 2 2 �
𝐷𝐷 = �𝑥𝑥1 + 1 𝑥𝑥2 + +1 … 𝑥𝑥𝑛𝑛 + 1�
� ⋮ ⋮ ⋱ ⋮ �
� 𝑥𝑥𝑛𝑛1 + 1 𝑥𝑥𝑛𝑛2 + 1 … 𝑥𝑥𝑛𝑛𝑛𝑛 + 1�

Giải.
1 0 0 … 0
𝑥𝑥1 + 1 𝑥𝑥2 + 1 … 𝑥𝑥𝑛𝑛 + 1 �1 𝑥𝑥 + 1
� 2 � 1 𝑥𝑥2 + 1 … 𝑥𝑥𝑛𝑛 + 1�
2 2 � �
𝐷𝐷 = �𝑥𝑥1 + 1 𝑥𝑥2 + +1 … 𝑥𝑥𝑛𝑛 + 1� = �1 𝑥𝑥21 + 1 𝑥𝑥22 + 1 … 𝑥𝑥2𝑛𝑛 + 1�
� ⋮ ⋮ ⋱ ⋮ �
�⋮ ⋮ ⋮ ⋱ ⋮ �
� 𝑥𝑥𝑛𝑛1 + 1 𝑥𝑥𝑛𝑛2 + 1 … 𝑥𝑥𝑛𝑛𝑛𝑛 + 1�
�1 𝑥𝑥𝑛𝑛1 + 1 𝑥𝑥𝑛𝑛2 + 1 … 𝑥𝑥𝑛𝑛𝑛𝑛 + 1�
0+1 −1 + 1 −1 + 1 … −1 + 1 1 −1 −1 … −1
�0 + 1 𝑥𝑥1 + 1 𝑥𝑥2 + 1 … 𝑥𝑥𝑛𝑛 + 1� �1 𝑥𝑥1 𝑥𝑥2 … 𝑥𝑥𝑛𝑛 �
� � � �
= �0 + 1 𝑥𝑥21 + 1 𝑥𝑥22 + 1 … 𝑥𝑥2𝑛𝑛 + 1� = �1 𝑥𝑥21 𝑥𝑥22 … 𝑥𝑥2𝑛𝑛 �
� ⋮ ⋮ ⋮ ⋱ ⋮ � �⋮ ⋮ ⋮ ⋱ ⋮ �
�0 + 1 𝑥𝑥𝑛𝑛1 + 1 𝑥𝑥𝑛𝑛2 + 1 … 𝑥𝑥𝑛𝑛𝑛𝑛 + 1� �1 𝑥𝑥𝑛𝑛1 𝑥𝑥𝑛𝑛2 … 𝑥𝑥𝑛𝑛𝑛𝑛 �

Đỗ Minh Triết IX-104 - MATHTASY -


Ôn thi Olympic Toán Đại số Phần IX: Định thức

2 −1 −1 … −1 −1 −1 −1 … −1
�0 𝑥𝑥 𝑥𝑥 … 𝑥𝑥 � � 1 𝑥𝑥 𝑥𝑥2 … 𝑥𝑥𝑛𝑛 �
1 2 𝑛𝑛 1
� � � �
= �0 𝑥𝑥21 𝑥𝑥22 … 𝑥𝑥2𝑛𝑛 � + � 1 𝑥𝑥21 𝑥𝑥22 … 𝑥𝑥2𝑛𝑛 �
�⋮ ⋮ ⋮ ⋱ ⋮ � � ⋮ ⋮ ⋮ ⋱ ⋮ �
�0 𝑥𝑥1 𝑥𝑥2 … 𝑥𝑥𝑛𝑛 � � 1 𝑥𝑥𝑛𝑛1
𝑛𝑛 𝑛𝑛 𝑛𝑛
𝑥𝑥𝑛𝑛2 … 𝑥𝑥𝑛𝑛𝑛𝑛 �
−1 −1 −1 … −1
� 0 𝑥𝑥 − 1 𝑥𝑥2 − 1 … 𝑥𝑥𝑛𝑛 − 1�
𝑛𝑛 1
� �
= 2 � 𝑥𝑥𝑖𝑖 � �𝑥𝑥𝑖𝑖 − 𝑥𝑥𝑗𝑗 � + � 0 𝑥𝑥21 − 1 𝑥𝑥22 − 1 … 𝑥𝑥2𝑛𝑛 − 1�
𝑖𝑖=1 1≤𝑗𝑗<𝑖𝑖≤𝑛𝑛 � ⋮ ⋮ ⋮ ⋱ ⋮ �
𝑛𝑛 𝑛𝑛 𝑛𝑛
� 0 𝑥𝑥1 − 1 𝑥𝑥2 − 1 … 𝑥𝑥𝑛𝑛 − 1�
1 1 … 1
� 𝑥𝑥1 + 1 𝑥𝑥2 + 1 … 𝑥𝑥𝑛𝑛 + 1 �
𝑛𝑛 𝑛𝑛
� �
= 2 � 𝑥𝑥𝑖𝑖 � �𝑥𝑥𝑖𝑖 − 𝑥𝑥𝑗𝑗 � − �(𝑥𝑥𝑖𝑖 − 1) � 𝑥𝑥21 + 𝑥𝑥1 + 1 𝑥𝑥22 + 𝑥𝑥2 + 1 … 𝑥𝑥2𝑛𝑛 + 𝑥𝑥 + 𝑛𝑛 + 1 �
𝑖𝑖=1 1≤𝑗𝑗<𝑖𝑖≤𝑛𝑛 𝑖𝑖=1 � ⋮ ⋮ ⋱ ⋮ �
𝑛𝑛−1
𝑥𝑥
� 1 + ⋯ + 𝑥𝑥1 + 1 𝑥𝑥𝑛𝑛−1
2 + ⋯ + 𝑥𝑥2 + 1 … 𝑥𝑥𝑛𝑛−1
𝑛𝑛 + ⋯ + 𝑥𝑥𝑛𝑛 + 1�
1 1 … 1
𝑛𝑛 � 𝑥𝑥 𝑛𝑛
𝑥𝑥2 … 𝑥𝑥𝑛𝑛 �
= 2 � 𝑥𝑥𝑖𝑖 � �𝑥𝑥𝑖𝑖 − 𝑥𝑥𝑗𝑗 � − �(𝑥𝑥𝑖𝑖 − 1) � 1 �
𝑖𝑖=1 1≤𝑗𝑗<𝑖𝑖≤𝑛𝑛 𝑖𝑖=1
� ⋮ ⋮ ⋱ ⋮ �
𝑛𝑛−1
�𝑥𝑥𝑛𝑛−1
1 𝑥𝑥𝑛𝑛−1
2 … 𝑥𝑥𝑛𝑛 �
𝑛𝑛 𝑛𝑛
= �2 � 𝑥𝑥𝑖𝑖 − �(𝑥𝑥𝑖𝑖 − 1)� � �𝑥𝑥𝑖𝑖 − 𝑥𝑥𝑗𝑗 �
𝑖𝑖=1 𝑖𝑖=1 1≤𝑗𝑗<𝑖𝑖≤𝑛𝑛

Phương pháp 4: Tách thành tích các định thức

Ví dụ 1 : Tính
𝑎𝑎 + 𝑏𝑏1 𝑎𝑎1 + 𝑏𝑏2 … 𝑎𝑎1 + 𝑏𝑏𝑛𝑛
� 1 �
𝑎𝑎 + 𝑏𝑏1 𝑎𝑎2 + 𝑏𝑏2 … 𝑎𝑎2 + 𝑏𝑏𝑛𝑛
� 2 �
𝐷𝐷 = � 𝑎𝑎3 + 𝑏𝑏1 𝑎𝑎3 + 𝑏𝑏2 … 𝑎𝑎3 + 𝑏𝑏𝑛𝑛 �
� ⋮ ⋮ ⋱ ⋮ �
�𝑎𝑎𝑛𝑛 + 𝑏𝑏1 𝑎𝑎𝑛𝑛 + 𝑏𝑏2 … 𝑎𝑎𝑛𝑛 + 𝑏𝑏𝑛𝑛 �
𝑎𝑎 + 𝑏𝑏1 𝑎𝑎1 + 𝑏𝑏2 … 𝑎𝑎1 + 𝑏𝑏𝑛𝑛 𝑎𝑎 1 0 … 0 1 1 … 1
� 1 � �⎛ 1 �
𝑎𝑎 + 𝑏𝑏1 𝑎𝑎2 + 𝑏𝑏2 … 𝑎𝑎2 + 𝑏𝑏𝑛𝑛 𝑎𝑎 1 0 … 0⎞ ⎛𝑏𝑏 𝑏𝑏2 … 𝑏𝑏𝑛𝑛 ⎞
� 2 � �⎜⎜ 2 ⎟
⎟⎜⎜ 1 ⎟
⎟�
𝐷𝐷 = � 𝑎𝑎3 + 𝑏𝑏1 𝑎𝑎3 + 𝑏𝑏2 … 𝑎𝑎3 + 𝑏𝑏𝑛𝑛 � = �⎜
⎜ 𝑎𝑎3 1 0 … 0⎟⎟ ⎜
⎜ 0 0 … 0⎟ ⎟�
⎜ ⎟
� ⋮ ⋮ ⋱ ⋮ ⎜
� � ⋮ ⋮ ⋮ ⋱ ⋮⎟⎜ ⋮ ⋮ ⋱ ⋮ ⎟�
�𝑎𝑎𝑛𝑛 + 𝑏𝑏1 𝑎𝑎𝑛𝑛 + 𝑏𝑏2 … 𝑎𝑎𝑛𝑛 + 𝑏𝑏𝑛𝑛 � �⎝𝑎𝑎𝑛𝑛 1 0 … 0⎠ ⎝ 0 0 … 0 ⎠�
Từ đó, ta có kết luận
𝑎𝑎1 + 𝑏𝑏1 nếu 𝑛𝑛 = 1
𝐷𝐷 = �(𝑎𝑎1 − 𝑎𝑎2 )(𝑏𝑏2 − 𝑏𝑏1 ) nếu 𝑛𝑛 = 2
0 nếu 𝑛𝑛 > 2

Ví dụ 2 : Tính

Đỗ Minh Triết IX-105 - MATHTASY -


Ôn thi Olympic Toán Đại số Phần IX: Định thức

cos(𝑥𝑥1 − 𝑦𝑦1 ) cos(𝑥𝑥1 − 𝑦𝑦2 ) … cos(𝑥𝑥1 − 𝑦𝑦𝑛𝑛 )


� �
cos(𝑥𝑥2 − 𝑦𝑦1 ) cos(𝑥𝑥2 − 𝑦𝑦2 ) … cos(𝑥𝑥2 − 𝑦𝑦𝑛𝑛 ) �
𝐷𝐷 = �
� ⋮ ⋮ ⋱ ⋮ �
�cos(𝑥𝑥𝑛𝑛 − 𝑦𝑦1 ) cos(𝑥𝑥𝑛𝑛 − 𝑦𝑦2 ) … cos(𝑥𝑥𝑛𝑛 − 𝑦𝑦𝑛𝑛 )�
cos(𝑥𝑥1 − 𝑦𝑦1 ) cos(𝑥𝑥1 − 𝑦𝑦2 ) … cos(𝑥𝑥1 − 𝑦𝑦𝑛𝑛 )
� �
cos(𝑥𝑥2 − 𝑦𝑦1 ) cos(𝑥𝑥2 − 𝑦𝑦2 ) … cos(𝑥𝑥2 − 𝑦𝑦𝑛𝑛 ) �
𝐷𝐷 = �
� ⋮ ⋮ ⋱ ⋮ �
�cos(𝑥𝑥𝑛𝑛 − 𝑦𝑦1 ) cos(𝑥𝑥𝑛𝑛 − 𝑦𝑦2 ) … cos(𝑥𝑥𝑛𝑛 − 𝑦𝑦𝑛𝑛 )�
cos 𝑥𝑥1 sin 𝑥𝑥1 0 … 0 cos 𝑦𝑦1 cos 𝑦𝑦2 cos 𝑦𝑦3 … cos 𝑦𝑦𝑛𝑛
�⎛ �
cos 𝑥𝑥2 sin 𝑥𝑥2 0 … 0⎞ ⎛ sin 𝑦𝑦1 sin 𝑦𝑦2 sin 𝑦𝑦3 … sin 𝑦𝑦𝑛𝑛 ⎞
�⎜⎜ ⎟
⎟⎜ ⎜ ⎟
⎟ �
= �⎜ ⎜ cos 𝑥𝑥3 sin 𝑥𝑥3 0 … 0⎟ ⎟ ⎜ 0 0 0 … 0 ⎟ �
⎜ ⎟⎜ ⎟
� ⎜ ⋮ ⋮ ⋮ ⋱ ⋮⎟⎜ ⋮ ⋮ ⋮ ⋱ ⋮ ⎟�
�⎝cos 𝑥𝑥𝑛𝑛 sin 𝑥𝑥𝑛𝑛 0 … 0⎠ ⎝ 0 0 0 … 0 ⎠�
Từ đó, ta có kết luận
cos(𝑥𝑥1 − 𝑦𝑦1 ) nếu 𝑛𝑛 = 1
𝐷𝐷 = �sin(𝑥𝑥1 − 𝑥𝑥2 ) sin(𝑦𝑦1 − 𝑦𝑦2 ) nếu 𝑛𝑛 = 2
0 nếu 𝑛𝑛 > 2

Ví dụ : Cho 𝐴𝐴 là ma trận vuông cấp 𝑛𝑛 có dạng 𝐴𝐴 = �𝑎𝑎𝑖𝑖𝑖𝑖 �, trong đó 𝑎𝑎𝑖𝑖𝑖𝑖 = min{𝑖𝑖, 𝑗𝑗}.
Tính det 𝐴𝐴.

Giải.

Ta có
1 1 1 … 1 1 0 0 … 0 1 1 1 … 1
�1 2 2 … 2 � �⎛1 1 0 … 0⎞ ⎛0 1 1 … 1⎞ �
⎜ ⎟⎜⎜ ⎟

det 𝐴𝐴 = ��1 2 3 … 3 �� = ��⎜

⎜1 1 1 … 0⎟⎟
⎟ ⎜
⎜0 0 1 … 1⎟⎟
�=1

� ⋮ ⋮ ⋮ ⋱ ⋮ � �⎜ ⋮ ⋮ ⋮ ⋱ ⋮ ⎟ ⎜ ⋮ ⋮ ⋮ ⋱ ⎟
⋮ �
�1 2 3 … 𝑛𝑛� �⎝1 1 1 … 1⎠ ⎝0 0 0 … 1⎠ �

Ví dụ 3 (Câu 2 đề dự tuyển <Không rõ 1> 2009) :


Cho 2009 đa thức 𝑓𝑓𝑗𝑗 (𝑥𝑥) = 𝑎𝑎0,𝑗𝑗 + 𝑎𝑎1,𝑗𝑗 𝑥𝑥+. . . +𝑎𝑎2007,𝑗𝑗 𝑥𝑥2007 với 𝑗𝑗 ∈ {1,2, … ,2009} và ma trận vuông
cấp 2009 như sau
𝑓𝑓1 (1) 𝑓𝑓1 (2) … 𝑓𝑓1 (2009)
⎛ 𝑓𝑓2 (1) 𝑓𝑓2 (2) … 𝑓𝑓2 (2009) ⎞
𝐴𝐴 = ⎜

⎜ ⋮



⋮ ⋱ ⋮
𝑓𝑓
⎝ 2009 (1) 𝑓𝑓2009 (2) … 𝑓𝑓2009 (2009) ⎠
Tính det 𝐴𝐴.

Giải.

Cách 1:

Đỗ Minh Triết IX-106 - MATHTASY -


Ôn thi Olympic Toán Đại số Phần IX: Định thức

Ta tách ma trận 𝐴𝐴 thành tích 2 ma trận như sau:


𝑎𝑎0,1 𝑎𝑎1,1 𝑎𝑎2,1 … 𝑎𝑎2007,1 0 1 1 1 … 1 1

⎜ 𝑎𝑎0,2 𝑎𝑎1,2 𝑎𝑎2,2 … 𝑎𝑎2007,2 0⎞⎟
⎜ ⎟⎜⎛1 2 3 … 2008 2009 ⎞ ⎟
⎜ 0⎟ ⎜

⎜ 𝑎𝑎0,3 𝑎𝑎1,3 𝑎𝑎2,3 … 𝑎𝑎2007,3 ⎟
⎟⎜⎜1 22 32 … 20082 20092 ⎟ ⎟


⎜ ⋮ ⋮ ⋮ ⋱ ⋮ ⋮⎟⎟ ⎜
⎜ ⋮ ⋮ ⋮ ⋱ ⋮ ⋮ ⎟

⎜ ⎟ ⎜
⎜ ⎟
⎜ ⎟ 2007 ⎟
⎜𝑎𝑎 0,2008 𝑎𝑎1,2008 𝑎𝑎2,2008 … 𝑎𝑎2007,2008 0⎟ 1 22007 32007 … 20082007 2009
⎝𝑎𝑎0,2009 𝑎𝑎1,2009 𝑎𝑎2,2009 … 𝑎𝑎2007,2009 0⎠ ⎝0 0 0 … 0 0 ⎠

Suy ra det 𝐴𝐴 = 0.

Cách 2:

Xét không gian vector ℝ2007 [𝑥𝑥] gồm các đa thức với hệ số thực và có bậc không quá 2007. Ta có
dim ℝ2007 [𝑥𝑥] = 2008. Do đó các vector 𝑓𝑓1 (𝑥𝑥), 𝑓𝑓2 (𝑥𝑥), … , 𝑓𝑓2009 (𝑥𝑥) là phụ thuộc tuyến tính, tức là tồn
tại các số thực 𝜆𝜆1 , 𝜆𝜆2 , … , 𝜆𝜆2009 không đồng thời bằng không sao cho
𝜆𝜆1 𝑓𝑓1 (𝑥𝑥) + 𝜆𝜆2 𝑓𝑓2 (𝑥𝑥)+. . . +𝜆𝜆2009 𝑓𝑓2009 (𝑥𝑥) = 0
từ đó suy ra det 𝐴𝐴 = 0.

Phương pháp 5: Truy hồi

Giả sử ta đang tính định thức cấp 𝑛𝑛 ≥ 2 là 𝐷𝐷𝑛𝑛 , sau các phép biến đổi sơ cấp và khai triển, ta
nhận được dãy truy hồi tuyến tính cấp 2:
𝐷𝐷𝑛𝑛 = 𝑝𝑝𝐷𝐷𝑛𝑛−1 + 𝑞𝑞𝐷𝐷𝑛𝑛−2
Nếu 𝑞𝑞 = 0 thì 𝐷𝐷𝑛𝑛 = 𝑝𝑝𝑛𝑛−1 𝐷𝐷1 (dễ thấy)
Nếu 𝑞𝑞 ≠ 0 thì 𝐷𝐷𝑛𝑛 có dạng 𝐷𝐷𝑛𝑛 = 𝑎𝑎𝑥𝑥𝑛𝑛1 + 𝑏𝑏𝑥𝑥𝑛𝑛2 với 𝑥𝑥1 , 𝑥𝑥2 là hai nghiệm của phương trình 𝑥𝑥2 − 𝑝𝑝𝑝𝑝 −
𝑞𝑞 = 0.

*Nếu 𝑥𝑥1 ≠ 𝑥𝑥2 thì


𝐷𝐷2 − 𝑥𝑥2 𝐷𝐷1 𝑛𝑛 𝐷𝐷2 − 𝑥𝑥1 𝐷𝐷1 𝑛𝑛
𝐷𝐷𝑛𝑛 = 𝑥𝑥 − 𝑥𝑥
𝑥𝑥1 (𝑥𝑥1 − 𝑥𝑥2 ) 1 𝑥𝑥2 (𝑥𝑥1 − 𝑥𝑥2 ) 2
Chứng minh
𝑥𝑥1 , 𝑥𝑥2 là hai nghiệm phân biệt của phương trình 𝑥𝑥2 − 𝑝𝑝𝑝𝑝 − 𝑞𝑞 = 0, áp dụng định lý Viete, ta có
𝑥𝑥1 + 𝑥𝑥2 = 𝑝𝑝

𝑥𝑥1 𝑥𝑥2 = −𝑞𝑞
nên 𝐷𝐷𝑛𝑛 = 𝑝𝑝𝐷𝐷𝑛𝑛−1 + 𝑞𝑞𝐷𝐷𝑛𝑛−2 = (𝑥𝑥1 + 𝑥𝑥2 )𝐷𝐷𝑛𝑛−1 − 𝑥𝑥1 𝑥𝑥2 𝐷𝐷𝑛𝑛−2 , ta được
𝐷𝐷 − 𝑥𝑥2 𝐷𝐷𝑛𝑛−1 = 𝑥𝑥1 (𝐷𝐷𝑛𝑛−1 − 𝑥𝑥2 𝐷𝐷𝑛𝑛−2 ) =. . . = 𝑥𝑥𝑛𝑛−2 (𝐷𝐷2 − 𝑥𝑥2 𝐷𝐷1 )
� 𝑛𝑛 1
𝐷𝐷𝑛𝑛 − 𝑥𝑥1 𝐷𝐷𝑛𝑛−1 = 𝑥𝑥2 (𝐷𝐷𝑛𝑛−1 − 𝑥𝑥1 𝐷𝐷𝑛𝑛−2 ) =. . . = 𝑥𝑥𝑛𝑛−2
2 (𝐷𝐷2 − 𝑥𝑥1 𝐷𝐷1 )
Nhân 𝑥𝑥1 vào hai vế đẳng thức đầu, nhân 𝑥𝑥2 vào hai vế đẳng thức sau, rồi trừ theo vế, ta được
đẳng thức cần chứng minh.

Đỗ Minh Triết IX-107 - MATHTASY -


Ôn thi Olympic Toán Đại số Phần IX: Định thức

Nhận xét: trong trường hợp này, công thức tính 𝐷𝐷𝑛𝑛 khá phức tạp, khó nhớ, vì thế trong thực hành,
ta nên lặp lại các bước như trong phần chứng minh.
Một cách khác, nếu 𝐷𝐷𝑛𝑛 có dạng 𝐷𝐷𝑛𝑛 = 𝑎𝑎𝑥𝑥𝑛𝑛1 + 𝑏𝑏𝑥𝑥𝑛𝑛2 với 𝑥𝑥1 , 𝑥𝑥2 là hai nghiệm của phương
trình 𝑥𝑥2 − 𝑝𝑝𝑝𝑝 − 𝑞𝑞 = 0 thì ta có thể coi 𝐷𝐷0 = 1 để công thức truy hồi đúng với 𝑛𝑛 ≥ 2, khi đó 𝑎𝑎, 𝑏𝑏
được xác định bởi hệ:
𝐷𝐷 = 1 = 𝑎𝑎𝑥𝑥01 + 𝑏𝑏𝑥𝑥02 = 𝑎𝑎 + 𝑏𝑏
� 0
𝐷𝐷1 = 𝑎𝑎𝑥𝑥11 + 𝑏𝑏𝑥𝑥12 = 𝑎𝑎𝑥𝑥1 + 𝑏𝑏𝑥𝑥2

*Nếu 𝑥𝑥1 = 𝑥𝑥2 thì


𝐷𝐷𝑛𝑛 = (𝑛𝑛 − 1)𝑥𝑥𝑛𝑛−2
1 𝐷𝐷2 − (𝑛𝑛 − 2)𝑥𝑥𝑛𝑛−1
1 𝐷𝐷1
Chứng minh
Nếu 𝑥𝑥1 = 𝑥𝑥2 thì từ trên, ta có 𝐷𝐷𝑛𝑛 − 𝑥𝑥1 𝐷𝐷𝑛𝑛−1 = 𝑥𝑥1 (𝐷𝐷𝑛𝑛−1 − 𝑥𝑥1 𝐷𝐷𝑛𝑛−2 ) =. . . = 𝑥𝑥𝑛𝑛−2
1 (𝐷𝐷2 − 𝑥𝑥1 𝐷𝐷1 )
Đặt 𝐴𝐴 = 𝐷𝐷2 − 𝑥𝑥1 𝐷𝐷1 và thay 𝑛𝑛 bởi 𝑛𝑛 − 1 trong đẳng thức trên, ta được
𝐷𝐷𝑛𝑛 = 𝑥𝑥1 𝐷𝐷𝑛𝑛−1 + 𝑥𝑥𝑛𝑛−2
1 𝐴𝐴 (1)
𝐷𝐷𝑛𝑛−1 = 𝑥𝑥1 𝐷𝐷𝑛𝑛−2 + 𝑥𝑥𝑛𝑛−3
1 𝐴𝐴 (2)
Thay (2) vào (1), ta được 𝐷𝐷𝑛𝑛 = 𝑥𝑥21 𝐷𝐷𝑛𝑛−2 + 2𝑥𝑥𝑛𝑛−2
1 𝐴𝐴.
Tiếp tục quá trình này, cuối cùng ta có
𝐷𝐷𝑛𝑛 = 𝑥𝑥𝑛𝑛−1
1 𝐷𝐷1 + (𝑛𝑛 − 1)𝑥𝑥𝑛𝑛−2
1 𝐴𝐴 = 𝑥𝑥𝑛𝑛−1
1 𝐷𝐷1 + (𝑛𝑛 − 1)𝑥𝑥𝑛𝑛−2
1 (𝐷𝐷2 − 𝑥𝑥1 𝐷𝐷1 )
𝑛𝑛−2 𝑛𝑛−1
= (𝑛𝑛 − 1)𝑥𝑥1 𝐷𝐷2 − (𝑛𝑛 − 2)𝑥𝑥1 𝐷𝐷1

Ví dụ 1 : Tính
1 1 0 0 … 0 0
� −1 1 1 0 … 0 0�
� 0��
𝐷𝐷𝑛𝑛 = � 0 −1 1 1 … 0
� ⋮ ⋮ ⋮ ⋮ … ⋮ ⋮�
� 0 0 0 0 … 1 1�
� 0 0 0 0 … −1 1�
Giải.

Cách 1:

Khai triển theo dòng đầu, ta được: 𝐷𝐷𝑛𝑛 = 𝐷𝐷𝑛𝑛−1 + 𝐷𝐷𝑛𝑛−2 .


Ta có 𝐷𝐷1 = 1, 𝐷𝐷2 = 2. (𝐷𝐷𝑛𝑛 chính là số Fibonacci thứ 𝑛𝑛)
Phương trình 𝑥𝑥2 − 𝑥𝑥 − 1 = 0 có 2 nghiệm phân biệt:

1± 5
𝑥𝑥1,2 =
2
Áp dụng chứng minh hoặc công thức, tính toán, ta được:

Đỗ Minh Triết IX-108 - MATHTASY -


Ôn thi Olympic Toán Đại số Phần IX: Định thức

√ 𝑛𝑛+1 √ 𝑛𝑛+1
1 1+ 5 1− 5
𝐷𝐷𝑛𝑛 = √ �� � −� � �
5 2 2

Cách 2:

Vì phương trình 𝑥𝑥2 − 𝑥𝑥 − 1 = 0 có 2 nghiệm phân biệt trên nên 𝐷𝐷𝑛𝑛 = 𝑎𝑎𝑥𝑥𝑛𝑛1 + 𝑏𝑏𝑥𝑥𝑛𝑛2
Coi 𝐷𝐷0 = 1 để công thức truy hồi đúng với 𝑛𝑛 ≥ 2, khi đó ta có hệ phương trình:
√ 0 √ 0
⎧ 1+ 5 1− 5

�𝐷𝐷0 = 𝑎𝑎 � 2 � + 𝑏𝑏 � 2 � = 1
⎨ √ 1 √ 1
� 1+ 5 1− 5
�𝐷𝐷1 = 𝑎𝑎 � � + 𝑏𝑏 � � =1
⎩ 2 2
Giải hệ tìm 𝑎𝑎, 𝑏𝑏, ta được kết quả trên.

Ví dụ 2 : Tính
𝑎𝑎 𝑏𝑏 0 … 0 𝑎𝑎 + 𝑏𝑏 𝑏𝑏 0 … 0
� 𝑐𝑐 𝑎𝑎 𝑏𝑏 … 0� � 𝑎𝑎 𝑎𝑎 + 𝑏𝑏 𝑏𝑏 … 0 �
𝑎𝑎) 𝐷𝐷𝑛𝑛 = ��0 𝑐𝑐 𝑎𝑎 … 0�� 𝑏𝑏) 𝐷𝐷𝑛𝑛 = �� 0 𝑎𝑎 𝑎𝑎 + 𝑏𝑏 … 0


�⋮ ⋮ ⋮ ⋱ ⋮� �⋮ ⋮ ⋮ ⋱ ⋮ �
� 0 0 0 … 𝑎𝑎� �0 0 0 … 𝑎𝑎 + 𝑏𝑏 �
𝑎𝑎 + 𝑏𝑏 𝑎𝑎𝑎𝑎 0 … 0
� 1 𝑎𝑎 + 𝑏𝑏 𝑎𝑎𝑎𝑎 … 0 �

𝑐𝑐) 𝐷𝐷𝑛𝑛 = � 0 �
1 𝑎𝑎 + 𝑏𝑏 … 0 � 𝑑𝑑) 𝐷𝐷𝑛𝑛
�⋮ ⋮ ⋮ ⋱ ⋮ �
�0 0 0 … 𝑎𝑎 + 𝑏𝑏 �
2 cos 𝜑𝜑 1 0 … 0
� 1 2 cos 𝜑𝜑 1 �
… 0
� �
=� 0 1 2 cos 𝜑𝜑 … 0 �
�⋮ ⋮ ⋮ ⋱ ⋮ �
�0 0 0 … 2 cos 𝜑𝜑 �
e) Chứng minh rằng
cos 𝜑𝜑 1 0 … 0
� 1 2 cos 𝜑𝜑 1 �
… 0
� �
𝐷𝐷 = � 0 1 2 cos 𝜑𝜑 … 0 � = cos(𝑛𝑛𝑛𝑛)
�⋮ ⋮ ⋮ ⋱ ⋮ �
�0 0 0 … 2 cos 𝜑𝜑 �

Giải.

a) Khai triển theo cột đầu, ta được


𝐷𝐷𝑛𝑛 = 𝑎𝑎𝐷𝐷𝑛𝑛−1 − 𝑏𝑏𝑏𝑏𝐷𝐷𝑛𝑛−2
*Nếu phương trình 𝑥𝑥2 − 𝑎𝑎𝑎𝑎 + 𝑏𝑏𝑏𝑏 = 0 có 2 nghiệm phân biệt 𝑥𝑥1 , 𝑥𝑥2
Coi 𝐷𝐷0 = 1 để công thức này đúng ∀𝑛𝑛 ≥ 2. Khi đó tồn tại hai số 𝛼𝛼, 𝛽𝛽 sao cho 𝐷𝐷𝑛𝑛 = 𝛼𝛼𝑥𝑥𝑛𝑛1 + 𝛽𝛽𝑥𝑥𝑛𝑛2 .

Đỗ Minh Triết IX-109 - MATHTASY -


Ôn thi Olympic Toán Đại số Phần IX: Định thức

Từ 𝐷𝐷0 = 𝛼𝛼𝑥𝑥01 + 𝛽𝛽𝑥𝑥02 = 1, 𝐷𝐷1 = 𝛼𝛼𝑥𝑥11 + 𝛽𝛽𝑥𝑥12 = 𝑎𝑎, ta có


𝑥𝑥1
⎧𝛼𝛼 =
� 𝑥𝑥1 − 𝑥𝑥2
⎨ 𝑥𝑥2
�𝛽𝛽 =
⎩ 𝑥𝑥2 − 𝑥𝑥1
Do đó
𝑛𝑛
1
𝐷𝐷𝑛𝑛 = (𝑥𝑥𝑛𝑛+1 − 𝑥𝑥 𝑛𝑛+1
) = � 𝑥𝑥𝑘𝑘1 𝑥𝑥𝑛𝑛−𝑘𝑘
2
𝑥𝑥1 − 𝑥𝑥2 1 2
𝑘𝑘=1

Tìm 𝑥𝑥1 , 𝑥𝑥2 rồi thay vào (nghiệm xấu).


* Nếu phương trình 𝑥𝑥2 − 𝑎𝑎𝑎𝑎 + 𝑏𝑏𝑏𝑏 = 0 có nghiệm kép
Áp dụng chứng minh hay công thức 𝐷𝐷𝑛𝑛 = (𝑛𝑛 − 1)𝑥𝑥𝑛𝑛−2
1 𝐷𝐷2 − (𝑛𝑛 − 2)𝑥𝑥𝑛𝑛−1
1 𝐷𝐷1

b) * Nếu 𝑎𝑎 ≠ 𝑏𝑏

Cách 1: Dùng lý luận trên

Cách 2: Áp dụng công thức

Ta có 𝐷𝐷𝑛𝑛 = (𝑎𝑎 + 𝑏𝑏)𝐷𝐷𝑛𝑛−1 − 𝑎𝑎𝑎𝑎𝐷𝐷𝑛𝑛−2


𝐷𝐷1 = 𝑎𝑎 + 𝑏𝑏, 𝐷𝐷2 = (𝑎𝑎 + 𝑏𝑏)2 − 𝑎𝑎𝑎𝑎 = 𝑎𝑎2 + 𝑏𝑏2 + 𝑎𝑎𝑎𝑎
Phương trình 𝑥𝑥2 − (𝑎𝑎 + 𝑏𝑏) + 𝑎𝑎𝑎𝑎 = 0 có 2 nghiệm phân biệt 𝑎𝑎, 𝑏𝑏
Áp dụng công thức, tính toán, ta được:
1
𝐷𝐷𝑛𝑛 = (𝑎𝑎𝑛𝑛+1 − 𝑏𝑏𝑛𝑛+1 )
𝑎𝑎 − 𝑏𝑏
Cách 3: Áp dụng chứng minh

𝐷𝐷𝑛𝑛 − 𝑎𝑎𝐷𝐷𝑛𝑛−1 = 𝑏𝑏(𝐷𝐷𝑛𝑛−1 − 𝑎𝑎𝐷𝐷𝑛𝑛−2 ) =. . . = 𝑏𝑏𝑛𝑛−2 (𝐷𝐷2 − 𝑎𝑎𝐷𝐷1 ) = 𝑏𝑏𝑛𝑛 1


� 𝑛𝑛−2 𝑛𝑛
⇒ 𝐷𝐷𝑛𝑛 = (𝑎𝑎𝑛𝑛+1 − 𝑏𝑏𝑛𝑛+1 )
𝐷𝐷𝑛𝑛 − 𝑏𝑏𝐷𝐷𝑛𝑛−1 = 𝑎𝑎(𝐷𝐷𝑛𝑛−1 − 𝑏𝑏𝐷𝐷𝑛𝑛−2 ) =. . . = 𝑎𝑎 (𝐷𝐷2 − 𝑏𝑏𝐷𝐷1 ) = 𝑎𝑎 𝑎𝑎 − 𝑏𝑏
(Chú ý: công thức tính 𝐷𝐷𝑛𝑛 đúng với mọi 𝑛𝑛 ∈ ℕ)
*Nếu 𝑎𝑎 = 𝑏𝑏
Theo trên, ta có 𝐷𝐷𝑛𝑛 − 𝑎𝑎𝐷𝐷𝑛𝑛−1 = 𝑎𝑎𝑛𝑛−2 (𝐷𝐷2 − 𝑎𝑎𝐷𝐷1 ) hay 𝐷𝐷𝑛𝑛 = 𝑎𝑎𝐷𝐷𝑛𝑛−1 + 𝑎𝑎𝑛𝑛−2 (𝐷𝐷2 − 𝑎𝑎𝐷𝐷1 )
Suy ra 𝐷𝐷𝑛𝑛−1 = 𝑎𝑎𝐷𝐷𝑛𝑛−2 + 𝑎𝑎𝑛𝑛−3 (𝐷𝐷2 − 𝑎𝑎𝐷𝐷1 )
Thay ngược lại 𝐷𝐷𝑛𝑛 = 𝑎𝑎[𝑎𝑎𝐷𝐷𝑛𝑛−2 + 𝑎𝑎𝑛𝑛−3 (𝐷𝐷2 − 𝑎𝑎𝐷𝐷1 )] + 𝑎𝑎𝑛𝑛−2 (𝐷𝐷2 − 𝑎𝑎𝐷𝐷1 ) = 𝑎𝑎2 𝐷𝐷𝑛𝑛−2 + 2𝑎𝑎𝑛𝑛−2 (𝐷𝐷2 −
𝑎𝑎𝐷𝐷1 )
Lặp lại quá trình trên, cuối cùng ta được
𝐷𝐷𝑛𝑛 = 𝑎𝑎𝑛𝑛−1 𝐷𝐷1 + (𝑛𝑛 − 1)𝑎𝑎𝑛𝑛−2 (𝐷𝐷2 − 𝑎𝑎𝐷𝐷1 ) = (𝑛𝑛 − 1)𝑎𝑎𝑛𝑛−2 𝐷𝐷2 − (𝑛𝑛 − 2)𝑎𝑎𝑛𝑛−1 𝐷𝐷1
𝑎𝑎 + 𝑏𝑏 𝑏𝑏
= (𝑛𝑛 − 1)𝑎𝑎𝑛𝑛−2 � � − (𝑛𝑛 − 2)𝑎𝑎𝑛𝑛−1 (𝑎𝑎 + 𝑏𝑏)
𝑎𝑎 𝑎𝑎 + 𝑏𝑏

Đỗ Minh Triết IX-110 - MATHTASY -


Ôn thi Olympic Toán Đại số Phần IX: Định thức

= (𝑛𝑛 − 1)𝑎𝑎𝑛𝑛−2 (𝑎𝑎2 + 𝑏𝑏2 + 𝑎𝑎𝑎𝑎) − (𝑛𝑛 − 2)𝑎𝑎𝑛𝑛−1 (𝑎𝑎 + 𝑏𝑏)


= 𝑎𝑎𝑛𝑛 + 𝑎𝑎𝑛𝑛−1 𝑏𝑏 + (𝑛𝑛 − 1)𝑎𝑎𝑛𝑛−2 𝑏𝑏2

c) 𝐷𝐷𝑛𝑛 = (𝑎𝑎 + 𝑏𝑏)𝐷𝐷𝑛𝑛−1 − 𝑎𝑎𝑎𝑎𝐷𝐷𝑛𝑛−2 , chính là định thức câu b.

d) Đặt 𝑧𝑧 = cos 𝜑𝜑 + 𝑖𝑖 sin 𝜑𝜑 , 𝑧𝑧 ̅ = cos 𝜑𝜑 − 𝑖𝑖 sin 𝜑𝜑 thì 𝑧𝑧 + 𝑧𝑧 ̅ = 2 cos 𝜑𝜑 , 𝑧𝑧𝑧𝑧 ̅ = 1, theo trên, ta có
1 2𝑖𝑖 sin[(𝑛𝑛 + 1)𝜑𝜑] sin[(𝑛𝑛 + 1)𝜑𝜑]
𝐷𝐷𝑛𝑛 = (𝑧𝑧 𝑛𝑛+1 − 𝑧𝑧 𝑛𝑛+1
̅ )= =
𝑧𝑧 − 𝑧𝑧 ̅ 2𝑖𝑖 sin 𝜑𝜑 sin 𝜑𝜑

e) Ta có 𝐷𝐷 = cos 𝜑𝜑 𝐷𝐷𝑛𝑛−1 − 𝐷𝐷𝑛𝑛−2 , với 𝐷𝐷𝑛𝑛 được xác định từ câu d, ta được
1
cos 𝜑𝜑 sin(𝑛𝑛𝑛𝑛) − sin[(𝑛𝑛 − 1)𝜑𝜑] 2 �[sin[(𝑛𝑛 + 1)𝜑𝜑] + sin[(𝑛𝑛 − 1)𝜑𝜑]] − sin[(𝑛𝑛 − 1)𝜑𝜑]�
𝐷𝐷 = =
sin 𝜑𝜑 sin 𝜑𝜑
1 [sin[(𝑛𝑛 + 1)𝜑𝜑] − sin[(𝑛𝑛 − 1)𝜑𝜑]] 1 . 2. cos 2𝑛𝑛 𝜑𝜑 . sin 2𝜑𝜑
=2 =2 2 2 = cos(𝑛𝑛𝑛𝑛)
sin 𝜑𝜑 sin 𝜑𝜑

Ví dụ 3 : Tính
1 − 𝑏𝑏1 𝑏𝑏2 0 0 … 0
� �
−1 1 − 𝑏𝑏2 𝑏𝑏3 0 … 0
� �
𝐷𝐷𝑛𝑛 = � 0 −1 1 − 𝑏𝑏3 𝑏𝑏4 … 0 �
�⋮ ⋮ ⋮ ⋮ … ⋮ �
�0 0 0 0 … 1 − 𝑏𝑏𝑛𝑛 �

Giải.

Sử dụng tính đa tuyến tính, tách định thức theo cột đầu rồi khai triển
1 𝑏𝑏2 0 0 … 0
� �
−1 1 − 𝑏𝑏2 𝑏𝑏3 0 … 0
� �
𝐷𝐷𝑛𝑛 = � 0 −1 1 − 𝑏𝑏3 𝑏𝑏4 … 0 � − 𝑏𝑏1 𝐷𝐷𝑛𝑛−1
� ⋮ ⋮ ⋮ ⋮ … ⋮ �
� 0 0 0 0 … 1 − 𝑏𝑏𝑛𝑛 �
Lấy hàng 1 cộng vào hàng 2, lấy hàng 2 mới cộng vào hàng 3,…, lấy hàng 𝑛𝑛 − 1 mới cộng vào
hàng 𝑛𝑛, ta được
𝐷𝐷𝑛𝑛 = 1 − 𝑏𝑏1 𝐷𝐷𝑛𝑛−1 = 1 − 𝑏𝑏1 (1 − 𝑏𝑏2 𝐷𝐷𝑛𝑛−2 ) =. . . = 1 − 𝑏𝑏1 + 𝑏𝑏1 𝑏𝑏2 − 𝑏𝑏1 𝑏𝑏2 𝑏𝑏3 +. . . +(−1)𝑛𝑛 𝑏𝑏1 𝑏𝑏2 … 𝑏𝑏𝑛𝑛

Ví dụ 4* : Tính

Đỗ Minh Triết IX-111 - MATHTASY -


Ôn thi Olympic Toán Đại số Phần IX: Định thức

𝑥𝑥 1 0 0 … 0 0
� 𝑛𝑛 − 1 𝑥𝑥 2 0 … 0 0 �
� 0 𝑛𝑛 − 2 𝑥𝑥 3 … 0 0 ��
𝐷𝐷𝑛𝑛 (𝑥𝑥) = �
� ⋮ ⋮ ⋮ ⋮ ⋱ ⋮ ⋮ �
� 0 0 0 0 … 𝑥𝑥 𝑛𝑛 − 1 �
� 0 0 0 0 … 1 𝑥𝑥 �
Giải.

Lấy 𝑛𝑛 − 𝑖𝑖 dòng sau cộng vào dòng 𝑖𝑖 với 𝑖𝑖 = 1, … , 𝑛𝑛 − 1


𝑥𝑥 + 𝑛𝑛 − 1 𝑥𝑥 + 𝑛𝑛 − 1 𝑥𝑥 + 𝑛𝑛 − 1 𝑥𝑥 + 𝑛𝑛 − 1 … 𝑥𝑥 + 𝑛𝑛 − 1 𝑥𝑥 + 𝑛𝑛 − 1
� 𝑛𝑛 − 1 𝑥𝑥 + 𝑛𝑛 − 2 𝑥𝑥 + 𝑛𝑛 − 1 𝑥𝑥 + 𝑛𝑛 − 1 … 𝑥𝑥 + 𝑛𝑛 − 1 𝑥𝑥 + 𝑛𝑛 − 1�
� 0 𝑛𝑛 − 2 𝑥𝑥 + 𝑛𝑛 − 3 𝑥𝑥 + 𝑛𝑛 − 1 … 𝑥𝑥 + 𝑛𝑛 − 1 𝑥𝑥 + 𝑛𝑛 − 1��
𝐷𝐷𝑛𝑛 (𝑥𝑥) = �
�⋮ ⋮ ⋮ ⋮ … ⋮ ⋮ �
�0 0 0 0 … 𝑥𝑥 + 1 𝑥𝑥 + 𝑛𝑛 − 1�
�0 0 0 0 … 1 𝑥𝑥 �
Lấy cột 𝑖𝑖 nhân -1 rồi cộng vào cột 𝑖𝑖 + 1 với 𝑖𝑖 = 𝑛𝑛 − 1, … ,1
𝑥𝑥 + 𝑛𝑛 − 1 0 0 0 … 0 0
� 𝑛𝑛 − 1 𝑥𝑥 − 1 1 0 … 0 0 �
� 0 𝑛𝑛 − 2 𝑥𝑥 − 1 2 … 0 0 ��
𝐷𝐷𝑛𝑛 (𝑥𝑥) = �
� ⋮ ⋮ ⋮ ⋮ … ⋮ ⋮ �
� 0 0 0 0 … 𝑥𝑥 − 1 𝑛𝑛 − 2�
� 0 0 0 0 … 1 𝑥𝑥 − 1�
𝑛𝑛−1
= (𝑥𝑥 + 𝑛𝑛 − 1)𝐷𝐷𝑛𝑛−1 (𝑥𝑥 − 1) = (𝑥𝑥 + 𝑛𝑛 − 1)(𝑥𝑥 − 1 + 𝑛𝑛 − 2)𝐷𝐷𝑛𝑛−2 (𝑥𝑥 − 2) =. . . = �(𝑥𝑥 + 𝑛𝑛 − 1 − 2𝑖𝑖)
𝑖𝑖=0

Ví dụ 5* : Tính
1 −𝑎𝑎1 −𝑎𝑎2 … −𝑎𝑎𝑛𝑛
� �
𝑎𝑎1 𝑏𝑏1 0 … 0
� �
𝐷𝐷𝑛𝑛 = � 𝑎𝑎2 0 𝑏𝑏2 … 0 �
� ⋮ ⋮ ⋮ ⋱ ⋮ �
�𝑎𝑎𝑛𝑛 0 0 … 𝑏𝑏𝑛𝑛 �

Giải.

Khai triển theo hàng cuối, ta được


−𝑎𝑎1 −𝑎𝑎2 … −𝑎𝑎𝑛𝑛−1 −𝑎𝑎𝑛𝑛
� 𝑏𝑏1 0 … 0 0�
𝐷𝐷𝑛𝑛 = 𝑏𝑏𝑛𝑛 𝐷𝐷𝑛𝑛−1 + (−1) 𝑎𝑎𝑛𝑛 ��
𝑛𝑛+1 0 𝑏𝑏2 … 0 0 ��
� ⋮ ⋮ ⋱ ⋮ ⋮�
� 0 0 … 𝑏𝑏𝑛𝑛−1 0�
= 𝑏𝑏𝑛𝑛 𝐷𝐷𝑛𝑛−1 + (−1)𝑛𝑛+1 𝑎𝑎𝑛𝑛 (−1)𝑛𝑛 (−𝑎𝑎𝑛𝑛 )𝑏𝑏1 𝑏𝑏2 … 𝑏𝑏𝑛𝑛−1 = 𝑏𝑏𝑛𝑛 𝐷𝐷𝑛𝑛−1 + 𝑏𝑏1 𝑏𝑏2 … 𝑏𝑏𝑛𝑛−1 𝑎𝑎2𝑛𝑛
Vì vậy ta có
𝐷𝐷𝑛𝑛 = 𝑏𝑏𝑛𝑛 𝐷𝐷𝑛𝑛−1 + 𝑏𝑏1 𝑏𝑏2 … 𝑏𝑏𝑛𝑛−1 𝑎𝑎2𝑛𝑛
𝐷𝐷𝑛𝑛−1 = 𝑏𝑏𝑛𝑛−1 𝐷𝐷𝑛𝑛−2 + 𝑏𝑏1 𝑏𝑏2 … 𝑏𝑏𝑛𝑛−2 𝑎𝑎2𝑛𝑛−1 (nhân hai vế cho 𝑏𝑏𝑛𝑛 )

Đỗ Minh Triết IX-112 - MATHTASY -


Ôn thi Olympic Toán Đại số Phần IX: Định thức

𝐷𝐷𝑛𝑛−2 = 𝑏𝑏𝑛𝑛−2 𝐷𝐷𝑛𝑛−3 + 𝑏𝑏1 𝑏𝑏2 … 𝑏𝑏𝑛𝑛−3 𝑎𝑎2𝑛𝑛−2 (nhân hai vế cho 𝑏𝑏𝑛𝑛−1 𝑏𝑏𝑛𝑛 )
…………...
𝐷𝐷2 = 𝑏𝑏2 𝐷𝐷1 + 𝑏𝑏1 𝑎𝑎22 (nhân hai vế cho 𝑏𝑏3 … 𝑏𝑏𝑛𝑛 )
𝐷𝐷1 = 𝑏𝑏1 + 𝑎𝑎21 (nhân hai vế cho 𝑏𝑏2 … 𝑏𝑏𝑛𝑛 )
Cộng theo vế các đẳng thức sau khi nhân theo vế một lượng đã chỉ ra, ta được
𝑛𝑛
𝐷𝐷𝑛𝑛 = 𝑏𝑏1 𝑏𝑏2 … 𝑏𝑏𝑛𝑛 + � 𝑎𝑎2𝑛𝑛 (𝑏𝑏1 … 𝑏𝑏𝑖𝑖−1 𝑏𝑏𝑖𝑖+1 … 𝑏𝑏𝑛𝑛 )
𝑖𝑖=1

Ví dụ 7 : Tính
𝑎𝑎 𝑥𝑥 𝑥𝑥 … 𝑥𝑥
�𝑦𝑦 𝑧𝑧 0 … 0�
𝐷𝐷𝑛𝑛 = ��𝑦𝑦 0 𝑧𝑧 … 0 ��
�⋮ ⋮ ⋮ ⋱ ⋮�
� 𝑦𝑦 0 0 … 𝑧𝑧 �
Giải:

Đây là trường hợp đặc biệt của ví dụ trên. Khai triển tương tự, ta có
𝐷𝐷𝑛𝑛 = 𝑧𝑧𝐷𝐷𝑛𝑛−1 + (−1)𝑛𝑛+1 𝑦𝑦(−1)𝑛𝑛 𝑥𝑥𝑧𝑧𝑛𝑛−2 = 𝑧𝑧𝐷𝐷𝑛𝑛−1 − 𝑥𝑥𝑥𝑥𝑧𝑧 𝑛𝑛−2 ⇒ 𝐷𝐷𝑛𝑛−1 = 𝑧𝑧𝐷𝐷𝑛𝑛−2 − 𝑥𝑥𝑥𝑥𝑧𝑧 𝑛𝑛−3
⇒ 𝐷𝐷𝑛𝑛 = 𝑧𝑧𝐷𝐷𝑛𝑛−1 − 𝑥𝑥𝑥𝑥𝑧𝑧 𝑛𝑛−2 = 𝑧𝑧 2 𝐷𝐷𝑛𝑛−2 − 2𝑥𝑥𝑥𝑥𝑧𝑧 𝑛𝑛−2 =. . . = 𝑧𝑧 𝑛𝑛−1 𝐷𝐷1 − (𝑛𝑛 − 1)𝑥𝑥𝑥𝑥𝑧𝑧 𝑛𝑛−2
= 𝑧𝑧 𝑛𝑛−1 𝑎𝑎 − (𝑛𝑛 − 1)𝑥𝑥𝑥𝑥𝑧𝑧 𝑛𝑛−2

Phương pháp 6: Rút nhân tử tuyến tính

Nếu mỗi phần tử của ma trận vuông 𝐴𝐴 cấp 𝑛𝑛 là một đa thức bậc nhất theo biến 𝑥𝑥 nào đó thì |𝐴𝐴|
là một đa thức bậc không quá 𝑛𝑛 với biến đó. Do vậy, nếu ta tìm được 𝑛𝑛 đa thức bậc nhất
𝑓𝑓1 (𝑥𝑥), . . , 𝑓𝑓𝑛𝑛 (𝑥𝑥) độc lập tuyến tính sao cho mỗi 𝑓𝑓𝑖𝑖 (𝑥𝑥) là một ước của |𝐴𝐴| thì ta kết luận |𝐴𝐴| là tích
𝑓𝑓1 (𝑥𝑥) … 𝑓𝑓𝑛𝑛 (𝑥𝑥) sai khác nhau một nhân tử hằng số.

Ví dụ 1 : Tính
1 2 3 … 𝑛𝑛
� 1 𝑥𝑥 + 1 3 … 𝑛𝑛 �
𝐷𝐷(𝑥𝑥) = �� 1 2 𝑥𝑥 + 1 … 𝑛𝑛 �

�⋮ ⋮ ⋮ ⋱ ⋮ �
�1 2 3 … 𝑥𝑥 + 1 �

Giải.

Đối với bài này, ta có thể đưa định thức về dạng bậc thang một cách đơn giản bằng các phép
biến đổi sơ cấp (lần lượt nhân cột đầu cho −2, … , −𝑛𝑛 rồi cộng vào cột 2, … , 𝑛𝑛).

Đỗ Minh Triết IX-113 - MATHTASY -


Ôn thi Olympic Toán Đại số Phần IX: Định thức

Tuy nhiên, dễ thấy định thức là một đa thức biến 𝑥𝑥 bậc tối đa là 𝑛𝑛 − 1, đó chính là luỹ thừa của
𝑥𝑥 trong tích các phần tử trên đường chéo chính (ứng với phép thế đồng nhất) và hệ số của 𝑥𝑥𝑛𝑛−1
là 1. Mặt khác, nếu ta lần lượt cho 𝑥𝑥 = 1, … , 𝑛𝑛 thì định thức bằng 0 do có 2 hàng tỉ lệ. Do đó
𝐷𝐷(𝑥𝑥) chia hết cho 𝑛𝑛 − 1 đa thức bậc nhất và độc lập tuyến tính 𝑥𝑥 − 1, … , 𝑥𝑥 − 𝑛𝑛 + 1.
Vậy 𝐷𝐷(𝑥𝑥) = (𝑥𝑥 − 1) … (𝑥𝑥 − 𝑛𝑛 + 1).

Ví dụ 2 : Tính
0 𝑥𝑥 𝑦𝑦 𝑧𝑧
�𝑥𝑥 0 𝑧𝑧 𝑦𝑦 �
𝐷𝐷 = � �
�𝑦𝑦 𝑧𝑧 0 𝑥𝑥�
� 𝑧𝑧 𝑦𝑦 𝑥𝑥 0�

Giải.

Cộng tất cả các cột vào cột đầu, ta thấy 𝐷𝐷 ⋮ (𝑥𝑥 + 𝑦𝑦 + 𝑧𝑧).
Nhân cột 2 cho -1, cột 3 cho -1, cột 4 cho 1 rồi cộng vào cột đầu, ta được 𝐷𝐷 ⋮ (𝑥𝑥 + 𝑦𝑦 − 𝑧𝑧)
Nhân cột 2 cho -1, cột 3 cho 1, cột 4 cho -1 rồi cộng vào cột đầu, ta được 𝐷𝐷 ⋮ (𝑥𝑥 − 𝑦𝑦 + 𝑧𝑧)
Nhân cột 2 cho 1, cột 3 cho -1, cột 4 cho -1 rồi cộng vào cột đầu, ta được 𝐷𝐷 ⋮ (−𝑥𝑥 + 𝑦𝑦 + 𝑧𝑧)
Từ đây, ta kết luận định thức sai khác tích của 4 biểu thức trên là một hằng số. Mặt khác, hệ số
của 𝑧𝑧 4 trong khai triển là 1, trong khi của tích 4 biểu thức là -1.
Vậy 𝐷𝐷 = −(𝑥𝑥 + 𝑦𝑦 + 𝑧𝑧)(𝑥𝑥 + 𝑦𝑦 − 𝑧𝑧)(𝑥𝑥 − 𝑦𝑦 + 𝑧𝑧)(−𝑥𝑥 + 𝑦𝑦 + 𝑧𝑧).

Ví dụ 3 : Tính
−𝑥𝑥 4 20 10
� 4 −𝑥𝑥 10 20 �
𝐷𝐷 = �
� 20 10 −𝑥𝑥 4��
� 10 20 4 −𝑥𝑥 �
Giải.

Làm tương tự trên, 34, 26, 6, 14 là nghiệm của 𝐷𝐷, hệ số của 𝑥𝑥4 trong khai triển là 1 nên
𝐷𝐷 = (𝑥𝑥 − 34)(𝑥𝑥 − 26)(𝑥𝑥 − 6)(𝑥𝑥 − 14)

Phương pháp 7: Quy nạp

Cách làm của phương pháp này là ban đầu tính định thức cấp thấp (2, 3) sau đó dự đoán rồi
chứng minh công thức bằng quy nạp, điều quan trọng ở đây là biết phán đoán chính xác.

Ví dụ (Tổng quát câu 3 đề chọn đội tuyển ĐH Kinh tế Quốc dân 2012) :

Đỗ Minh Triết IX-114 - MATHTASY -


Ôn thi Olympic Toán Đại số Phần IX: Định thức

Tính định thức


1 2 3 … 𝑛𝑛 − 1 𝑛𝑛
� −𝑥𝑥 𝑥𝑥 0 … 0 0�
𝐷𝐷𝑛𝑛 = �� 0 −𝑥𝑥 𝑥𝑥 … 0 0 ��
� ⋮ ⋮ ⋮ … ⋮ ⋮�
� 0 0 0 … −𝑥𝑥 𝑥𝑥�
Ta có

1.2 0 2.3 1 2 3 3.4 2


1 2
𝐷𝐷1 = |1| = 1 = 𝑥𝑥 ; 𝐷𝐷2 = � � = 3𝑥𝑥 = 𝑥𝑥 ; 𝐷𝐷3 = � −𝑥𝑥 𝑥𝑥 0 � = 6𝑥𝑥2 = 𝑥𝑥
2 −𝑥𝑥 𝑥𝑥 2 2
0 −𝑥𝑥 𝑥𝑥
Dự đoán
𝑘𝑘(𝑘𝑘 + 1) 𝑘𝑘−1
𝐷𝐷𝑘𝑘 = 𝑥𝑥 ∀𝑘𝑘 ≥ 1
2
Ta chứng minh công thức cũng đúng với 𝑘𝑘 + 1, thật vậy, khai triển định thức 𝐷𝐷𝑘𝑘+1 theo cột cuối,
ta có
1 2 3 … 𝑘𝑘 − 1 𝑘𝑘 𝑘𝑘 + 1
�−𝑥𝑥 𝑥𝑥 0 … 0 0 0 �
� �
0 −𝑥𝑥 𝑥𝑥 … 0 0 0 �
𝐷𝐷𝑘𝑘+1 = � = 𝑥𝑥𝐷𝐷𝑘𝑘 + (−1)𝑘𝑘+2 (𝑘𝑘 + 1)(−𝑥𝑥)𝑘𝑘
� ⋮ ⋮ ⋮ … ⋮ ⋮ 0 �
� 0 0 0 … −𝑥𝑥 𝑥𝑥 0 �
� 0 0 0 … 0 −𝑥𝑥 𝑥𝑥 �
𝑘𝑘(𝑘𝑘 + 1) 𝑘𝑘 𝑘𝑘 (𝑘𝑘 + 1)(𝑘𝑘 + 2) 𝑘𝑘
= 𝑥𝑥 + (𝑘𝑘 + 1)𝑥𝑥𝑘𝑘 = � + 1� (𝑘𝑘 + 1)𝑥𝑥𝑘𝑘 = 𝑥𝑥
2 2 2
𝑛𝑛(𝑛𝑛 + 1) 𝑛𝑛−1
Vậy 𝐷𝐷𝑛𝑛 = 𝑥𝑥 .
2

Trên đây là một số phương pháp tính định thức, trong một số trường hợp, ta có thể sử dụng một
hay nhiều phương pháp tính đối với một định thức.

Ví dụ (Câu 3 đề chọn đội tuyển ĐH Kinh tế Quốc dân 2005) : Tính định thức
cấp 𝑛𝑛 sau
1 2 3 4 … 𝑛𝑛 − 1 𝑛𝑛
�𝑥𝑥 1 2 3 … 𝑛𝑛 − 2 𝑛𝑛 − 1�
�𝑥𝑥 𝑥𝑥 1 2 … 𝑛𝑛 − 3 𝑛𝑛 − 2�
� �
�𝑥𝑥 𝑥𝑥 𝑥𝑥 1 … 𝑛𝑛 − 4 𝑛𝑛 − 3�
�⋮ ⋮ ⋮ ⋮ … ⋮ ⋮ �
�𝑥𝑥 𝑥𝑥 𝑥𝑥 𝑥𝑥 … 2 3 �
�𝑥𝑥 𝑥𝑥 𝑥𝑥 𝑥𝑥 … 1 2 �
�𝑥𝑥 𝑥𝑥 𝑥𝑥 𝑥𝑥 … 𝑥𝑥 1 �
Giải. (Sử dụng tính đa tuyến tính và biến đổi sơ cấp Gauss)

Nhân cột 𝑖𝑖 với -1 rồi cộng vào cột 𝑖𝑖 + 1, 𝑖𝑖 = 𝑛𝑛 − 1, . . ,1, ta được

Đỗ Minh Triết IX-115 - MATHTASY -


Ôn thi Olympic Toán Đại số Phần IX: Định thức

1 1 1 1 … 1 1
�𝑥𝑥 1 − 𝑥𝑥 1 1 … 1 1 �
�𝑥𝑥 0 1 − 𝑥𝑥 1 … 1 1 �
� �
�𝑥𝑥 0 0 1 − 𝑥𝑥 … 1 1 �
�⋮ ⋮ ⋮ ⋮ … ⋮ ⋮ �
�𝑥𝑥 0 0 0 … 1 1 �
�𝑥𝑥 0 0 0 … 1 − 𝑥𝑥 1 �
�𝑥𝑥 0 0 0 … 0 1 − 𝑥𝑥�
Sử dụng tính đa tuyến tính, ta tách định thức theo cột đầu như sau
1 − 𝑥𝑥 1 1 1 … 1 1 𝑥𝑥 1 1 1 … 1 1
� 0 1 − 𝑥𝑥 1 1 … 1 1 � �𝑥𝑥 1 − 𝑥𝑥 1 1 … 1 1 �
� 0 0 1 − 𝑥𝑥 1 … 1 1 � �𝑥𝑥 0 1 − 𝑥𝑥 1 … 1 1 �
� � � �
� 0 0 0 1 − 𝑥𝑥 … 1 1 � �𝑥𝑥 0 0 1 − 𝑥𝑥 … 1 1 �
+
� ⋮ ⋮ ⋮ ⋮ … ⋮ ⋮ � �⋮ ⋮ ⋮ ⋮ … ⋮ ⋮ �
� 0 0 0 0 … 1 1 � �𝑥𝑥 0 0 0 … 1 1 �
� 0 0 0 0 … 1 − 𝑥𝑥 1 � �𝑥𝑥 0 0 0 … 1 − 𝑥𝑥 1 �
� 0 0 0 0 … 0 1 − 𝑥𝑥� �𝑥𝑥 0 0 0 … 0 1 − 𝑥𝑥�
Định thức đầu bằng (1 − 𝑥𝑥)𝑛𝑛 và ở định thức sau, ta đưa 𝑥𝑥 ở cột đầu ra ngoài
1 1 1 1 … 1 1
�1 1 − 𝑥𝑥 1 1 … 1 1 �
�1 0 1 − 𝑥𝑥 1 … 1 1 �
� �
1 0 0 1 − 𝑥𝑥 … 1 1 �
(1 − 𝑥𝑥)𝑛𝑛 + 𝑥𝑥 �
�⋮ ⋮ ⋮ ⋮ … ⋮ ⋮ �
�1 0 0 0 … 1 1 �
�1 0 0 0 … 1 − 𝑥𝑥 1 �
�1 0 0 0 … 0 1 − 𝑥𝑥�
Ở định thức sau, nhân hàng 𝑖𝑖 với -1 rồi cộng vào hàng 𝑖𝑖 + 1, 𝑖𝑖 = 𝑛𝑛 − 1, … ,1, sau đó khai triển nó
theo cột đầu, ta được
1 1 1 1 … 1 1
�0 −𝑥𝑥 0 0 … 0 0 �
�0 𝑥𝑥 − 1 −𝑥𝑥 0 … 0 0 �
� �
0 0 𝑥𝑥 − 1 −𝑥𝑥 … 0 0 �
(1 − 𝑥𝑥)𝑛𝑛 + 𝑥𝑥 �
�⋮ ⋮ ⋮ ⋮ … ⋮ ⋮ �
�0 0 0 0 … 0 0 �
�0 0 0 0 … −𝑥𝑥 0 �
�0 0 0 0 … 𝑥𝑥 − 1 −𝑥𝑥�
𝑛𝑛 𝑛𝑛−1
= (1 − 𝑥𝑥) + 𝑥𝑥(−𝑥𝑥) = (−1) (𝑥𝑥 − 1) + (−1)𝑛𝑛−1 𝑥𝑥𝑛𝑛 = (−1)𝑛𝑛 [(𝑥𝑥 − 1)𝑛𝑛 − 𝑥𝑥𝑛𝑛 ]
𝑛𝑛 𝑛𝑛

Ví dụ 5 (Câu 1b Olympic 1993) : Tính


1 + 𝑎𝑎1 𝑏𝑏1 𝑎𝑎2 𝑏𝑏2 … 𝑎𝑎𝑛𝑛 𝑏𝑏𝑛𝑛
� �
𝑎𝑎1 𝑏𝑏1 1 + 𝑎𝑎2 𝑏𝑏2 … 𝑎𝑎𝑛𝑛 𝑏𝑏𝑛𝑛
𝐷𝐷𝑛𝑛 = � �
� ⋮ ⋮ ⋱ ⋮ �
� 𝑎𝑎1 𝑏𝑏1 𝑎𝑎2 𝑏𝑏2 … 1 + 𝑎𝑎𝑛𝑛 𝑏𝑏𝑛𝑛 �
Giải: (Sử dụng tính đa tuyến tính và truy hồi)

Đỗ Minh Triết IX-116 - MATHTASY -


Ôn thi Olympic Toán Đại số Phần IX: Định thức

Dùng tính chất đa tuyến tính, tách định thức theo cột cuối, ta có
1 + 𝑎𝑎1 𝑏𝑏1 𝑎𝑎2 𝑏𝑏2 … 0 1 + 𝑎𝑎1 𝑏𝑏1 𝑎𝑎2 𝑏𝑏2 … 𝑎𝑎1
� � � 𝑎𝑎2 �
𝑎𝑎1 𝑏𝑏1 1 + 𝑎𝑎2 𝑏𝑏2 … 0 𝑎𝑎1 𝑏𝑏1 1 + 𝑎𝑎2 𝑏𝑏2 …
𝐷𝐷𝑛𝑛 = � � + 𝑏𝑏𝑛𝑛 � ⋮ ��
� ⋮ ⋮ ⋱ ⋮� � ⋮ ⋮ ⋱
� 𝑎𝑎1 𝑏𝑏1 𝑎𝑎2 𝑏𝑏2 … 1� � 𝑎𝑎1 𝑏𝑏1 𝑎𝑎2 𝑏𝑏2 … 𝑎𝑎𝑛𝑛 �
Ở định thức sau, nhân cột cuối cho −𝑏𝑏𝑗𝑗 rồi cộng vào cột 𝑗𝑗 với 𝑗𝑗 = 1, … , 𝑛𝑛 − 1 thì ta được
𝐷𝐷𝑛𝑛 = 𝐷𝐷𝑛𝑛−1 + 𝑎𝑎𝑛𝑛 𝑏𝑏𝑛𝑛 = 𝐷𝐷𝑛𝑛−2 + 𝑎𝑎𝑛𝑛−1 𝑏𝑏𝑛𝑛−1 + 𝑎𝑎𝑛𝑛 𝑏𝑏𝑛𝑛 =. . . = 𝐷𝐷1 + 𝑎𝑎2 𝑏𝑏2 +. . . +𝑎𝑎𝑛𝑛 𝑏𝑏𝑛𝑛
= 1 + 𝑎𝑎1 𝑏𝑏1 +. . . +𝑎𝑎𝑛𝑛 𝑏𝑏𝑛𝑛 .

9.1.2. Sử dụng định nghĩa định thức giải một số bài toán liên quan
Định nghĩa phép thế: Một hoán vị (hay phép thế) của tập 𝑋𝑋 = {1, … , 𝑛𝑛} là một song ánh từ
tập này vào chính nó. Tập tất cả các hoán vị của 𝑋𝑋 ký hiệu là 𝑆𝑆𝑛𝑛 .
Như vậy, một hoán vị của 𝑋𝑋 chính là một cách sắp xếp có thứ tự của {1, … , 𝑛𝑛}. Thông thường
ta ký hiệu một hoán vị 𝜋𝜋 ∈ 𝑆𝑆𝑛𝑛 như sau
1 2 … 𝑛𝑛
𝜋𝜋 = � 𝜋𝜋(𝑛𝑛)�
𝜋𝜋(1) 𝜋𝜋(2) …
Nếu ta xem tích của 2 phép thế 𝜎𝜎, 𝜋𝜋 ∈ 𝑆𝑆𝑛𝑛 là hợp thành của 2 ánh xạ 𝜎𝜎, 𝜋𝜋 thì khi đó 𝑆𝑆𝑛𝑛 là một
nhóm (phép Toán kết hợp, có phần tử đơn vị là phép thế đồng nhất, phần tử nghịch đảo của 𝜋𝜋
là ánh xạ ngược 𝜋𝜋−1 ).
Cho 1 ≤ 𝑖𝑖 < 𝑗𝑗 ≤ 𝑛𝑛. Phép thế
1 … 𝑖𝑖 − 1 𝑖𝑖 𝑖𝑖 + 1 … 𝑗𝑗 − 1 𝑗𝑗 𝑗𝑗 + 1 … 𝑛𝑛
� �
1 … 𝑖𝑖 − 1 𝑗𝑗 𝑖𝑖 + 1 … 𝑗𝑗 − 1 𝑖𝑖 𝑗𝑗 + 1 … 𝑛𝑛
Đổi chỗ 2 phần tử 𝑖𝑖 < 𝑗𝑗 cho nhau đồng thời giữ nguyên các phần tử còn lại được gọi là một chyển
vị (hay chuyển trí hay phép thế sơ cấp). Ký hiệu là (𝑖𝑖, 𝑗𝑗).
Cho 𝑎𝑎1 , … , 𝑎𝑎𝑘𝑘 là các phần tử khác nhau của {1, … , 𝑛𝑛}. Hoán vị 𝜋𝜋 giữ nguyên các phần tử khác
𝑎𝑎1 , … , 𝑎𝑎𝑘𝑘 và thoả mãn 𝜋𝜋(𝑎𝑎1 ) = 𝑎𝑎2 , 𝜋𝜋(𝑎𝑎2 ) = 𝑎𝑎3 , … , 𝜋𝜋(𝑎𝑎𝑛𝑛 ) = 𝑎𝑎1 được gọi là một xích độ dài 𝑘𝑘, ký
hiệu (𝑎𝑎1 , … , 𝑎𝑎𝑘𝑘 ). Rõ ràng ta có
(𝑎𝑎1 , … , 𝑎𝑎𝑘𝑘 ) = (𝑎𝑎1 , … , 𝑎𝑎𝑘𝑘−1 ) ∘ (𝑎𝑎𝑘𝑘−1 , 𝑎𝑎𝑘𝑘 )
Do đó:
Tính chất: Một hoán vị của 𝑎𝑎1 , … , 𝑎𝑎𝑛𝑛 đều là tích các chuyển vị (của tập hợp này).
Định nghĩa nghịch thế: Ta gọi cặp {𝑖𝑖, 𝑗𝑗} ⊆ {1, . . , 𝑛𝑛} là một nghịch thế của hoán vị 𝜋𝜋 nếu 𝑖𝑖 − 𝑗𝑗
trái dấu với 𝜋𝜋(𝑖𝑖) − 𝜋𝜋(𝑗𝑗). Một hoán vị với số các nghịch thế là số chẵn (lẻ) được gọi là một hoán
vị chẵn (lẻ). Dấu của 𝜋𝜋, ký hiệu sign(𝜋𝜋), nhận giá trị 1 (-1) nếu 𝜋𝜋 là hoán vị chẵn (lẻ). Như vậy,
nếu gọi 𝑁𝑁(𝜋𝜋) là số các nghịch thế của 𝜋𝜋 thì sign(𝜋𝜋) = (−1)𝑁𝑁(𝜋𝜋) .
Ví dụ:

Đỗ Minh Triết IX-117 - MATHTASY -


Ôn thi Olympic Toán Đại số Phần IX: Định thức

1. Số các nghịch thế của chuyển vị (𝑖𝑖 − 1, 𝑖𝑖) chỉ có 1, đó là {(𝑖𝑖, 𝑖𝑖 − 1)}.
2. Số các chuyển vị (1,3) là 3: {3,1}, {3,2}, {2,1}.
3. Tổng quát, số các nghịch thế của chuyển vị (𝑖𝑖, 𝑗𝑗) là 2(𝑗𝑗 − 𝑖𝑖) − 1.
Bổ đề (Dấu của phép thế):
𝜋𝜋(𝑖𝑖) − 𝜋𝜋(𝑗𝑗)
∀𝜋𝜋 ∈ 𝑆𝑆𝑛𝑛 : sign(𝜋𝜋) = �
1≤𝑖𝑖<𝑗𝑗≤𝑛𝑛
𝑖𝑖 − 𝑗𝑗

Định lí: ∀ 𝜎𝜎, 𝜋𝜋 ∈ 𝑆𝑆𝑛𝑛 : sign(𝜎𝜎𝜎𝜎) = sign(𝜎𝜎)sign(𝜋𝜋)


Hệ quả: ∀ 𝜋𝜋 ∈ 𝑆𝑆𝑛𝑛 : sign(𝜋𝜋) = sign(𝜋𝜋−1 )
Định nghĩa: Định thức của ma trận vuông 𝐴𝐴 = �𝑎𝑎𝑖𝑖𝑖𝑖 � cấp 𝑛𝑛, kí hiệu det 𝐴𝐴 hoặc |𝐴𝐴|, là tổng
𝑛𝑛
|𝐴𝐴| = � sign(𝜋𝜋𝑖𝑖 )𝑎𝑎1𝜋𝜋𝑖𝑖 (1) … 𝑎𝑎𝑛𝑛𝜋𝜋𝑖𝑖 (𝑛𝑛) trong đó 𝜋𝜋𝑖𝑖 ∈ 𝑆𝑆𝑛𝑛 với 𝑖𝑖 = 1, … , 𝑛𝑛
𝑖𝑖=1

Như vậy, định thức của ma trận 𝐴𝐴 = �𝑎𝑎𝑖𝑖𝑖𝑖 � là tổng của |𝑆𝑆𝑛𝑛 | = 𝑛𝑛! số hạng, mỗi số hạng là tích của
𝑛𝑛 phần tử của 𝐴𝐴 nằm trên các hàng khác nhau và trên các cột khác nhau ứng với dấu của phép
thế các chỉ số của phần tử này (trong mỗi tích không có hai thành phần nào cùng dòng hoặc cùng
cột). Tuy nhiên, việc tính định thức theo định nghĩa là quá phức tạp vì khi 𝑛𝑛 lớn thì 𝑛𝑛! số hạng
của |𝐴𝐴| là rất lớn, chưa kể đến việc phải đi tìm dấu của phép thế tương ứng. Trên thực tế, người
ta chỉ áp dụng định nghĩa định thức để chứng minh các tính chất của định thức hay khi 𝐴𝐴 có
dạng đặc biệt.

𝑎𝑎 𝑎𝑎12
Ví dụ: Sử dụng định nghĩa định thức, chứng minh rằng nếu 𝐴𝐴 = �𝑎𝑎11 𝑎𝑎22 � thì |𝐴𝐴| = 𝑎𝑎11 𝑎𝑎22 −
21
𝑎𝑎12 𝑎𝑎21 .
Ta có 𝑆𝑆𝑛𝑛 = {𝜋𝜋1 , 𝜋𝜋2 }, trong đó
1 2 1 2 1 2 1 2
𝜋𝜋1 = � �=� � , 𝜋𝜋2 = � �=� �
𝜋𝜋1 (1) 𝜋𝜋1 (2) 1 2 𝜋𝜋2 (1) 𝜋𝜋2 (2) 2 1
𝜋𝜋 (1) − 𝜋𝜋1 (2) 𝜋𝜋 (1) − 𝜋𝜋1 (2)
sign(𝜋𝜋1 ) = 1 = 1 , sign(𝜋𝜋2 ) = 1 = −1
1−2 1−2
Áp dụng định nghĩa định thức, ta có
|𝐴𝐴| = sign(𝜋𝜋1 )𝑎𝑎1𝜋𝜋1 (1) 𝑎𝑎2𝜋𝜋1 (2) + sign(𝜋𝜋2 )𝑎𝑎1𝜋𝜋2 (1) 𝑎𝑎2𝜋𝜋2 (2) = 𝑎𝑎11 𝑎𝑎22 − 𝑎𝑎12 𝑎𝑎21

9.2. Bài tập

Bài 9.1. (Câu 4 Olympic 2004) Cho ma trận vuông 𝐴𝐴 = �𝑎𝑎𝑖𝑖𝑖𝑖 � cấp 𝑛𝑛 thoả mãn

Đỗ Minh Triết IX-118 - MATHTASY -


Ôn thi Olympic Toán Đại số Phần IX: Định thức

0 nếu 𝑖𝑖 = 𝑗𝑗
𝑎𝑎𝑖𝑖𝑖𝑖 = �
±1 nếu 𝑖𝑖 ≠ 𝑗𝑗
Chứng minh rằng
a) Nếu 𝑛𝑛 = 3 thì tồn tại ma trận 𝐴𝐴 sao cho det 𝐴𝐴 = 0.
b) Với 𝑛𝑛 = 4 ta luôn có det 𝐴𝐴 ≠ 0.

Chứng minh.
0 −1 1
a) Chọn 𝐴𝐴 = �1 0 1� thì det 𝐴𝐴 = 0
1 1 0
0 1 1 1
⎛1 0 1 1⎞
b) Xét ma trận 𝐵𝐵 = ⎜
⎜1 1 ⎟ thì det 𝐵𝐵 = −3.
0 1⎟
⎝1 1 1 0⎠
Theo định nghĩa định thức thì
det 𝐵𝐵 = � sign(𝜋𝜋)𝑏𝑏1𝜋𝜋(1) 𝑏𝑏2𝜋𝜋(2) 𝑏𝑏3𝜋𝜋(3) 𝑏𝑏4𝜋𝜋(4) = −3
𝜋𝜋∈𝑆𝑆4

Xét một ma trận 𝐴𝐴 vuông cấp 4 thoả mãn điều kiện bài Toán, ta có
det 𝐴𝐴 = � sign(𝜋𝜋)𝑎𝑎1𝜋𝜋(1) 𝑎𝑎2𝜋𝜋(2) 𝑎𝑎3𝜋𝜋(3) 𝑎𝑎4𝜋𝜋(4)
𝜋𝜋∈𝑆𝑆4

Để ý rằng số hạng ứng với 𝜋𝜋 ∈ 𝑆𝑆4 trong khai triển det 𝐴𝐴 khác 0 nếu và chỉ nếu nó cũng khác 0
trong khai triển của det 𝐵𝐵. Vì các số hạng này bằng ±1 và det 𝐵𝐵 = −3 nên số các số hạng khác 0
trong khai triển của det 𝐵𝐵, cũng như det 𝐴𝐴 là một số lẻ. Suy ra det 𝐴𝐴 ≠ 0.

Bài 9.2. Tìm giá trị lớn nhất của định thức cấp 3 có các phần tử là 1 hoặc -1.

Giải.

Khai triển định thức bằng định nghĩa thì rõ ràng cả 6 số hạng của nó khác 0. Mặt khác
(𝑎𝑎11 𝑎𝑎22 𝑎𝑎33 )(𝑎𝑎12 𝑎𝑎23 𝑎𝑎31 )(𝑎𝑎13 𝑎𝑎21 𝑎𝑎32 ) = −(−𝑎𝑎11 𝑎𝑎23 𝑎𝑎32 )(−𝑎𝑎13 𝑎𝑎22 𝑎𝑎31 )(−𝑎𝑎12 𝑎𝑎21 𝑎𝑎33 )
nên ít nhất trong 6 số hạng này phải có ít nhất 2 số hạng đối nhau. Do đó định thức có trị tuyệt
đối lớn nhất là 4. Thực tế, đó là định thức có các phần tử trên đường chéo chính là -1, các phần
tử còn lại là 1.

Bài 9.3. Chứng minh ma trận sau không suy biến


1 2 … 2008 2009

⎜ 22 32 … 20092 20102 ⎞ ⎟
𝐴𝐴 = ⎜
⎜ 33 43 … 20103 20103 ⎟ ⎟
⎜ ⋮ ⋮ … ⋮ ⋮ ⎟
⎝ 20092009 20102009 … 20102009 2010 2009 ⎠

Đỗ Minh Triết IX-119 - MATHTASY -


Ôn thi Olympic Toán Đại số Phần IX: Định thức

Chứng minh.

Trong khai triển |𝐴𝐴| chỉ có một số hạng là lẻ, đó là tích các phần tử trên đường chéo phụ (ứng
với phép thế có số nghịch thế lớn nhất), các số hạng còn lại đều chẵn do có chứa nhân tử là luỹ
thừa của 2010. Như vậy |𝐴𝐴| là một số lẻ, nghĩa là |𝐴𝐴| ≠ 0.
(Cách khác: xét trên modun 2, |𝐴𝐴| đồng dư với định thức mà các phần tử trên đường chéo phụ
bằng 1, các phần tử phía dưới đường chéo phụ bằng 0, do đó |𝐴𝐴| ≡ 1 (𝑚𝑚𝑚𝑚𝑚𝑚 2) ⇒ |𝐴𝐴| ≠ 0)

Bài 9.4. (Tổng quát câu 2 Olympic 2001) Cho các ma trận vuông thực 𝐴𝐴, 𝐵𝐵 thoả mãn
𝐴𝐴𝑛𝑛 = 0, 𝐴𝐴𝐴𝐴 = 𝐴𝐴 + 𝐵𝐵. Chứng minh det 𝐵𝐵 = 0.
Chứng minh
Cách 1:

Từ giả thiết 𝐴𝐴𝑛𝑛 = 0, suy ra det 𝐴𝐴 = 0. Từ giả thiết 𝐴𝐴𝐴𝐴 = 𝐴𝐴 + 𝐵𝐵, ta có


det 𝐵𝐵 = det(𝐴𝐴𝐴𝐴 − 𝐴𝐴) = det 𝐴𝐴 det(𝐵𝐵 − 𝐼𝐼) = 0.

Cách 2:

Giả sử det 𝐵𝐵 ≠ 0 tức là 𝐵𝐵 khả nghịch. Từ giả thiết, ta có


0 = 𝐴𝐴𝑛𝑛 = 𝐴𝐴𝑛𝑛 𝐵𝐵 = 𝐴𝐴𝑛𝑛−1 (𝐴𝐴𝐴𝐴) = 𝐴𝐴𝑛𝑛−1 (𝐴𝐴 + 𝐵𝐵) = 𝐴𝐴𝑛𝑛 + 𝐴𝐴𝑛𝑛−1 𝐵𝐵 = 𝐴𝐴𝑛𝑛−1 𝐵𝐵
Do đó 0 = 0𝐵𝐵−1 = 𝐴𝐴𝑛𝑛−1 .
Lặp lại quá trình trên, cuối cùng ta thu được 𝐴𝐴 = 0 kết hợp giả thiết 𝐴𝐴𝐴𝐴 = 𝐴𝐴 + 𝐵𝐵 suy ra 𝐵𝐵 = 0,
mâu thuẫn . Vậy det 𝐵𝐵 = 0.

Cách 3:

Từ giả thiết 𝐴𝐴𝑛𝑛 = 0, suy ra 𝐴𝐴𝑛𝑛 – 𝐼𝐼 𝑛𝑛 = (𝐴𝐴 − 𝐼𝐼)(𝐴𝐴𝑛𝑛−1 + 𝐴𝐴𝑛𝑛−2 +. . . +𝐴𝐴 + 𝐼𝐼) = −𝐼𝐼. Do đó 𝐴𝐴– 𝐼𝐼 khả
nghịch.
Mặt khác 𝐴𝐴𝐴𝐴 = 𝐴𝐴 + 𝐵𝐵 hay (𝐴𝐴 − 𝐼𝐼)𝐵𝐵 = 𝐴𝐴. Vì 𝐴𝐴– 𝐼𝐼 khả nghịch nên rank[(𝐴𝐴 − 𝐼𝐼)𝐵𝐵] = rank 𝐵𝐵 =
rank 𝐴𝐴 ≤ 𝑛𝑛 − 1 do det 𝐴𝐴 = 0. Suy ra điều phải chứng minh.

Bài 9.5. Cho 𝑎𝑎1 , 𝑎𝑎2 , … , 𝑎𝑎𝑛𝑛 ∈ ℝ. Chứng minh rằng định thức
𝑎𝑎1 𝑎𝑎2 … 𝑎𝑎𝑛𝑛
�𝑎𝑎𝑛𝑛 𝑎𝑎1 … 𝑎𝑎𝑛𝑛−1 �
�� ⋮ ⋮ ⋱ ⋮ �� = 𝑓𝑓(𝜖𝜖1 )𝑓𝑓(𝜖𝜖2 ) … 𝑓𝑓(𝜖𝜖𝑛𝑛 )
� 𝑎𝑎2 𝑎𝑎3 … 𝑎𝑎1 �
trong đó 𝜖𝜖𝑖𝑖 (𝑖𝑖 = 1, … , 𝑛𝑛) là các căn bậc 𝑛𝑛 của đơn vị, 𝑓𝑓(𝑥𝑥) = 𝑎𝑎1 + 𝑎𝑎2 𝑥𝑥+. . . +𝑎𝑎𝑛𝑛 𝑥𝑥𝑛𝑛−1 .

Chứng minh.

Giả sử 𝜖𝜖1 = 1. Ta có

Đỗ Minh Triết IX-120 - MATHTASY -


Ôn thi Olympic Toán Đại số Phần IX: Định thức

𝑎𝑎1 𝑎𝑎2 … 𝑎𝑎𝑛𝑛 1 1 … 1 𝑓𝑓(1) 𝑓𝑓(𝜖𝜖2 ) … 𝑓𝑓(𝜖𝜖𝑛𝑛 )


�𝑎𝑎𝑛𝑛 𝑎𝑎1 … 𝑎𝑎𝑛𝑛−1 � �1 𝜖𝜖2 … 𝜖𝜖𝑛𝑛 � � �
𝑓𝑓(1) 𝜖𝜖2 𝑓𝑓(𝜖𝜖2 ) … 𝜖𝜖𝑛𝑛 𝑓𝑓(𝜖𝜖𝑛𝑛 ) �
�� ⋮ ⋮ ⋱ ⋮ �� � ⋮ � =�
� ⋮ ⋱ ⋮ � � ⋮ ⋮ ⋱ ⋮ �
� 𝑎𝑎2 𝑎𝑎3 … 𝑎𝑎1 � � 1 𝜖𝜖𝑛𝑛−1
2 … 𝜖𝜖𝑛𝑛−1𝑛𝑛 � 𝑛𝑛−1
�𝑓𝑓(1) 𝜖𝜖2 𝑓𝑓(𝜖𝜖2 ) … 𝑛𝑛 𝜖𝜖𝑛𝑛−1
𝑓𝑓(𝜖𝜖 )
𝑛𝑛 �
1 1 … 1
�1 𝜖𝜖2 … 𝜖𝜖𝑛𝑛 �
= 𝑓𝑓(𝜖𝜖1 )𝑓𝑓(𝜖𝜖2 ) … 𝑓𝑓(𝜖𝜖𝑛𝑛 ) � �
� ⋮ ⋮ ⋱ ⋮ �
� 1 𝜖𝜖𝑛𝑛−1
2 … 𝜖𝜖𝑛𝑛−1
𝑛𝑛 �
1 1 … 1
�1 𝜖𝜖2 … 𝜖𝜖𝑛𝑛 �
Do các 𝜖𝜖𝑖𝑖 (𝑖𝑖 = 1, … , 𝑛𝑛) đôi một khác nhau nên định thức Vandermonde � �≠
� ⋮ ⋮ ⋱ ⋮ �
� 1 𝜖𝜖𝑛𝑛−1
2 … 𝜖𝜖𝑛𝑛−1
𝑛𝑛 �
0, suy ra điều phải chứng minh.

Bài 9.6. (Câu 4 ngày thứ hai Olympic Quốc tế 2007)* Cho 𝑛𝑛 là số nguyên dương lẻ lớn hơn 1.
Xét ma trận 𝐴𝐴 = �𝑎𝑎𝑖𝑖𝑖𝑖 � vuông cấp 𝑛𝑛 thoả:

2 nếu 𝑖𝑖 = 𝑗𝑗
𝑎𝑎𝑖𝑖𝑖𝑖 = �1 nếu 𝑖𝑖 − 𝑗𝑗 ≡ 2 (mod 𝑛𝑛)
0 với các phần tử còn lại
Chứng minh |𝐴𝐴| = 4.

Chứng minh.

1 nếu 𝑖𝑖 − 𝑗𝑗 ≡ ±1 (mod 𝑛𝑛)


Để ý rằng 𝐴𝐴 = 𝐵𝐵2 , trong đó 𝐵𝐵 = �𝑏𝑏𝑖𝑖𝑖𝑖 � = �
0 với các phần tử còn lại
Ta tính
0 1 0 … 0 0 1
�1 0 1 … 0 1 0�
�0 1 0 … 1 0 0��

|𝐵𝐵| = � ⋮ ⋮ ⋮ ⋱ ⋮ ⋮ ⋮�
�0 0 1 … 0 1 0�
�0 1 0 … 1 0 1�
�1 0 0 … 0 1 0�
Khai triển theo dòng đầu
1 1 1 0 1
�0 0 1 � � 1 0 1 �
� 1 0 1 � � 1 0 1 �
� � � �
|𝐵𝐵| = − � 1 ⋱ ⋱ +
� � ⋱ ⋱ ⋱ �
� ⋱ 0 1 � � 1 0 1�
� 1 0 1� � 1 0�
�1 1 0� �1 1�
Khai triển theo cột đầu cả hai định thức

Đỗ Minh Triết IX-121 - MATHTASY -


Ôn thi Olympic Toán Đại số Phần IX: Định thức

0 1 1
⎛ �1 0 1 � �0 1 �⎞

⎜ � � �1 �⎟


⎜ � 1 0 1 � � 0 1 0 �⎟

= −⎜
⎜ � ⋱ ⋱ ⋱ �−� ⋱ ⋱ ⋱ ⎟
�⎟

⎜ ⎟
⎜ � 1 0 1 � � 1 0 1 �⎟

⎜� 1 0 1� � 1 0 1 �⎟
⎝� 1 0� � 1 0 1�⎠
1 0 1 0 1
⎛ � 1 0 1 � �1 0 1 �⎞

⎜ � � � 1 0 1 �⎟


⎜ 1 0 1 �⎟

⎜ � � � ⎟ = −(0 − 1) + (1 − 0)
+ ⎜� ⋱ ⋱ ⋱ �−� ⋱ ⋱ ⋱ �⎟

⎜ ⎟
⎜ � 0 1 0 1� � 1 0 1 �⎟

⎜� 1 0� � 1 0 1�⎟
⎝� 1� � 1 0�⎠
Ma trận thứ hai, thứ ba là ma trận tam giác trên, dưới với các phần tử trên đường chéo là 1, còn
với ma trận thứ nhất, thứ tư ta lấy 𝑑𝑑1 − 𝑑𝑑3 + 𝑑𝑑5 −. . . +𝑑𝑑𝑛𝑛−2 thì được dòng 0.
Vậy det 𝐵𝐵 = 2 suy ra det 𝐴𝐴 = 4.
2 0 1 1 0
2 1 1 �0 2 0 1 1�
Ví dụ minh hoạ cho 𝑛𝑛 = 3, 5: �1 2 1� = 4 ; ��1 0 2 0 1�� = 4.
1 1 2 �1 1 0 2 0�
�0 1 0 0 2�

Bài 9.7. Cho 𝐴𝐴 là ma trận vuông cấp 𝑛𝑛. Chứng minh rằng det(𝐴𝐴𝐴𝐴𝑇𝑇 + 𝐼𝐼) > 0.

Chứng minh.

𝐴𝐴𝐴𝐴𝑇𝑇 + 𝐼𝐼 là ma trận đối xứng nên nó là ma trận của một dạng toàn phương, hơn nữa dạng toàn
phương này xác định dương. Thật vậy, với mọi 𝑥𝑥 ≠ 0, sử dụng tích vô hướng trên ℂ, ta có
〈(𝐴𝐴𝐴𝐴𝑇𝑇 + 𝐼𝐼)𝑥𝑥, 𝑥𝑥〉 = 〈𝐴𝐴𝐴𝐴𝑇𝑇 𝑥𝑥, 𝑥𝑥〉 + 〈𝑥𝑥, 𝑥𝑥〉 = 〈𝐴𝐴𝑇𝑇 𝑥𝑥, 𝐴𝐴𝑇𝑇 𝑥𝑥〉 + 〈𝑥𝑥, 𝑥𝑥〉 > 0
Do đó mà các giá trị riêng của 𝐴𝐴𝐴𝐴𝑇𝑇 + 𝐼𝐼 đều dương vì vậy tích các giá trị riêng dương đó bằng
định thức của 𝐴𝐴𝐴𝐴𝑇𝑇 + 𝐼𝐼 cũng dương.
Nhận xét:
Trên đây là cách giải tổng quát, đúng với cả trường ℂ, ℝ. Tuy nhiên, đối với trường ℝ, ta còn có
thể giải bằng tích vô hướng thông thường:
Với mọi 𝑥𝑥 ∈ ℝ𝑛𝑛 , 𝑥𝑥 ≠ 0, ta có: 𝑥𝑥𝑇𝑇 (𝐴𝐴𝐴𝐴𝑇𝑇 + 𝐼𝐼)𝑥𝑥 = 𝑥𝑥𝑇𝑇 𝐴𝐴𝐴𝐴𝑇𝑇 𝑥𝑥 + 𝑥𝑥𝑇𝑇 𝑥𝑥 = (𝐴𝐴𝑇𝑇 𝑥𝑥)𝑇𝑇 𝐴𝐴𝑇𝑇 𝑥𝑥 + 𝑥𝑥𝑇𝑇 𝑥𝑥 > 0

Bài 9.8. Cho 𝐴𝐴, 𝐵𝐵 là hai ma trận vuông, thực, cấp 𝑛𝑛. Chứng minh rằng det(𝐴𝐴𝐴𝐴𝑇𝑇 + 𝐵𝐵𝐵𝐵𝑇𝑇 ) ≥ 0.

Chứng minh.

Hoàn toàn tương tự như trên.

Đỗ Minh Triết IX-122 - MATHTASY -


Ôn thi Olympic Toán Đại số Phần IX: Định thức

𝑥𝑥𝑇𝑇 (𝐴𝐴𝐴𝐴𝑇𝑇 + 𝐵𝐵𝐵𝐵𝑇𝑇 )𝑥𝑥 = 𝑥𝑥𝑇𝑇 𝐴𝐴𝐴𝐴𝑇𝑇 𝑥𝑥 + 𝑥𝑥𝑇𝑇 𝐵𝐵𝐵𝐵𝑇𝑇 𝑥𝑥 = (𝐴𝐴𝑇𝑇 𝑥𝑥)𝑇𝑇 𝐴𝐴𝑇𝑇 𝑥𝑥 + (𝐵𝐵𝑇𝑇 𝑥𝑥)𝑇𝑇 𝐵𝐵𝑇𝑇 𝑥𝑥 ≥ 0
đẳng thức xảy ra khi 𝐴𝐴𝑇𝑇 𝑥𝑥 = 𝐵𝐵𝑇𝑇 𝑥𝑥 = 0.

Bài 9.9. Cho ma trận 𝐴𝐴 thoả mãn 𝐴𝐴 + 𝐴𝐴𝑇𝑇 = 0. Chứng minh 𝐼𝐼 + 𝐴𝐴 khả nghịch.

Chứng minh.

Cách 1:

Ma trận 𝐼𝐼 + 𝐴𝐴𝑇𝑇 𝐴𝐴 xác định dương, thật vậy ∀𝑥𝑥 ≠ 0, ta có


〈(𝐼𝐼 + 𝐴𝐴𝑇𝑇 𝐴𝐴)𝑥𝑥, 𝑥𝑥〉 = 〈𝑥𝑥, 𝑥𝑥〉 + 〈𝐴𝐴𝑇𝑇 𝐴𝐴𝐴𝐴, 𝑥𝑥〉 = 〈𝑥𝑥, 𝑥𝑥〉 + 〈𝐴𝐴𝐴𝐴, 𝐴𝐴𝐴𝐴〉 > 0
Do đó det(𝐼𝐼 + 𝐴𝐴𝑇𝑇 𝐴𝐴) > 0. Mặt khác, từ giả thiết, ta có 𝐴𝐴 = −𝐴𝐴𝑇𝑇 nên 𝐴𝐴, 𝐴𝐴𝑇𝑇 giao hoán
(𝐼𝐼 + 𝐴𝐴𝑇𝑇 𝐴𝐴) = (𝐼𝐼 + 𝐴𝐴)(𝐼𝐼 + 𝐴𝐴𝑇𝑇 )
Suy ra
det(𝐼𝐼 + 𝐴𝐴𝑇𝑇 𝐴𝐴) = det(𝐼𝐼 + 𝐴𝐴) det(𝐼𝐼 + 𝐴𝐴𝑇𝑇 ) > 0
Vậy det(𝐼𝐼 + 𝐴𝐴) ≠ 0 hay 𝐼𝐼 + 𝐴𝐴 khả nghịch.

Cách 2:

Từ giả thiết, ta có 𝐴𝐴 = −𝐴𝐴𝑇𝑇 nên 𝐴𝐴, 𝐴𝐴𝑇𝑇 giao hoán.


Nếu 𝐼𝐼 + 𝐴𝐴 suy biến thì tồn tại 𝑥𝑥 ≠ 0 sao cho (𝐼𝐼 + 𝐴𝐴)𝑥𝑥 = 0 hay 𝐴𝐴𝐴𝐴 = −𝑥𝑥 hay 𝐴𝐴𝑇𝑇 𝑥𝑥 = 𝑥𝑥 ,vậy
𝐴𝐴𝑇𝑇 𝐴𝐴𝐴𝐴 = −𝑥𝑥. Suy ra
〈𝑥𝑥, 𝐴𝐴𝑇𝑇 𝐴𝐴𝐴𝐴〉 = −〈𝑥𝑥, 𝑥𝑥〉
Mặt khác ∀𝑥𝑥 ≠ 0, ta có 〈𝑥𝑥, 𝐴𝐴𝑇𝑇 𝐴𝐴𝐴𝐴〉 = 〈𝐴𝐴𝐴𝐴, 𝐴𝐴𝐴𝐴〉 ≥ 0 nhưng −〈𝑥𝑥, 𝑥𝑥〉 < 0, mâu thuẫn.
Vậy 𝐼𝐼 + 𝐴𝐴 khả nghịch.

Bài 9.10. (Câu 1 Olympic 2010) Tìm tất cả các ma trận vuông 𝐴𝐴 cấp 𝑛𝑛 ≥ 2 sao cho với mọi
ma trận 𝐵𝐵 vuông cấp 𝑛𝑛, ta đều có det(𝐴𝐴 + 𝐵𝐵) = det 𝐴𝐴 + det 𝐵𝐵.

Giải.

Chọn 𝐵𝐵 = 𝐴𝐴 thì từ giả thiết, ta có det(2𝐴𝐴) = 2 det 𝐴𝐴, mặt khác det(2𝐴𝐴) = 2𝑛𝑛 det 𝐴𝐴. Suy ra
det 𝐴𝐴 = 0.
Giả sử 𝐴𝐴 ≠ 0, khi đó tồn tại cột 𝐴𝐴𝑖𝑖 ≠ 0 (1 ≤ 𝑖𝑖 ≤ 𝑛𝑛).
Chọn ma trận 𝐵𝐵 có các cột 𝐵𝐵1 , … , 𝐵𝐵𝑖𝑖−1 , −𝐴𝐴𝑖𝑖 , 𝐵𝐵𝑖𝑖+1 , … , 𝐵𝐵𝑛𝑛 sao cho det 𝐵𝐵 ≠ 0 khi đó
det(𝐴𝐴 + 𝐵𝐵) = det 𝐴𝐴 + det 𝐵𝐵 = det 𝐵𝐵
Mặt khác, cột thứ 𝑖𝑖 của ma trận 𝐴𝐴 + 𝐵𝐵 là 0 nên det(𝐴𝐴 + 𝐵𝐵) = det 𝐵𝐵 = 0, mâu thuẫn.
Vậy 𝐴𝐴 = 0.

Đỗ Minh Triết IX-123 - MATHTASY -


Ôn thi Olympic Toán Đại số Phần IX: Định thức

Bài 9.11. (Câu 1 Olympic 2010) Cho 𝐴𝐴, 𝐵𝐵 là các ma trận vuông cấp 2010 với hệ số thực
sao cho
det 𝐴𝐴 = det(𝐴𝐴 + 𝐵𝐵) = det(𝐴𝐴 + 2𝐵𝐵) =. . . = det(𝐴𝐴 + 2010𝐵𝐵) = 0.
a) Chứng minh rằng det(𝑥𝑥𝑥𝑥 + 𝑦𝑦𝑦𝑦) = 0 ∀𝑥𝑥, 𝑦𝑦 ∈ ℝ.
b) Tìm ví dụ chứng tỏ kết luận trên không còn đúng nếu chỉ có
det 𝐴𝐴 = det(𝐴𝐴 + 𝐵𝐵) = det(𝐴𝐴 + 2𝐵𝐵) =. . . = det(𝐴𝐴 + 2009𝐵𝐵) = 0.

Chứng minh.

a) Nhận xét rằng định thức 𝑃𝑃 (𝑡𝑡) = det(𝐴𝐴 + 𝑡𝑡𝑡𝑡) là một đa thức bậc 2010 của 𝑡𝑡 có 2011 nghiệm
0, 1, … , 2010 nên 𝑃𝑃 (𝑡𝑡) ≡ 0. Định thức 𝑄𝑄(𝑡𝑡) = det(𝑡𝑡𝑡𝑡 + 𝐵𝐵) cũng là đa thức bậc 2010 của 𝑡𝑡. Mà
𝑄𝑄(𝑡𝑡) = 𝑡𝑡2010 det(𝐴𝐴 + 1𝑡𝑡 𝐵𝐵) = 𝑃𝑃 (𝑡𝑡−1 ), 𝑡𝑡 ≠ 0. Do đó ta cũng có 𝑄𝑄(𝑡𝑡) ≡ 0.
- Với 𝑥𝑥 = 0, 𝑦𝑦 ≠ 0 thì det(𝐴𝐴 + 0𝐵𝐵) = det(𝐴𝐴) = 𝑃𝑃 (0) = 0
- Với 𝑥𝑥 ≠ 0, 𝑦𝑦 = 0 thì det(0𝐴𝐴 + 𝑦𝑦𝑦𝑦) = det(𝑦𝑦𝑦𝑦) = 𝑄𝑄(0) = 0
- Với 𝑥𝑥 ≠ 0, 𝑦𝑦 ≠ 0 thì ta có det(𝑥𝑥𝑥𝑥 + 𝑦𝑦𝑦𝑦) = 𝑥𝑥2010 det(𝐴𝐴 + 𝑥𝑥𝑦𝑦 𝐵𝐵) = 𝑃𝑃 (𝑥𝑥𝑦𝑦 ) = 0
Vậy det(𝑥𝑥𝑥𝑥 + 𝑦𝑦𝑦𝑦) = 0 ∀𝑥𝑥, 𝑦𝑦 ∈ ℝ.
0 0 0 … 0

⎜0 1 0 … 0⎞ ⎟
b) Chọn 𝐴𝐴 = diag(0,1,2, … ,2009) = ⎜
⎜ 0 0 2 … 0 ⎟
⎟ và 𝐵𝐵 = −𝐼𝐼
⎜⋮ ⋮ ⋮ ⋱ ⋮ ⎟
⎝0 0 0 … 2009⎠
Khi đó det 𝐴𝐴 = det(𝐴𝐴 + 𝐵𝐵) = det(𝐴𝐴 + 2𝐵𝐵) =. . . = det(𝐴𝐴 + 2009𝐵𝐵) = 0 nhưng det(𝐴𝐴 + 2010𝐵𝐵) ≠
0.

Bài 9.12. Cho ma trận cấp 10 có các phần tử nguyên, trong đó ít nhất 92 phần tử lẻ. Chứng
minh det 𝐴𝐴 ⋮ 2.

Chứng minh.

Ma trận có ít nhất 92 phần tử lẻ nên nó có ít nhất hai dòng mà tất cả các phần tử đều lẻ, trừ
dòng này cho dòng kia, ta được một dòng mới mà tất cả các phần tử đều chẵn. Từ đó det 𝐴𝐴 ⋮ 2.

Đỗ Minh Triết IX-124 - MATHTASY -


PHẦN X. LUỸ THỪA MA TRẬN (Power of the Matrix)
Bài Toán tính luỹ thừa ma trận 𝐴𝐴 vuông (cấp 2, 3) cho trước có 2 dạng: một là tính 𝐴𝐴𝑛𝑛 ∀𝑛𝑛 ∈ ℕ∗ ,
hai là tính 𝐴𝐴𝑚𝑚 với 𝑚𝑚 ∈ ℕ∗ cho trước. Bài Toán tính 𝐴𝐴𝑚𝑚 có thể đưa về tính 𝐴𝐴𝑛𝑛 sau đó thay 𝑛𝑛 =
𝑚𝑚. Sau đây là một số phương pháp cho bài Toán này:

10.1. Luỹ thừa ma trận cấp 3

Phương pháp 1: ∃𝑘𝑘 ∈ ℕ∗ (không quá lớn) sao cho 𝐴𝐴𝑘𝑘 = ±𝐼𝐼 hoặc 𝐴𝐴𝑘𝑘 = 0.

Nếu 𝐴𝐴𝑘𝑘+1 = 𝐴𝐴𝑘𝑘 thì 𝐴𝐴𝑛𝑛 = 𝐴𝐴𝑘𝑘 ∀𝑛𝑛 ≥ 𝑘𝑘.

0 −2 1 𝑛𝑛
Ví dụ 1 : Tính 𝐴𝐴 = � −1
𝑛𝑛
0 0� .
−2 0 0
Giải.

Ta có 𝐴𝐴3 = 0.
Kết luận (luôn nhớ phải kết luận đầy đủ):
𝐴𝐴1 =. . ., 𝐴𝐴2 =. . ., 𝐴𝐴𝑛𝑛 = 0 ∀𝑛𝑛 ≥ 3
2002
1 0 0
⎛ 0 √1 −1⎞

Ví dụ 2 : Tính 𝐴𝐴2002 =⎜
⎜ 2 2⎟

1 1
⎝0 √
2

2⎠

Giải.
1 0 0
Ta có 𝐴𝐴2 =. . . , 𝐴𝐴3 =. . . , 𝐴𝐴4 = �0 −1 0� ⇒ 𝐴𝐴8 = 𝐼𝐼
0 0 −1
Suy ra 𝐴𝐴2002 = (𝐴𝐴8 )250 . 𝐴𝐴2 = 𝐴𝐴2 =. ..
1 0 0 2009
Ví dụ 3 : Tính 𝐴𝐴2009 = �1 0 0�
0 1 0
Giải.
1 0 0 1 0 0
𝐴𝐴2 = �1 0 0� = 𝐴𝐴3 ⇒ 𝐴𝐴2009 = 𝐴𝐴2 = �1 0 0�
1 0 0 1 0 0

Phương pháp 2: Tách 𝐴𝐴𝑛𝑛 = (𝐵𝐵 + 𝑎𝑎𝑎𝑎)𝑛𝑛 , trong đó luỹ thừa 𝐵𝐵 dễ tính.

Đỗ Minh Triết X-125 - MATHTASY -


Ôn thi Olympic Toán Đại số Phần X: Luỹ thừa ma trận

1 −2 1 𝑛𝑛
Ví dụ 1 : Tính 𝐴𝐴 = � −1
𝑛𝑛
1 0� .
−2 0 0
Giải.
0 −2 1 1 0 0
Tách 𝐴𝐴 = � −1 0 0� + �0 1 0� = 𝐵𝐵 + 𝐼𝐼 thì 𝐵𝐵𝑘𝑘 = 0 ∀𝑘𝑘 ≥ 3. Do đó
−2 0 0 0 0 1
𝑛𝑛(𝑛𝑛 − 1) 2
𝐴𝐴𝑛𝑛 = (𝐵𝐵 + 𝐼𝐼)𝑛𝑛 = 𝐼𝐼 + 𝑛𝑛𝑛𝑛 + 𝐵𝐵 = ⋯
2

Phương pháp 3: Tách 𝐴𝐴𝑛𝑛 = (𝐵𝐵 + 𝐶𝐶)𝑛𝑛 với 𝐵𝐵𝐵𝐵 = 𝐶𝐶𝐶𝐶, luỹ thừa 𝐵𝐵, 𝐶𝐶 dễ tính

Phương pháp này rất ít khi dùng.


2 0 0 𝑛𝑛
Ví dụ 1 : Tính 𝐴𝐴 = �0 3
𝑛𝑛
1� .
0 0 3
Giải.
2 0 0 0 0 0
Tách 𝐴𝐴 = �0 3 0� + �0 0 1� = 𝐵𝐵 + 𝐶𝐶
0 0 3 0 0 0
2𝑛𝑛 0 0
Ta có 𝐵𝐵𝑛𝑛 = � 0 3𝑛𝑛 0 � , 𝐶𝐶 2 = 0
0 0 3𝑛𝑛
Suy ra 𝐴𝐴𝑛𝑛 = (𝐵𝐵 + 𝐶𝐶)𝑛𝑛 = 𝐶𝐶𝑛𝑛𝑛𝑛−1 𝐵𝐵𝑛𝑛−1 𝐶𝐶 + 𝐶𝐶𝑛𝑛𝑛𝑛 𝐵𝐵𝑛𝑛 = ⋯

Phương pháp 4: Quy nạp

Đầu tiên ta tính một số luỹ thừa bậc thấp (2,3) của 𝐴𝐴, sau đó dự đoán công thức tổng quát
rồi chứng minh bằng quy nạp.

𝑎𝑎 0 0 𝑛𝑛
Ví dụ 1 : Tính 𝐴𝐴 = � 𝑏𝑏 0
𝑛𝑛
𝑎𝑎�
0 0 𝑎𝑎
Giải.

𝑎𝑎2 0 0
Ta có 𝐴𝐴 = �𝑎𝑎𝑎𝑎 0 𝑎𝑎2 � từ đó dự đoán và chứng minh quy nạp rằng
2

0 0 𝑎𝑎2
𝑎𝑎𝑛𝑛 0 0
𝑛𝑛
𝐴𝐴 = �𝑎𝑎 𝑏𝑏 0 𝑎𝑎𝑛𝑛 � ∀𝑛𝑛 ≥ 2
𝑛𝑛−1

0 0 𝑎𝑎𝑛𝑛

Đỗ Minh Triết X-126 - MATHTASY -


Ôn thi Olympic Toán Đại số Phần X: Luỹ thừa ma trận

𝑎𝑎 0 0 𝑛𝑛 𝑎𝑎𝑛𝑛 0 0
Ví dụ 2 : Tính 𝐴𝐴 = �0 𝑎𝑎 0 � = �0
𝑛𝑛
𝑎𝑎𝑛𝑛 0 �.
𝑏𝑏 0 𝑎𝑎 𝑛𝑛𝑎𝑎𝑛𝑛−1 𝑏𝑏 0 𝑎𝑎𝑛𝑛

Phương pháp 5: Chéo hoá

Phương pháp này rất hay dùng, tức là ta có 𝐴𝐴 = 𝑃𝑃𝑃𝑃𝑃𝑃 −1 trong đó 𝐵𝐵 là ma trận đường chéo.
Khi đó 𝐴𝐴𝑛𝑛 = 𝑃𝑃 𝐵𝐵 𝑛𝑛 𝑃𝑃 −1 , luỹ thừa 𝐵𝐵 chỉ cần luỹ thừa các phần tử trên đường chéo chính, từ đó
tính được 𝐴𝐴𝑛𝑛 dễ dàng.

0 1 0 𝑛𝑛
Ví dụ 1 : Tính 𝐴𝐴 = �0
𝑛𝑛
0 1� .
6 −11 6
Giải.

Đa thức đặc trưng của A:


−𝜆𝜆 1 0
� 0 −𝜆𝜆 1 � = −(𝜆𝜆 − 1)(𝜆𝜆 − 2)(𝜆𝜆 − 3)
6 −11 6 − 𝜆𝜆
⇒ A có 3 giá trị riêng phân biệt 𝜆𝜆 = 1, 𝜆𝜆 = 2, 𝜆𝜆 = 3 nên A chéo hoá được.
1
- Với 𝜆𝜆 = 1, ta được một vector riêng �1�
1
1
- Với 𝜆𝜆 = 2, ta được một vector riêng �2�
4
1
- Với 𝜆𝜆 = 3, ta được một vector riêng �3�
9
Ký hiệu
−5 1
⎛ 3
1 0 0 1 1 1 ⎜ 2 2⎞⎟
𝐵𝐵 = �0 2 0 � ; 𝐶𝐶 = �1 −1 ⎜
2 3� ⇒ 𝐶𝐶 = ⎜ −3 4 −1 ⎟


⎜ ⎟
0 0 3 1 4 9 −3 1⎟
⎝ 1 2 2⎠
thì 𝐴𝐴 = 𝐶𝐶𝐶𝐶𝐶𝐶 −1 , suy ra
−5 3 1 1
⎛ 3 − 3.2𝑛𝑛 + 3𝑛𝑛 + 4.2𝑛𝑛 − . 3𝑛𝑛 − 2𝑛𝑛 + . 3𝑛𝑛 ⎞
⎜ 2 2 2 2 ⎟

⎜ −5 3 1 1 ⎟

𝐴𝐴𝑛𝑛 = 𝐶𝐶𝐵𝐵𝑛𝑛 𝐶𝐶 −1 =⎜
⎜ 3 − 3.2𝑛𝑛+1 + 3𝑛𝑛+1 + 4.2𝑛𝑛+1 − . 3𝑛𝑛+1 − 2𝑛𝑛+1 + . 3𝑛𝑛+1 ⎟


⎜ 2 2 2 2 ⎟

⎜ −5 3 1 1 ⎟
𝑛𝑛+2
⎝ 3 − 3.2 + 3𝑛𝑛+2 + 4.2𝑛𝑛+2 − . 3𝑛𝑛+2 − 2𝑛𝑛+2 + . 3𝑛𝑛+2 ⎠
2 2 2 2

Đỗ Minh Triết X-127 - MATHTASY -


Ôn thi Olympic Toán Đại số Phần X: Luỹ thừa ma trận

Phương pháp 6a: Tam giác hoá

1. Thuật Toán tam giác hoá

Không phải ma trận 𝐴𝐴 ∈ 𝑀𝑀3 (ℝ) nào cũng chéo hoá được, tuy nhiên, luôn có thể tam giác hoá
𝐴𝐴, tức là tìm được ma trận 𝑃𝑃 khả nghịch sao cho 𝐴𝐴 = 𝑃𝑃𝑃𝑃𝑃𝑃 −1 , trong đó, 𝐵𝐵 là ma trận tam
giác trên. Khi 𝐴𝐴 không chéo hoá được thì thường xảy ra hai trường hợp sau:

- 𝐴𝐴 có một giá trị riêng kép, một giá trị riêng đơn (giá trị riêng kép chỉ cho một vector riêng).

- 𝐴𝐴 có một giá trị riêng bội ba (chỉ cho 2 hoặc 1 vector riêng).

Ví dụ 1: 𝑨𝑨 có một giá trị riêng kép, một giá trị riêng đơn (giá trị riêng kép chỉ cho
một vector riêng)
5 −17 25
Tam giác hoá 𝐴𝐴 = �2 −9 16�
1 −5 9
Đa thức đặc trưng của 𝐴𝐴:
𝑃𝑃𝐴𝐴 (𝜆𝜆) = −(𝜆𝜆 − 2)2 (𝜆𝜆 − 1)
𝐴𝐴 có một giá trị riêng kép 𝜆𝜆1 = 2 và một giá trị riêng đơn 𝜆𝜆2 = 1.
3
Với 𝜆𝜆1 = 2, ta được một vector riêng tương ứng 𝑣𝑣1 = �2�
1
11
Với 𝜆𝜆2 = 1, ta được một vector riêng tương ứng 𝑣𝑣3 = � 7 �
3
1
Ta cần tìm 𝑣𝑣2 sao cho 𝐴𝐴𝑣𝑣2 = 𝜆𝜆1 𝑣𝑣2 + 𝑣𝑣1 chẳng hạn 𝑣𝑣2 = �0�
0
3 1 11 0 −3 7
Ký hiệu 𝑃𝑃 = �2 0 7 � , 𝑃𝑃 −1 = �1 −2 1�. Khi đó
1 0 3 0 1 −2
2 1 0
𝐵𝐵 = 𝑃𝑃 −1 𝐴𝐴𝐴𝐴 = �0 2 0�
0 0 1
là ma trận tam giác trên.

Ví dụ 2: 𝑨𝑨 có một giá trị riêng bội ba chỉ cho 2 vector riêng.


−2 2 −1
Tam giác hoá 𝐴𝐴 = �−1 1 −1�
−1 2 −2

Đỗ Minh Triết X-128 - MATHTASY -


Ôn thi Olympic Toán Đại số Phần X: Luỹ thừa ma trận

Đa thức đặc trưng của 𝐴𝐴:


𝑃𝑃𝐴𝐴 (𝜆𝜆) = −(𝜆𝜆 + 1)3
𝐴𝐴 có một giá trị riêng bội ba 𝜆𝜆1 = −1.
1 0
Với 𝜆𝜆1 = −1, ta được hai vector riêng tương ứng 𝑣𝑣1 = � 0� và 𝑣𝑣2 = �1�
−1 2
1
Ta cần tìm 𝑣𝑣3 sao cho 𝑣𝑣1 , 𝑣𝑣2 , 𝑣𝑣3 độc lập tuyến tính, chẳng hạn 𝑣𝑣3 = �0�
0
1 0 1 0 2 −1
Ký hiệu 𝑃𝑃 = � 0 1 0� , 𝑃𝑃 −1 = �0 1 0�. Khi đó
−1 2 0 1 −2 1
−1 0 −1
𝐵𝐵 = 𝑃𝑃 −1 𝐴𝐴𝐴𝐴 = � 0 −1 −1�
0 0 −1
là ma trận tam giác trên.

Ví dụ 3: 𝑨𝑨 có một giá trị riêng bội ba chỉ cho 1 vector riêng.


2 0 1
Tam giác hoá 𝐴𝐴 = � 1 1 0�
−1 1 3
Đa thức đặc trưng của 𝐴𝐴:
𝑃𝑃𝐴𝐴 (𝜆𝜆) = (2 − 𝜆𝜆)3
1
𝐴𝐴 có một giá trị riêng bội ba 𝜆𝜆 = 2 và chỉ được một vector riêng tương ứng 𝑣𝑣1 = �1�
0
Ta cần tìm 𝑣𝑣2 sao cho 𝐴𝐴𝑣𝑣2 = 𝜆𝜆𝑣𝑣2 + 𝑣𝑣1 rồi chọn 𝑣𝑣3 sao cho 𝑣𝑣1 , 𝑣𝑣2 , 𝑣𝑣3 độc lập tuyến tính.
1 1
Chẳng hạn 𝑣𝑣2 = �0� , 𝑣𝑣3 = �0�
1 0
1 1 1 0 1 0
Ký hiệu 𝑃𝑃 = �1 0 0� , 𝑃𝑃 = �0
−1
0 1�. Khi đó
0 1 0 1 −1 −1
2 1 1
𝐵𝐵 = 𝑃𝑃 −1 𝐴𝐴𝐴𝐴 = �0 2 −1 �
0 0 2
là ma trận tam giác trên.

Phương pháp 6b: Tam giác hoá

2. Áp dụng tính luỹ thừa ma trận:

Đỗ Minh Triết X-129 - MATHTASY -


Ôn thi Olympic Toán Đại số Phần X: Luỹ thừa ma trận

4 6 −15 𝑛𝑛
Ví dụ 1 : Tính 𝐴𝐴 = �1
𝑛𝑛
3 −5 � .
1 2 −4
Giải.

Đa thức đặc trưng của 𝐴𝐴: 𝑃𝑃𝐴𝐴 (𝜆𝜆) = −(𝜆𝜆 − 1)3


−2 5 1
Với 𝜆𝜆 = 1, hệ (𝐴𝐴 − 𝜆𝜆𝜆𝜆)𝑥𝑥 = 0 cho ta 2 vector riêng 𝑣𝑣1 = � 1� và 𝑣𝑣2 = �0�, chọn 𝑣𝑣3 = �0�
0 1 0
1 0 0 −2 5 1
Đặt 𝐵𝐵 = �0 1 0� và 𝑃𝑃 = � 1 0 0� → 𝑃𝑃 −1 =. .. thì ta có 𝐵𝐵 = 𝑃𝑃 −1 𝐴𝐴𝐴𝐴
0 0 1 0 1 0
1 0 𝑛𝑛
Dễ dàng tính được 𝐵𝐵𝑛𝑛 = �0 1 𝑛𝑛�, từ đó 𝐴𝐴𝑛𝑛 = 𝑃𝑃 𝐵𝐵𝑛𝑛 𝑃𝑃 −1 =. ..
0 0 1

Phương pháp 7: Sử dụng đa thức đặc trưng, đa thức tối tiểu của 𝐴𝐴

4 6 −15 2010
Ví dụ 1 (Câu 10 đề dự tuyển ĐH KH Huế 2010) : Tính �1 3 −5 � .
1 2 −4
Giải.
4 6 −15
Kí hiệu 𝐴𝐴 = �1 3 −5 �. Đa thức đặc trưng và đa thức tối tiểu của 𝐴𝐴 là
1 2 −4
𝑃𝑃𝐴𝐴 (𝑥𝑥) = (1 − 𝑥𝑥)3 ; 𝛱𝛱𝐴𝐴 (𝑥𝑥) = (𝑥𝑥 − 1)2
Chia 𝑥𝑥2010 cho 𝛱𝛱𝐴𝐴 (𝑥𝑥) = (𝑥𝑥 − 1)2 , ta được:
𝑥𝑥2010 = (𝑥𝑥 − 1)2 𝑄𝑄(𝑥𝑥) + (𝑎𝑎𝑎𝑎 + 𝑏𝑏)
Đạo hàm
2010𝑥𝑥2009 = 2(𝑥𝑥 − 1)𝑄𝑄(𝑥𝑥) + (𝑥𝑥 − 1)2 𝑄𝑄′ (𝑥𝑥) + 𝑎𝑎
Thay 𝑥𝑥 = 1 vào hai đẳng thức trên, ta được 𝑎𝑎 + 𝑏𝑏 = 1, 𝑎𝑎 = 2010. Từ đó
6031 12060 −30150
𝐴𝐴2010 = 2010𝐴𝐴 − 2009𝐼𝐼 = �2010 4021 −10050�
2010 4020 −10049

Phương pháp 8: Luỹ thừa của một ma trận đối xứng là một ma trận đối xứng.

2 1 1 𝑛𝑛
Ví dụ 1 : Tính 𝐴𝐴 = �1 2
𝑛𝑛
1� .
1 1 2

Đỗ Minh Triết X-130 - MATHTASY -


Ôn thi Olympic Toán Đại số Phần X: Luỹ thừa ma trận

Do 𝐴𝐴 là ma trận đối xứng nên 𝐴𝐴𝑛𝑛 cũng là ma trận đối xứng, nó có dạng như sau
2 1 1 𝑛𝑛 𝑎𝑎𝑛𝑛 𝑏𝑏𝑛𝑛 𝑏𝑏𝑛𝑛
�1 ⎛
2 1� = ⎜ 𝑏𝑏𝑛𝑛 𝑎𝑎𝑛𝑛 𝑏𝑏𝑛𝑛 ⎞

1 1 2 ⎝ 𝑏𝑏𝑛𝑛 𝑏𝑏𝑛𝑛 𝑎𝑎𝑛𝑛 ⎠
Xét đẳng thức 𝐴𝐴𝑛𝑛+1 = 𝐴𝐴𝑛𝑛 𝐴𝐴 ta được 𝑎𝑎𝑛𝑛+1 = 2𝑎𝑎𝑛𝑛 + 2𝑏𝑏𝑛𝑛 và 𝑏𝑏𝑛𝑛+1 = 𝑎𝑎𝑛𝑛 + 3𝑏𝑏𝑛𝑛 . Do đó ta có
𝑎𝑎𝑛𝑛+1 − 𝑏𝑏𝑛𝑛+1 = 𝑎𝑎𝑛𝑛 − 𝑏𝑏𝑛𝑛 = ⋯ = 𝑎𝑎1 − 𝑏𝑏1 = 1

𝑎𝑎𝑛𝑛+1 + 2𝑏𝑏𝑛𝑛+1 = 4(𝑎𝑎𝑛𝑛 + 2𝑏𝑏𝑛𝑛 ) = ⋯ = 4 (𝑎𝑎1 + 2𝑏𝑏1 ) = 4𝑛𝑛+1
𝑛𝑛

Giải hệ trên, tìm được 𝑎𝑎𝑛𝑛+1 = 4 . Vậy


𝑛𝑛+1 +2 𝑛𝑛+1 −1
3
, 𝑏𝑏𝑛𝑛+1 = 4 3
𝑛𝑛 𝑛𝑛
1 4𝑛𝑛 + 2 4𝑛𝑛 − 1 4𝑛𝑛 − 1
𝑛𝑛
𝐴𝐴 = �4 − 1 4 + 2 4𝑛𝑛 − 1�
3
4𝑛𝑛 − 1 4𝑛𝑛 − 1 4𝑛𝑛 + 2

10.2. Luỹ thừa ma trận cấp 2

1. Tính luỹ thừa ma trận cấp 2 tổng quát


𝑎𝑎 𝑏𝑏 𝑛𝑛
Giả sử ta cần tính 𝐴𝐴𝑛𝑛 = � �
𝑐𝑐 𝑑𝑑
Xét đa thức đặc trưng và biệt thức ∆
𝑃𝑃𝐴𝐴 (𝑥𝑥) = 𝑥𝑥2 − (𝑎𝑎 + 𝑑𝑑)𝑥𝑥 + (𝑎𝑎𝑎𝑎 − 𝑏𝑏𝑏𝑏)
∆ = (𝑎𝑎 + 𝑑𝑑)2 − 4(𝑎𝑎𝑎𝑎 − 𝑏𝑏𝑏𝑏) = (𝑎𝑎 − 𝑑𝑑)2 + 4𝑏𝑏𝑏𝑏
+) Nếu ∆ > 0, đa thức đặc trưng có 2 nghiệm phân biệt hay 𝐴𝐴 có 2 giá trị riêng phân biệt nên
𝐴𝐴 chéo hoá được trên ℝ và việc tính luỹ thừa 𝐴𝐴 là đơn giản.
2
+) Nếu ∆ = 0 thì 𝑎𝑎𝑎𝑎 − 𝑏𝑏𝑏𝑏 = (𝑎𝑎+𝑑𝑑)
4
, ta có
(𝑎𝑎 + 𝑑𝑑)2 𝑎𝑎 + 𝑑𝑑 2
𝑃𝑃𝐴𝐴 (𝐴𝐴) = 𝐴𝐴2 − (𝑎𝑎 + 𝑑𝑑)𝐴𝐴 + 𝐼𝐼 = �𝐴𝐴 − 𝐼𝐼� = 0
4 2
Do đó
𝑎𝑎 + 𝑑𝑑 𝑘𝑘
�𝐴𝐴 − 𝐼𝐼� = 0 ∀ 𝑘𝑘 ≥ 2
2
Vậy nên
𝑎𝑎 + 𝑑𝑑 𝑎𝑎 + 𝑑𝑑 𝑛𝑛
𝐴𝐴𝑛𝑛 = ��𝐴𝐴 − 𝐼𝐼� + 𝐼𝐼�
2 2
𝑎𝑎 + 𝑑𝑑 0 𝑎𝑎 + 𝑑𝑑 𝑛𝑛 𝑎𝑎 + 𝑑𝑑 1 𝑎𝑎 + 𝑑𝑑 𝑛𝑛−1
= 𝐶𝐶𝑛𝑛0 �𝐴𝐴 − 𝐼𝐼� � 𝐼𝐼� + 𝐶𝐶𝑛𝑛1 �𝐴𝐴 − 𝐼𝐼� � 𝐼𝐼�
2 2 2 2
𝑎𝑎 + 𝑑𝑑 𝑛𝑛 𝑎𝑎 + 𝑑𝑑 𝑎𝑎 + 𝑑𝑑 𝑛𝑛−1
=� 𝐼𝐼� + 𝑛𝑛 �𝐴𝐴 − 𝐼𝐼� � 𝐼𝐼�
2 2 2
Tóm lại trong trường hợp này

Đỗ Minh Triết X-131 - MATHTASY -


Ôn thi Olympic Toán Đại số Phần X: Luỹ thừa ma trận

𝑎𝑎 + 𝑑𝑑 𝑛𝑛 𝑎𝑎 + 𝑑𝑑 𝑎𝑎 + 𝑑𝑑 𝑛𝑛−1
𝐴𝐴𝑛𝑛 = � 𝐼𝐼� + 𝑛𝑛 �𝐴𝐴 − 𝐼𝐼� � 𝐼𝐼�
2 2 2
+) Nếu ∆< 0 thì 𝑃𝑃𝐴𝐴 (𝑥𝑥) có hai nghiệm phức liên hợp
(𝑎𝑎 + 𝑑𝑑) ± 𝑖𝑖�|∆|
𝑥𝑥1,2 =
2
𝑥𝑥1 0
𝐴𝐴 chéo hoá được trên ℂ và 𝐴𝐴 = 𝑃𝑃𝑃𝑃𝑃𝑃 −1 = 𝑃𝑃 � � 𝑃𝑃 −1 .
0 𝑥𝑥2
Biểu diễn 𝑥𝑥1,2 dưới dạng lượng giác:

𝑥𝑥1,2 = 𝑟𝑟(cos 𝜑𝜑 ± 𝑖𝑖 sin 𝜑𝜑)
Khi đó
√ 𝑛𝑛
𝑛𝑛 𝑟𝑟(cos 𝜑𝜑 + 𝑖𝑖 sin 𝜑𝜑) 0 √ 𝑛𝑛 cos 𝑛𝑛𝑛𝑛 + 𝑖𝑖 sin 𝑛𝑛𝑛𝑛 0
𝐵𝐵 = � √ � = 𝑟𝑟 � �
0 𝑟𝑟(cos 𝜑𝜑 − 𝑖𝑖 sin 𝜑𝜑) 0 cos 𝑛𝑛𝑛𝑛 − 𝑖𝑖 sin 𝑛𝑛𝑛𝑛

Vậy trong trường hợp này


√ 𝑛𝑛 cos 𝑛𝑛𝑛𝑛 + 𝑖𝑖 sin 𝑛𝑛𝑛𝑛 0
𝐴𝐴𝑛𝑛 = 𝑟𝑟 𝑃𝑃 � � 𝑃𝑃 −1
0 cos 𝑛𝑛𝑛𝑛 − 𝑖𝑖 sin 𝑛𝑛𝑛𝑛

Trên đây là cách tính luỹ thừa ma trận cấp 2, mọi ma trận cấp 2 luôn có thể áp dụng cách làm
này. Tuy nhiên, có những khi quá trình tính toán khá là phức tạp (trường hợp nghiệm của phương
trình đặc trưng “xấu” hay chéo hoá ma trận trên ℂ). Sau đây là một phương pháp khác:

2. Đưa ma trận về dạng lượng giác


𝐜𝐜𝐜𝐜𝐜𝐜 𝜶𝜶 ± 𝐬𝐬𝐬𝐬𝐬𝐬 𝜶𝜶 𝒏𝒏 𝐜𝐜𝐜𝐜𝐜𝐜 𝒏𝒏𝒏𝒏 ± 𝐬𝐬𝐬𝐬𝐬𝐬 𝒏𝒏𝒏𝒏
� � =� �
∓ 𝐬𝐬𝐬𝐬𝐬𝐬 𝜶𝜶 𝐜𝐜𝐜𝐜𝐜𝐜 𝜶𝜶 ∓ 𝐬𝐬𝐬𝐬𝐬𝐬 𝒏𝒏𝒏𝒏 𝐜𝐜𝐜𝐜𝐜𝐜 𝒏𝒏𝒏𝒏
Riêng với ma trận cấp 2, một phương pháp nữa để tính luỹ thừa đó là đưa ma trận về dạng lượng
giác trên (nếu có thể). Phương pháp này cũng khá phổ biến, nó làm cho quá trình tính toán đơn
giản hơn nhiều so với dạng đại số.
Ví dụ (Câu 2 Olympic 2002) : Cho ma trận

3 5
⎛ +1 − ⎞
⎜ 2 ⎟
𝐴𝐴 = ⎜

⎜ 1 √ 2 ⎟ ⎟

3
⎝ 2 − 1 ⎠
2
Tính 𝐴𝐴2002 .

Giải.

Để ý rằng có thể đưa ma trận sau đây về dạng lượng giác:

Đỗ Minh Triết X-132 - MATHTASY -


Ôn thi Olympic Toán Đại số Phần X: Luỹ thừa ma trận


3 1 1 𝜋𝜋 𝜋𝜋 𝜋𝜋
⎛ + −2. ⎞ cos + sin −2 sin

⎜ 2 2 √ 2 ⎟ ⎟ ⎛ 6 6 6 ⎞
⎜ ⎟ =⎜ 𝜋𝜋 𝜋𝜋 𝜋𝜋⎟
⎜ 1 3 1 ⎟
− ⎠ ⎝ sin cos − sin
⎝ 2 2 2 6 6 6⎠

Bằng quy nạp, ta chứng minh được là


𝜋𝜋 𝜋𝜋 𝜋𝜋 𝑛𝑛 𝑛𝑛𝑛𝑛 𝑛𝑛𝑛𝑛 𝑛𝑛𝑛𝑛
cos + sin −2 sin cos + sin −2 sin

⎜ 6 6 6 ⎞ ⎛ 6 6 6 ⎞
𝜋𝜋 𝜋𝜋 𝜋𝜋⎟ = ⎜ 𝑛𝑛𝑛𝑛 𝑛𝑛𝑛𝑛 𝑛𝑛𝑛𝑛⎟
sin cos − sin ⎠ sin cos − sin ⎠
⎝ 6 6 6 ⎝ 6 6 6
Một mặt, ta biến đổi ma trận này như sau
√ √ √ √ √
3 1 3 3 3 3 3 5
⎛ + −1 ⎞ ⎛ +1 − ⎞ ⎛ +1 − ⎞
⎜ 2 2 √ ⎟ 𝑑𝑑 → 𝑑𝑑 ⎜ 2 ⎟ −𝑐𝑐 → 𝑐𝑐 ⎜ 2⎟

⎜ ⎟
⎟ 2 1 ⎜
⎜ √2 2⎟

1 2 ⎜ 2
⎜ √2 ⎟

⎜ 1 3 1⎟ − − − − −→ ⎜ 1 3 1 ⎟ − − −−→ ⎜ 1 3 ⎟
⎝ 2 − ⎠ ⎝ 2 − ⎠ ⎝ 2 − 1 ⎠
2 2 2 2 2
Từ cách biến đổi đó, nếu đặt

3 1
⎛ + −1 ⎞
1 1 ⎜
⎜ 2 2 √ ⎟

𝑆𝑆 = � � ; 𝐵𝐵 = ⎜ ⎟
0 1 ⎜ 1 3 1⎟
⎝ 2 − ⎠
2 2
thì ta có 𝐴𝐴 = 𝑆𝑆𝑆𝑆𝑆𝑆 −1 , do đó 𝐴𝐴𝑛𝑛 = 𝑆𝑆𝐵𝐵𝑛𝑛 𝑆𝑆 −1 , thay 𝑛𝑛 = 2002:
2002𝜋𝜋 2002𝜋𝜋 2002𝜋𝜋
cos + sin −2 sin
1 1 ⎛ 6 6 6 ⎞ 1 −1
𝐴𝐴2002 = 𝑆𝑆𝐵𝐵2002 𝑆𝑆 −1 =� �⎜⎜



⎟ � �
0 1 2002𝜋𝜋 2002𝜋𝜋 2002𝜋𝜋 0 1
sin cos − sin
⎝ 6 6 6 ⎠
√ √
1 3 √ 1 √ 5 3
− 3 ⎞ − 3
1 1 ⎛
⎜2 2 ⎟ 1 −1 ⎛
⎜ 2 √ 2 ⎟

=� �⎜
⎜ √ √ ⎟
⎟ � � = ⎜
⎜ ⎟

0 1 ⎜ 3 1 3⎟ 0 1 ⎜ 3 1 √ ⎟
⎝ − + − + 3
2 2 2 ⎠ ⎝ 2 2 ⎠

sinφ −cosφ
Ví dụ : Cho 𝐴𝐴 = � �. Tính 𝐴𝐴𝑛𝑛 .
cosφ sinφ

Giải.

Đặt 𝜑𝜑 = 𝜋𝜋2 − 𝛼𝛼 thì


𝜋𝜋 𝜋𝜋 𝑛𝑛
sin𝜑𝜑 −cos𝜑𝜑 𝑛𝑛 sin � − 𝛼𝛼� −cos � − 𝛼𝛼�
⎡ 2 2 ⎤ cos𝛼𝛼 −sin𝛼𝛼 𝑛𝑛
𝐴𝐴𝑛𝑛 = � � =⎢ 𝜋𝜋 𝜋𝜋 ⎥ =� �
cos𝜑𝜑 sin𝜑𝜑 cos � − 𝛼𝛼� sin � − 𝛼𝛼� sin𝛼𝛼 cos𝛼𝛼
⎣ 2 2 ⎦
=�
cos 𝑛𝑛𝑛𝑛 − sin 𝑛𝑛𝑛𝑛 �
sin 𝑛𝑛𝑛𝑛 cos 𝑛𝑛𝑛𝑛

Đỗ Minh Triết X-133 - MATHTASY -


Ôn thi Olympic Toán Đại số Phần X: Luỹ thừa ma trận

3. Ngoài ra, các phương pháp tính luỹ thừa ma trận cấp 3 nêu trên vẫn có
thể áp dụng được cho ma trận cấp 2.
3 −5
Ví dụ: Tính 𝐴𝐴2010 với 𝐴𝐴 = � �.
1 −1
Cách 1: (Quy nạp)

Ta có
4 −10 2 −10
𝐴𝐴2 = � � , 𝐴𝐴3 = � � , 𝐴𝐴4 = −4𝐼𝐼
2 −4 2 −6
Bằng quy nạp, dễ dàng chứng minh rằng
⎧ (−4)𝑘𝑘 𝐼𝐼nếu 𝑛𝑛 = 4𝑘𝑘

� (−4)𝑘𝑘 𝐴𝐴
𝑛𝑛 nếu 𝑛𝑛 = 4𝑘𝑘 + 1
𝐴𝐴 = ⎨
� nếu 𝑛𝑛 = 4𝑘𝑘 + 2
(−4)𝑘𝑘 𝐴𝐴2

⎩(−4)𝑘𝑘 𝐴𝐴3nếu 𝑛𝑛 = 4𝑘𝑘 + 3
2 −5
Suy ra 𝐴𝐴2010 = 𝐴𝐴4.502+2 = (−4)502 𝐴𝐴2 = 21005 � �
1 −2
Cách 2: (Chéo hoá)

Đa thức đặc trưng của 𝐴𝐴 là


𝑃𝑃𝐴𝐴 (𝜆𝜆) = 𝜆𝜆2 − 2𝜆𝜆 + 2 = (𝜆𝜆 − 1)2 + 1
𝐴𝐴 chéo hoá được trên ℂ, 𝑃𝑃𝐴𝐴 (𝜆𝜆) có hai nghiệm phức liên hợp 𝜆𝜆1,2 = 1 ± 𝑖𝑖 và hai vector riêng liên
hợp.
Với 𝑥𝑥 = 𝜆𝜆1 , ta có hệ
(2 − 𝑖𝑖)𝑥𝑥1 − 5𝑥𝑥2 = 0
� ⇔ 𝑥𝑥1 = (2 + 𝑖𝑖)𝑥𝑥2
𝑥𝑥1 − (2 + 𝑖𝑖)𝑥𝑥2 = 0
Chọn 𝑥𝑥2 = 1, ta được 1 vector riêng tương ứng (2 + 𝑖𝑖, 1)
Suy ra với 𝑥𝑥 = 𝜆𝜆2 , ta được 1 vector riêng tương ứng (2 − 𝑖𝑖, 1)
1 + 𝑖𝑖 0 2 + 𝑖𝑖 2 − 𝑖𝑖 1 −2 + 𝑖𝑖
Đặt 𝐵𝐵 = � � , 𝑃𝑃 = � � thì 𝑃𝑃 −1 = 2𝑖𝑖
1
� � và
0 1 − 𝑖𝑖 1 1 −1 2 + 𝑖𝑖
1 2 + 𝑖𝑖 2 − 𝑖𝑖 1 + 𝑖𝑖 0 1 −2 + 𝑖𝑖
𝐴𝐴 = 𝑃𝑃𝑃𝑃𝑃𝑃 −1 = � �� �� �
2𝑖𝑖 1 1 0 1 − 𝑖𝑖 −1 2 + 𝑖𝑖
1 2 + 𝑖𝑖 2 − 𝑖𝑖 (1 + 𝑖𝑖)2010 0 1 −2 + 𝑖𝑖
𝐴𝐴2010 = 𝑃𝑃 𝐵𝐵2010 𝑃𝑃 −1 = � �� �� �
2𝑖𝑖 1 1 0 (1 − 𝑖𝑖) 2010 −1 2 + 𝑖𝑖
2010𝜋𝜋 2010𝜋𝜋
√ 1 2 + 𝑖𝑖 ⎛ cos + 𝑖𝑖 sin 0 ⎞
2010 2 − 𝑖𝑖 ⎜ 4 4 ⎟ 1 −2 + 𝑖𝑖
= 2 � �⎜
⎜ ⎟
⎟ � �
2𝑖𝑖 1 1 7.2010𝜋𝜋 7.2010𝜋𝜋 −1 2 + 𝑖𝑖
⎝ 0 cos + 𝑖𝑖 sin
4 4 ⎠
1 2 + 𝑖𝑖 2 − 𝑖𝑖 𝑖𝑖 0 1 −2 + 𝑖𝑖
= 21005 � �� �� �
2𝑖𝑖 1 1 0 −𝑖𝑖 −1 2 + 𝑖𝑖

Đỗ Minh Triết X-134 - MATHTASY -


Ôn thi Olympic Toán Đại số Phần X: Luỹ thừa ma trận

−𝑖𝑖 −1 + 2𝑖𝑖 −1 − 2𝑖𝑖 1 −2 + 𝑖𝑖


= 21005 � �� �
2 𝑖𝑖 −𝑖𝑖 −1 2 + 𝑖𝑖
1 4𝑖𝑖 −10𝑖𝑖 2 −5
= 21005 � � = 21005 � �
2𝑖𝑖 2𝑖𝑖 −4𝑖𝑖 1 −2

2 1 𝑛𝑛
Ví dụ : Tính 𝐴𝐴𝑛𝑛 = � �
1 2
Giải.
0 1 1 0
Tách 𝐴𝐴 = � � + 2� � = 𝐵𝐵 + 2𝐼𝐼 thì 𝐵𝐵𝑘𝑘 = 𝐼𝐼 ∀𝑘𝑘 ≥ 2. Do đó
1 0 0 1
𝐴𝐴𝑛𝑛 = (𝐵𝐵 + 2𝐼𝐼)𝑛𝑛 = 2𝑛𝑛 𝐼𝐼 + 𝑛𝑛2𝑛𝑛−1 𝐴𝐴 + (3𝑛𝑛 − 2𝑛𝑛 − 𝑛𝑛2𝑛𝑛−1 )𝐼𝐼 = ⋯

1 1 2010
Ví dụ (Câu 2 đề dự tuyển CĐSP BR-VT 2010) : Tính 𝐴𝐴2010 = � � .
−1 3
Giải.

Thực hiện phép chia 𝑥𝑥2010 cho đa thức đặc trưng của 𝐴𝐴 là (𝑥𝑥 − 2)2 , ta có:
𝑥𝑥2010 = (𝑥𝑥 − 2)2 𝑄𝑄(𝑥𝑥) + (𝑎𝑎𝑎𝑎 + 𝑏𝑏)
Đạo hàm, ta được: 2010𝑥𝑥2009 = 2(𝑥𝑥 − 2)𝑄𝑄(𝑥𝑥) + (𝑥𝑥 − 2)2 𝑄𝑄′ (𝑥𝑥) + 𝑎𝑎
2010
Thay 𝑥𝑥 = 2 vào hai đẳng thức trên, ta có: � 2 = 2𝑎𝑎 + 𝑏𝑏 ⇔ �𝑎𝑎 = 2010.22009
2010.22009 = 𝑎𝑎 𝑏𝑏 = −2009.22010
−1004 1005
Vậy 𝐴𝐴2010 = 2010.22009 𝐴𝐴 − 2009.22010 𝐼𝐼 = 22010 (1005𝐴𝐴 − 2009𝐼𝐼) = 22010 � �
−1005 1006

10.3. Luỹ thừa ma trận cấp cao (𝒏𝒏 ≥ 𝟒𝟒)


1 1 1 1

⎜ 1 1 −1 −1 ⎞
⎟. Tính 𝐴𝐴2012 .
Câu : Cho 𝐴𝐴 = ⎜
1 −1 1 −1 ⎟
⎝ 1 −1 −1 1⎠
Giải.

Tính toán cho ta 𝐴𝐴2 = 4𝐼𝐼 ⇒ 𝐴𝐴3 = 4𝐴𝐴 ⇒ 𝐴𝐴4 = 4𝐴𝐴2 = 16𝐼𝐼 ⇒ 𝐴𝐴5 = 16𝐴𝐴 ⇒ 𝐴𝐴6 = 16𝐴𝐴2 = 64𝐼𝐼.
Dễ dàng chứng minh được bằng quy nạp rằng
𝑘𝑘
𝐴𝐴𝑘𝑘 = � 2𝑘𝑘−1𝐼𝐼 nếu 𝑘𝑘 = 2𝑚𝑚
2 𝐼𝐼 nếu 𝑘𝑘 = 2𝑚𝑚 + 1
Do đó 𝐴𝐴2012 = 22012 𝐼𝐼.

Câu : Cho ma trận:

Đỗ Minh Triết X-135 - MATHTASY -


Ôn thi Olympic Toán Đại số Phần X: Luỹ thừa ma trận

−1 0 1 1 0 0

⎜ 1 −1 −3 0 1 0⎞⎟

⎜ 0 1 2 0 0 1⎟⎟
𝐵𝐵 = ⎜
⎜ ⎟ . Tı́nh 𝐵𝐵2012 .
⎜ 0 0 0 −1 0 1⎟⎟
⎜ 0 0 0 1 −1 −3 ⎟
⎝ 0 0 0 0 1 2⎠
Giải.

𝐵𝐵2 = �𝐴𝐴 𝐼𝐼 � �𝐴𝐴 𝐼𝐼 � = �𝐴𝐴2 2𝐴𝐴� ⇒ 𝐵𝐵3 = 𝐵𝐵2 𝐵𝐵 = �𝐴𝐴2 2𝐴𝐴� �𝐴𝐴 𝐼𝐼 � = �𝐴𝐴3 𝐴𝐴2 �
0 𝐴𝐴 0 𝐴𝐴 0 𝐴𝐴2 0 𝐴𝐴2 0 𝐴𝐴 0 𝐴𝐴3
2
=� 0 𝐴𝐴 �
0 0
2 3
⇒ 𝐵𝐵4 = 𝐵𝐵3 𝐵𝐵 = �0 𝐴𝐴 � �𝐴𝐴 𝐼𝐼 � = �0 𝐴𝐴 � = 0 ⇒ 𝐵𝐵2012 = 0
0 0 0 𝐴𝐴 0 0

Câu (Câu 1b Olympic 1999) : Cho 𝑓𝑓(𝑥𝑥) = 𝑥𝑥1999 + 𝑥𝑥2 − 1, và ma trận


4 3 0 0
⎛2 3 0 0⎞
𝐶𝐶 = ⎜
⎜4 ⎟
9 −1 0⎟
⎝1 2 5 2⎠
Tính det[𝑓𝑓(𝐶𝐶)].

Cách 1:

Đặt 𝐶𝐶 = �𝐴𝐴 0 �, ta có
𝐵𝐵 𝐷𝐷
2
𝐶𝐶 2 = � 𝐴𝐴 0 � ⇒ 𝐶𝐶 3 = � 𝐴𝐴3 0�
2
𝐵𝐵𝐵𝐵 + 𝐷𝐷𝐷𝐷 𝐷𝐷 𝐵𝐵𝐴𝐴 + 𝐷𝐷𝐷𝐷𝐷𝐷 + 𝐷𝐷2 𝐵𝐵
2
𝐷𝐷3
𝐴𝐴4 0
⇒ 𝐶𝐶4 = � 3 2 �
𝐵𝐵𝐴𝐴 + 𝐷𝐷𝐷𝐷𝐴𝐴 + 𝐷𝐷 𝐵𝐵𝐵𝐵 + 𝐷𝐷 𝐵𝐵 𝐷𝐷4
2 3

𝐴𝐴𝑛𝑛 0
𝑛𝑛
… ⇒ 𝐶𝐶 = ⎜ ⎛ 𝑛𝑛−1 ⎞

� 𝐷𝐷𝑖𝑖 𝐵𝐵𝐴𝐴𝑛𝑛−1−𝑖𝑖 𝐷𝐷𝑛𝑛
⎝ 𝑖𝑖=0 ⎠
Suy ra
𝐴𝐴1999 + 𝐴𝐴2 − 𝐼𝐼2 0
⎛1998 ⎞
𝑓𝑓(𝐶𝐶) = 𝐶𝐶 1999
+ 𝐶𝐶 − 𝐼𝐼4 = ⎜
2
⎜ ⎟

� 𝐷𝐷𝑖𝑖 𝐵𝐵𝐴𝐴1998−𝑖𝑖 + 𝐵𝐵𝐵𝐵 + 𝐷𝐷𝐷𝐷 𝐷𝐷1999 + 𝐷𝐷2 − 𝐼𝐼2
⎝ 𝑖𝑖=0 ⎠
Từ đó
det[𝑓𝑓(𝐶𝐶)] = det(𝐴𝐴1999 + 𝐴𝐴2 − 𝐼𝐼2 ) det(𝐷𝐷1999 + 𝐷𝐷2 − 𝐼𝐼2 )
Mặt khác
|𝐴𝐴 − 𝜆𝜆𝜆𝜆| = (𝜆𝜆 − 1)(𝜆𝜆 − 6) ; |𝐷𝐷 − 𝜆𝜆𝜆𝜆| = (𝜆𝜆 + 1)(𝜆𝜆 − 2)
tức là 𝐴𝐴, 𝐷𝐷 đều chéo hoá được:

Đỗ Minh Triết X-136 - MATHTASY -


Ôn thi Olympic Toán Đại số Phần X: Luỹ thừa ma trận

1 0 −1 0 −1
𝐴𝐴 = 𝑃𝑃 𝐴𝐴′ 𝑃𝑃 −1 = 𝑃𝑃 � � 𝑃𝑃 −1 ; 𝐷𝐷 = 𝑇𝑇 𝐷𝐷′ 𝑇𝑇 −1 = 𝑇𝑇 � � 𝑇𝑇
0 6 0 2
Vậy
det[𝑓𝑓(𝐶𝐶)] = det(𝐴𝐴′1999 + 𝐴𝐴′2 − 𝐼𝐼2 ) det�𝐷𝐷′ 1999 + 𝐷𝐷′ 2 − 𝐼𝐼2 �
1+1−1 0 −1 + 1 − 1 0
= det � 1999 2 � det � 1999 �
0 6 +6 −1 0 2 + 22 − 1
1 0 −1 0
= det � 1999 2 � det � 1999 � = −(61999 + 35)(21999 + 3)
0 6 +6 −1 0 2 + 22 − 1
Cách 2:

Như đã chứng minh nếu 𝜆𝜆 là một giá trị riêng của 𝐶𝐶 thì 𝑓𝑓(𝜆𝜆) sẽ là giá trị riêng của 𝑓𝑓(𝐶𝐶) và định
thức của một ma trận bằng tích các giá trị riêng của nó. Từ đó
4 − 𝜆𝜆 3
det(𝐶𝐶 − 𝜆𝜆𝜆𝜆) = (2 − 𝜆𝜆)(−1 − 𝜆𝜆) � � = (2 − 𝜆𝜆)(−1 − 𝜆𝜆)(1 − 𝜆𝜆)(6 − 𝜆𝜆)
2 3 − 𝜆𝜆
𝐶𝐶 có 4 giá trị riêng 2, -1, 1, 6 suy ra các giá trị riêng của 𝑓𝑓(𝐶𝐶) là
𝑓𝑓(2) = 21999 + 3 , 𝑓𝑓(−1) = −1 , 𝑓𝑓(1) = 1 , 𝑓𝑓(6) = 61999 + 35
Vậy det[𝑓𝑓(𝐶𝐶)] = −(61999 + 35)(21999 + 3).

Câu (Câu 4 Olympic 2009) : Tính 𝐴𝐴2009 , trong đó


0 0 0 0 −1

⎜0 −7 5 3 0⎞⎟
𝐴𝐴 = ⎜

⎜0 −5 4 2 0⎟


⎜0 −9 6 4 0⎟
⎝1 0 0 0 0⎠
Giải.

Ta biến đổi ma trận 𝐴𝐴 về dạng tam giác khối như sau


0 0 0 0 −1 0 −9 6 4 0 4 −9 6 0 0
⎛0 −7 5 3 0⎞ 𝑑𝑑 ⟷ 𝑑𝑑 ⎛ 0 −7 5 3 0⎞ 𝑐𝑐1 ⟷ 𝑐𝑐4 ⎛ 3 −7 5 0 0⎞

⎜ ⎟
⎟ − − − − −→ ⎜
1 4
⎜ ⎟
⎟ − − − − −→ ⎜ ⎜ ⎟


⎜0 −5 4 2 0⎟⎟ ⎜
⎜ 0 −5 4 2 0⎟
⎟ ⎜
⎜ 2 −5 4 0 0⎟⎟
⎜0 −9 6 4 0 ⎟ 𝑉𝑉1 ⎜ 0 0 0 0 −1 ⎟ 𝑉𝑉2 ⎜0 0 0 0 −1 ⎟
⎝1 0 0 0 0⎠ ⎝1 0 0 0 0⎠ ⎝0 0 0 1 0⎠
2 −5 4 0 0 4 −5 2 0 0
𝑑𝑑1 ⟷ 𝑑𝑑3 ⎛ 3 −7 5 0 0⎞ 𝑐𝑐1 ⟷ 𝑐𝑐3 ⎛ 5 −7 3 0 0⎞
⎜ ⎟ ⎜ ⎟
− − − − −→ ⎜ ⎜
⎜ 4 −9 6 0


0⎟ − − − − −→ 𝐵𝐵 = ⎜

⎜ 6 −9 4 0 0⎟


𝑉𝑉3 ⎜0 0 0 0 −1 ⎟ 𝑉𝑉4 ⎜0 0 0 0 −1 ⎟
⎝0 0 0 1 0 ⎠ ⎝ 0 0 0 1 0⎠
Từ phép biến đổi đó, ta có
𝐴𝐴 → 𝑉𝑉1 𝐴𝐴 → 𝑉𝑉1 𝐴𝐴𝑉𝑉2 → 𝑉𝑉3 𝑉𝑉1 𝐴𝐴𝑉𝑉2 → 𝑉𝑉3 𝑉𝑉1 𝐴𝐴𝑉𝑉2 𝑉𝑉4 = 𝐵𝐵
trong đó

Đỗ Minh Triết X-137 - MATHTASY -


Ôn thi Olympic Toán Đại số Phần X: Luỹ thừa ma trận

0 0 0 1 0 0 0 0 1 0

⎜0 1 0 0 0⎞⎟ ⎛
⎜0 1 0 0 0⎞⎟
𝑉𝑉1 = ⎜

⎜ 0 0 1 0 0⎟⎟
⎟ ; 𝑉𝑉 2 = ⎜

⎜0 0 1 0 0⎟⎟

⎜1 0 0 0 0 ⎟ ⎜1 0 0 0 0⎟
⎝0 0 0 0 1⎠ ⎝0 0 0 0 1⎠
0 0 1 0 0 0 0 1 0 0

⎜0 1 0 0 0⎞⎟ ⎛
⎜0 1 0 0 0⎞⎟
𝑉𝑉3 = ⎜

⎜ 1 0 0 0 0⎟⎟
⎟ ; 𝑉𝑉 4 = ⎜

⎜1 0 0 0 0⎟⎟

⎜0 0 0 1 0⎟ ⎜0 0 0 1 0⎟
⎝0 0 0 0 1⎠ ⎝0 0 0 0 1⎠
Ký hiệu
0 0 0 1 0 0 0 1 0 0
⎛ 0⎞ ⎛ 0 1 0 0 0⎞
⎜0 1 0 0
⎟ → 𝑃𝑃 −1 = 𝑉𝑉 𝑉𝑉 = ⎜
⎟ ⎜ ⎟
𝑃𝑃 = 𝑉𝑉2 𝑉𝑉4 = ⎜

⎜1 0 0 0 0⎟⎟ 3 1 ⎜
⎜ 1 0 0 0 0⎟⎟

⎜0 0 1 0⎟ ⎜0 0 0
0 1 0⎟
⎝0 0 0 0 1⎠ ⎝0 0 0 0 1⎠
Suy ra
𝐴𝐴 = 𝑃𝑃𝑃𝑃𝑃𝑃 −1 = 𝑃𝑃 diag(𝐶𝐶, 𝐷𝐷)𝑃𝑃 −1
trong đó
4 −5 2
0 1
𝐶𝐶 = �5 −7 3� và 𝐷𝐷 = � �
−1 0
6 −9 4
Ta có
3 −3 1
𝐶𝐶 3 = 𝐶𝐶 2 = �3 −3 1� ⇒ 𝐶𝐶 2009 = 𝐶𝐶 2
3 −3 1
⎧ 𝐼𝐼 nếu 𝑛𝑛 = 4𝑘𝑘
� nếu 𝑛𝑛 = 4𝑘𝑘 + 1
𝐷𝐷2 = −𝐼𝐼 ⇒ 𝐷𝐷3 = −𝐷𝐷 ⇒ 𝐷𝐷4 = 𝐼𝐼 ⇒ 𝐷𝐷𝑛𝑛 = ⎨ 𝐷𝐷 ⇒ 𝐷𝐷2009 = 𝐷𝐷4.502+1 = 𝐷𝐷
� −𝐼𝐼 nếu 𝑛𝑛 = 4𝑘𝑘 + 2
⎩ −𝐷𝐷 nếu 𝑛𝑛 = 4𝑘𝑘 + 3

Từ đó
𝐴𝐴2009 = 𝑃𝑃 diag(𝐶𝐶 2009 , 𝐷𝐷2009 )𝑃𝑃 −1
0 0 0 1 0 3 −3 1 0 0 0 0 1 0 0 0 0 0 0 1

⎜0 1 0 0 0⎟⎞ ⎛
⎜ 3 −3 1 0 ⎞ ⎛
0⎟ ⎜0 1 0 0 ⎞ ⎛
0⎟ ⎜ 0 −3 3 1 0⎞

=⎜

⎜1 0 0 0 0

⎟ ⎜
⎜ 3 −3 1 0
⎟⎜
0⎟
⎟⎜⎜0 0 0 1 0⎟
⎟=⎜
⎟ ⎜
⎜ 0 −3 3

1 0⎟
⎜0 0 1 0 0⎟⎟⎜⎜0 0 0 0 1⎟ ⎜1 0 0 0 0⎟ ⎜ 0 −3 3

1 0⎟
⎝0 0 0 0 1⎠ ⎝0 0 0 −1 0⎠ ⎝0 0 0 0 1⎠ ⎝ −1 0 0 0 0⎠

10.4. Luỹ thừa ma trận qua các bài toán về dãy truy hồi tuyến tính

1. Các dãy truy hồi tuyến tính đồng thời cấp 1 với hệ số không đổi

Ví dụ 1 : Giả sử (𝑢𝑢𝑛𝑛 )𝑛𝑛∈ℕ , (𝑣𝑣𝑛𝑛 )𝑛𝑛∈ℕ , (𝑤𝑤𝑛𝑛 )𝑛𝑛∈ℕ là các dãy số thực được xác định bởi

Đỗ Minh Triết X-138 - MATHTASY -


Ôn thi Olympic Toán Đại số Phần X: Luỹ thừa ma trận

⎧𝑢𝑢0 = 0, 𝑣𝑣0 = 22, 𝑤𝑤0 = 22


� 1
� ⎧ 𝑢𝑢
� � 𝑛𝑛+1 = (2𝑢𝑢𝑛𝑛 + 𝑣𝑣𝑛𝑛 + 𝑤𝑤𝑛𝑛 )
� � 4
� 1
⎨ ∀𝑛𝑛 ∈ ℕ
� ⎨ 𝑣𝑣𝑛𝑛+1 = 3 (𝑢𝑢𝑛𝑛 + 𝑣𝑣𝑛𝑛 + 𝑤𝑤𝑛𝑛 )
� �
� �
� �𝑤𝑤𝑛𝑛+1 = 1 (𝑢𝑢𝑛𝑛 + 𝑣𝑣𝑛𝑛 + 2𝑤𝑤𝑛𝑛 )
⎩ ⎩ 4
Tính 𝑢𝑢𝑛𝑛 , 𝑣𝑣𝑛𝑛 , 𝑤𝑤𝑛𝑛 và nghiên cứu sự hội tụ của ba dãy này.

Giải.
1 1 1

⎜2 4 4⎞⎟
𝑢𝑢𝑛𝑛 ⎜
⎜1 1 1⎟⎟
Kí hiệu 𝑋𝑋𝑛𝑛 = � 𝑣𝑣𝑛𝑛 � ∀𝑛𝑛 ∈ ℕ và 𝐴𝐴 = ⎜
⎜ ⎟
⎟ thì ta có 𝑋𝑋𝑛𝑛+1 = 𝐴𝐴𝑋𝑋𝑛𝑛 suy ra 𝑋𝑋𝑛𝑛
𝑤𝑤𝑛𝑛 ⎜
⎜3 3 3⎟⎟
⎜1 1 1⎟
⎝4 4 2⎠
𝑛𝑛
= 𝐴𝐴 𝑋𝑋0 .
Đa thức đặc trưng của 𝐴𝐴 là 𝑃𝑃𝐴𝐴 (𝑥𝑥) = (1 − 𝑥𝑥)(12
1
− 𝑥𝑥)(14 − 𝑥𝑥)
𝐴𝐴 có 3 giá trị riêng phân biệt nên chéo hoá được, ta có
1 0 0
1 1 3 ⎛⎜0 1 ⎞ 8 6 8
0⎟ 1
𝐴𝐴 = 𝑃𝑃𝑃𝑃𝑃𝑃 −1 = �1 0 −8 � ⎜

⎜ 4 ⎟

⎟ �11 0 −11�
1 −1 3 ⎜ 1 ⎟ 22 1 −2 1
⎝0 0
12⎠
Từ đó, ta có

1 1 1 3 1 0 0 8 6 8 0
𝑋𝑋𝑛𝑛 = 𝐴𝐴𝑛𝑛 𝑋𝑋0 = �1 0 −8 � �0 4−𝑛𝑛 0 � �11 0 −11� �22�
22
1 −1 3 0 0 12−𝑛𝑛 1 −2 1 22
Vậy
⎧ 𝑢𝑢 = 14 − 11.4−𝑛𝑛 − 3.12−𝑛𝑛
� 𝑛𝑛
∀𝑛𝑛 ∈ ℕ: 𝑣𝑣𝑛𝑛 = 14 + 8.12−𝑛𝑛


⎩ 𝑤𝑤𝑛𝑛 = 14 + 11.4−𝑛𝑛 − 3.12−𝑛𝑛
Rõ ràng (𝑢𝑢𝑛𝑛 )𝑛𝑛∈ℕ , (𝑣𝑣𝑛𝑛 )𝑛𝑛∈ℕ , (𝑤𝑤𝑛𝑛 )𝑛𝑛∈ℕ hội tụ về 14.

Ví dụ 2 (Câu 1 Olympic 1997) : Giả sử 𝑥𝑥0 , 𝑦𝑦0 , 𝑧𝑧0 là các số thực cho trước. Hãy xác
định tất cả các số thực 𝑥𝑥𝑛𝑛 , 𝑦𝑦𝑛𝑛 , 𝑧𝑧𝑛𝑛 (𝑛𝑛 = 0,1, … ) thoả mãn hệ phương trình
𝑥𝑥𝑛𝑛+1 = −𝑥𝑥𝑛𝑛 + 𝑦𝑦𝑛𝑛 + 𝑧𝑧𝑛𝑛
� 𝑦𝑦𝑛𝑛+1 = 𝑥𝑥𝑛𝑛 − 𝑦𝑦𝑛𝑛 + 𝑧𝑧𝑛𝑛 (𝑛𝑛 = 0,1, … )
𝑧𝑧𝑛𝑛+1 = 𝑥𝑥𝑛𝑛 + 𝑦𝑦𝑛𝑛 − 𝑧𝑧𝑛𝑛

Giải.

Đỗ Minh Triết X-139 - MATHTASY -


Ôn thi Olympic Toán Đại số Phần X: Luỹ thừa ma trận

𝑥𝑥𝑛𝑛 −1 1 1
Ký hiệu 𝑋𝑋𝑛𝑛 = � 𝑦𝑦𝑛𝑛 � ∀𝑛𝑛 ∈ ℕ và 𝐴𝐴 = � 1 −1 1� thì ta có 𝑋𝑋𝑛𝑛+1 = 𝐴𝐴𝑋𝑋𝑛𝑛 suy ra 𝑋𝑋𝑛𝑛 =
𝑧𝑧𝑛𝑛 1 1 −1
𝐴𝐴𝑛𝑛 𝑋𝑋0 .
*Ta tính 𝐴𝐴𝑛𝑛 như sau.

Cách 1:
1 1 1
Tách 𝐴𝐴 = 𝐵𝐵 − 2𝐼𝐼, trong đó 𝐵𝐵 = �1 1 1�, khi đó 𝐵𝐵𝑛𝑛 = 3𝑛𝑛−1 𝐵𝐵 suy ra
1 1 1
𝐴𝐴 = (𝐵𝐵 − 2𝐼𝐼) = 𝐶𝐶𝑛𝑛 𝐵𝐵 (−2𝐼𝐼)0 + 𝐶𝐶𝑛𝑛1 𝐵𝐵𝑛𝑛−1 (−2𝐼𝐼)1 +. . . +𝐶𝐶𝑛𝑛𝑛𝑛−1 𝐵𝐵(−2𝐼𝐼)𝑛𝑛−1 + 𝐶𝐶𝑛𝑛𝑛𝑛 𝐵𝐵0 (−2𝐼𝐼)𝑛𝑛
𝑛𝑛 𝑛𝑛 0 𝑛𝑛

= 𝐶𝐶𝑛𝑛0 3𝑛𝑛−1 (−2)0 𝐵𝐵 + 𝐶𝐶𝑛𝑛1 3𝑛𝑛−2 . (−2)1 𝐵𝐵+. . . +𝐶𝐶𝑛𝑛𝑛𝑛−1 30 (−2)𝑛𝑛−1 𝐵𝐵 + 𝐶𝐶𝑛𝑛𝑛𝑛 (−2)𝑛𝑛 𝐼𝐼
1
= [𝐶𝐶𝑛𝑛0 3𝑛𝑛 (−2)0 𝐵𝐵 + 𝐶𝐶𝑛𝑛1 3𝑛𝑛−1 . (−2)1 𝐵𝐵+. . . +𝐶𝐶𝑛𝑛𝑛𝑛−1 31 (−2)𝑛𝑛−1 𝐵𝐵 + 𝐶𝐶𝑛𝑛𝑛𝑛 3(−2)𝑛𝑛 𝐼𝐼]
3
1 1
= [((3 − 2)𝑛𝑛 − 𝐶𝐶𝑛𝑛𝑛𝑛 30 (−2)𝑛𝑛 )𝐵𝐵 + 𝐶𝐶𝑛𝑛𝑛𝑛 3(−2)𝑛𝑛 𝐼𝐼] = [(1 − (−2)𝑛𝑛 )𝐵𝐵 + 3(−2)𝑛𝑛 𝐼𝐼]
3 3
𝑛𝑛 𝑛𝑛
1 + 2(−2) 1 − (−2) 1 − (−2)𝑛𝑛
1⎛
= ⎜ 1 − (−2)𝑛𝑛 1 + 2(−2)𝑛𝑛 1 − (−2)𝑛𝑛 ⎞ ⎟
3 𝑛𝑛 𝑛𝑛 𝑛𝑛
⎝ 1 − (−2) 1 − (−2) 1 + 2(−2) ⎠

Cách 2:

Do 𝐴𝐴 là ma trận đối xứng nên 𝐴𝐴𝑛𝑛 cũng là ma trận đối xứng, nó có dạng như sau
−1 1 1 𝑛𝑛 𝑎𝑎𝑛𝑛 𝑏𝑏𝑛𝑛 𝑏𝑏𝑛𝑛
� 1 −1 1 � = ⎛
⎜ 𝑏𝑏𝑛𝑛 𝑎𝑎𝑛𝑛 𝑏𝑏𝑛𝑛 ⎞

1 1 −1 ⎝ 𝑏𝑏𝑛𝑛 𝑏𝑏𝑛𝑛 𝑎𝑎𝑛𝑛 ⎠
Xét đẳng thức 𝐴𝐴𝑛𝑛+1 = 𝐴𝐴𝑛𝑛 𝐴𝐴 ta được 𝑎𝑎𝑛𝑛+1 = −𝑎𝑎𝑛𝑛 + 2𝑏𝑏𝑛𝑛 và 𝑏𝑏𝑛𝑛+1 = 𝑎𝑎𝑛𝑛 . Do đó ta có
𝑎𝑎𝑛𝑛+1 − 𝑏𝑏𝑛𝑛+1 = (−2)(𝑎𝑎𝑛𝑛 − 𝑏𝑏𝑛𝑛 ) =. . . = (−2)𝑛𝑛 (𝑎𝑎1 − 𝑏𝑏1 ) = (−2)𝑛𝑛+1

𝑎𝑎𝑛𝑛+1 + 2𝑏𝑏𝑛𝑛+1 = 𝑎𝑎𝑛𝑛 + 2𝑏𝑏𝑛𝑛 = ⋯ = 𝑎𝑎1 + 2𝑏𝑏1 = 1
𝑛𝑛+1 𝑛𝑛+1
Giải hệ trên, tìm được 𝑎𝑎𝑛𝑛+1 = 1+2(−2)
3
, 𝑏𝑏𝑛𝑛+1 = 1−(−2)
3
, từ đó
1 + 2(−2)𝑛𝑛 1 − (−2)𝑛𝑛 1 − (−2)𝑛𝑛
1⎛
𝐴𝐴 = ⎜ 1 − (−2)𝑛𝑛
𝑛𝑛
1 + 2(−2)𝑛𝑛 1 − (−2)𝑛𝑛 ⎞⎟
3
⎝ 1 − (−2)𝑛𝑛 1 − (−2)𝑛𝑛 𝑛𝑛
1 + 2(−2) ⎠
⎧𝑥𝑥 = 𝑥𝑥0 + 𝑦𝑦0 + 𝑧𝑧0 + (−2)𝑛𝑛 �2𝑥𝑥0 − 𝑦𝑦0 − 𝑧𝑧0 �


𝑛𝑛
3 3
� 𝑥𝑥0 + 𝑦𝑦0 + 𝑧𝑧0 2𝑦𝑦 − 𝑧𝑧0 − 𝑧𝑧0
Vậy ∀𝑛𝑛 ∈ ℕ: ⎨ 𝑦𝑦𝑛𝑛 = + (−2)𝑛𝑛 � 0 �
� 3 3

� 𝑧𝑧𝑛𝑛 = 𝑥𝑥0 + 𝑦𝑦0 + 𝑧𝑧0 + (−2)𝑛𝑛 �2𝑧𝑧0 − 𝑥𝑥0 − 𝑦𝑦0 �
⎩ 3 3

Ví dụ 3 (Câu 2 Olympic 2011) : Cho 3 dãy số {𝑥𝑥𝑛𝑛 }, {𝑦𝑦𝑛𝑛 }, {𝑧𝑧𝑛𝑛 } xác định như sau:

Đỗ Minh Triết X-140 - MATHTASY -


Ôn thi Olympic Toán Đại số Phần X: Luỹ thừa ma trận

⎧ 𝑥𝑥 = 4𝑥𝑥𝑛𝑛 − 𝑦𝑦𝑛𝑛 − 5𝑧𝑧𝑛𝑛


� 𝑛𝑛+1
𝑥𝑥0 = 𝑦𝑦0 = 𝑧𝑧0 và 𝑦𝑦𝑛𝑛+1 = 2𝑥𝑥𝑛𝑛 − 2𝑧𝑧𝑛𝑛


⎩𝑧𝑧𝑛𝑛+1 = 𝑥𝑥𝑛𝑛 − 2𝑧𝑧𝑛𝑛
Tính 𝑥𝑥2011 .

Giải.
𝑥𝑥𝑛𝑛 4 −1 −5
Ký hiệu 𝑋𝑋𝑛𝑛 = � 𝑦𝑦𝑛𝑛 � ∀𝑛𝑛 ∈ ℕ và 𝐴𝐴 = �2 0 −2� thì ta có 𝑋𝑋𝑛𝑛+1 = 𝐴𝐴𝑋𝑋𝑛𝑛 suy ra 𝑋𝑋𝑛𝑛 =
𝑧𝑧𝑛𝑛 1 0 −2
𝐴𝐴𝑛𝑛 𝑋𝑋0 .
Đa thức đặc trưng của 𝐴𝐴 là 𝑝𝑝(𝑥𝑥) = −(𝑥𝑥 − 2)(𝑥𝑥 − 1)(𝑥𝑥 + 1)
Ta tính 𝐴𝐴2011 như sau
Thực hiện phép chia 𝑥𝑥2011 cho 𝑝𝑝(𝑥𝑥), ta được
𝑥𝑥2011 = 𝑝𝑝(𝑥𝑥)𝑞𝑞(𝑥𝑥) + (𝑎𝑎𝑥𝑥2 + 𝑏𝑏𝑏𝑏 + 𝑐𝑐)
Từ đó, lần lược chọn 𝑥𝑥 = 2, 1, −1 ta được hệ phương trình
⎧ 22011 − 2
� 𝑎𝑎 =
4𝑎𝑎 + 2𝑏𝑏 + 𝑐𝑐 = 22011 � 3
� 𝑎𝑎 + 𝑏𝑏 + 𝑐𝑐 = 1 ⇔ ⎨𝑏𝑏 = 1
� 2011
𝑎𝑎 − 𝑏𝑏 + 𝑐𝑐 = −1 � 𝑐𝑐 = 2 − 2
⎩ 3
Mặt khác, do 𝑝𝑝(𝐴𝐴) = 0 nên

22011 − 2 9 −4 −8 4 −1 −5 2 − 22011 1 0 0
𝐴𝐴2011 = 𝑎𝑎𝐴𝐴2 + 𝑏𝑏𝑏𝑏 + 𝑐𝑐𝑐𝑐 = �6 −2 −6� + �2 0 −2� + �0 1 0�
3 3
2 −1 −1 1 0 −2 0 0 1
2011
1 8. 22011 − 4 −4. 22011 + 5 −8. 22011 + 1
= �6. 2 −6 −3. 22011 + 2 −6. 22011 + 6�
3 2011
2. 2 −1 −22011 + 2 −2. 22011 − 2
Suy ra
2011 2011 2011 𝑥𝑥0
1 8. 22011 − 4 −4. 22011 + 5 −8. 22011 + 1
𝑋𝑋2011 = 𝐴𝐴 2011
𝑋𝑋0 = �6. 2 − 6 −3. 2 + 2 −6. 2 + 6� �𝑥𝑥0 �
3 𝑥𝑥0
2. 22011 − 1 −22011 + 2 −2. 22011 − 2
2
Vậy 𝑥𝑥2011 = (1 − 22012 )𝑥𝑥0 .
3

2. Các dãy truy hồi tuyến tính với hệ số không đổi


𝑢𝑢0 = 1, 𝑢𝑢1 = 1, 𝑢𝑢2 = 1
Ví dụ : Tính (𝑢𝑢𝑛𝑛 )𝑛𝑛∈ℕ biết �
∀𝑛𝑛 ∈ ℕ: 𝑢𝑢𝑛𝑛+3 = 45𝑢𝑢𝑛𝑛 − 39𝑢𝑢𝑛𝑛+1 + 11𝑢𝑢𝑛𝑛+2

Giải.

Đỗ Minh Triết X-141 - MATHTASY -


Ôn thi Olympic Toán Đại số Phần X: Luỹ thừa ma trận

𝑢𝑢𝑛𝑛 0 1 0
𝑢𝑢
Ký hiệu 𝑋𝑋𝑛𝑛 = � 𝑛𝑛+1 � ∀𝑛𝑛 ∈ ℕ và 𝐴𝐴 = � 0 0 1 � thì ta có 𝑋𝑋𝑛𝑛+1 = 𝐴𝐴𝑋𝑋𝑛𝑛 suy ra 𝑋𝑋𝑛𝑛 =
𝑢𝑢𝑛𝑛+2 45 −39 11
𝐴𝐴 𝑋𝑋0 .
𝑛𝑛

Đa thức đặc trưng của 𝐴𝐴 là 𝑃𝑃𝐴𝐴 (𝑥𝑥) = −(𝑥𝑥 − 3)2 (𝑥𝑥 − 5)


1
Với 𝑥𝑥1 = 3 ta được 1 vector riêng tương ứng 𝑣𝑣1 = �3�
9
1
Với 𝑥𝑥2 = 5 ta được 1 vector riêng tương ứng 𝑣𝑣3 = � 5 �
25
𝐴𝐴 không chéo hoá được nhưng ta sẽ tam giác hoá 𝐴𝐴 bằng cách chọn 𝑣𝑣2 thoả 𝐴𝐴𝑣𝑣2 = 𝑥𝑥1 𝑣𝑣2 + 𝑣𝑣1 ,
0
chẳng hạn 𝑣𝑣2 = �1�. Khi đó ta có
6
1 −5 6 −1 0 1 0 1 0 1 3 1 0
𝐵𝐵 = 𝑃𝑃 −1 𝐴𝐴𝐴𝐴 = �−30 16 −2 � � 0 0 1 � �3 1 5 � = �0 3 0�
4
9 −6 1 45 −39 11 9 6 25 0 0 5
là ma trận tam giác trên.
Dễ dàng tính được
3𝑛𝑛 𝑛𝑛3𝑛𝑛−1 0
𝑛𝑛
𝐵𝐵 = � 0 3𝑛𝑛 0 � ∀ 𝑛𝑛 ∈ ℕ
0 0 5𝑛𝑛
Suy ra

𝑛𝑛 𝑛𝑛 −1 1 1 0 1 3𝑛𝑛 𝑛𝑛3𝑛𝑛−1 0 −5 6 −1 0
𝑋𝑋𝑛𝑛 = 𝐴𝐴 𝑋𝑋0 = 𝑃𝑃 𝐵𝐵 𝑃𝑃 𝑋𝑋0 = �3 1 5 �� 0 3𝑛𝑛 0 � �−30 16 −2 � �1�
4
9 6 25 0 0 5𝑛𝑛 9 −6 1 1
Vậy ∀𝑛𝑛 ∈ ℕ: 𝑢𝑢𝑛𝑛 = −4𝑛𝑛3𝑛𝑛−1 + 5𝑛𝑛 .

⎧𝑢𝑢 = 1, 𝑢𝑢1 = 2
� 0
Ví dụ : Cho (𝑢𝑢𝑛𝑛 )𝑛𝑛∈ℕ được xác định bởi ⎨ 𝑢𝑢2𝑛𝑛+1 = 𝑢𝑢2𝑛𝑛 + 𝑢𝑢2𝑛𝑛−1
� ∀𝑛𝑛 ∈ ℕ∗ : �
⎩ 𝑢𝑢2𝑛𝑛 = 𝑢𝑢2𝑛𝑛−1 + 2𝑢𝑢2𝑛𝑛−2

Giải.

⎧ 𝑢𝑢 = 1, 𝑢𝑢1 = 2
� 0
Từ giả thiết, ta có ⎨ 𝑢𝑢 = 2𝑢𝑢2𝑛𝑛−2 + 𝑢𝑢2𝑛𝑛−1
� ∀𝑛𝑛 ∈ ℕ∗ : � 2𝑛𝑛
⎩ 𝑢𝑢2𝑛𝑛+1 = 2𝑢𝑢2𝑛𝑛−2 + 2𝑢𝑢2𝑛𝑛−1
𝑢𝑢 2 1
Ký hiệu 𝑋𝑋𝑛𝑛 = �𝑢𝑢2𝑛𝑛−2 � ∀𝑛𝑛 ∈ ℕ∗ và 𝐴𝐴 = � � thì ta có 𝑋𝑋𝑛𝑛+1 = 𝐴𝐴𝑋𝑋𝑛𝑛 suy ra 𝑋𝑋𝑛𝑛 = 𝐴𝐴𝑛𝑛−1 𝑋𝑋1 .
2𝑛𝑛−1 2 2
√ √
Đa thức đặc trưng của 𝐴𝐴 là 𝑃𝑃𝐴𝐴 (𝑥𝑥) = �𝑥𝑥 − �2 + 2���𝑥𝑥 − �2 − 2��
𝐴𝐴 có 2 giá trị riêng phân biệt nên chéo hoá được, ta có

Đỗ Minh Triết X-142 - MATHTASY -


Ôn thi Olympic Toán Đại số Phần X: Luỹ thừa ma trận

√ √ √
1 1 2 + 2 0 − 2 − 2 −1
𝐴𝐴 = 𝑃𝑃𝑃𝑃𝑃𝑃 −1
= �√ √ �� √ �. � √ �
2 − 2 0 2− 2 4 − 2 1
Từ đó, ta có
√ √ √
− 2 √1 1 �2 + 2�𝑛𝑛−1 0 − 2 −1 1
𝑛𝑛−1
𝑋𝑋𝑛𝑛 = 𝐴𝐴 𝑋𝑋1 = � √ �� √ 𝑛𝑛−1 � � √ �� �
4 2 − 2 0 �2 − 2� − 2 1 2
√ √ 𝑛𝑛−1 √ 𝑛𝑛−1 √
− 2 �2 + 2� �2 − 2� −2 − 2
= �√ √ 𝑛𝑛−1 √ √ 𝑛𝑛−1 � � √ �
4 2�2 + 2� − 2�2 − 2� 2− 2
√ √ √
− 2 �2 − 2�𝑛𝑛 − �2 + 2�𝑛𝑛
= � √ √ √ √ �
4 − 2�2 + 2�𝑛𝑛 − 2�2 − 2�𝑛𝑛

⎧ − 2 √ √
� 𝑢𝑢2𝑛𝑛−2 = ��2 − 2�𝑛𝑛 − �2 + 2�𝑛𝑛 �
Vậy ∀𝑛𝑛 ∈ ℕ∗ : 4 .
⎨ 1 √ √
�𝑢𝑢2𝑛𝑛−1 = ��2 + 2�𝑛𝑛 + �2 − 2�𝑛𝑛 �
⎩ 2

Đỗ Minh Triết X-143 - MATHTASY -


PHẦN XI. BÀI TOÁN XÁC ĐỊNH MA TRẬN

11.1. Tìm mọi ma trận 𝑿𝑿 giao hoán hoặc phản giao hoán với một ma
trận 𝑨𝑨 cho trước.

Phương pháp 1: Đưa bài Toán về tìm mọi ma trận 𝑋𝑋 giao hoán với ma trận 𝐴𝐴 − 𝑘𝑘𝑘𝑘 đơn giản
hơn.

𝐴𝐴𝐴𝐴 ± 𝑋𝑋𝑋𝑋 = 0 ⇔ (𝐴𝐴 − 𝑘𝑘𝑘𝑘)𝑋𝑋 ± 𝑋𝑋(𝐴𝐴 − 𝑘𝑘𝑘𝑘) = 0


2009 −1 0
Ví dụ 1 : Cho ma trận 𝐴𝐴 = � 0 2009 −1 �. Tìm tất cả các ma trận B cấp 3 giao
0 0 2009
hoán được với ma trận A.

Giải.
0 −1 0 0 −1 0
𝐴𝐴𝐴𝐴 = 𝐵𝐵𝐵𝐵 ⇔ (𝐴𝐴 − 2009𝐼𝐼)𝐵𝐵 = 𝐵𝐵(𝐴𝐴 − 2009𝐼𝐼) ⇔ �0 0 −1 � 𝐵𝐵 = 𝐵𝐵 �0 0 −1 �.
0 0 0 0 0 0
𝑎𝑎11 𝑎𝑎12 𝑎𝑎13
Ký hiệu 𝐵𝐵 = �𝑎𝑎21 𝑎𝑎22 𝑎𝑎23 �, ta có
𝑎𝑎31 𝑎𝑎32 𝑎𝑎33
0 −1 0 𝑎𝑎11 𝑎𝑎12 𝑎𝑎13 𝑎𝑎11 𝑎𝑎12
𝑎𝑎13 0 −1 0
�0 0 −1 � � 21𝑎𝑎 𝑎𝑎 22 𝑎𝑎 23 � = �𝑎𝑎21 𝑎𝑎22
𝑎𝑎23 � �0 0 −1 �
0 0 0 𝑎𝑎31 𝑎𝑎32 𝑎𝑎33 𝑎𝑎31 𝑎𝑎32
𝑎𝑎33 0 0 0
−𝑎𝑎21 −𝑎𝑎22 −𝑎𝑎23 0 −𝑎𝑎11 −𝑎𝑎12 ⎧𝑎𝑎21 = 𝑎𝑎32 = 0
�𝑎𝑎 = 0
⇔ �−𝑎𝑎31 −𝑎𝑎32 ⎛
−𝑎𝑎33 � = ⎜0 −𝑎𝑎21 −𝑎𝑎22 ⎞
⎟ ⇔ ⎨ 31
𝑎𝑎 = 𝑎𝑎33 = 𝑎𝑎11
0 0 0 ⎝0 −𝑎𝑎31 −𝑎𝑎32 ⎠ � 22
⎩𝑎𝑎12 = 𝑎𝑎23
𝑎𝑎 𝑏𝑏 𝑐𝑐
Vậy 𝐵𝐵 = �0 𝑎𝑎 𝑏𝑏 �. Ngược lại, mọi ma trận có dạng này đều giao hoán được với 𝐴𝐴.
0 0 𝑎𝑎

Ví dụ 2 (Câu 3 đề dự tuyển ĐH Quy Nhơn 2009) :


Xác định tất cả các ma trận vuông cấp 3 giao hoán với ma trận
0 −1 −1
𝐴𝐴 = � −1 0 −1 �
1 1 2
Giải.

Đỗ Minh Triết XI-144 - MATHTASY -


Ôn thi Olympic Toán Đại số Phần XI: Bài Toán xác định ma trận

𝑥𝑥1 𝑥𝑥2 𝑥𝑥3


Gọi ma trận cần tìm là 𝑋𝑋 = � 4
𝑥𝑥 𝑥𝑥5 𝑥𝑥6 �. Khi đó 𝐴𝐴𝐴𝐴 = 𝑋𝑋𝑋𝑋 ⇔ (𝐼𝐼 − 𝐴𝐴)𝑋𝑋 = 𝑋𝑋(𝐼𝐼 − 𝐴𝐴)
𝑥𝑥7 𝑥𝑥8 𝑥𝑥9
Ta có
𝑥𝑥1 𝑥𝑥2 𝑥𝑥3 1 1 1 𝑥𝑥1 + 𝑥𝑥2 − 𝑥𝑥3 𝑥𝑥1 + 𝑥𝑥2 − 𝑥𝑥3 𝑥𝑥1 + 𝑥𝑥2 − 𝑥𝑥3
𝑋𝑋(𝐼𝐼 − 𝐴𝐴) = �𝑥𝑥4 𝑥𝑥5 𝑥𝑥6 � � 1 1 1� = � 4 + 𝑥𝑥5 − 𝑥𝑥6
𝑥𝑥 𝑥𝑥4 + 𝑥𝑥5 − 𝑥𝑥6 𝑥𝑥4 + 𝑥𝑥5 − 𝑥𝑥6 �
𝑥𝑥7 𝑥𝑥8 𝑥𝑥9 −1−1 −1 𝑥𝑥7 + 𝑥𝑥8 − 𝑥𝑥9 𝑥𝑥7 + 𝑥𝑥8 − 𝑥𝑥9 𝑥𝑥7 + 𝑥𝑥8 − 𝑥𝑥9
1 1 1 𝑥𝑥 1 𝑥𝑥 2 𝑥𝑥3
(𝐼𝐼 − 𝐴𝐴)𝑋𝑋 = � 1 1 1� �𝑥𝑥4 𝑥𝑥5 𝑥𝑥6 �
−1 −1 −1 𝑥𝑥7 𝑥𝑥8 𝑥𝑥9
𝑥𝑥1 + 𝑥𝑥4 + 𝑥𝑥7 𝑥𝑥2 + 𝑥𝑥5 + 𝑥𝑥8 𝑥𝑥3 + 𝑥𝑥6 + 𝑥𝑥9
=� 𝑥𝑥1 + 𝑥𝑥 4 + 𝑥𝑥 7 𝑥𝑥2 + 𝑥𝑥5 + 𝑥𝑥8 𝑥𝑥3 + 𝑥𝑥6 + 𝑥𝑥9 �
−(𝑥𝑥1 + 𝑥𝑥4 + 𝑥𝑥7 ) −(𝑥𝑥2 + 𝑥𝑥5 + 𝑥𝑥8 ) −(𝑥𝑥3 + 𝑥𝑥6 + 𝑥𝑥9 )
Suy ra 𝑥𝑥1 + 𝑥𝑥2 − 𝑥𝑥3 = 𝑥𝑥1 + 𝑥𝑥4 + 𝑥𝑥7 = 𝑥𝑥2 + 𝑥𝑥5 + 𝑥𝑥8 = 𝑥𝑥3 + 𝑥𝑥6 + 𝑥𝑥9 = 𝑥𝑥4 + 𝑥𝑥5 − 𝑥𝑥6 = −𝑥𝑥7 − 𝑥𝑥8 +
𝑥𝑥9
⎧ 𝑥𝑥1 + 𝑥𝑥2 − 𝑥𝑥3 = 𝑥𝑥3 + 𝑥𝑥6 + 𝑥𝑥9 ⎧ 𝑥𝑥1 + 𝑥𝑥2 − 2𝑥𝑥3 = 𝑥𝑥6 + 𝑥𝑥9
�𝑥𝑥
� 3 + 𝑥𝑥 6 + 𝑥𝑥 9 = −𝑥𝑥 7 − 𝑥𝑥 8 + 𝑥𝑥 9

� 𝑥𝑥3 = −𝑥𝑥6 − 𝑥𝑥7 − 𝑥𝑥8
hay ⎨−𝑥𝑥7 − 𝑥𝑥8 + 𝑥𝑥9 = 𝑥𝑥2 + 𝑥𝑥5 + 𝑥𝑥8 ⇔ ⎨𝑥𝑥2 + 𝑥𝑥5 = −𝑥𝑥7 − 2𝑥𝑥8 + 𝑥𝑥9
� 𝑥𝑥 + 𝑥𝑥5 + 𝑥𝑥8 = 𝑥𝑥4 + 𝑥𝑥5 − 𝑥𝑥6 � 𝑥𝑥2 − 𝑥𝑥4 = −𝑥𝑥6 − 𝑥𝑥8
� 2 �
⎩ 𝑥𝑥4 + 𝑥𝑥5 − 𝑥𝑥6 = 𝑥𝑥1 + 𝑥𝑥4 + 𝑥𝑥7 ⎩ 𝑥𝑥1 − 𝑥𝑥5 = −𝑥𝑥6 − 𝑥𝑥7
𝑥𝑥3 = −𝑥𝑥6 − 𝑥𝑥7 − 𝑥𝑥8 𝑥𝑥3 = −𝑥𝑥6 − 𝑥𝑥7 − 𝑥𝑥8
⎧ ⎧
� 𝑥𝑥
� 1 + 𝑥𝑥 2 = −𝑥𝑥 6 − 2𝑥𝑥 7 − 2𝑥𝑥 8 + 𝑥𝑥 9 � 𝑥𝑥
� 1 + 𝑥𝑥 2 = −𝑥𝑥6 − 2𝑥𝑥7 − 2𝑥𝑥8 + 𝑥𝑥9
⇔ ⎨ 𝑥𝑥2 + 𝑥𝑥5 = −𝑥𝑥7 − 2𝑥𝑥8 + 𝑥𝑥9 ⇔ 𝑥𝑥1 + 𝑥𝑥2 = −𝑥𝑥6 − 2𝑥𝑥7 − 2𝑥𝑥8 + 𝑥𝑥9

� 𝑥𝑥2 − 𝑥𝑥4 = −𝑥𝑥6 − 𝑥𝑥8 � 𝑥𝑥2 − 𝑥𝑥4 = −𝑥𝑥6 − 𝑥𝑥8
� �
⎩ 𝑥𝑥1 − 𝑥𝑥5 = −𝑥𝑥6 − 𝑥𝑥7 ⎩ 𝑥𝑥1 − 𝑥𝑥5 = −𝑥𝑥6 − 𝑥𝑥7
Dễ thấy 𝑥𝑥3 độc lập với các phương trình còn lại, hệ có 2 phương trình giống nhau, chuyển 𝑥𝑥5 làm
số hạng tự do, cuối cùng ta được nghiệm của hệ
𝑥𝑥 = 𝑥𝑥5 − 𝑥𝑥6 − 𝑥𝑥7
⎧ 1

� 𝑥𝑥2 = −𝑥𝑥5 − 𝑥𝑥7 − 2𝑥𝑥8 + 𝑥𝑥9
𝑥𝑥 = −𝑥𝑥6 − 𝑥𝑥7 − 𝑥𝑥8
⎨ 3
� 𝑥𝑥4 = −𝑥𝑥5 + 𝑥𝑥6 − 𝑥𝑥7 − 𝑥𝑥8 + 𝑥𝑥9

⎩ 𝑥𝑥𝑖𝑖 ∈ ℝ, 𝑖𝑖 = 5,6,7,8,9
Ma trận cần tìm có dạng
𝑎𝑎 − 𝑏𝑏 − 𝑐𝑐 −𝑎𝑎 − 𝑐𝑐 − 2𝑑𝑑 + 𝑒𝑒 −𝑏𝑏 − 𝑐𝑐 − 𝑑𝑑
𝑋𝑋 = �−𝑎𝑎 + 𝑏𝑏 − 𝑐𝑐 − 𝑑𝑑 + 𝑒𝑒 𝑎𝑎 𝑏𝑏 �
𝑐𝑐 𝑑𝑑 𝑒𝑒
Ngược lại, mọi ma trận có dạng trên đều giao hoán được với 𝐴𝐴.

Ví dụ 3 (Câu 5 Olympic 2007) : Cho ma trận


2 −1 0 0
⎛ 0 2 −1 0⎞
𝐴𝐴 = ⎜
⎜0 ⎟
0 2 −1 ⎟
⎝0 0 0 2⎠

Đỗ Minh Triết XI-145 - MATHTASY -


Ôn thi Olympic Toán Đại số Phần XI: Bài Toán xác định ma trận

Tìm tất cả các ma trận vuông 𝑋𝑋 cấp 4 sao cho 𝐴𝐴𝐴𝐴 = 𝑋𝑋𝑋𝑋.

Giải.

Ta có 𝐴𝐴𝐴𝐴 = 𝑋𝑋𝑋𝑋 ⇔ (𝐴𝐴 − 2𝐼𝐼)𝑋𝑋 = 𝑋𝑋(𝐴𝐴 − 2𝐼𝐼)


0 −1 0 0 0 −1 0 0

⎜ 0 0 −1 0⎞⎟ 𝑋𝑋 = 𝑋𝑋 ⎛
⎜ 0 0 −1 0⎞⎟
⎜0 (1)
0 0 −1 ⎟ ⎜ 0 0 0 −1 ⎟
⎝0 0 0 0⎠ ⎝ 0 0 0 0⎠
Kí hiệu:
0 −1 0 0
⎛ 0 0 −1 0⎞
𝐵𝐵 = ⎜
⎜0 ⎟ = �𝑏𝑏𝑖𝑖𝑖𝑖 �; 𝐶𝐶 = 𝐵𝐵𝐵𝐵 = �𝑐𝑐𝑖𝑖𝑖𝑖 �; 𝐷𝐷 = 𝑋𝑋𝐵𝐵 = �𝑑𝑑𝑖𝑖𝑖𝑖 �
0 0 −1 ⎟
⎝0 0 0 0⎠
Khi đó (1) tương đương 𝐶𝐶 = 𝐷𝐷 hay 𝑐𝑐𝑖𝑖𝑖𝑖 = 𝑑𝑑𝑖𝑖𝑖𝑖 , 1 ≤ 𝑖𝑖, 𝑗𝑗 ≤ 4. Ta thấy 𝑐𝑐𝑖𝑖𝑖𝑖 = 0 ∀𝑗𝑗 và 𝑑𝑑𝑖𝑖𝑖𝑖 = 0 ∀𝑖𝑖. Mặt
khác với 𝑖𝑖 ≤ 3 và 𝑗𝑗 ≥ 2 ta có: 𝑐𝑐𝑖𝑖𝑖𝑖 = 𝑑𝑑𝑖𝑖𝑖𝑖 . Do đó
4 4
� 𝑏𝑏𝑖𝑖𝑖𝑖 𝑥𝑥𝑘𝑘𝑘𝑘 = � 𝑥𝑥𝑖𝑖𝑖𝑖 𝑏𝑏𝑘𝑘𝑘𝑘 hay 𝑥𝑥(𝑖𝑖+1)𝑗𝑗 = 𝑥𝑥𝑖𝑖(𝑗𝑗−1)
𝑘𝑘=1 𝑘𝑘=1

Tóm lại, ta thu được 𝑥𝑥𝑖𝑖𝑖𝑖 = 𝑥𝑥𝑖𝑖+1,𝑗𝑗+1 , 1 ≤ 𝑖𝑖, 𝑗𝑗 ≤ 3. Vậy ma trận 𝑋𝑋 có dạng
𝑎𝑎 𝑏𝑏 𝑐𝑐 𝑑𝑑
⎛ 0 𝑎𝑎 𝑏𝑏 𝑐𝑐 ⎞
𝑋𝑋 = ⎜
⎜0 ⎟, 𝑎𝑎, 𝑏𝑏, 𝑐𝑐, 𝑑𝑑 ∈ ℝ
0 𝑎𝑎 𝑏𝑏 ⎟
⎝0 0 0 𝑎𝑎⎠
Ngược lại, mọi ma trận có dạng trên đều giao hoán được với 𝐴𝐴.

Phương pháp 2: Ứng dụng chéo hoá (hoặc tam giác hoá) ma trận 𝐴𝐴 = 𝑃𝑃𝑃𝑃𝑃𝑃 −1 .

𝐴𝐴𝐴𝐴 = 𝑋𝑋𝑋𝑋 ⇔ 𝑃𝑃𝑃𝑃𝑃𝑃 −1 𝑋𝑋 = 𝑋𝑋𝑋𝑋𝑋𝑋𝑃𝑃 −1 ⇔ 𝐵𝐵(𝑃𝑃 −1 𝑋𝑋𝑋𝑋 ) = (𝑃𝑃 −1 𝑋𝑋𝑋𝑋 )𝐵𝐵

Bài Toán đã cho quy về bài Toán đơn giản hơn đó là tìm mọi ma trận 𝑄𝑄 = 𝑃𝑃 −1 𝑋𝑋𝑋𝑋 giao hoán với
ma trận 𝐵𝐵 (ma trận đường chéo hoặc ma trận tam giác). Tìm được 𝑄𝑄 ⇒ 𝑋𝑋 = 𝑃𝑃𝑃𝑃𝑃𝑃 −1 .
Chú ý:
- Dễ dàng thấy rằng mọi ma trận giao hoán với ma trận chéo là một ma trận chéo.
- Trong trường hợp trên, nếu 𝐵𝐵 là ma trận chéo với các phần tử chéo khác nhau thì 𝑄𝑄 cũng là
ma trận chéo và với 𝑋𝑋 = 𝑃𝑃𝑃𝑃𝑃𝑃 −1 thì 𝑋𝑋 cũng đã được chéo hoá với cùng cơ sở đã chéo hoá 𝐴𝐴.
Như vậy, ta có kết luận: nếu một ma trận 𝑋𝑋 giao hoán với một ma trận 𝐴𝐴 có các giá trị riêng
đôi một phân biệt thì 𝑋𝑋 cũng chéo hoá được với cùng cơ sở đó, nói khác đi thì 𝑋𝑋 có cùng vector
riêng với 𝐴𝐴.
Ví dụ 1 (Câu 3 đề dự tuyển ĐH Quy Nhơn 2009) :
Xác định tất cả các ma trận vuông cấp 3 giao hoán với ma trận

Đỗ Minh Triết XI-146 - MATHTASY -


Ôn thi Olympic Toán Đại số Phần XI: Bài Toán xác định ma trận

0 −1 −1
𝐴𝐴 = � −1 0 −1 �
1 1 2
Giải.

Chéo hoá 𝐴𝐴, ta được:


1 −1 −1 0 0 0 1 1 1
𝐴𝐴 = 𝐶𝐶𝐶𝐶 𝐶𝐶 −1 = � 1 0 1� �0 1 0� � 1 1 2�
−1 1 0 0 0 1 −1 0 −1
Khi đó nếu 𝑋𝑋 giao hoán được với 𝐴𝐴
𝑋𝑋𝑋𝑋 = 𝐴𝐴𝐴𝐴 ⇔ 𝑋𝑋𝑋𝑋𝑋𝑋 𝐶𝐶 −1 = 𝐶𝐶𝐶𝐶 𝐶𝐶 −1 𝑋𝑋 ⇔ (𝐶𝐶 −1 𝑋𝑋𝑋𝑋)𝑃𝑃 = 𝑃𝑃 (𝐶𝐶 −1 𝑋𝑋𝑋𝑋)
tương đương 𝑄𝑄 = 𝐶𝐶 −1 𝑋𝑋𝑋𝑋 giao hoán được với 𝑃𝑃 .
Thử trực tiếp thì 𝑄𝑄 có dạng
𝑎𝑎 0 0
𝑄𝑄 = �0 𝑏𝑏 𝑑𝑑�
0 𝑒𝑒 𝑐𝑐
1 −1 −1 𝑎𝑎 0 0 1 1 1
Vậy ma trận 𝑋𝑋 cần tìm có dạng 𝑋𝑋 = 𝐶𝐶𝑄𝑄𝐶𝐶 −1 = � 1 0 1� �0 𝑏𝑏 𝑑𝑑� � 1 1 2�
−1 1 0 0 𝑒𝑒 𝑐𝑐 −1 0 −1
𝑎𝑎 − 𝑏𝑏 + 𝑐𝑐 + 𝑑𝑑 − 𝑒𝑒 𝑎𝑎 − 𝑏𝑏 − 𝑒𝑒 𝑎𝑎 − 2𝑏𝑏 + 𝑐𝑐 + 𝑑𝑑 − 2𝑒𝑒
=� 𝑎𝑎 − 𝑐𝑐 + 𝑒𝑒 𝑎𝑎 + 𝑒𝑒 𝑎𝑎 − 𝑐𝑐 + 2𝑒𝑒 �
−𝑎𝑎 + 𝑏𝑏 − 𝑑𝑑 −𝑎𝑎 + 𝑏𝑏 −𝑎𝑎 + 2𝑏𝑏 − 𝑑𝑑
Ngược lại, mọi ma trận có dạng trên đều giao hoán được với 𝐴𝐴.

Phương pháp 3: Đưa bài Toán về tìm mọi ma trận 𝑋𝑋 giao hoán với ma trận 𝐵𝐵 tự khả nghịch
(nếu có thể).

Ví dụ 1 (Câu 1 Olympic 2001) : Cho ma trận


1 0 1
𝐴𝐴 = �0 1 2�
0 0 −1
Tìm tất cả các ma trận 𝑋𝑋 vuông cấp 3 sao cho 𝐴𝐴𝐴𝐴 + 𝑋𝑋𝑋𝑋 = 0.

Giải.

Ta có 𝐴𝐴2 = 𝐼𝐼. Giả sử 𝑋𝑋 là ma trận thoả mãn 𝐴𝐴𝐴𝐴 + 𝑋𝑋𝑋𝑋 = 0, suy ra 𝑋𝑋 = −𝐴𝐴𝐴𝐴𝐴𝐴, từ đó
1 1 1 1
𝑋𝑋 = (𝑋𝑋 + 𝑋𝑋) = (𝐴𝐴2 𝑋𝑋 − 𝐴𝐴𝐴𝐴𝐴𝐴) = 𝐴𝐴 � 𝐴𝐴𝐴𝐴� − � 𝐴𝐴𝐴𝐴� 𝐴𝐴 = 𝐴𝐴𝐴𝐴 − 𝐵𝐵𝐵𝐵
2 2 2 2
trong đó 𝐵𝐵 = 12 𝐴𝐴𝐴𝐴. Ngược lại mọi ma trận 𝑋𝑋 có dạng
𝑋𝑋 = 𝐴𝐴𝐴𝐴 − 𝐵𝐵𝐵𝐵
đều thoả mãn 𝐴𝐴𝐴𝐴 + 𝑋𝑋𝑋𝑋 = 0, thật vậy
𝐴𝐴𝐴𝐴 + 𝑋𝑋𝑋𝑋 = 𝐴𝐴(𝐴𝐴𝐴𝐴 − 𝐵𝐵𝐵𝐵) − (𝐴𝐴𝐴𝐴 − 𝐵𝐵𝐵𝐵)𝐴𝐴 = 𝐴𝐴2 𝐵𝐵 − 𝐴𝐴𝐴𝐴𝐴𝐴 + 𝐴𝐴𝐴𝐴𝐴𝐴 − 𝐵𝐵𝐴𝐴2 = 𝐵𝐵 − 𝐵𝐵 = 0

Đỗ Minh Triết XI-147 - MATHTASY -


Ôn thi Olympic Toán Đại số Phần XI: Bài Toán xác định ma trận

Vậy ma trận 𝑋𝑋 cần tìm là 𝐴𝐴𝐴𝐴 − 𝐵𝐵𝐵𝐵 với 𝐵𝐵 là ma trận tuỳ ý.

Dựa vào cách làm này, quay trở lại ví dụ trước, ta còn có thể làm như sau:
Ví dụ 2 (Câu 3 đề dự tuyển ĐH Quy Nhơn 2009) :
Xác định tất cả các ma trận vuông cấp 3 giao hoán với ma trận
0 −1 −1
𝐴𝐴 = � −1 0 −1 �
1 1 2
Giải.

Dễ thấy 𝐴𝐴2 = 𝐴𝐴 hay 𝐴𝐴(𝐴𝐴 − 𝐼𝐼) = 0.


Đặt 𝐴𝐴 = 12 (𝐵𝐵 + 𝐼𝐼) thì ta có 𝐴𝐴(𝐴𝐴 − 𝐼𝐼) = 12 (𝐵𝐵 + 𝐼𝐼) 12 (𝐵𝐵 − 𝐼𝐼) = 0, suy ra 𝐵𝐵2 = 𝐼𝐼.
Mặt khác, ma trận 𝑋𝑋 giao hoán với 𝐴𝐴 nếu và chỉ nếu 𝑋𝑋 giao hoán với 𝐵𝐵, thật vậy
1 1
𝐴𝐴𝐴𝐴 = 𝑋𝑋𝑋𝑋 ⇔ (𝐵𝐵 + 𝐼𝐼)𝑋𝑋 = 𝑋𝑋(𝐵𝐵 + 𝐼𝐼) ⇔ 𝐵𝐵𝐵𝐵 = 𝑋𝑋𝑋𝑋
2 2
Ta có 𝑋𝑋 = 𝐵𝐵𝐵𝐵𝐵𝐵, từ đó
1 1 1 1
𝑋𝑋 = (𝑋𝑋 + 𝑋𝑋) = (𝐵𝐵2 𝑋𝑋 + 𝐵𝐵𝐵𝐵𝐵𝐵) = 𝐵𝐵 � 𝐵𝐵𝐵𝐵� + � 𝐵𝐵𝐵𝐵� 𝐵𝐵 = 𝐵𝐵𝐵𝐵 + 𝑍𝑍𝑍𝑍
2 2 2 2
trong đó 𝑍𝑍 = 12 𝐵𝐵𝐵𝐵.
Ngược lại mọi ma trận 𝑋𝑋 có dạng 𝑋𝑋 = 𝐵𝐵𝐵𝐵 + 𝑍𝑍𝑍𝑍 đều giao hoán với 𝐵𝐵, thật vậy
𝐵𝐵𝐵𝐵 − 𝑍𝑍𝑍𝑍 = 𝐵𝐵(𝐵𝐵𝐵𝐵 + 𝑍𝑍𝑍𝑍) − (𝐵𝐵𝐵𝐵 + 𝑍𝑍𝑍𝑍)𝐵𝐵 = 𝐵𝐵 2 𝑍𝑍 + 𝐵𝐵𝐵𝐵𝐵𝐵 − 𝐵𝐵𝐵𝐵𝐵𝐵 − 𝑍𝑍𝐵𝐵2 = 𝑍𝑍 − 𝑍𝑍 = 0
Vậy ma trận 𝑋𝑋 cần tìm là 𝐵𝐵𝐵𝐵 + 𝑍𝑍𝑍𝑍, trong đó
−1 −2 −2
𝐵𝐵 = 2𝐴𝐴 − 𝐼𝐼 = � −2 −1 −2 � , 𝑍𝑍 ∈ 𝑀𝑀3 (ℝ).
2 2 3

11.2. Phương trình ma trận

Câu (Câu 2 đề chọn đội tuyển ĐH Kinh tế Quốc dân 2005) :


Tìm ma trận 𝑋𝑋 thoả mãn
1 −1 1
6 2 −7
𝑋𝑋 �1 0 −1 � = � �
15 2 −13
1 1 −2
Giải.
1 2 3
Ma trận thoả mãn đẳng thức là 𝑋𝑋 = � �
4 5 6

Đỗ Minh Triết XI-148 - MATHTASY -


Ôn thi Olympic Toán Đại số Phần XI: Bài Toán xác định ma trận

−1 0
Câu : Giải phương trình 𝑋𝑋 2 − 2𝑋𝑋 = � � trong đó 𝑋𝑋 là ma trận vuông cấp 2.
6 3
Giải.
−1 0 𝑎𝑎 𝑏𝑏
Ta có 𝑋𝑋 2 − 2𝑋𝑋 = � �. Đặt 𝑋𝑋– 𝐼𝐼 = 𝑌𝑌 và 𝑌𝑌 = � �.
6 3 𝑐𝑐 𝑑𝑑
⎧𝑎𝑎2 + 𝑏𝑏𝑏𝑏 = 0 ⎧ 𝑎𝑎 = 0

� 𝑏𝑏(𝑎𝑎 + 𝑑𝑑) = 0 � 𝑏𝑏 = 0
0 0
Suy ra 𝑌𝑌 2 = � �⇔⎨ ⇔
⎨𝑐𝑐𝑐𝑐 = 6
.
6 4 � 𝑐𝑐(𝑎𝑎 + 𝑑𝑑) = 6 �
� ⎩𝑑𝑑2 = 4
⎩𝑏𝑏𝑏𝑏 + 𝑑𝑑2 = 4
1 0 1 0
Vậy 𝑋𝑋 = � � hoặc 𝑋𝑋 = � �.
3 3 −3 −1

⎧(tr 𝑋𝑋)𝑌𝑌 + (tr 𝑌𝑌 )𝑋𝑋 = �4 8



� 4 −4
Câu : Giải hệ ⎨ , trong đó 𝑋𝑋, 𝑌𝑌 là các ma trận
� 1 1
⎩ 𝑋𝑋𝑋𝑋 = � �
4 −2
vuông cấp 2.

Giải.

Từ hệ đã cho ta có
4 8 tr 𝑋𝑋 = 0
tr[(tr 𝑋𝑋)𝑌𝑌 + (tr 𝑌𝑌 )𝑋𝑋] = 2 tr 𝑋𝑋 tr 𝑌𝑌 = tr � �=0⇒�
4 −4 tr 𝑌𝑌 = 0
⎧(tr 𝑌𝑌 )𝑋𝑋 = �4 8

� 4 −4
+) Nếu tr 𝑋𝑋 = 0 = 0 thì
⎨ 1 1
� 𝑋𝑋𝑋𝑋 = � �
⎩ 4 −2
4 8
Đặt tr 𝑌𝑌 = 𝜆𝜆1 , 𝜆𝜆 ≠ 0 thì 𝑋𝑋 = 𝜆𝜆 � �, ta được
4 −4
⎧ tr 𝑌𝑌 =
1 ⎧tr 𝑌𝑌 = 1
� � 𝜆𝜆 1 3 −1
𝜆𝜆 ⇔ ⇒ 𝑌𝑌 = � �
⎨ 1 2 1 1 ⎨ 1 1 2 −1
1 1 4𝜆𝜆 −1 1
�4𝜆𝜆 � � 𝑌𝑌 = � � � 𝑌𝑌 = � � � �
⎩ 1 −1 4 −2 ⎩ 4𝜆𝜆 1 −1 4 −2
⎧𝑋𝑋 = 𝜆𝜆 �4 8 �
� 4 −4
Vậy nghiệm của hệ là: ⎨ , 𝜆𝜆 ∈ ℝ∗
�𝑌𝑌 = 4𝜆𝜆 �3
1 −1

⎩ −1 1
⎧(tr 𝑋𝑋)𝑌𝑌 = �4 8

� 4 −4
+) Nếu tr 𝑌𝑌 = 0 thì ⎨
� 1 1
⎩ 𝑋𝑋𝑋𝑋 = � �
4 −2
4 8
Đặt 𝑉𝑉 (𝑋𝑋) = 𝜇𝜇1 , 𝜇𝜇 ≠ 0 thì 𝑌𝑌 = 𝜇𝜇 � �, ta được
4 −4

Đỗ Minh Triết XI-149 - MATHTASY -


Ôn thi Olympic Toán Đại số Phần XI: Bài Toán xác định ma trận

⎧ 1 ⎧tr 𝑋𝑋 = 1
� tr 𝑋𝑋 = �
� 𝜇𝜇 1 2 1
𝜇𝜇 ⇔ ⇒ 𝑋𝑋 = � �
⎨ ⎨ 1 1 1 1 2 −1 12𝜇𝜇 2 10
�4𝜇𝜇𝜇𝜇 �1 2 � = �1 1 � �
� 𝑋𝑋 = � �� �
⎩ 1 −1 4 −2 ⎩ 4𝜇𝜇 4 −2 1 −1
⎧𝑋𝑋 = 1 �2 1 �
� 12𝜇𝜇 2 10
Vậy nghiệm của hệ là: ⎨ , 𝜇𝜇 ∈ ℝ∗
�𝑌𝑌 = 𝜇𝜇 � 4 8
⎩ �
4 −4

⎧𝑋𝑋𝑋𝑋 = �1 0


� 1 2
� 2 2
Câu: Cho 𝑋𝑋, 𝑌𝑌 , 𝑍𝑍 là các ma trận vuông cấp 2. Giải hệ phương trình 𝑌𝑌𝑌𝑌 = � �.
⎨ 1 1


�𝑍𝑍𝑍𝑍 = �2 0

⎩ 2 0
Giải.

⎧𝐴𝐴 = �1 0�

� 1 2
� 𝑋𝑋𝑋𝑋 = 𝐴𝐴
2 2
Đặt ⎨𝐵𝐵 = � � ⇒ �𝑌𝑌𝑌𝑌 = 𝐵𝐵 ⇒ 𝐴𝐴𝐴𝐴 = 𝑋𝑋𝑋𝑋𝑋𝑋𝑋𝑋 = 𝑋𝑋𝑋𝑋𝑋𝑋.
1 1

� 𝑍𝑍𝑍𝑍 = 𝐶𝐶
�𝐶𝐶 = � 0�
2
⎩ 2 0
𝑥𝑥 𝑦𝑦
Đặt 𝑋𝑋 = � � thì từ 𝑋𝑋𝑋𝑋𝑋𝑋𝑋𝑋 = 𝑋𝑋𝑋𝑋𝑋𝑋 ta có
𝑧𝑧 𝑡𝑡
⎧(2𝑥𝑥 + 𝑦𝑦)(𝑥𝑥 + 𝑧𝑧) = 2 𝑦𝑦 + 𝑡𝑡 = 0

�(2𝑥𝑥 + 𝑦𝑦)(𝑦𝑦 + 𝑡𝑡) = 0 ⎧

⇔ ⎨(2𝑥𝑥 + 𝑦𝑦)(𝑥𝑥 + 𝑧𝑧) = 2
⎨(2𝑧𝑧 + 𝑡𝑡)(𝑥𝑥 + 𝑧𝑧) = 6 �

� ⎩(2𝑧𝑧 + 𝑡𝑡)(𝑥𝑥 + 𝑧𝑧) = 6
⎩(2𝑧𝑧 + 𝑡𝑡)(𝑦𝑦 + 𝑡𝑡) = 0
⎧ 𝑦𝑦 + 𝑡𝑡 = 0 ⎧ 𝑡𝑡 = −𝑦𝑦
� �
⇔ (2𝑥𝑥 + 𝑦𝑦)(𝑥𝑥 + 𝑧𝑧) = 2 ⇔ (2𝑥𝑥 + 𝑦𝑦) = ±1

� ⎨

⎩2(𝑥𝑥 + 𝑧𝑧)2 = 8 ⎩(𝑥𝑥 + 𝑧𝑧) = ±2
𝑥𝑥 ±1 − 2𝑥𝑥
Vậy 𝑋𝑋 = � �
±2 − 𝑥𝑥 ∓1 + 2𝑥𝑥
Ta chứng minh được rằng 𝑋𝑋 khả nghịch khi và chỉ khi ±4𝑥𝑥 − 2 ≠ 0. Khi đó
1 ∓1 + 2𝑥𝑥 ∓1 + 2𝑥𝑥
𝑋𝑋 −1 = � �
±4𝑥𝑥 − 2 ∓2 + 𝑥𝑥 𝑥𝑥
Vậy nghiệm của hệ là
⎧𝑋𝑋 = � 𝑥𝑥 ±1 − 2𝑥𝑥
� �
� ±2 − 𝑥𝑥 ∓1 + 2𝑥𝑥
� 1 ∓1 + 2𝑥𝑥 ∓1 + 2𝑥𝑥
⎨𝑌𝑌 = ±2𝑥𝑥 − 1 �∓1 + 𝑥𝑥 𝑥𝑥



�𝑍𝑍 = �±1 ±1�
⎩ ±1 ±1

Đỗ Minh Triết XI-150 - MATHTASY -


Ôn thi Olympic Toán Đại số Phần XI: Bài Toán xác định ma trận

11.3. Các dạng khác

Câu (Câu 6 Olympic 1994) : Cho 𝐴𝐴 là ma trận vuông cấp hai luỹ đẳng. Chứng minh
rằng để ma trận 𝑋𝑋 giao hoán với 𝐴𝐴, điều kiện cần và đủ là tồn tại ma trận 𝑋𝑋0 sao cho 𝑋𝑋 =
𝐴𝐴𝑋𝑋0 + 𝑋𝑋0 𝐴𝐴 − 𝑋𝑋0 .

Giải.

Giả sử ta có ma trận 𝑋𝑋 giao hoán với ma trận 𝐴𝐴 luỹ đẳng. Xét 𝑋𝑋0 = 2𝐴𝐴𝐴𝐴 − 𝑋𝑋, ta có
𝐴𝐴𝑋𝑋0 + 𝑋𝑋0 𝐴𝐴 − 𝑋𝑋0 = 𝐴𝐴(2𝐴𝐴𝐴𝐴 − 𝑋𝑋) + (2𝐴𝐴𝐴𝐴 − 𝑋𝑋)𝐴𝐴 − 𝑋𝑋0 = 𝑋𝑋
Ngược lại mọi ma trận 𝑋𝑋 có dạng 𝑋𝑋 = 𝐴𝐴𝑋𝑋0 + 𝑋𝑋0 𝐴𝐴 − 𝑋𝑋0 đều giao hoán được với 𝐴𝐴 luỹ đẳng.

Câu (Câu 3 đề dự tuyển ĐH Thuỷ Lợi 2009) :



𝑎𝑎 −𝑏𝑏 4 3 −1
Tìm tất cả các số thực 𝑎𝑎, 𝑏𝑏 sao cho � � =� √ �
𝑏𝑏 𝑎𝑎 1 3

Giải.
√ √
Đặt 𝑎𝑎 = 𝑥𝑥 4 2, 𝑏𝑏 = 𝑦𝑦 4 2, phương trình đã cho được đưa về dạng

3 1 𝜋𝜋 𝜋𝜋
𝑥𝑥 −𝑦𝑦 4 ⎡ 2 − ⎤ cos − sin
⎡ 6⎤
�𝑦𝑦 𝑥𝑥� = ⎢⎢ 1 √2 ⎥ = ⎢ 𝜋𝜋6 𝜋𝜋⎥
3⎥ sin cos ⎦
⎣ 2 ⎣ 6 6
2 ⎦
Định thức vế trái cho ta 𝑥𝑥2 + 𝑦𝑦 2 = 1 nên đặt 𝑥𝑥 = cos 𝛼𝛼 , 𝑦𝑦 = sin 𝛼𝛼, ta có
𝜋𝜋 𝜋𝜋
cos − sin
cos 4𝛼𝛼 − sin 4𝛼𝛼 ⎡ 6⎤
� � = ⎢ 𝜋𝜋6 𝜋𝜋⎥
sin 4𝛼𝛼 cos 4𝛼𝛼 sin cos
⎣ 6 6⎦
Từ đó ta có 𝛼𝛼 = 24
𝜋𝜋
+ 𝑘𝑘𝑘𝑘
2
với 𝑘𝑘 = 0,1,2,3
√ 𝜋𝜋 𝑘𝑘𝑘𝑘 √ 𝜋𝜋 𝑘𝑘𝑘𝑘
Vậy 𝑎𝑎 = 4
2 cos � + � , 𝑏𝑏 = 4 2 sin � + � với 𝑘𝑘 = 0,1,2,3.
24 2 24 2

Câu (Câu 3b đề chọn đội tuyển vòng 1 ĐH An Giang 2009) : Tìm tất cả các ma
trận 𝐴𝐴 ∈ 𝑀𝑀𝑛𝑛 (ℝ) có mọi phần tử không âm và ma trận khả nghịch của nó cũng có mọi phần tử
không âm.

Giải.

Ký hiệu 𝐴𝐴 = �𝑎𝑎𝑖𝑖𝑖𝑖 �, 𝐴𝐴−1 = �𝑏𝑏𝑖𝑖𝑖𝑖 �, 𝑖𝑖, 𝑗𝑗 = ���������


1, 𝑛𝑛; 𝑎𝑎𝑖𝑖𝑖𝑖 ≥ 0, 𝑏𝑏𝑖𝑖𝑖𝑖 ≥ 0 ∀𝑖𝑖, 𝑗𝑗. Khi đó, ta có
𝑛𝑛
𝐴𝐴𝐴𝐴−1 = �𝑐𝑐𝑖𝑖𝑖𝑖 � = �� 𝑎𝑎𝑖𝑖𝑖𝑖 𝑏𝑏𝑘𝑘𝑘𝑘 � = 𝐼𝐼
𝑘𝑘=1

Đỗ Minh Triết XI-151 - MATHTASY -


Ôn thi Olympic Toán Đại số Phần XI: Bài Toán xác định ma trận

Suy ra ∀𝑖𝑖, 𝑗𝑗, ta có


𝑛𝑛
⎧� 𝑎𝑎 𝑏𝑏 = 0 nếu 𝑖𝑖 ≠ 𝑗𝑗
𝑎𝑎𝑖𝑖𝑖𝑖 𝑏𝑏𝑘𝑘𝑘𝑘 = 0 nếu 𝑖𝑖 ≠ 𝑗𝑗 ⎧ �𝑎𝑎𝑖𝑖𝑖𝑖 = 0 nếu 𝑖𝑖 ≠ 𝑗𝑗

� 𝑖𝑖𝑖𝑖 𝑘𝑘𝑘𝑘 ⎧ �
� 𝑏𝑏𝑘𝑘𝑘𝑘 = 0
𝑘𝑘=1
� 𝑛𝑛
⇒ ⇒ 𝑛𝑛
⎨ 𝑛𝑛 ⎨� 𝑎𝑎𝑖𝑖𝑖𝑖 𝑏𝑏𝑘𝑘𝑘𝑘 = 1 nếu 𝑖𝑖 ≠ 𝑗𝑗 ⎨

�� 𝑎𝑎𝑖𝑖𝑖𝑖 𝑏𝑏𝑘𝑘𝑘𝑘 = 1 nếu 𝑖𝑖 ≠ 𝑗𝑗 � �� 𝑎𝑎𝑖𝑖𝑖𝑖 𝑏𝑏𝑘𝑘𝑘𝑘 = 1 nếu 𝑖𝑖 ≠ 𝑗𝑗

⎩ 𝑘𝑘=1
⎩ 𝑘𝑘=1 ⎩ 𝑘𝑘=1
���������
Suy ra 𝑎𝑎𝑖𝑖𝑖𝑖 𝑏𝑏𝑖𝑖𝑖𝑖 = 1, 𝑖𝑖 = 1, 𝑛𝑛. Vậy chỉ có ma trận đường chéo với các phần tử chéo không âm là thoả
mãn yêu cầu đề bài
𝑎𝑎11 … 0
𝐴𝐴 = � ⋮ ⋱ ⋮ � , 𝑎𝑎𝑖𝑖𝑖𝑖 > 0, 𝑖𝑖 = ���������
1, 𝑛𝑛
0 … 𝑎𝑎𝑛𝑛𝑛𝑛

Đỗ Minh Triết XI-152 - MATHTASY -


PHẦN XII. HỆ PHƯƠNG TRÌNH TUYẾN TÍNH

Bài 12.1. (Câu 6a Olympic 2009) Giải hệ phương trình:


⎧ 2𝑥𝑥1 + 𝑥𝑥2 − 𝑥𝑥3 + 2𝑥𝑥4 + 𝑥𝑥5 − 𝑥𝑥6 = 1
� −𝑥𝑥 + 2𝑥𝑥2 + 2𝑥𝑥3 + 𝑥𝑥4 + 𝑥𝑥5 − 𝑥𝑥6 = 1
� 1
� 𝑥𝑥1 − 2𝑥𝑥2 + 2𝑥𝑥3 + 𝑥𝑥4 + 𝑥𝑥5 − 𝑥𝑥6 = 1
⎨ −2𝑥𝑥1 − 𝑥𝑥2 − 𝑥𝑥3 + 2𝑥𝑥4 + 𝑥𝑥5 − 𝑥𝑥6 = 1

� 2𝑥𝑥1 + 𝑥𝑥2 + 𝑥𝑥3 − 𝑥𝑥4 − 𝑥𝑥5 + 2𝑥𝑥6 = 1

⎩ −𝑥𝑥1 + 2𝑥𝑥2 + 𝑥𝑥3 − 𝑥𝑥4 + 2𝑥𝑥5 + 𝑥𝑥6 = 1

Giải.

Từ hai phương trình đầu, ta có: 3𝑥𝑥1 − 𝑥𝑥2 = 3𝑥𝑥3 − 𝑥𝑥4 .


Từ phương trình 3, 4, ta có: 3𝑥𝑥1 − 𝑥𝑥2 = 𝑥𝑥4 − 3𝑥𝑥3
⇒ 3𝑥𝑥1 − 𝑥𝑥2 = 0
Từ phương trình 1, 3, ta có: 𝑥𝑥1 + 3𝑥𝑥2 = 3𝑥𝑥3 − 𝑥𝑥4 .
Từ phương trình 2, 4, ta có: 𝑥𝑥1 + 3𝑥𝑥2 = 𝑥𝑥4 − 3𝑥𝑥3
⇒ 𝑥𝑥1 + 3𝑥𝑥2 = 0
Vậy ta có 𝑥𝑥1 = 𝑥𝑥2 = 0 ⇒ 𝑥𝑥4 = 𝑥𝑥6 = 1, 𝑥𝑥3 = 𝑥𝑥5 = 13

Bài 12.2. (Câu 4 Olympic 1999) Giải hệ phương trình


⎧ 𝑥𝑥1 + 2𝑥𝑥2 + 3𝑥𝑥3 +. . . +𝑛𝑛𝑥𝑥𝑛𝑛 = 1
� 𝑥𝑥 + 2𝑥𝑥 + 3𝑥𝑥 +. . . +𝑛𝑛𝑥𝑥 = 2
2 3 4 1
⎨ ⋮

⎩𝑥𝑥𝑛𝑛 + 2𝑥𝑥1 + 3𝑥𝑥2 +. . . +𝑛𝑛𝑥𝑥𝑛𝑛−1 = 𝑛𝑛

Giải.

Cộng theo vế tất cả phương trình, ta được


𝑥𝑥1 + 𝑥𝑥2 +. . . +𝑥𝑥𝑛𝑛 = 1
Lấy phương trình thứ 𝑘𝑘 trừ theo vế phương trình thứ 𝑘𝑘 + 1 (𝑘𝑘 < 𝑛𝑛)
(𝑥𝑥1 + 𝑥𝑥2 +. . . +𝑥𝑥𝑛𝑛 ) − 𝑛𝑛𝑥𝑥𝑘𝑘 = 𝑘𝑘 − (𝑘𝑘 + 1)
𝑥𝑥1 + 𝑥𝑥2 +. . . +𝑥𝑥𝑛𝑛 1 2
⇒ 𝑥𝑥𝑘𝑘 = + = (𝑘𝑘 = 1, … , 𝑛𝑛 − 1)
𝑛𝑛 𝑛𝑛 𝑛𝑛
Lấy phương trình thứ 𝑛𝑛 trừ theo vế phương trình thứ nhất, ta được
(𝑥𝑥1 + 𝑥𝑥2 +. . . +𝑥𝑥𝑛𝑛 ) − 𝑛𝑛𝑥𝑥𝑛𝑛 = 𝑛𝑛 − 1
𝑛𝑛 − 2
⇒ 𝑥𝑥𝑛𝑛 = −
𝑛𝑛

Đỗ Minh Triết XII-153 - MATHTASY -


Ôn thi Olympic Toán Đại số Phần XII: Hệ phương trình tuyến tính

Bài 12.3. (Câu 3 Olympic 1996) Cho hệ phương trình tuyến tính gồm 10 phương trình và 11
ẩn số. Biết rằng:
i/ (1992, 1993, …, 2002) là một nghiệm của hệ.
i// Khi xoá cột thứ 𝑗𝑗 trong ma trận hệ số thì ta được một ma trận vuông cấp 10 có định thức đúng
bằng 𝑗𝑗 (𝑗𝑗 = 1, … ,11).
Hãy tìm nghiệm tổng quát của hệ đã cho.

Giải.

Ký hiệu 𝐴𝐴 = �𝑎𝑎𝑖𝑖𝑖𝑖 �10×11 là ma trận hệ số của hệ phương trình.


Từ giả thiết 2), thấy ngay rank 𝐴𝐴 = 10 nên hệ phương trình thuần nhất tương ứng 𝐴𝐴𝐴𝐴 = 0 chỉ có
một nghiệm độc lập tuyến tính.
Từ giả thiết 1), (1992, 1993, … , 2002) là một nghiệm riêng của hệ, do đó, nghiệm tổng quát của
nó có dạng sau
(𝑥𝑥1 , 𝑥𝑥2 , … , 𝑥𝑥11 ) = (1992, 1993, … , 2002) + (𝑎𝑎1 , 𝑎𝑎2 , … , 𝑎𝑎11 )𝑡𝑡 , ∀ 𝑡𝑡 ∈ ℝ
trong đó (𝑎𝑎1 , 𝑎𝑎2 , … , 𝑎𝑎11 ) là một nghiệm riêng của hệ thuần nhất tương ứng 𝐴𝐴𝐴𝐴 = 0 và vấn đề đặt
ra là ta cần tìm một nghiệm riêng này.
Với mỗi 𝑗𝑗 = 1, … ,11, ký hiệu 𝐵𝐵𝑗𝑗 là ma trận vuông cấp 11 có dòng đầu là dòng thứ 𝑗𝑗 của ma trận
𝐴𝐴, 10 dòng còn lại tương ứng vẫn là các dòng của 𝐴𝐴, nghĩa là
𝑎𝑎𝑗𝑗1 𝑎𝑎𝑗𝑗2 … 𝑎𝑎𝑗𝑗10 𝑎𝑎𝑗𝑗11
⎛ 𝑎𝑎11 𝑎𝑎12 … 𝑎𝑎1,10 𝑎𝑎1,11 ⎞

⎜ ⎟
⎜ ⋮ ⋮ … ⋮ ⋮ ⎟ ⎟

⎜ ⎟
𝐵𝐵𝑗𝑗 = ⎜ 𝑎𝑎 𝑗𝑗1 𝑎𝑎 𝑗𝑗2 … 𝑎𝑎𝑗𝑗10 𝑎𝑎𝑗𝑗11 ⎟


⎜ ⎟
⎜ ⋮ ⋮ … ⋮ ⋮ ⎟ ⎟

⎜ 𝑎𝑎91 ⎟
𝑎𝑎92 … 𝑎𝑎9,10 𝑎𝑎9,11 ⎟
⎝𝑎𝑎10,1 𝑎𝑎10,2 … 𝑎𝑎10,10 𝑎𝑎10,11 ⎠
Như vậy �𝐵𝐵𝑗𝑗 � = 0 do có 2 dòng giống nhau. Mặt khác, khai triển �𝐵𝐵𝑗𝑗 � theo dòng đầu và áp dụng
giả thiết 2), ta có
𝑎𝑎𝑗𝑗1 . 1 + 𝑎𝑎𝑗𝑗2 . (−2) + 𝑎𝑎𝑗𝑗3 . 3+. . . +𝑎𝑎𝑗𝑗10 . (−10) + 𝑎𝑎𝑗𝑗11 . 11 = 0 ∀ 𝑗𝑗 = 1, … ,11
Suy ra (1, −2, 3, −4, 5, −6, 7, −8, 9, −10, 11) là một nghiệm riêng của hệ thuần nhất 𝐴𝐴𝐴𝐴 = 0.
Vậy hệ đã cho có nghiệm tổng quát
(𝑥𝑥1 , 𝑥𝑥2 , … , 𝑥𝑥11 ) = (1992, 1993, … , 2002) + (1, −2, 3, −4, 5, −6, 7, −8, 9, −10, 11)𝑡𝑡
= (1992 + 𝑡𝑡, 1993 − 2𝑡𝑡, … , 2002 + 11𝑡𝑡)∀ 𝑡𝑡 ∈ ℝ.

Bài 12.4. Giả sử 𝑎𝑎 ∈ ℝ∗ . Chứng minh rằng hệ phương trình sau luôn có nghiệm với mọi
𝑏𝑏, 𝑐𝑐, 𝑑𝑑 ∈ ℝ

Đỗ Minh Triết XII-154 - MATHTASY -


Ôn thi Olympic Toán Đại số Phần XII: Hệ phương trình tuyến tính

⎧ 𝑎𝑎𝑎𝑎 + (1 − 𝑏𝑏)𝑦𝑦 + 𝑐𝑐𝑐𝑐 + (1 − 𝑑𝑑)𝑡𝑡 = 𝑎𝑎



� (𝑏𝑏 − 1)𝑥𝑥 + 𝑎𝑎𝑎𝑎 + (𝑑𝑑 − 1)𝑧𝑧 + 𝑐𝑐𝑐𝑐 = 𝑏𝑏
⎨−𝑐𝑐𝑐𝑐 + (1 − 𝑑𝑑)𝑦𝑦 + 𝑎𝑎𝑎𝑎 + (𝑏𝑏 − 1)𝑡𝑡 = 𝑐𝑐


⎩ (𝑑𝑑 − 1)𝑥𝑥 − 𝑐𝑐𝑐𝑐 + (1 − 𝑏𝑏)𝑧𝑧 + 𝑎𝑎𝑎𝑎 = 𝑑𝑑

Chứng minh.

Gọi 𝐴𝐴 là ma trận các hệ số của hệ phương trình, 𝐴𝐴𝑇𝑇 là ma trận chuyển vị của ma trận 𝐴𝐴, ta có

𝐴𝐴𝐴𝐴𝑇𝑇 = [𝑎𝑎2 + (1 − 𝑏𝑏)2 + 𝑐𝑐2 + (1 − 𝑑𝑑)2 ]𝐼𝐼

Do đó:

det 𝐴𝐴 = [𝑎𝑎2 + (1 − 𝑏𝑏)2 + 𝑐𝑐2 + (1 − 𝑑𝑑)2 ] ≠ 0 ∀𝑎𝑎 ∈ ℝ∗ , 𝑏𝑏, 𝑐𝑐, 𝑑𝑑 ∈ ℝ

Vậy hệ luôn có nghiệm với mọi 𝑏𝑏, 𝑐𝑐, 𝑑𝑑 ∈ ℝ.

Đỗ Minh Triết XII-155 - MATHTASY -


PHẦN XIII. MỘT SỐ BÀI TOÁN KHÁC

Bài 13.1. (Câu 3 Olympic 1996) Cho


2 0 0 𝑛𝑛 𝑎𝑎11 (𝑛𝑛) 𝑎𝑎12 (𝑛𝑛) 𝑎𝑎13 (𝑛𝑛)
�0 3 0� = ⎜𝑎𝑎21 (𝑛𝑛) 𝑎𝑎22 (𝑛𝑛) 𝑎𝑎23 (𝑛𝑛)⎞
⎛ ⎟
0 1 2 𝑎𝑎
⎝ 31 (𝑛𝑛) 𝑎𝑎 32 (𝑛𝑛) 𝑎𝑎 33 (𝑛𝑛) ⎠
𝑎𝑎22 (𝑛𝑛)
Tính lim
𝑛𝑛→∞ 𝑎𝑎32 (𝑛𝑛)

Giải.

Ta có
𝑎𝑎11 (𝑛𝑛 + 1) 𝑎𝑎12 (𝑛𝑛 + 1) 𝑎𝑎13 (𝑛𝑛 + 1) 2 0 0 𝑛𝑛+1 2 0 0 2 0 0 𝑛𝑛

⎜𝑎𝑎21 (𝑛𝑛 + 1) 𝑎𝑎22 (𝑛𝑛 + 1) 𝑎𝑎23 (𝑛𝑛 + 1)⎞ ⎟ = �0 3 0� = �0 3 0� �0 3 0�
⎝𝑎𝑎31 (𝑛𝑛 + 1) 𝑎𝑎32 (𝑛𝑛 + 1) 𝑎𝑎33 (𝑛𝑛 + 1)⎠ 0 1 2 0 1 2 0 1 2
2 0 0 𝑎𝑎11 (𝑛𝑛) 𝑎𝑎12 (𝑛𝑛) 𝑎𝑎13 (𝑛𝑛)

= �0 3 0� ⎜𝑎𝑎21 (𝑛𝑛) 𝑎𝑎22 (𝑛𝑛) 𝑎𝑎23 (𝑛𝑛)⎞⎟
0 1 2 ⎝𝑎𝑎31 (𝑛𝑛) 𝑎𝑎32 (𝑛𝑛) 𝑎𝑎33 (𝑛𝑛)⎠
Đồng nhất, ta thu được
𝑎𝑎 (𝑛𝑛 + 1) = 3𝑎𝑎22 (𝑛𝑛)
� 22
𝑎𝑎32 (𝑛𝑛 + 1) = 𝑎𝑎22 (𝑛𝑛) + 2𝑎𝑎32 (𝑛𝑛)
𝑎𝑎 (𝑛𝑛)
3. 22
𝑎𝑎22 (𝑛𝑛) 𝑎𝑎22 (𝑛𝑛 + 1) 3𝑎𝑎22 (𝑛𝑛) 𝑎𝑎32 (𝑛𝑛) 3𝑢𝑢𝑛𝑛
Đặt 𝑢𝑢𝑛𝑛 = ⇒ 𝑢𝑢1 = 3, 𝑢𝑢𝑛𝑛+1 = = = =
𝑎𝑎32 (𝑛𝑛) 𝑎𝑎32 (𝑛𝑛 + 1) 𝑎𝑎22 (𝑛𝑛) + 2𝑎𝑎32 (𝑛𝑛) 𝑎𝑎22 (𝑛𝑛) 𝑢𝑢𝑛𝑛 + 2
+2
𝑎𝑎32 (𝑛𝑛)
Bằng quy nạp, ta chứng minh được rằng dãy {𝑢𝑢𝑛𝑛 } đơn điệu giảm và 𝑢𝑢𝑛𝑛 ≥ 1. Suy ra tồn tại giới
hạn lim 𝑢𝑢𝑛𝑛 = 𝑎𝑎 ≥ 1, chuyển qua giới hạn đẳng thức cuối cùng, ta được 𝑎𝑎 = 𝑎𝑎+2
3𝑎𝑎
, tức 𝑎𝑎 = 1.
𝑛𝑛→∞

Bài 13.2. (Câu 4 Olympic Quốc tế 1997) Cho 𝐴𝐴, 𝐵𝐵 ∈ 𝑀𝑀𝑛𝑛 (ℝ) thoả 𝐴𝐴2 + 𝐵𝐵2 = 𝐴𝐴𝐴𝐴. Chứng
minh rằng nếu 𝐵𝐵𝐵𝐵 − 𝐴𝐴𝐴𝐴 khả nghịch thì 𝑛𝑛 chia hết cho 3.

Chứng minh.

Đặt 𝑆𝑆 = 𝐴𝐴 + 𝑧𝑧𝑧𝑧, trong đó 𝑧𝑧 = − 12 + 𝑖𝑖 2
3
, ta có
�������������������
𝑆𝑆𝑆𝑆 ̅ = (𝐴𝐴 + 𝑧𝑧𝑧𝑧)�𝐴𝐴 ̅ ) = 𝐴𝐴2 + 𝑧𝑧𝑧𝑧𝑧𝑧 + 𝑧𝑧 𝐴𝐴
+ 𝑧𝑧𝑧𝑧� = (𝐴𝐴 + 𝑧𝑧𝑧𝑧)(𝐴𝐴 + 𝑧𝑧 𝐵𝐵 ̅ 𝐴𝐴 + 𝐵𝐵2
= 𝐴𝐴𝐴𝐴 + 𝑧𝑧𝑧𝑧𝑧𝑧 + 𝑧𝑧 𝐴𝐴
̅ 𝐴𝐴 = 𝑧𝑧𝑧𝑧𝑧𝑧 + (1 + 𝑧𝑧 )̅ 𝐴𝐴𝐴𝐴 = 𝑧𝑧(𝐵𝐵𝐵𝐵 − 𝐴𝐴𝐴𝐴)
Ta có det�𝑆𝑆𝑆𝑆 �̅ = det 𝑆𝑆 det 𝑆𝑆 ̅ = det 𝑆𝑆 det
�������������
𝑆𝑆 = |det 𝑆𝑆|2 là một số thực.

Đỗ Minh Triết XIII-156 - MATHTASY -


Ôn thi Olympic Toán Đại số Một số thuật ngữ Anh - Việt

Mặt khác, nếu 𝐵𝐵𝐵𝐵 − 𝐴𝐴𝐴𝐴 khả nghịch thì det�𝑆𝑆𝑆𝑆 �̅ = det[𝑧𝑧(𝐵𝐵𝐵𝐵 − 𝐴𝐴𝐴𝐴)] = 𝑧𝑧 𝑛𝑛 det(𝐵𝐵𝐵𝐵 − 𝐴𝐴𝐴𝐴) ≠ 0.
det�𝑆𝑆𝑆𝑆 �̅ , det(𝐵𝐵𝐵𝐵 − 𝐴𝐴𝐴𝐴) là số thực nên 𝑧𝑧 𝑛𝑛 cũng phải là số thực, tức là phần ảo của nó phải bằng
0, mà
2𝜋𝜋 2𝜋𝜋 𝑛𝑛 2𝑛𝑛𝑛𝑛 2𝑛𝑛𝑛𝑛
𝑧𝑧 𝑛𝑛 = �cos + 𝑖𝑖 sin � = cos + 𝑖𝑖 sin
3 3 3 3
Suy ra phần ảo của 𝑧𝑧 𝑛𝑛 bằng 0 khi và chỉ khi 𝑛𝑛 chia hết cho 3.

Bài 13.3. (Tính chất ma trận luỹ linh – Wikipedia)


Mọi ma trận suy biến đều có thể viết thành tích của các ma trận lũy linh.

Chứng minh.

Bài 13.4. (Câu 5 đề chọn đội tuyển ĐH Kinh tế Quốc dân 2005)
Cho 𝐴𝐴, 𝐵𝐵 là các ma trận vuông cấp 𝑛𝑛 ≥ 2 thoả mãn 𝐴𝐴𝐴𝐴 = 𝐴𝐴 + 𝐵𝐵 và 𝐴𝐴2005 = 0. Khi đó hệ phương
trình tuyến tính thuần nhất 𝑛𝑛 ẩn số 𝑥𝑥1 , … , 𝑥𝑥𝑛𝑛 nhận 𝐵𝐵 làm ma trận hệ số có bao nhiêu nghiệm? Giải
thích.

Giải.

Những thứ rút ra được:


Từ giả thiết 𝑨𝑨𝑨𝑨 = 𝑨𝑨 + 𝑩𝑩 dễ dàng suy ra 𝑨𝑨𝑨𝑨 = 𝑩𝑩𝑩𝑩 và 𝐫𝐫𝐫𝐫𝐫𝐫𝐫𝐫 𝑨𝑨 = 𝐫𝐫𝐫𝐫𝐫𝐫𝐫𝐫 𝑩𝑩.
Giả sử 𝑥𝑥0 là nghiệm của hệ 𝐵𝐵𝐵𝐵 = 0, khi đó 0 = 𝐵𝐵𝑥𝑥0 = (𝐴𝐴𝐴𝐴 − 𝐴𝐴)𝑥𝑥0 = 𝐴𝐴𝐴𝐴𝑥𝑥0 − 𝐴𝐴𝑥𝑥0 = −𝐴𝐴𝑥𝑥0 tức
là 𝑥𝑥0 cũng là nghiệm của hệ 𝐴𝐴𝐴𝐴 = 0.
Ngược lại, giả sử 𝑥𝑥0 là nghiệm của hệ 𝐴𝐴𝐴𝐴 = 0, khi đó 𝐵𝐵𝑥𝑥0 = (𝐴𝐴𝐴𝐴 − 𝐴𝐴)𝑥𝑥0 = 𝐴𝐴𝐴𝐴𝑥𝑥0 − 𝐴𝐴𝑥𝑥0 =
𝐵𝐵(𝐴𝐴𝑥𝑥0 ) = 0.
Từ đó suy ra hai hệ 𝑨𝑨𝑨𝑨 = 𝟎𝟎 và 𝑩𝑩𝑩𝑩 = 𝟎𝟎 có cùng tập nghiệm.
𝐴𝐴2005 = 0 nên det 𝐴𝐴 = 0 và 𝐼𝐼 − 𝐴𝐴 khả nghịch (𝐼𝐼 − 𝐴𝐴)−1 = (𝐼𝐼 + 𝐴𝐴+. . . +𝐴𝐴2004 )
Từ 𝐴𝐴 = 𝐴𝐴𝐴𝐴 − 𝐵𝐵 = −(𝐼𝐼 − 𝐴𝐴)𝐵𝐵 ⇒ −𝐴𝐴 − 𝐴𝐴2 −. . . −𝐴𝐴2004 − 𝐴𝐴2005 = 𝐵𝐵
rank 𝐴𝐴 = rank 𝐵𝐵 = rank(𝐴𝐴 + 𝐴𝐴2 +. . . +𝐴𝐴2005 )

Bài 13.5. (Câu 2b đề chọn đội tuyển ĐH Giao thông vận tải 2011)
Cho 𝐴𝐴, 𝐵𝐵 ∈ 𝑀𝑀𝑛𝑛 (ℤ), 𝐴𝐴𝐴𝐴 = 𝐵𝐵𝐵𝐵, det 𝐵𝐵 = 1. Chứng minh rằng nếu det(𝐴𝐴3 + 𝐵𝐵3 ) = 1 thì 𝐴𝐴2 = 𝐼𝐼.

Chứng minh.

Nếu 1 = det(𝐴𝐴3 + 𝐵𝐵3 ) = det(𝐴𝐴 + 𝐵𝐵) det(𝐴𝐴2 − 𝐴𝐴𝐴𝐴 + 𝐵𝐵2 )

Đỗ Minh Triết XIII-157 - MATHTASY -


Ôn thi Olympic Toán Đại số Một số thuật ngữ Anh - Việt

⇒ det(𝐴𝐴 + 𝐵𝐵) = det(𝐴𝐴2 − 𝐴𝐴𝐴𝐴 + 𝐵𝐵2 ) = ±1


1 = det(𝐴𝐴3 + 𝐵𝐵3 ) det 𝐵𝐵 = det(𝐴𝐴 + 𝐵𝐵) det(𝐴𝐴2 − 𝐴𝐴𝐴𝐴 + 𝐵𝐵2 ) det 𝐵𝐵
1 = det(𝐴𝐴3 + 𝐵𝐵3 ) det 𝐵𝐵 = det(𝐴𝐴 + 𝐵𝐵) det�𝐴𝐴2 − 𝐵𝐵(𝐴𝐴 + 𝐵𝐵)� det 𝐵𝐵

Bài 13.6. *Cho hệ phương trình tuyến tính tổng quát dưới dạng 𝐴𝐴𝐴𝐴 = 𝐵𝐵, trong đó 𝐴𝐴 = �𝑎𝑎𝑖𝑖𝑖𝑖 �𝑛𝑛×𝑚𝑚 ,
𝑋𝑋 = (𝑥𝑥1 … 𝑥𝑥𝑛𝑛 )𝑇𝑇 và 𝐵𝐵 = (𝑏𝑏1 … 𝑏𝑏𝑚𝑚 )𝑇𝑇 . Giả sử 𝑋𝑋0 là một nghiệm riêng của hệ 𝐴𝐴𝐴𝐴 = 𝐵𝐵. Chứng minh
rẳng 𝑋𝑋 là nghiệm của hệ 𝐴𝐴𝐴𝐴 = 𝐵𝐵 nếu và chỉ nếu nó có dạng 𝑋𝑋 = 𝑋𝑋0 + 𝑋𝑋 ′ với 𝑋𝑋 ′ là nghiệm tổng
quát của phương trình thuần nhất 𝐴𝐴𝐴𝐴 = 0.
1 −2 2 −1 1
*Với 𝐴𝐴 = �2 −4 3 1 � , 𝐵𝐵 = �3� như câu trên, hãy biểu diễn mọi nghiệm của hệ 𝐴𝐴𝐴𝐴 =
3 −6 4 3 5
𝐵𝐵 dưới dạng tổng của một nghiệm riêng của nó với nghiệm tổng quát của phương trình thuần
nhất tương ứng.

Trong Đại số tuyến tính (Linear Algebra) có hai ngôn ngữ biểu đạt vấn đề, đó là ngôn ngữ ma
trận (Matrix) và ngôn ngữ ánh xạ tuyến tính (Linear Mapping, Linear Transformation). Đôi khi
việc đưa ngôn ngữ này sang ngôn ngữ khác để giải quyết vấn đề sẽ trở nên đơn giản hơn và thường
có những vấn đề phải giải quyết bằng cả hai ngôn ngữ đan xen, do đó người làm Toán phải nắm
vững cả hai cách biểu đạt. Trong kì thi Olympic Toán Sinh viên toàn quốc, ta thường dùng nhiều
đến ngôn ngữ ma trận hơn nhưng nếu sử dụng đến ngôn ngữ ánh xạ tuyến tính, có những vấn đề
trở nên đơn giản cực kỳ.

Đỗ Minh Triết XIII-158 - MATHTASY -


MỘT SỐ THUẬT NGỮ VIỆT - ANH
A
ảnh của ánh xạ tuyến tính image of a linear mapping
B
bù vuông góc của không gian con orthogonal complement of a subspace
C
cơ sở của không gian vector basis for a vector space
D
độ dài của vector norm of a vector
độc lập tuyến tính linear independent set
G
giao của hai không gian con intersection of two subspaces
góc giữa hai vector angle between two vectors
H

hình chiếu vuông góc của vector xuống không orthogonal projection of a vector onto
gian con subspace
K
khoảng cách giữa hai vector distance of two vectors
khoảng cách từ vector đến không gian con distance from the vecto v to the subspace
không gian con subspace
không gian con riêng eigenspace
không gian Euclide Euclidean space
không gian nghiệm của hệ AX=b null space of A
không gian vector vector space
M
ma trận chéo diagonal matrix
ma trận chuyển cơ sở từ E sang F the change of bases from E to F
ma trận của ánh xạ t/tính trong cơ sở E, F matrix of a linear mapping with respect to two
bases E and F
ma trận dạng bậc thang echelon form, rref = row reduced echelon form
ma trận đơn vị identity matrix

Đỗ Minh Triết XIII-159 - MATHTASY -


Ôn thi Olympic Toán Đại số Một số thuật ngữ Anh - Việt

ma trận hệ số (của hệ AX =b) coefficient matrix


ma trận không zero matrix
ma trận mở rộng (A|b) augmented matrix
ma trận tam giác dưới lower triangular matrix
ma trận tam giác trên upper triangular matrix
ma trận vuông square matrix
N
nhân của ánh xạ tuyến tính kernel of a linear mapping
H
hệ cramer cramer’s system
hệ phương trình tuyến tính system of linear equations, linear system
hệ thuần nhất homogenuous system
P
phân tích LU của ma trận A LU factorization
phân tích QR của ma trận AQ R factorization
phụ thuộc tuyến tính linear dependent set
phương pháp khử Gauss Gaussian elimination
T
tập sinh của không gian vector spanning set for a vector space
tọa độ của vector trong cơ sở E coordinates of a vector with respect to the
basis
tổ hợp tuyến tính linear combination
tổng của hai không gian con sum of two subspaces
S
số chiều của không gian vector dimension of a vector space

Đỗ Minh Triết XIII-160 - MATHTASY -

You might also like